You are on page 1of 114

nephSAP

Nephrology Self-Assessment Program

Volume 20  Number 2  January 2022

Electrolytes and Acid-Base Disorders

Guest Editor:
Julian L. Seifter, MD

Editorial Director:
Alice M. Sheridan, MD

Deputy Editor:
Martina M. McGrath, MBChB, FASN
nephSAP
EDITORIAL DIRECTOR, nephSAP Preface
Alice M. Sheridan, MD Launched in 2002, the Nephrology Self-Assessment Program (nephSAP®) is one of the premier educational
Brigham and Women’s Hospital activities of the American Society of Nephrology (ASN) Alliance for Kidney Health. nephSAP focuses on
Boston, MA
self-assessment and education and provides Continuing Medical Education (CME) credits and Maintenance
DEPUTY EDITOR, nephSAP of Certification (MOC) points for individuals certified by the American Board of Internal Medicine (ABIM).
Martina M. McGrath, MBChB, FASN Starting with Volume 19 (2020), nephSAP content is available electronically through the nephSAP website
Brigham and Women’s Hospital (nephsap.org). Content for Volume 18 and earlier, and all archived nephSAP content, are available on the
Boston, MA main ASN website (asn-online.org/nephsap).
GUEST EDITOR PROGRAM MISSION AND OBJECTIVES: nephSAP® provides a learning vehicle for physicians,
Julian L. Seifter, MD scientists, advanced practitioners, nurses, pharmacists, and other health professionals working and training in
Brigham and Women’s Hospital the field of nephrology to renew and refresh their clinical knowledge, diagnostic, and therapeutic skills. This
Boston, MA enduring material provides learners challenging, clinically-oriented questions based on case vignettes, detailed
learning objectives, a series of articles that review recent publications, and an editorial on an important
SENIOR PROGRAM MANAGER FOR
and evolving topic. This combination of materials enables learners to rigorously assess their strengths and
SELF-ASSESSMENT
weaknesses in the broad domain of nephrology.
Suzanne P. Armstrong, MA
American Society of Nephrology EDUCATION: nephSAP® supports continuous improvement and lifelong learning by helping busy clinicians
Washington, DC and trainees at all levels stay current with the latest scientific and medical advances in kidney care. Each issue
of nephSAP is dedicated to a specific theme, i.e., to a specific area of clinical nephrology, hypertension,
PAST EDITORS
dialysis, and transplantation, and consists of an editorial, topical articles, and self-assessment questions. All
Jeffrey S. Berns, MD, FASN
clinically relevant and key elements of nephrology are reviewed and updated every two years. The authors of
Ronald J. Falk, MD, FASN
each issue digest and interpret key studies published since the release of the previous issue and integrate this
Stanley Goldfarb, MD, PhD
new material with the body of existing information. Occasionally a special edition is produced to cover an
Gerald A. Hladik, MD, FASN
Raymond R. Townsend, MD area not ordinarily addressed by core issues of nephSAP.
Jerry Yee, MD, FASN SELF-ASSESSMENT: An examination with thirty, single-best-answer questions follow the 60 to 100
FOUNDING EDITORS pages of topical articles. This examination is available online with immediate feedback which includes answers
to all questions, brief discussions, and an updated bibliography.
Richard J. Glassock, MD
Editor-in-Chief Emeritus CME and MOC: Those answering 75% correctly can claim up to 10 CME credits and MOC points per issue.
Robert G. Narins, MD

CONTRIBUTING AUTHORS
Robert Jay Alpern, MD Solomiia Savedchuk, MD
Yale University School of Medicine Duke University Medical Center
New Haven, CT Durham, NC

Tushar Chopra, MD, FASN Zhabiz Solhjou, MD


University of Virginia Health System Brigham and Women’s Hospital
Charlottesville , VA Boston, MA

Thomas D. DuBose, Jr., MD, FASN Andrew S. Terker, MD, PhD


Wake Forest School of Medicine Vanderbilt University Medical Center
Winston Salem, NC Nashville, TN

David H. Ellison, MD, FASN Naoto Tominaga, MD, PhD


Oregon Health & Science University Kawasaki Municipal Tama Hospital
Portland, OR Kawasaki Kanagawa
Japan
Namrata Krishnan, MD
Yale School of Medicine Joseph G. Verbalis, MD
New Haven, CT Georgetown University
Washington, DC
Ruediger W. Lehrich, MD
Duke University Medical Center
Durham, NC

Martina M. McGrath,
MBChB, FASN
Brigham and Women’s Hospital
Boston, MA

Send Your Feedback nephsap@asn-online.org nephSAP® ©2022 by The American Society of Nephrology
nephSAP Volume 20 4 Number 2 4 January 2022

Editorial Hypotonic Hyponatremia (<280 mOsm/kg


H2O) ......................................................................... 78
Teaching Renal Physiology at the Bedside ............................. 67
Julian L. Seifter Assess ECF Volume ............................................................ 78
Common Causes of Hyponatremia related to Nonosmotic
Case 1: Why Would a Healthy Subject in a Research Study, in and/or Inappropriate AVP Release .......................................... 80
the Midst of a Water Diuresis, Suddenly Become Anuric?....... 67
Syndrome of Inappropriate Antidiuretic Hormone Secretion
Case 2: What Stimulates Release of Atrial Natriuretic Peptide versus Syndrome of Inappropriate Antidiuresis .................... 80
from the Heart? ......................................................................... 68
Drug-Induced Hyponatremia ............................................. 80
Case 3: Can the Urine Be Concentrated in the Absence of
Antidiuretic Hormone?............................................................. 69 Iatrogenic Hyponatremia .................................................... 81

Case 4: Can a Patient Have Diabetes Insipidus of Pregnancy Symptoms and Adverse Outcomes of Hyponatremia .............. 82
Manifesting Only in the Postpartum Period? .......................... 71 Acute Hyponatremia .......................................................... 82
CASE 5: Can Lithium Cause Both Central and Nephrogenic Chronic Hyponatremia ...................................................... 82
Diabetes Insipidus?................................................................... 72
Treatment of Hyponatremia .................................................... 82
References ................................................................................ 73
Current Recommendations regarding Rate of Serum [Na+]
Articles Correction .......................................................................... 82
Pathophysiology, Evaluation, Outcomes, and Treatment of Osmotic Demyelination Syndrome ..................................... 85
Hyponatremia........................................................................... 74
Overly Rapid Correction of Serum [Na+]: Risks and
Naoto Tominaga and Joseph G. Verbalis Prevention .......................................................................... 85
Learning Objectives.................................................................. 74 Conventional, Specific, and Potential Future Treatment
Options for Hyponatremia .................................................. 86
Pathophysiology of Hyponatremia ......................................... 74
Fluid Restriction ....................................................... 86
Definition and Prevalence of Hyponatremia ........................ 74
Furosemide with NaCl in Combination with Fluid
Sodium Concentration and Sodium Content: Two Related Restriction ................................................................. 87
but Different Concepts........................................................ 74
Vasopressin Receptor Antagonists............................. 87
Total Plasma Osmolality and Effective Plasma Osmolality
(Tonicity)............................................................................ 74 Urea ........................................................................... 87
Two Defense Systems Protecting against Body Water Sodium-Glucose Cotransporter-2 Inhibitors in
Losses.................................................................................. 75 Combination with Fluid Restriction ........................ 87
Arginine Vasopressin: Appropriate Secretion ....................... 75 Conclusion ............................................................................... 87
Nonosmotic AVP Stimuli: Inappropriate Secretion.............. 76 References ................................................................................ 88
+
Etiologies of Disorders of Serum [Na ] ................................ 76 Pathophysiology, Evaluation, and Treatment of
Evaluation of Hyponatremia ................................................... 77 Hypernatremia ......................................................................... 90
Solomiia Savedchuk and Ruediger W. Lehrich
Check Plasma Osmolality (Posm) ......................................... 77
Isotonic Hyponatremia (280–295 mOsm/kg Learning Objectives.................................................................. 90
H2O) ......................................................................... 77
Physiology of Osmoregulation, Thirst, and Urinary
Hypertonic Hyponatremia (>295 mOsm/kg Concentration .......................................................................... 90
H2O) ......................................................................... 78
Regulation of Thirst ............................................................ 90
nephSAP Volume 20 4 Number 2 4 January 2022

Homeostatic and Anticipatory Thirst .................................. 90 Potassium Absorption in Distal Convoluted Tubule 2
(DCT2) and Collecting Duct (CD) by the H+,
Urinary Concentration ........................................................ 91
K+-ATPases ....................................................................... 104
Incidence of and Risk Factors for Hypernatremia ................... 91
Role of Hypokalemia in Maintenance of Metabolic
Hospital-Acquired Hypernatremia ...................................... 91 Alkalosis.................................................................................. 104
Intensive Care Unit Hypernatremia..................................... 91 Clinical Approach to the Hypokalemic Patient .................. 105
Community-Acquired Hypernatremia................................. 92 Causes of Hypokalemia ..................................................... 106
Salt Intoxication .................................................................. 92 Low Dietary K+ Intake ........................................................... 106
Athletic Activity as a Risk Factor for Hypernatremia ............ 92 Potassium Intake in CKD Patients .................................... 107
Prognosis of Hypernatremia .................................................... 92 Redistribution of Potassium .............................................. 107
Permissive Hypernatremia ....................................................... 94 Extrarenal K+ Loss ............................................................ 109
Diabetes Insipidus .................................................................... 94 Increased Renal K+ Excretion ................................................. 110
Medication-Induced Diabetes Insipidus .............................. 94 Increased Distal Sodium Delivery ...................................... 110
Diagnosis of Diabetes Insipidus ........................................... 95 Hypokalemia and Nongap Metabolic Acidosis................... 110
Treatment of Hypernatremia ................................................... 96 The Urine Anion Gap and the Urine Osmolar Gap............ 111
Donor and Recipient Hypernatremia in Solid Organ Hypokalemia with Metabolic Alkalosis, Volume Depletion,
Transplantation ........................................................................ 99 and Normal or Low BP—High Urine [Cl]: Bartter
Syndrome and Gitelman Syndrome ................................... 111
Heart................................................................................... 99
Assessment of Renal Potassium Excretion .......................... 113
Liver.................................................................................... 99
Treatment of Hypokalemia ............................................... 113
Lung ................................................................................... 99
Complications and Outcomes ................................................ 113
Kidney and Pancreas............................................................ 99
Acute Complications ......................................................... 113
References ................................................................................. 99
Muscle Weakness..................................................... 113
Pathophysiology, Evaluation, and Treatment of
Hypokalemia .......................................................................... 102 Rhabdomyolysis ...................................................... 114
Tushar Chopra and Thomas D. DuBose Jr Cardiac Arrhythmia........................................................... 114

Learning Objectives................................................................ 102 Long-Term Outcomes....................................................... 114

Introduction ........................................................................... 102 Hypokalemic Nephropathy (Kaliopenic


Nephropathy) .......................................................... 114
Total Body Potassium Distribution ....................................... 102
New-Onset Diabetes ................................................ 114
Regulation of Potassium Homeostasis ................................... 102
References .............................................................................. 114
Cell Shifts ......................................................................... 102
Renal Potassium Absorption and Secretion ........................ 103 Pathophysiology, Evaluation, and Treatment of
Hyperkalemia ......................................................................... 117
Proximal Tubule ..................................................... 103
Andrew S. Terker and David H. Ellison
Thick Ascending Limb of Henle (TALH) ............... 103
Learning Objectives................................................................ 117
Distal Convoluting Tubule (DCT) ......................... 103
Introduction ........................................................................... 117
nephSAP Volume 20 4 Number 2 4 January 2022

Cardiovascular Outcomes and Mortality ............................... 117 Chronic Metabolic Acidosis in CKD...................................... 135
Total Body Potassium Distribution ....................................... 117 Background ...................................................................... 135
Regulation of Potassium Homeostasis ................................... 117 Eubicarbonatemic Metabolic Acidosis and Urine Citrate ... 135
Gastrointestinal Absorption............................................... 117 Mechanisms of Injury Associated with Chronic Metabolic
Acidosis in CKD ............................................................... 136
Renal Potassium Reabsorption and Secretion..................... 117
Treatment of Metabolic Acidosis in CKD.............................. 137
Proximal Tubule...................................................... 117
Veverimer ............................................................................... 140
Thick Ascending Limb of Henle.............................. 118
Metabolic Acidosis and Kidney Transplant Outcomes.......... 141
Distal Convoluted Tubule....................................... 118
References ............................................................................... 142
Connecting Tubule and Collecting Duct ................ 118
Integrated Renal Potassium Excretion: The Potassium Metabolic Alkalosis ................................................................ 145
Switch ..................................................................................... 118 Namrata Krishnan and Robert Jay Alpern
Diagnosis and Evaluation....................................................... 120
Learning Objectives ............................................................... 145
Causes and Risk Factors .................................................... 121
Introduction .......................................................................... 145
Prevention............................................................................... 122
Mechanisms That Prevent Metabolic Alkalosis ..................... 145
Treatment ............................................................................... 122
Buffering .......................................................................... 145
Acute Hyperkalemia .......................................................... 122
Respiratory Compensation ............................................... 145
Stabilize the Membrane ........................................... 123
Renal Excretion ................................................................. 145
Shift Potassium into Cells ....................................... 123
Generation and Maintenance of Sustained Metabolic
Remove Potassium from the Body .......................... 123 Alkalosis ................................................................................. 146
Gastrointestinal Potassium Removal ...................... 124 Factors That Result in Maintenance of Metabolic
Chronic Hyperkalemia ..................................................... 124 Alkalosis ........................................................................... 146

References .............................................................................. 126 Volume Contraction ................................................ 146


Potassium Depletion ............................................... 147
Metabolic Acidosis ................................................................. 130
Enhanced Distal Acidification Due to Imbalances
Zhabiz Solhjou and Martina McGrath
in Distal Nephron Na+Absorptive Capacity and
Distal Delivery of Na+ and Water............................ 147
Learning Objectives................................................................ 130
Defects in Distal Bicarbonate Secretion.................. 147
Introduction .......................................................................... 130
Factors That Result in Generation of Metabolic Alkalosis... 147
Lactic Acidosis ........................................................................ 130
External H+ Loss ..................................................... 148
Ketoacidosis............................................................................ 131
Exogenous Alkali Gain ............................................ 148
Diabetic Ketoacidosis ........................................................ 131
Metabolic Alkalosis in the Absence of H+ Loss or
Euglycemic Ketoacidosis with Sodium-Glucose
Alkali Gain (potassium depletion) .......................... 148
Transporter-2 Inhibitors.................................................... 131
Continuous RRT and Ketoacidosis.................................... 132 Clinical Conditions That Cause Metabolic Alkalosis ............ 148
Drug-Induced Metabolic Acidosis ..................................... 132 Exogenous Alkali Gain ...................................................... 148
Sodium Thiosulfate ........................................................... 132 Decreased Effective Arterial Blood Volume........................ 148
Metformin ........................................................................ 132 Gastric HCl Loss (Gastric Alkalosis) ....................... 148
nephSAP Volume 20 4 Number 2 4 January 2022

Lower GI HCl Loss .................................................. 149 Kidney Replacement Therapy............................................ 155


Increased Distal Delivery of Sodium....................... 150 Acetazolamide ................................................................... 155
Diuretic Induced ..................................................... 150 Non-sodium Based Acid Solutions .................................... 155
Distal Delivery of Nonreabsorbable Anions ........... 150 Treatment of Specific Clinical Scenarios ............................ 156
Bartter Syndrome .................................................... 150 Diuretic-Induced Metabolic Alkalosis ................... 156
Gitelman Syndrome ................................................ 150 Genetic Disorders Resulting in Secondary
Mineralocorticoid Excess......................................... 156
Magnesium Depletion ............................................. 150
Syndromes of Primary Mineralocorticoid
Hypercalcemia ........................................................ 150
Excess ....................................................................... 156
Post–Hypercapneic Alkalosis .................................. 150
Posthypercapneic Metabolic Alkalosis.................... 156
Decreased Distal Bicarbonate Secretion ............................ 150
References ............................................................................... 157
Pendred Syndrome .................................................. 150
Self-Assessment Questions
Cystic Fibrosis ......................................................... 151
nephSAP Volume 20, Number 2, January 2022..................... 160
Increased Distal Transport Activity.................................... 151
High Renin, High Aldosterone ............................... 151 Planned Issues
Low Renin, High Aldosterone ................................. 151 Acute Kidney Injury and Critical Care Nephrology
Low Renin, Low Aldosterone ................................. 151 Guest Editor: Paul M. Palevsky, MD, FASN
March 2022
Increase in a Nonaldosterone Mineralocorticoid
Receptor Agonist ..................................................... 151 Chronic Kidney Disease
Activation of the Mineralocorticoid Receptor ........ 151 Guest Editor: Valerie A. Luyckx, MD, MS, PhD
April 2022
Activation of Transport Mechanisms Downstream
of the Receptor......................................................... 151 Transplantation
Unclear Mechanism: Postfasting alkalosis ......................... 151 Guest Editor: Sumit Mohan, MD, MPH, FASN
May 2022
Evaluation......................................................................... 151
Epidemiology and Outcomes ................................................ 153 Hypertension
Treatment ............................................................................... 153 Guest Editor: George L. Bakris, MD, FASN
June 2022
Indications for Intervention............................................... 153
Correct Causes of Generation and Maintenance................. 153
nephSAP Volume 2044Number
Volume19 Number00
244 October
January 2022
2020

The nephSAP editorial team and the KSAP editorial board extend their sincere appreciation to the following
reviewers. Their efforts and insights help improve the quality of these postgraduate education offerings.

nephSAP Review Panel


Mustafa Ahmad, MD, FASN Armando Coca, MD, MPH Susie L. Hu, MD, FASN
King Fahad Medical City Hospital Clınico Universitario Warren Alpert Medical School of Brown University
SAUDI ARABIA SPAIN UNITED STATES
Jafar Al-Said, MD, FASN Scott D. Cohen, MD, MPH, FASN Ekambaram Ilamathi, MD, FASN
Bahrain Specialist Hospital George Washington University Northwell Health - Southside Hospital
UNITED STATES UNITED STATES UNITED STATES
Carmichael Angeles, MD, FASN Beatrice P. Concepcion, MD, FASN
Talha Hassan Imam, MD
Pharmaceutical Product Development, LLC Vanderbilt University Medical Center
Kaiser Permanente
UNITED STATES UNITED STATES
UNITED STATES
Kisra Anis, MD Gabriel Contreras, MD, MPH
University of Miami Health System Joshua Kaplan, MD, FASN
Jacobi Medical Center, Albert Einstein College of
UNITED STATES Rutgers New Jersey Medical School
Medicine
UNITED STATES
UNITED STATES Patrick Cunningham, MD, FASN
Naheed Ansari, MD, FASN University of Chicago Medical Center Amir Kazory, MD, FASN
Jacobi Medical Center, Albert Einstein College of UNITED STATES University of Florida
Medicine UNITED STATES
Kevin A. Curran, MD
UNITED STATES Kevin A. Curran, MD, PA Quresh T. Khairullah, MD, FASN
UNITED STATES St. Clair Nephrology
Nabeel Aslam, MD, FASN
Mayo Clinic Florida Rajiv K. Dhamija, MD UNITED STATES
UNITED STATES Rancho Los Amigos National Rehabilitation Center
Yong-Lim Kim, MD, PhD
UNITED STATES
Nisha Bansal, MD, FASN Kyungpook National University Hospital
Kidney Research Institute Alejandro Diez, MD, FASN SOUTH KOREA
UNITED STATES Ohio Sate University
UNITED STATES Nitin V. Kolhe, MD, FASN
Krishna M. Baradhi, MD, FASN Royal Derby Hospital
University of Oklahoma Health Sciences Center John J. Doran, MD, FASN UNITED KINGDOM
UNITED STATES Emory University School of Medicine
UNITED STATES Farrukh M. Koraishy, MD, PhD, FASN
Gopal Basu, MD, MBBS, FASN Stony Brook University New York
The Alfred Hospital Lynda A. Frassetto, MD, FASN
UNITED STATES
AUSTRALIA University of California San Francisco
UNITED STATES Eugene C. Kovalik, MD, FASN
Emmy Klip Bell, MD, MPH Duke University Medical Center
University of Alabama at Birmingham Tibor Fulop, MD, PhD, FASN
Medical University of South Carolina UNITED STATES
UNITED STATES
UNITED STATES Steven W. Kraft, MD
Mona B. Brake, MD, FASN
Duvuru Geetha, MD, FASN Western Nephrology
VA Medical Center
John Hopkins Bayview Medical Center UNITED STATES
UNITED STATES
UNITED STATES
Ruth C. Campbell, MD, FASN Vineeta Kumar, MD
Ilya Glezerman, MD University of Alabama at Birmingham
Medical University of South Carolina
Memorial Sloan Kettering Cancer Center UNITED STATES
UNITED STATES
UNITED STATES
Chia-Ter Chao, MD, PhD, FASN Norbert H. Lameire, MD, PhD
Carl S. Goldstein, MD, FASN
National Taiwan University Hospital University Hospital
Medical Diagnostic Associates, PA
TAIWAN BELGIUM
UNITED STATES
Joline L.T. Chen, MD, MS, FASN Steven M. Gorbatkin, MD, PhD, FASN Sheron Latcha, MD, FASN
University of California, Irvine Emory University School of Medicine Memorial Sloan Kettering Cancer Center
UNITED STATES UNITED STATES UNITED STATES
Karen I. Ching, MD, FASN Aditi Gupta, MD, FASN Vincent W.S. Lee, MBBS, PhD
Hawaii Permanente Medical Group University of Kansas Medical Center Westmead Hospital
UNITED STATES UNITED STATES AUSTRALIA
nephSAP Volume 2044Number
Volume19 Number00
244 October
January 2020
2022

Paolo Lentini, MD, PhD, FASN Phuong-Thu T. Pham, MD, FASN Stephen M. Sozio, MD, FASN
Ospedale San Bassiano,Bassano del Grappa (VI) UCLA Medical Center Johns Hopkins University School of Medicine
ITALY UNITED STATES UNITED STATES
Tingting Li, MD, MS, FASN Pairach Pintavorn, MD, FASN Ignatius Yun-Sang Tang, MD, PharmD, FASN
Washington University in St. Louis East Georgia Kidney Group University of Illinois Hospital and Health Sciences
UNITED STATES UNITED STATES System
Roberto Pisoni, MD UNITED STATES
Orfeas Liangos, MD, DrMed, FASN
Kuratorium f€ur Dialyse und Nierentransplantation Medical University of South Carolina Ahmad R. Tarakji, MD, FASN
GERMANY UNITED STATES King Saud University
Paul H. Pronovost, MD, FASN SAUDI ARABIA
Ajit Mahapatra, MD
Waterbury Hospital
The Permanente Medical Group Hung-Bin Tsai, MD, MS, PhD
UNITED STATES
UNITED STATES National Taiwan University Hospital
Mohammad A. Quasem, MD TAIWAN
A. Bilal Malik, MD United Health Services Hospitals-Binghamton
University of Washington Medical Center UNITED STATES Kausik Umanath, MD, MS, FASN
UNITED STATES Henry Ford Health System
Wajeh Y. Qunibi, MD UNITED STATES
Ernest I. Mandel, MD, MS, FASN University of Texas Health Science Center
Brigham and Women's Hospital UNITED STATES Puchimada M. Uthappa, MD, MBBS, FASN
UNITED STATES Columbia Asia Hospital, Mysore
Pawan K. Rao, MD, FASN
INDIA
Naveed N. Masani, MD, FASN Nephrology Hypertension Associates
Wintrop University Hospital UNITED STATES Anthony M. Valeri, MD
UNITED STATES Hernan Rincon-Choles, MD, MS, FASN Columbia University Medical Center
Cleveland Clinic UNITED STATES
Ellen T. McCarthy, MD
University of Kansas Medical Center UNITED STATES Allen W. Vander, MD, FASN
UNITED STATES Dario Roccatello, MD Kidney Center of South Louisiana
Ospedale San GIovanni Bosco UNITED STATES
Kirtida Mistry, MBChB, FASN
ITALY
US Food and Drug Administration Jon R. Von Visger, MD, PhD
UNITED STATES Ehab R. Saad, MA, MD, FASN Jacobs School of Medicine & Biomedical Sciences
Medical College of Wisconsin UNITED STATES
Lawrence S. Moffatt, MD UNITED STATES
Charlotte Medical Clinic South Park Nand K. Wadhwa, MD, FASN
UNITED STATES Mark C. Saddler, MBChB New York Health Center for Nephrology
Durango Nephrology Associates UNITED STATES
Thangamani Muthukumar, MD, FASN UNITED STATES
New York Presbyterian Hospital Weill Cornell Connie J. Wang, MD
Medical Center Mohammad N. Saqib, MD Hennepin County Medical Center
UNITED STATES Valley Kidney Specialists, PC UNITED STATES
UNITED STATES
Mohanram Narayanan, MD, FASN Maura A. Watson, DO, FASN, MPH
Hitesh H. Shah, MD, FASN
Baylor Scott & White Healthcare Walter Reed National Military Center
Zucker School of Medicine at Hofstra/Northwell
UNITED STATES UNITED STATES
UNITED STATES
Macaulay A. Onuigbo, MD, MS, MBA, FASN Michiko Shimada, MD, PhD, FASN Dawn F. Wolfgram, MD, FASN
University of Vermont Medical Center Hirosaki University Medical College of Wisconsin
UNITED STATES JAPAN UNITED STATES
Rosemary Ouseph, MD Shayan Shirazian, MD Brian Y. Young, MD
St. Louis University Winthrop University Hospital UC Davis Medical Center
UNITED STATES UNITED STATES UNITED STATES
Todd E. Pesavento, MD, FASN Arif Showkat, MD, MBBS, MPH, FASN Mario Javier Zarama, MD, FASN
Ohio State University Wexner Medical Center University of Tennessee Kidney Specialists of Minnesota
UNITED STATES UNITED STATES UNITED STATES
nephSAP Volume 20 4 Number 2 4 January 2022
Volume 19 4 Number 4 October 2020

Learning Objectives
1. To examine recent scientific advances in our understanding of the pathophysiology of disorders of fluid, electrolytes, and
acid-base balance.
2. To review how the understanding of pathophysiology can be applied to the care of patients.
3. To analyze how recent studies related to fluid, electrolyte, and acid-base disorders can be applied to clinical decision making.

Target Audience:
Physicians, scientists, advanced practitioners, nurses, pharmacists, and other health professionals working and training in the
field of nephrology.

Accreditation and AMA Credit Designation Statements


In support of improving patient care, the American Society of Nephrology is jointly accredited by the
Accreditation Council for Continuing Medical Education (ACCME), the Accreditation Council for
Pharmacy Education (ACPE), and the American Nurses Credentialing Center (ANCC), to provide
continuing education for the healthcare team.

The American Society of Nephrology designates this internet activity (enduring material) for a maximum of 10.00 AMA PRA
TM
Category 1 Credits . Physicians should claim only the credit commensurate with the extent of their participation in the activity.

ABIM MOC Statement


Successful completion of this CME activity, which includes participation in the evaluation component, enables the participant to
earn up to 10.00 MOC points in the American Board of Internal Medicine’s (ABIM) Maintenance of Certification (MOC) pro-
gram. Participants will earn MOC points equivalent to the amount of CME credits claimed for the activity. It is the CME activity
provider’s responsibility to submit participant completion information to Joint Accreditation for the purpose of granting ABIM
MOC credit.

MOC points will be applied to only those ABIM candidates who have enrolled in the MOC program. It is your responsibility
to complete the ABIM MOC enrollment process.

Original Release Date


January 2022

CME/MOC Termination Date


February 28, 2023

Examination Available Online


On or before January 30, 2022

Estimated Time for Completion


10 hours

Self-Assessment Examination
4 Answers with explanations are provided with a passing score after the first and/or second attempt.
4 This issue will be archived in March 1, 2023, and answers with explanations will be posted on the ASN website.
nephSAP Volume 20 4 Number 2 4 January 2022
Volume 19 4 Number 4 October 2020

Method of Participation
4 Read the entire issue that is supplemented by original articles in the reference lists.
4 Complete the online self-assessment examination.
4 Each participant is allowed two attempts to pass the examination (75% correct) for CME and MOC.
4 Upon completion, review your score and incorrect answers and print your certificate.

Activity Evaluation, CME, and MOC


1. Go to the ASN eLEARNING CENTER (education.asn-online.org)
2. Select MY ACTIVITIES from the left-hand navigation.
3. Select the corresponding issue.
4. Read all information on the Overview screen.
5. Follow the onscreen instructions to:
a. Complete the Exam (i.e., post test),
b. Complete the Evaluation,
c. Claim credit/MOC, and
d. Print a Certificate

Your complete ASN Transcript is available on the ASN eLEARNING CENTER:


4 Select TRANSCRIPT on the left-hand navigation.
4 View course status, credits claimed, or print a certificate for completed learning activities.

System Requirements
Compatible Browser and Software
The nephSAP website (nephsap.org) is formatted for cross-browser functionality and should display correctly in all currently
supported web browsers. Individual nephSAP articles are available as dynamic HTML or in Portable Document Format (PDF),
which requires Adobe Reader or comparable PDF viewing software.

Monitor Settings
The nephSAP website was designed to be viewed in a 1024  768 or higher resolution.
Medium or Combination of Media Used
The media used include content provided online as dynamic HTML or downloadable PDF, in addition to an online evaluation
and self-assessment examination.
Technical Support
If you have difficulty viewing any of the pages, please refer to the nephSAP technical support page for possible solutions. If you
continue having problems, contact ASN at nephsap@asn-online.org.
nephSAP Volume 20 4 Number 2 4 January 2022
Volume 19 4 Number 4 October 2020

ASN Disclosure Policy


It is the policy of American Society of Nephrology (ASN) that all individuals in a position to control the content of ASN's educational activities are expected to
disclose all their and their life spouse/partner's financial relationships with “ineligible companies” within the prior 24 months as well as other non-financial
interests. (“Ineligible companies” are those whose primary business is producing, marketing, selling, re-selling, or distributing health care products used by or on
patients.) Participants should note that the use of products outside U.S. Food and Drug Administration (FDA) approved labeling should be considered
experimental and are advised to consult current prescribing information for approval indications.
The intent of this policy is not to prevent expert faculty with relevant relationship(s) with commercial interest(s) from involvement in educational activities,
but rather to ensure that these activities promote quality and safety, are effective in improving medical practice, are based on valid content, and are inde-
pendent of control from commercial interests and free of commercial bias. In addition, all faculty were instructed to provide balanced, scientifically rigor-
ous, and evidence-based presentations.
ASN requires all individuals in a position to control content for this activity to complete disclosure forms on the ASN website. Responses are listed below.
Disclosures have been reviewed and assessed by ASN. All relationships with ineligible companies have been identified and all relevant financial relationships
have been mitigated.

EDITORS:
Alice M. Sheridan, MD—Employer(s): Brigham and Women's Hospital; No additional relevant financial relationship(s) with ineligible companies to disclose.
Martina M. McGrath, MBChB, FASN—Employer(s): Brigham and Women's Hospital; VA Boston Medical Center; No additional relevant financial
relationship(s) with ineligible companies to disclose.
EDITORIAL AUTHOR:
Julian L. Seifter, MD—No additional relevant financial relationship(s) with ineligible companies to disclose.

AUTHORS:
Robert Jay Alpern, MD—Employer(s): Yale University School of Medicine; Ownership Interest: Abbott, AbbVie, Tricida, Apple, Berkshire Hathaway,
IBM, Icahn Enterprises, Ford, Intel, Johnson and Johnson, Kraft Heinz, Merck, Pfizer, Verizon, Coca-Cola, UPS, Microsoft, Newell Brands, Organon,
Peoples United Financial, Texas Pacific Land Corp, First Trust Nasdaq Bank, T. Rowe Price Real Estate Investor; Advisory/Leadership Role: Director of
Abbott, AbbVie and Tricida. All are publicly traded for-profit; Other Interests/Relationships: Director of NewHYTES, a non-profit that uses tennis and
education to help underprivileged children in New Haven.
Tushar Chopra, MD, FASN—Employer(s): University of Virginia Health System; Patents/Royalties: UpToDate
Thomas D. DuBose, Jr., MD, FASN—Employer(s): Wake Forest School of Medicine; No additional relevant financial relationship(s) with ineligible
companies to disclose.
David H. Ellison, MD, FASN—Employer(s): Oregon Health & Science University, Portland VA Medical Center; Patents/Royalties: UpToDate
Namrata Krishnan, MD—Employer(s): Veterans Affairs Medical Center, West Haven, CT; Yale School of Medicine, New Haven, CT; Advisory/Leadership
Role: Leadership roles within my institution, no outside paid roles; Other Interests/Relationships: NKF education committee, KSAP question writing committee.
Ruediger W. Lehrich, MD—Employer(s): Duke University Medical Center; No additional relevant financial relationship(s) with ineligible companies to disclose.
Martina M. McGrath, MBChB, FASN—Employer(s): Brigham and Women's Hospital, VA Boston Medical Center; Other Interests/Relationships: Deputy
Editor, nephSAP; No additional relevant financial relationship(s) with ineligible companies to disclose.
Solomiia Savedchuk, MD—Employer(s): Duke University; No additional relevant financial relationship(s) with ineligible companies to disclose.
Zhabiz Solhjou, MD—Employer(s): Brigham and Women's Hospital; Patents/Royalties: Microneedle-Based Platform for Simultaneous Local Delivery of
Drugs and Institial Skin Fluid Sampling for Management of Immune Mediated Skin Diseases; Other Interests/Relationships: ASN member
Andrew S. Terker, MD, PhD—Employer(s): Vanderbilt University Medical Center; Consultancy: Andrew Terker consultancy fees by Ampio
Pharmaceuticals; Ownership Interest: Ampio Pharmaceuticals
Naoto Tominaga, MD, PhD—Employer(s): Kawasaki Municipal Tama Hospital; Research Funding: Bayer Yakuhin, Ltd.; Chugai Pharmaceutical Co.,
Ltd.; Kyowa Kirin Co., Ltd.; Mitsubishi Tanabe Pharma Corp.; Otsuka Pharmaceutical Co., Ltd.; Takeda Pharmaceutical Co., Ltd.; Teijin Pharma Co.,
Ltd.; Honoraria: Astellas Pharmaceutical Inc.; Bayer Yakuhin, Ltd.; Baxter Ltd.; Chugai Pharmaceutical Co., Ltd.; Daiichi Sankyo Co., Ltd.; Edwards
Lifesciences Corp.; Fuso Pharmaceutical Industries, Ltd.; Kissei Pharmaceutical Co., Ltd.; Kyowa Kirin Co., Ltd.; Mitsubishi Tanabe Pharma Corp.;
Mochida Pharmaceutical Co., Ltd.; Otsuka Pharmaceutical Co., Ltd.; Sanwa Kagaku Kenkyusho Co., Ltd.
Joseph G. Verbalis, MD—Employer(s): Georgetown University; Consultancy: Corcept, EUSA, Otsuka; Honoraria: Corcept, EUSA, Otsuka
ASN STAFF: Suzanne P. Armstrong, MA—Employer(s): American Society of Nephrology; No additional relevant financial relationship(s) with ineligible
companies to disclose.

Commercial Support
There is no commercial support for this issue.
Nephrology Self-Assessment Program - Vol 20, No 2, January 2022 67

Editorial
Teaching Renal Physiology at the Bedside
Julian L. Seifter, MD
Brigham and Women's Hospital, Boston, Massachusetts

“He who studies medicine without books sails an uncharted sea, voided 6 hours later, the urine osmolality had increased from ,50
but he who studies medicine without patients does not go to sea to 1000 mOsm/kg (Figure 1).
at all.”
Sir William Osler When a healthy subject with a normal GFR is given a water
load of 15 ml/kg of body weight, 90% of that water load will be
In the clinical environment, underlying renal pathophysiology is
excreted within 4 hours in a maximally dilute urine with high uri-
often evident in a patient’s symptoms and findings. However, fur-
nary flow rate (i.e., water diuresis) (1). This occurs because the small
ther investigation is often required to identify the precise nature of
dilution of the plasma osmolality due to the water intake suppresses
the disease. When researchers investigate a question, they design a
the secretion of antidiuretic hormone (ADH), allowing the remain-
means to solve the problem through experimentation or the gather-
ing ADH to disappear quickly from the circulation given its very
ing of data. A clinician, like a researcher, can ask fundamental
short half-life of approximately 3 minutes. Thus, its antidiuretic
questions and develop tests that reveal answers to puzzling findings,
effect is reversed rapidly when physiologic stimuli such as hyperto-
particularly the clinician who is undeterred by complexity and feels
nicity or hypovolemia are removed (2,3).
free to explore different scenarios. If the excreted water is replaced, as in this case, the water diure-
The cases that I will be discussing below feature unusual, para-
sis is maintained and ADH levels remain low. Decreased binding of
doxical, or seemingly insignificant findings that invite an experimental ADH to the basolateral V2 receptor on the principal cells of the col-
bent of mind. In my experience, it is usually some “outlier” piece of lecting duct, on the connecting tubule and the distal convoluted
information that stimulates the sleuthing instinct and engages learners tubule, and on the cortical thick ascending limb results in the net
in the process of solving the problem at hand. All of these cases endocytosis and intracellular localization of aquaporin-2 (1–3). The
prompted open-ended questions: Why are we seeing this? Is there a signaling mechanism underlying aquaporin-2 endocytosis is less well
precedent for this? Have you seen this in another context? Can you defined than that underlying its translocation to the apical mem-
imagine what’s happening within the kidney? How would you design brane, but likely involves protein kinase C—induced ubiquitination
a plan to address the questions that you would like answered? of aquaporin (4).
Each case below is followed by a discussion that puts together In this subject, urinary flow rates of almost 1 L/h were
existing data or adds new data to enhance understanding of the achieved. The administration of hypotonic fluids sufficient to cause
patient’s illness. The representation of data in tabular or graphic form a water diuresis achieves the highest possible urinary flow rates—
organizes information as it accumulates and changes over time and much higher than that associated with isotonic fluids. Equal vol-
illustrates the correlation between laboratory data and the clinical pro- umes of saline expansion take longer to excrete and at lower urinary
gress of the patient, including the effects of interventions. Visual rep- flow rates. The different responses to water and saline were observed
resentation frequently reveals important interrelationships between by Homer Smith and others over 70 years ago and is a result of the
various pieces of data and directs the clinician toward further tests. independent regulation of solute and water excretion (5). A liter of
These salt-and-water encounters suggest that an experimental saline might take a day to be eliminated because of the time required
approach, involving stepwise assessment, can inculcate a deeper to decrease the new sodium transporters in the membranes from the
understanding of physiology and increase curiosity, creativity, and steroid mineralocorticoid receptors under aldosterone regulation.
scientific rigor in the clinical setting. If the majority of the excreted volume is non–solute-containing
water, replacement of the output with an equal amount of water
sustains the diuresis. Though it may sound counterintuitive in the
Case 1: Why Would a Healthy Subject in a testing of a diuretic (or in this case a natriuretic peptide), a water
diuresis is useful because it allows urine to be produced about every
Research Study, in the Midst of a Water Diuresis,
30 minutes without necessitating a catheter in the bladder. In this
Suddenly Become Anuric? case the goal was to determine a time course of electrolyte excretion
A healthy 25-year-old male volunteer entered a research study rates. In a rare patient with hyponatremia, another reason to perform
involving an infusion of a synthetic natriuretic peptide. Before he a water load might be as a test of diluting capacity. Before a water
was given the drug, a water diuresis was initiated by infusing hypo- load is given to any patient, the serum [Na1] must be known. It is
tonic 5% dextrose in water and replacing urine volume with oral important to predict a worst-case scenario of hyponatremia in case
water, millimeter for millimeter. After 3 hours, in the midst of a all added water is retained instead of being eliminated in the urine.
brisk water diuresis, his intravenous (IV) catheter infiltrated and had This subject stopped making urine despite the hypotonic fluids
to be replaced. He did not void for the next 6 hours. When he because of a nonosmotic stimulus to ADH secretion, which had

Copyright # 2022 by the American Society of Nephrology nephsap.org


68 Nephrology Self-Assessment Program - Vol 20, No 2, January 2022

Water load in normal volunteer


1200
D5W 400ml/hr lv replaced

Urine osm in mOsm/kg, (red) 1000


Urine volume, ml (blue)

800

600

400

200

0
1 2 3 4 5 6 7 8 9
Hour of test

Figure 1. Water load in a normal volunteer. This figure shows the urine volume (blue bars) and urine osmolality (red bars) over time.

been osmotically suppressed. In this case the cause of ADH secretion of ANP release was transmural stretch, then ANP would increase
was the pain of an infiltrated IV catheter. Other physiologic stimuli after stripping. In heart failure, not involving pericardial constric-
that cause ADH secretion include physical or emotional stress, rage, tion, both Pa and distending pressure are increased.
nausea, and hypoxemia (6). Although many have heard of these Figures 2 A–C show the effects of pericardial stripping on right
stimuli to ADH secretion, they are underappreciated in terms of atrial pressure, ANP levels, and urine flow rates during and after the
severity and duration. For the subject’s safety, the water intake was surgery. Right atrial pressure fell from very high levels to significantly
immediately discontinued to prevent hyponatremia. lower, but still abnormally elevated, levels (Figure 2A). The ANP
level before surgery was moderately elevated (though not as high as
reported in heart failure) and increased two- to three-fold during sur-
Case 2: What Stimulates Release of Atrial gery. The increase of ANP in the course of the surgery is consistent
Natriuretic Peptide from the Heart? with the hypothesis that the hormone is released according to atrial
A 67-year-old woman with a history of tuberculosis was admitted stretch and transmural pressure gradients (8) (Figure 2B).
with massive peripheral edema, distended neck veins, and a pulsatile Figure 2C shows the pattern of diuresis associated with
liver. She had a calcified pericardium. She was admitted for pericar- improvement in cardiac function after release of the calcified pericar-
dial stripping. dium. As mentioned in Case 1, the highest urinary flow rates occur
in a hypotonic diuresis that follows the suppression and rapid degra-
An astute medical resident had read in the literature that dation of ADH. However, given that ANP increased in our patient,
patients with heart failure with edema had very high atrial natri- we should consider interactions between ANP on ADH effect. ANP
uretic peptide (ANP) levels. Some claimed the mechanism of inhibits ADH release and decreases cortical collecting duct response
peptide release from granules of cardiac myocytes to be high intra- to ADH (9,10). It has been observed that normal volunteers, when
chamber atrial pressures (Pa), whereas others suggested myocardial given isotonic saline while supine, excrete hypotonic urine with urine
stretch as the mechanism. Myocardial stretch is a distention of the chemistries that could be interpreted as showing diabetes insipidus
myocyctes due to an increasing gradient between Pa and intrapleural (DI) (10). The experiments that demonstrated the effect of volume
pressure (Pip). The gradient, then, is (Pa2Pip) (7). expansion on free water excretion were done years before the oppo-
We expected that Pa would decrease during removal of the non- site effect of hypovolemia to stimulate ADH release (11), although
distensible calcium barrier and, similarly, that the transmural pres- the effect of hypovolemia on ADH gets much more attention.
sure gradient would also decrease. However, despite the fall in the We suggest that the high urinary flow rates seen in Figure 2C
transmural pressure gradient, the removal of the calcium encasement represent a volume-expanded ANP-assisted diuresis, with ANP-
would allow the difference in pressures, still positive, to stretch the mediated suppression of ADH. By the end of the pericardiectomy
heart muscle. If the initial cause of ANP release was due to increased the urine flow rate was 35 ml/min—a rate even higher than that in
Pa, then ANP levels would decrease after stripping. But if the cause Case 1.
Nephrology Self-Assessment Program - Vol 20, No 2, January 2022 69

A B
30 280

PCWP

Atrial Natriuretic Peptide PG / ML


240
25 RA MEAN After
Stripping
200
20 Before
Stripping
160
mmHg

15
120
10
80

5
40

0 0
0 2 4 6 8 10 12 14 16 18 -24 0 4 24 48
HOURS Hours

C
40

35

30
Urinary Flow ( ml/min )

25

20

15

10

0
0 2 4 6 8 10 12 14 16 18
Hours ( 0 = start of pericardiectomy )

Figure 2. Atrial natriuretic peptide after pericardiectomy (A) Right atrial (RA) mean pressure and pulmonary capillary wedge pressure (PCWP)
during and after pericardiectomy, which started at 0 hour and ended at the sixth hour. There was a near parallel decrease in pressures from 29
to 15 mm Hg. (B) Plasma atrial natriuretic peptide (ANP) levels increased and remained increased after pericardiectomy. Supine peripheral
blood samples were collected at 24 hours before surgery, just after induction of anesthesia (0 time), at 4 hours when a significant portion of the
heavily calcified pericardium had been removed, and at 24, 48, and 240 hours postoperatively. Supine ANP levels were obtained from a brachial
arterial line intraoperatively; otherwise, samples were from the right antecubital vein. Only the mean value is reported. Values for six normal
human subjects were 28.562.6 pg/ml. (C) Urine flow rates after pericardiectomy showed a dramatic postoperative increase to nearly 35 ml/min
despite decreases in central pressures and increase in ANP. The high flow state, however, appeared to be only short lived. Used with permission
from reference 8 (Wolozin MW, Ortola FV, Spodick DH, Seifter JL: Release of atrial natriuretic factor after pericardiectomy for chronic con-
strictive pericarditis. Am J Cardiol 62: 1323–1325, 1988).

In addition, ANP inhibits adrenal aldosterone production (9). Case 3: Can the Urine Be Concentrated in the
This effect was shown in the urine indices (not shown); the urine [Na] Absence of Antidiuretic Hormone?
was 48 meq/L and the urine [K] was 20 meq/L—a ratio of sodium to
An institutionalized 33-year-old woman with a history of trau-
potassium consistent with suppressed aldosterone. Most importantly,
matic brain injury was admitted to the intensive care unit in
those urine chemistries with a serum [Na] of 139 meq/L suggest that coma. Her admission [Na1] was 170 meq/L, and her urine
the postoperative urine was a mixed saline and water diuresis. In the 4 osmolality was 550 mOsm/kg. After 2 days of treatment, she was
liters excreted during the first 6 hours, approximately 50% was awake; her serum [Na1] was 156 meq/L and her urine osmolal-
osmole-free water clearance and 50% was isotonic saline diuresis. ity was ,150 mOsm/kg.
70 Nephrology Self-Assessment Program - Vol 20, No 2, January 2022

ARGININE VASOPRESSION (PERCENT REMAINING)


180
100
170
Serum (Na+), meq/L

160 80 CONTROL

150
60
140
PATIENT
130 40

120
20
1 2 3 4 5
days
600 0
0 10 20 30
TIME (MIN)
500
Urine Osm. mOsm/Kg

400
Figure 4. Postpartum serum versus control serum. This figure shows
300 AVP levels in patient's postpartum serum and in control serum after
the in vitro addition of AVP. The patient's serum compared to con-
200 trol serum shows a decrease in level of AVP, suggesting the presence
of vasopressin in the patient's serum.
100 her urine in the absence of ADH. Is there a mechanism by which
the urine can be concentrated in the absence of ADH?
0 The first step in solving this puzzle requires thinking through
1 2 3 4 5 the renal processes required to put out a maximally dilute urine.
days First, there must be ample isotonic filtration of plasma into the
renal tubules without excessive proximal reabsorption of that fluid.
This enables a high rate of delivery into the loop of Henle (i.e.,
the delivery phase). Second, the water-impermeable diluting seg-
Figure 3. Serum sodium and urine osmolality. The serum sodium
ment, consisting of the thick ascending limb in the medulla and
and urinary osmolal concentration were plotted versus days in the
parts of the cortical distal nephron, must be able to reabsorb
hospital The graph shows profound hypernatremia with increased
sodium in the absence of water, thus producing a dilute fluid. It
urine osmolality initially, suggesting retained ability of the kidney to
follows that the greater the delivery from the proximal tubule, the
concentrate the urine to protect from further water losses.
more separation of salt from water will occur, and maximal
Graphing the serum sodium and urinary osmolal concentration amounts of dilute tubular fluid will be available (i.e., the separa-
versus days in the hospital (Figure 3) yields an interesting pattern. tion phase). In the absence of ADH, large quantities of
The initial observation is profound hypernatremia with increased electrolyte-free water can be excreted as dilute urine (the regulation
urine osmolality, suggesting retained ability of the kidney to concen- phase).
trate the urine to protect from further water losses. An increased urine However, in conditions of excessive extracellular volume deple-
osmolality could be due to increased ADH in either hypertonic tion, filtration decreases and proximal reabsorption increases, which
expansion or hypertonic depletion of the body water volume, as well sharply limits fluid delivery to the diluting segments. The reabsorp-
as in extracellular volume depletion regardless of osmolality. However, tion of sodium in the thick ascending limb and corresponding water
on day 2, a clear diagnosis emerged. The serum sodium concentration reabsorption in the descending limb of Henle results in a very low
decreased, although it remained in a hypertonic range. However, the flow rate of tubular fluid to the ADH-sensitive portions of the neph-
urine concentration decreased to a significantly hypotonic level despite ron in the cortex and medulla.
the serum hypertonicity, which is characteristic of DI. A key fact is that even in the absence of ADH, the collecting
It would be logical to assume that the DI was of central origin duct is slightly permeable to water. This contrasts with the water-
because of the patient’s history of traumatic brain injury, although impermeable thick ascending limb in the medulla and parts of the
this was not tested. A trial on day 2 to see whether the urine could cortical distal nephron. Even if only slightly permeable, when tubu-
concentrate after administration of vasopressin would have been lar flow is very low, the collecting duct reabsorbs enough water to
diagnostic. If the patient clearly, on day 2, had DI, the obvious raise the osmolality of the tubular fluid (12). However, despite the
question is why on presentation she had the ability to concentrate relatively high osmolality of tubular fluid in such cases, the absolute
Nephrology Self-Assessment Program - Vol 20, No 2, January 2022 71

A B
400
Urine Osmolality (mOsm/liter)

Urine Osmolality (mOsm/liter)


Urine Output (ml/hr)

300 300

Urine Output (ml/hr)


200 200

100 100

0 0
3-hr mean 1 2 3 3-hr mean 1 2 3

Hour Hour

Figure 5. (A) Nephrogenic diabetes insipidus response to indomethacin plus dDAVP. Lightly shaded bars indicate urine osmolality, and heavily
shaded bars indicate urine output. Arrows denote the time at which a single dose of subcutaneous desamino-D-arginine vasopressin (dDAVP)
was administered. (B) In the absence of indomethacin plus dDAVP. Lightly shaded bars indicate urine osmolality, and heavily shaded bars urine
output. Arrows denote the time at which a single dose of subcutaneous desamino-D-arginine vasopressin (dDAVP) was administered. Used with
permission from reference 17 (Stasior DS, Kikeri D, Duel B, Seifter JL: Nephrogenic diabetes insipidus responsive to indomethacin plus
dDAVP. N Engl J Med 324: 850–851, 1991).

amount of absorbed water is low, and not nearly enough to decrease (13). The Berliner–Davidson effect is demonstrated in multiple clin-
the serum sodium. ical scenarios, including the following:
By comparison, under high flow rate conditions this ADH-
 DI is more profound in patients who are given a high-salt diet
independent reabsorption would not be enough to concentrate the
compared with a low-salt diet.
tubular fluid, although more free water is absorbed in the collecting  A loss of posterior pituitary function creates more water loss
duct during a water diuresis in the absence of ADH than when there than panhypopituitarism. In the latter case, hypocortisolemia
is low tubular delivery and non–ADH-dependent water reabsorption. resulting from decreased adrenocorticotrophic hormone
In this case it is notable that the osmolality of the urine was not (ACTH) causes hypotension and limits the excretion of large
maximal at 550 mOsm/kg. This was likely because of her chronic DI. volumes of hypotonic urine.
Inasmuch as she had sufficient water loss to drive her extreme hyper-  In patients with nephrogenic DI, urinary flow can be
natremia, there would also be enough non-ADH water reabsorption decreased by a thiazide diuretic, which, by causing extracellu-
to wash out the existing gradient between the interstitium and the lar volume depletion, decreases the GFR, enhances proximal
tubular lumen. With any persistently high urine flow rate, enough reabsorption, and decreases distal delivery. This limits the
production of electrolyte-free water and controls the polyuria.
water is generally reabsorbed to wash out the interstitial solute gradi-
ent so that the maximum interstitial fluid osmolality, and accordingly The following two cases also demonstrate the Berliner–Davidson
the maximum urine osmolality, is lowered until the gradient can be effect, among other physiologic findings.
re-established. Maximal urine osmolality will be achieved only when
the polyuric process is corrected—whether by treating low volume
states or by administering ADH in central DI—and enough solutes Case 4: Can a Patient Have Diabetes Insipidus of
such as salt and urea are available to restore medullary hyperosmolal- Pregnancy Manifesting Only in the
ity. This usually takes days of repletion of solutes. Postpartum Period?
Sixty years ago experiments were done to test the presence of A 28-year-old woman began to excrete high urine volumes, as much
non–ADH-dependent urinary concentrating ability. In one experi- as 1 L an hour, 1 to 2 days after the delivery of healthy twins. She had
ment, the bladder was ligated in dogs hypophysectomized to remove no history of polyuria. She had experienced third-trimester preeclamp-
ADH and produce DI (13). A pipette was placed into each hemi- sia, with marked edema and hypertension, which resolved immediately
bladder so that functionally the two urinary systems consisted of a after delivery. On postpartum day 2, the result of physical examination
kidney, ureter, and half-bladder. In these animals with DI, both was notable only for trace edema. Laboratory evaluation revealed
hemibladders excreted maximally dilute urine at baseline. One renal serum [Na1] 143 meq/L. Serum osmolality was 296 mOsm/kg, urine
artery was clamped to create renal artery constriction, which unilat- osmolality was 110 mOsm/kg, and UNa1 was 20 meq/L.
erally decreased the GFR. This decreased the flow rate of tubular
fluid from the proximal nephron into the collecting duct. Compared Polyuria regularly occurs in the postpartum period as a result of
with the contralateral unclamped kidney, the kidney with the artery the mobilization of the large volume of extracellular and intracellular
constriction yielded concentrated urine. The contralateral control fluid accumulated during pregnancy. At termination of pregnancy,
urinary system continued to generate dilute urine consistent with hormonal balance changes. For example, natriuretic peptides
DI. This phenomenon is known as the Berliner–Davidson effect increase salt excretion, normal sensitivity to vasopressin returns, and
72 Nephrology Self-Assessment Program - Vol 20, No 2, January 2022

the water excretion corrects the normal hyponatremia of pregnancy. have gestational polyuria when the enzyme level would be expected
However, this patient experienced a much greater loss of fluid than to be even higher?
would be expected. The edema decreased, suggesting a decrease in The answer lies in an interaction between salt and water reminis-
extracellular fluid, but the rapid rise in serum sodium concentration cent of the Berliner–Davidson effect discussed earlier. Pre-eclampsia
to 143 meq/L suggested excessive loss of hypotonic fluid. The high is a sodium-avid disorder characterized by edema. Therefore, in her
urine flow rate at 1 L/h was investigated by evaluating the urine pre-eclamptic state she was reabsorbing excessive amounts of isotonic
chemistry. The urine osmolality was hypotonic at 110 mOsm/kg, fluid in the proximal tubule and the loop of Henle, limiting the deliv-
approximately one third of the serum osmolality. Thus, two thirds ery of hypotonic water to the collecting duct. Thus, even in the
of the urine output was solute-free water, whereas one third was absence of vasopressin, water excretion was limited, preventing the
isosmotic. Multiplying the urine osmolar concentration by the vol- manifestation of DI. However, once delivery occurred, the pre-
ume of urine over time yields the solute excretion rate per time: eclampsia resolved, and the postpartum saline diuresis followed. The
increased flow rate to the distal nephron markedly increased free
_ CH O 1 Cosm
V5 2 water excretion because AVP was absent.
and
Cosm 5 ½Uosm 3 1L=hr=Posm: CASE 5: Can Lithium Cause Both Central and
Rearranging for free water clearance: Nephrogenic Diabetes Insipidus?
A 44-year-old woman taking long-term lithium was admitted from
_ Cosm
CH2 O 5 V2 a nursing home with cholecystitis. She was noted to be polyuric,
or with serum [Na1] of 150 meq/L. Assuming the patient had nephro-
genic DI, the surgical resident started the nonsteroidal anti-
_
CH2 O 5 V½1– ðUosm =Posm Þ inflammatory drug (NSAID) indomethacin, having read reports
where the total flow rate of urine, V, _ is compartmentalized into an that indomethacin decreased urine volume in the setting of DI (15).
isosmotic clearance, Cosm, and a solute-free water clearance, CH2O. When the patient remained polyuric, indicating that NSAIDs alone
In our patient, V_ was 1 L/h, and Cosm was (110/296). Cosm  did not correct the polyuria, a renal consultation was requested, and
0.372 L/hr. CH2O 5 1.0-0.372 5 0.628 L/h. a diagnostic test was done.
The urine sodium concentration, 20 meq/L, multiplied by 1 We administered a single dose of dDAVP to test whether the
L/h, indicates the loss in the urine of 1 L of isotonic salt solution every urine could be concentrated. Notably, the patient was already dehy-
seven hours, which, while indicating the presence of an osmotic diure- drated, with serum sodium of 150 meq/L; thus, water restriction
sis, also indicates that the polyuria is excessive for the usual postpar- before dDAVP was not required. The results are shown in Figure 5A.
tum natriuresis and suggests a large electrolyte-free component. The The dDAVP increased the urinary concentration and decreased urine
excretion of water is inappropriately high for the serum sodium con- volume, suggesting central DI.
centration of 143 meq/L, suggesting a water diuresis characteristic of The test was repeated after the NSAID was stopped (Figure 5B).
DI. In this patient, the differential diagnosis of DI includes pituitary The serum sodium concentration was again 150 meq/L, so that water
infarction (Sheehan syndrome) and, more likely, given the large pla- restriction was not required. After the NSAID was stopped, dDAVP
cental mass of a twin pregnancy, DI of pregnancy due to the presence did not concentrate the urine or decrease urinary volume further.
of excessive placental vasopressinase (14). The absence of an effect of dDAVP in the setting of dehydra-
However, one question relating to this diagnosis is this: Why is tion and polyuria usually indicates nephrogenic DI (16). However,
this condition presenting for the first time post partum, when she in this case, the NSAID plus the supraphysiologic addition of ADH
did not have gestational polyuria? (dDAVP) concentrated the urine, but ADH alone did not (17).
The enzyme vasopressinase (oxytocinase) is present in the placenta There are two possible explanations for this observation. The first
early in gestation to prevent premature labor and is elevated in multi- is that indomethacin potentiated the ADH effect in the collecting
gestational pregnancies or with large placental mass. Vasopressinase duct by inhibiting prostaglandins (18). Prostaglandins such as PGE2
continues to be present for days to weeks after removal of the placenta diminish the effect of ADH. The other possibility is that the NSAID
because of the slow clearance of the enzyme; therefore, ADH continues decreased renal plasma flow and therefore GFR. The decreased GFR
to be suppressed, and DI not only becomes evident after delivery of would increase proximal and loop of Henle reabsorption of Na1,
the placenta but continues for a period of time post partum (14). The which would decrease flow to the medullary collecting duct.
diagnosis is tested by administering an IV dose of arginine vasopressin This case is another example of the Berliner–Davidson effect.
(AVP), the normal ADH peptide, and measuring the urine osmolality. Again we see that decreasing Na1 delivery limits the output of dilute
If vasopressinase is present, the administered AVP fails to concentrate urine and allows for concentration of urine. Increasing Na1 delivery
the urine. By contrast, the administration of desamino-D-arginine improves water excretion and urinary dilution when ADH is absent
vasopressin (dDAVP), which is a synthetic nondegraded analogue of or nonfunctional. A vasopressin level from blood drawn before the
vasopressin, will concentrate the urine. Figure 4 shows an in vitro test second dehydration test later came back showing high levels of ADH.
of the patient’s postpartum serum with AVP added to it, which, com- In conclusion, fluid and electrolyte balance in the body is an
pared with control serum, shows a decrease in level of AVP. integrated system. Examples of the interconnections are observable
Another question relating to the diagnosis is this: If the patient in each of the cases presented. A common theme is how the regula-
had DI of pregnancy with vasopressinase present, why didn’t she tion of sodium balance by the renal tubules is connected to the
Nephrology Self-Assessment Program - Vol 20, No 2, January 2022 73

handling of water. These investigations illustrate critical thinking at 9. Inoue T, Nonoguchi H, Tomita K: Physiological effects of vasopressin
the bedside, illuminate connections between basic science and clini- and atrial natriuretic peptide in the collecting duct. Cardiovasc Res 51:
470–480, 2001 PubMed
cal medicine, and refine diagnoses so as to improve patient care.
10. Strauss MB, Davis RK, Rosenbaum JD, Rossmeisl EC: Water diuresis
produced during recumbency by the intravenous infusion of isotonic
References saline solution. J Clin Invest 30: 862–868, 1951 PubMed
1. Demers L: Pituitary function. In: Tietz Textbook of Clinical Chemis- 11. Orloff J, Burg M: Kidney. Annu Rev Physiol 33: 83–130, 1971 PubMed
try, 3rd Ed., edited by Burtis CA, Ashwood ER, Philadelphia, W.B. 12. Nielsen S, Chou CL, Marples D, Christensen EI, Kishore BK, Knepper
Saunders, 1999, pp 1470–1499. MA: Vasopressin increases water permeability of kidney collecting duct
2. Cheung PW, Bouley R, Brown D: Targeting the trafficking of kidney by inducing translocation of aquaporin-CD water channels to plasma
water channels for therapeutic benefit. Annu Rev Pharmacol Toxicol 60: membrane. Proc Natl Acad Sci U S A 92: 1013–1017, 1995 PubMed
175–194, 2020 PubMed 13. Berliner RW, Davidson DG: Production of hypertonic urine in the
3. Knepper MA, Inoue T: Regulation of aquaporin-2 water channel absence of pituitary antidiuretic hormone. J Clin Invest 36: 1416–1427,
trafficking by vasopressin. Curr Opin Cell Biol 9: 560–564, 1997 PubMed 1957 PubMed
4. Kamsteeg EJ, Hendriks G, Boone M, Konings IB, Oorschot V, van der 14. Marques P, Gunawardana K, Grossman A: Transient diabetes insipidus in
Sluijs P, et al: Short-chain ubiquitination mediates the regulated endocy- pregnancy. Endocrinol Diabetes Metab Case Rep 2015: 150078, 2015 PubMed
tosis of the aquaporin-2 water channel. Proc Natl Acad Sci U S A 103: 15. Allen HM, Jackson RL, Winchester MD, Deck LV, Allon M: Indometh-
18344–18349, 2006 PubMed acin in the treatment of lithium-induced nephrogenic diabetes insipidus.
5. Smith HW: Salt and water volume receptors: an exercise in physiologic Arch Intern Med 149: 1123–1126, 1989 PubMed
apologetics. Am J Med 23: 623–652, 1957 PubMed 16. Erden A, Karag€oz H, Başak M, Karahan S, Cetinkaya A, Avci D, et al:
6. Kamoi K, Ishibashi M, Yamaji T: Interaction of osmotic and nonosmotic Lithium intoxication and nephrogenic diabetes insipidus: a case report
stimuli in regulation of vasopressin secretion in hypoosmolar state of and review of literature. Int J Gen Med 6: 535–539, 2013 PubMed
man. Endocr J 44: 311–317, 1997 PubMed 17. Stasior DS, Kikeri D, Duel B, Seifter JL: Nephrogenic diabetes insipidus
7. Ruskoaho H, Lang RE, Toth M, Ganten D, Unger T: Release and regulation responsive to indomethacin plus dDAVP. N Engl J Med 324: 850–851,
of atrial natriuretic peptide (ANP). Eur Heart J 8: 99–109, 1987 PubMed 1991 PubMed
8. Wolozin MW, Ortola FV, Spodick DH, Seifter JL: Release of atrial 18. Berl T, Raz A, Wald H, Horowitz J, Czaczkes W: Prostaglandin synthe-
natriuretic factor after pericardiectomy for chronic constrictive pericardi- sis inhibition and the action of vasopressin: Studies in man and rat. Am J
tis. Am J Cardiol 62: 1323–1325, 1988 PubMed Physiol 232: F529–F537, 1977 PubMed
74 Nephrology Self-Assessment Program - Vol 20, No 2, January 2022

Article
Pathophysiology, Evaluation, Outcomes, and Treatment of
Hyponatremia
Naoto Tominaga, MD, PhD
Division of Nephrology and Hypertension, Kawasaki Municipal Tama Hospital, Kawasaki, Kanagawa, Japan and
Division of Nephrology and Hypertension, Department of Internal Medicine, St. Marianna University School of
Medicine, Kawasaki, Kanagawa, Japan
Joseph G. Verbalis, MD
Division of Endocrinology and Metabolism, Department of Medicine, Georgetown University, Washington, DC

Sodium Concentration and Sodium Content: Two


Learning Objectives Related but Different Concepts
1. To determine the differential diagnosis of hyponatremic states In this review, Na1 represents exchangeable Na1 in the body.
and the pathophysiology underlying each Serum [Na1] and Na1 content are related but different concepts.
Na1 is the predominant cation in extracellular fluid (ECF) (Figure
2. To identify the causes of nonosmotic and inappropriate 1) and is the major component of plasma osmolality (Posm); differ-
arginine vasopressin secretion and how they relate to the ences in effective Posm cause movement of water between the intra-
causation of hyponatremia cellular fluid (ICF) and ECF compartments. Thus, serum [Na1] is
tightly regulated to maintain water balance. Pathologic hyponatre-
3. To appreciate the symptoms of hyponatremia and how they mia or hypernatremia occurs when electrolyte-free water becomes
differ according to the chronicity of the hyponatremia excessive or insufficient relative to the Na1 content in the ECF, and
4. To explain the treatment options for hyponatremia and how it can occur regardless of the total body Na1 content. In other
words, hyponatremia can develop with any ECF volume status or
they differ according to the neurologic symptoms of the
body Na1 content.
hyponatremia
5. To determine the risk factors for the osmotic demyelination Total Plasma Osmolality and Effective Plasma
syndrome and the correction limits that should be observed in Osmolality (Tonicity)
the treatment for chronic hyponatremia As discussed, an abnormality in serum [Na1] is usually caused by an
imbalance in the amount of water in the ECF. How is the effective
Posm (tonicity), which regulates the amount of water, defined? Osmolal-
ity reflects the concentration of all solutes in a solution, but effective
osmolality reflects only solutes that cannot easily pass through cell
membranes. Movement of water between the ICF and ECF compart-
Pathophysiology of Hyponatremia ments is caused by differences in effective Posm. In other words, effective
Definition and Prevalence of Hyponatremia osmolality can most simply be understood as osmolality by solutes effec-
tive at moving water across body fluid compartments.
In most cases, hyponatremia is a disorder of water balance in the
For example, K1 in the ICF and Na1 and glucose in ECF are all
body. Hyponatremia is defined as a serum sodium concentration
substances that generate effective Posm. In contrast, urea is an osmotic
([Na1]) below the normal limits. Although most clinicians regard
substance but can move freely between the ICF and ECF compart-
hyponatremia as a serum [Na1] of ,135 mmol/L, minor variations
ments; therefore, it does not generate effective Posm. In the ECF, K1
in definition ranging from [Na1] of ,136 mmol/L (1–3) to [Na1]
concentration ([K1]) is negligible compared with [Na1]; thus, most
of ,135 mmol/L (4–7) exist in the medical literature.
of the effective Posm generated can be considered to be due to the
Hyponatremia is the most common electrolyte abnormality in
effective osmoles of Na1 and corresponding anions
hospitalized and critically ill patients, with a prevalence of 15% to
Posm and effective Posm are expressed by the following formulae:
30% (8,9). Incidences of hyponatremia from 7% to 53% have been
reported in institutionalized geriatric patients, even with the use of
strict criteria to define hypo-osmolality (10,11). Up to 40% to 70% Posm ðmOsm=kg H 2 OÞ 5 2 3 serum ½Na1  ðmmol=LÞ
of hyponatremia cases are iatrogenic/hospital-acquired, as reported 1 glucose ðmg=dLÞ=18 1 BUN ðmg=dLÞ=2:8
by some studies (12,13).

nephsap.org Copyright © 2022 by the American Society of Nephrology


Nephrology Self-Assessment Program - Vol 20, No 2, January 2022 75

<Increased free water in the ECF compartment> Plasma


Posm AVP concentration
(mOsm/kg H2O) (pg/ml)
+
++
Na
Na
Na
Na+ +
+
Na
Na
Na
++
Na+ 296 9
Na

Na
Na
++
+ 294 8
+
Na
Na+ ++ +
Na
Na
Na
Na+ 7 Uosm
+ + +
++ Thirst 292
Na
Na
Na+ Na
Na
Na
Na+ (mOsm/kg H2O)
osmotic 6
++ +
Na
Na
Na
Na+ threshold 290
5 1000
ICF ECF
n free water in the ECF Shift of free water 288 4
from ECF to ICF 800
286 3
pserum [Na ]
+
intracellular edema 600
284 2
400
<Decreased free water in the ECF compartment>
282 1 300
++ + AVP 200
Na
Na
Na
Na+
osmotic 280 0 100
+ +
Na
Na
Na
Na
++
+
++ +
Na
Na
Na
Na
++
+
threshold 0
Na
Na
Na
Na+ 278 0 250 500 750 1000
++ +
Na
Na
Na
++ +
Na
Na
Na
++
Na+
+ Na
Na
Na
Na+ Urine volume (ml/h)
Na+
++ +
Na
Na
Na
++ + 276
Na
Na
Na
Na+ Na+

ICF ECF p free water in the ECF Shift of free water


from ICF to ECF
nserum [Na+] dehydration
Figure 2. Arginine vasopressin and renal responses to increased
plasma osmolality. When Posm increases above 285 mOsm/kg H2O,
AVP secretion from the posterior pituitary gland is stimulated. As a
Figure 1. Na1 is an effective osmole and serum [Na1] contribution result, plasma AVP concentrations increase, more free water is reab-
to effective Posm (tonicity). Na1 content and serum [Na1] are related sorbed from the renal collecting ducts, and Uosm is increased, result-
but different concepts. Na1 is the predominant cation in ECF and is ing in a net decrease in urine output. However, when Posm continues
the major component of Posm. Differences in effective Posm cause to rise, it increases AVP secretion further and activates thirst when
movement of water between the ICF and ECF compartments. Path- Posm exceeds approximately 295 mOsm/kg H2O. Abbreviations:
ologic hyponatremia or hypernatremia occurs when electrolyte-free AVP, arginine vasopressin; ml/h, milliliters per hour; mOsm/kg
water becomes excessive or insufficient relative to the Na1 content in H2O, milliosmoles per kilogram of water; pg/ml, picograms per milli-
the ECF, and can occur regardless of the total body Na1 content. liter; Posm, plasma osmolality; Uosm, urine osmolality. Reprinted with
Dotted lines indicate normal body fluid status in the ICF and ECF permission from reference 14 (Robinson AG: Disorders of antidi-
compartments. Light blue boxes indicate ECF water. Circles indicate uretic hormone secretion. Clin Endocrinol Metab 14: 55–88, 1985).
Na1 molecules. Abbreviations: ECF, extracellular fluid; ICF, intra-
cellular fluid; Posm [Na1], ECF sodium concentration; Na1, sodium
ion. younger individuals (Figure 3) (15). This has been attributed to
cerebral cortex dysfunction, based on a study using positron emis-
Effective Posm ðmOsm=kg H 2 OÞ 5 2 3 serum ½Na1  sion tomography (16). Simply speaking, thirst in older adults is
ðmmol=LÞ 1 glucose ðmg=dLÞ=18: quenched with relatively smaller amounts of water, which can be
insufficient to compensate for body water deficiency in some
In the absence of hyperglycemia, effective Posm can be estimated
circumstances.
as two times the serum [Na1] 1 10 (which represents the osmolality
of other effective solutes in the ECF). From these formulae, it should
be clear that serum [Na1] is the major determinant of effective Posm. Arginine Vasopressin: Appropriate Secretion
AVP is a 9-amino acid neuropeptide, which acts as a hormone
Two Defense Systems Protecting against Body involved in water homeostasis as well as a neurotransmitter modulat-
Water Losses ing nerve transmission. AVP is often referred to as antidiuretic hor-
When Posm increases above 285 mOsm/kg H2O, arginine vasopres- mone (ADH). However, considering that there are three types of
sin (AVP) is secreted by the posterior pituitary gland (Figure 2) (14). AVP receptors—V1a, V1b, and V2 receptors—and that these recep-
As a result, plasma AVP concentrations increase, and more free water tors exist in various organs (17), the more appropriate terminology
is reabsorbed from the renal collecting ducts, resulting in increased for the hormone is AVP. AVP is secreted when the effective Posm
urine osmolality (Uosm) and a net decrease in urine output. When increases or when the effective circulating plasma volume decreases.
Posm exceeds approximately 295 mOsm/kg H2O, thirst is provoked, Homeostasis of body fluids is maintained via these two major AVP
leading to increased drinking. AVP secretion and thirst represent the secretory stimuli (Table 1). In a dehydrated state, osmoreceptors in
two major body defenses against water losses. the hypothalamus can detect an increase of just 1% to 2% in the
Special attention must be provided to older adults because they effective Posm, triggering AVP secretion from the posterior pituitary
demonstrate unique characteristics. Older adults are less likely to gland. AVP acts on AVP V2 receptors in the renal collecting ducts
feel thirsty and to have a narrower dynamic range of thirst than to decrease the excretion of free water in urine (Figure 4). As noted
76 Nephrology Self-Assessment Program - Vol 20, No 2, January 2022

14 volume rather than on Posm (18). This is an important physiologic


point because AVP secretion is enhanced when the ECF volume
12
65+ decreases even while Posm is not elevated. This represents the most
YOUNG common type of hypovolemic hyponatremia seen in clinical prac-
10
Control tice, especially in the older population. Because of this mechanism,
Thirst rating, cm

AVP secretion will persist until the ECF losses are corrected or
8
improved, resulting in no net improvement of hyponatremia. Con-
6
sequently, the secretion of AVP in response to situations where the
effective Posm is increased or the effective circulating plasma volume
4 is decreased represents "appropriate” AVP secretion.

2 Nonosmotic AVP Stimuli: Inappropriate Secretion


In addition to the two physiologic regulatory systems described, there
0
278 283 288 293 298
are both physiologic and pathologic conditions in which nonosmotic
Posm, mosmol · kg H2O–1 AVP secretion occurs. These include malignant tumors, lung diseases,
central nervous system diseases, drugs, and transient causes (nausea,
vomiting, pain, general anesthesia, prolonged strenuous exercise).
Hyponatremia has occasionally been observed in patients infected
Figure 3. Differences in thirst perception between young and elderly with SARS-CoV-2 (COVID-19), which has spread worldwide in a
individuals. Older adults are less likely to feel thirsty with a higher short time since December 2019. The potential causes of hyponatre-
set point of Posm of approximately 287 mOsm/kg H2O, and they mia with COVID-19 infection include pneumonitis, vomiting, diar-
have a narrower dynamic range of thirst than young individuals. rhea, inadequate oral intake, the administration of natriuretics, and
Abbreviations: cm, centimeter; mosmol・kg H2O21, milliosmoles per
the syndrome of inappropriate antidiuresis (SIAD) (19).
kilogram of water; mOsm/kg H2O, milliosmoles per kilogram of
Because these conditions usually are not related to physiologic
water; Posm, plasma osmolality; 651, 65 years and older. Reprinted
conditions that appropriately stimulate AVP secretion, i.e., hyperos-
with permission from reference 15 (Mack GW, Weseman CA, Lan-
molality or hypovolemia, they are often termed “inappropriate”
ghans GW, Scherzer H, Gillen CM, Nadel ER. Body fluid balance
stimuli of AVP secretion. Interleukin-6 (IL-6) released by monocytes
in dehydrated healthy older men: Thirst and renal osmoregulation. J
and macrophages during inflammation may play an important role
Appl Physiol 76: 1615–1623, 1985).
in the pathogenesis of some cases of hyponatremia (20). IL-6 crosses
the blood–brain barrier and acts in the circumventricular organs,
previously, with further increases in Posm, thirst is activated and
such as the subfornical organ and the organum vasculosum, trigger-
stimulates drinking behavior. The combination of these responses
ing nonosmotic AVP release from both the supraoptic and the para-
leads to the retention of ingested free water and normalization of
ventricular nuclei and inducing thirst (20).
the elevated Posm.
In all cases of nonosmotic stimulation of AVP secretion, elimi-
In cases of volume depletion, the decrease in the effective circu-
nating the root cause of increased AVP secretion will allow excretion
lating plasma volume triggers the activation of both the sympathetic
of excess retained water and correct the hyponatremia; however, if
nervous system and the renin-angiotensin system to maintain the
this is not possible, then direct treatment of the hyponatremia may
plasma volume by reducing urine Na1 excretion. However, when
be necessary.
there is a large decrease in circulating plasma volume (loss of 5% to
10% of body fluid volume) that lowers blood pressure (BP), AVP is
secreted independently of Posm by activation of arterial barorecep- Etiologies of Disorders of Serum [Na1]
tors, and the excretion of free water is suppressed, thus maintaining Plasma [Na1] is the main determinant of effective osmolality (tonic-
ECF volume. When ECF volume is severely reduced, AVP secretion ity). Plasma [Na1] is nearly equal to the total number of cations in
is overwhelmingly dependent on the effective circulating plasma the body (total Na1 content 1 total K1 content), which cannot

Table 1. Physiologic regulatory systems controlling AVP secretion

Osmoregulatory System Volume Regulatory System

Input Posm Effective circulating arterial volume


Sensors Hypothalamic osmoreceptive neurons Carotid sinus, atrial, and renal afferent arteriole baroreceptors
Effectors AVP, and thirst Sympathetic nervous system, renin-angiotensin-aldosterone
system, natriuretic peptide system, and AVP
Outputs Urine free water excretion, and fluid Urine Na1 and free water excretion
intake (drinking behavior)

AVP, arginine vasopressin; Posm, plasma osmolality.


Nephrology Self-Assessment Program - Vol 20, No 2, January 2022 77

ISOTONIC VOLUME DEPLETION Serum [Na1] is affected both by the tonicity of fluid ([Na1] 1
1
ISOVOLEMIC OSMOTIC INCREASE [K ]) entering the body and by the tonicity of fluid (mainly urine)
50 ([Na1] 1 [K1]) discharged from the body. Serum [Na1] is regu-
lated by the balance of cations, such as Na1, K1, and free water.
PLASMA AVP CONCENTRATION (pg/ml)

45 PAVP = 1.3e–0.17 VOL


Conditions affecting each part of the Edelman equation are illus-
40 trated in Figure 5 (22).
35 Thus, whenever an abnormality in serum [Na1] is observed, it
is necessary to assess the tonicity of both the infusion solution and
30
the excreted urine. If urine ([Na1] 1 [K1]) is higher than infusion
25 ([Na1] 1 [K1]), then serum [Na1] tends to decrease; conversely, if
PAVP = 2.5 OSM + 2.0 urine ([Na1] 1 [K1]) is lower, serum [Na1] tends to increase.
20
Therefore, when evaluating abnormalities in serum [Na1], urine
15 tonicity mainly consisting of urine ([Na1] 1 [K1]) cannot be
10
ignored because, unlike the negligible plasma [K1], urine [K1] con-
tributes significantly as an effective osmole.
5

0
Evaluation of Hyponatremia
When evaluating patients with hyponatremia, one must first exclude
0 5 10 15 20
isotonic and hypertonic hyponatremia.
PERCENT CHANGE

Check Plasma Osmolality (Posm)


Most patients with hyponatremia have a Posm of ,280 mOsm/kg
Figure 4. Changes in plasma AVP concentrations in response to iso-
H2O, but exceptions are cases of isotonic and hypertonic hyponatre-
volemic increases in Posm and isotonic volume depletion. AVP is
mia, so it is important to exclude these two conditions first because
secreted when the effective Posm (tonicity) increases or when the
they do not require specific treatment of the hyponatremia
effective circulating plasma volume decreases. In a dehydrated state,
osmoreceptors in the hypothalamus can detect increases of 1%–2%
in the effective Posm, triggering AVP secretion from the posterior Isotonic Hyponatremia (280–295 mOsm/kg H2O). It is possible
pituitary gland. In a severely volume-depleted state (loss of 5%–10% for true hyponatremia to exist with normal Posm.
of body fluid volume) that lowers BP, AVP is secreted independently Causes include hyperglycemia, administration of osmotic agents
of Posm by activation of arterial baroreceptors, and the excretion of (mannitol, sorbitol, glycine, glycerol), and radiographic contrast
free water is suppressed, thus maintaining ECF volume. When ECF agents (23–25). The serum contains additional osmoles that increase
volume is severely reduced, AVP secretion is overwhelmingly depen- effective osmolality (tonicity) and reduce the serum [Na1] by moving
dent on the effective circulating plasma volume rather than on Posm. water from the ICF to the ECF compartments. The most common
Abbreviations: AVP, arginine vasopressin; blood pressure, BP; ECF, cause is hyperglycemia due to uncontrolled diabetes mellitus. The
extracellular fluid; pg/ml, picograms per milliliter; DOSM, percent resulting hyponatremia is called a translocational hyponatremia.
change in plasma osmolality; PAVP, plasma arginine vasopressin con- Despite true hyponatremia, albeit with a normal Posm, hyperglycemia-
centration; Posm, plasma osmolality; DVOL, percent change in blood induced hyponatremia is frequently erroneously included as one of
volume. Reprinted with permission from reference 18 (Dunn FL, the forms of pseudohyponatremia.
Brennan TJ, Nelson AE, Robertson GL: The role of blood osmolal- Pseudohyponatremia occurs when serum [Na1] is seemingly
ity and volume in regulating vasopressin secretion in the rat. J Clin low by measurement, but is actually normal. This occurs when the
Invest 52: 3212–3219, 1973). amount of water in the plasma decreases because of solid compo-
nents such as increased protein or triglycerides that occupy part of
the measured serum volume at the time of measurement. Conse-
quently, the concentration of Na1 per volume of serum is low, but
normally be measured, divided by the total body water, as is demon-
the concentration of Na1 in the serum water is really normal. Pseu-
strated by Edelman’s equation (21):
dohyponatremia is therefore an artifact of measurement and does
 
Plasma Na1 ðmmol=LÞ 5 1:11 not represent true hyponatremia. It is usually caused by hypertrigly-

3 total body Na1 e 1 K 1 e ÞðmmolÞ=total body water ðLÞg 2 25:6: ceridemia or paraproteinemia from multiple myeloma. Because
serum osmolality is normal, no shifts of water from the ICF to the
ECF compartments occur, and no direct treatment of the hypona-
This can be simplified conceptually as follows:
tremia is necessary. If there is uncertainty about a diagnosis of pseu-
Plasma ½Na1  ðmmol=LÞ dohyponatremia (such as when low serum [Na1] is detected in a

5 total body Na1 e 1 K 1 e ðmmolÞ=total body water ðLÞ: patient with known myeloma and increased serum light chains, or a
patient is known to have hypertriglyceridemia), direct measurement
In the above formula, Na1e and K1e represent exchangeable of Posm or a blood gas measurement should be performed because
ions, that is, ions not affixed to bone or tissue. neither is affected by dilution of the blood.
78 Nephrology Self-Assessment Program - Vol 20, No 2, January 2022

DECREASED
Hypovolemic hyponatremia DECREASED
• Poor Na+ intake Vast majority of hypovolemic
• Renal or extrarenal Na+ losses and hypervolemic
hyponatremia
NORMAL • Poor K+ intake
Euvolemic hyponatremia • Renal or extrarenal K+ losses
Nae+ Ke+
• In the SIAD, small renal Na+
losses occur NORMAL
Euvolemic hyponatremia
INCREASED
• In the SIAD, small renal K+
Hypervolemic hyponatremia losses occur
• Na+ retention

Nae+ + Ke+
serum [Na+] =
TBW

TBW

DECREASED NORMAL INCREASED


Hypovolemic hyponatremia Hypovolemic hyponatremia Euvolemic hyponatremia
• Impaired aquaresis • Impaired aquaresis • Impaired aquaresis
Water retention falls short of Water retention normalizes Water retention expands TBW
normalizing TBW TBW • Excessive water intake

Hypervolemic hyponatremia
• Impaired aquaresis
Water retention expands TBW

Figure 5. Factors influencing serum [Na1] based on the Edelman equation. Hyponatremia represents an excess of water relative to the Na1
and K1 stores in the body. Hypotonic hyponatremia can show decreased, normal, or increased Nae1; decreased or normal Ke1; and decreased,
normal, or increased TBW, which means hyponatremia can occur with any ECF volume status. Abbreviations: Ke1, exchangeable potassium
ion; Nae1, exchangeable sodium ion; SIAD, syndrome of inappropriate antidiuresis; [Na1] sodium concentration; TBW, total body water.
Reprinted with permission from reference 22 (Adrogue HJ, Madias NE: The challenge of hyponatremia. J Am Soc Nephrol 23: 1140–1148,
2012).

Hypertonic Hyponatremia (>295 mOsm/kg H2O). Glucose, man- in the urine when needed, and a maximum of 15 to 20 L of water
nitol, sorbitol, glycerol, and radiographic contrast media are effective can be excreted daily in individuals with normal renal function and
osmotic substances and can move water from the ICF to the ECF, maximum urinary dilution ability (e.g., Uosm 50 mOsm/kg H2O) if
leading to a dilutional (i.e., translocational) hyponatremia. These are solute intake is adequate (e.g., 600–900 mOsm/day).
the same osmotically active molecules that can cause isotonic hypona- Hyponatremia due to water intoxication can occur when very
tremia, but when concentrations in the plasma exceed levels that can large volumes (i.e., 15–20 L/day) of fluids are consumed. However,
be compensated for by shifts of water from the ICF, then hyperosmo- insufficient solute intake can also lead to hypotonic hyponatremia.
lality occurs. Hyperglycemia decreases serum [Na1] by 1.6 mmol/L Solutes responsible for driving renal free water excretion include
for every 100 mg/dL increase in glucose above the normal 100 mg/dL electrolytes, such as Na1 and K1, and urea (26). The excretion of
blood concentration. However, when the blood glucose concentration these solutes decreases when oral intake is reduced; therefore, if daily
is $400 mg/dL, serum [Na1] decreases by 2.4 mmol/L for every 100 solute intake is decreased, the ability of the kidney to excrete free
mg/dL increase in glucose above normal. Thus, recalculation is water is decreased as well. If solute intake is limited, the ingestion of
required to ascertain the actual serum [Na1] (23). lesser amounts of water than 15 to 20 L/day can result in dilutional
A blood glucose concentration should be checked in all hypo- hyponatremia because decreased renal free water excretion will cause
natremic patients to have an accurate assessment of serum [Na1]. If water retention.
the blood glucose concentration is normal, recent administration of Evaluation of hypotonic hyponatremia requires further investi-
mannitol, sorbitol, glycerol, or radiographic contrast medium should gation to determine the cause (Figure 6) (27).
be considered.

Hypotonic Hyponatremia (<280 mOsm/kg H2O). In patients Assess ECF Volume


with hypotonic hyponatremia, urine osmolality and electrolytes When there are obvious signs of increased ECF volume status (such
should be measured to rule out water intoxication or inadequate sol- as edema or ascites), then heart failure, cirrhosis, nephrotic syn-
ute intake. The kidney has a very high capacity to excrete free water drome, or renal failure should be excluded. Conversely, when
Nephrology Self-Assessment Program - Vol 20, No 2, January 2022 79

Hyponatremia
Serum [Na+] < 135 mmol per L

Symptomatic (e.g., confusion, ataxia, seizures, Asymptomatic or mild symptoms


obtundation, coma, respiratory depression) (e.g., headache, lethargy, dizziness)

Severe hyponatremia Determine Posm

Normal Low High


280 to 295 mOsm per kg H2O < 280 mOsm per kg H2O > 295 mOsm per kg H2O

Isotonic hyponatremia Hypotonic hyponatremia Hypertonic hyponatremia


(e.g., pseudohyponatremia)

Assess volume status Assess for hyperglycemia, check for


Assess for hyperproteinemia mannitol or sorbitol use or recent
or hyperlipidemia administration of radiocontrast media
Evaluate vital signs, orthostatics, jugular venous
pressure, skin turgor, mucous membranes, peripheral
edema, and blood urea nitrogen and uric acid concentrations

Hypovolemic (decreased total Euvolemic (increased total body water, Hypervolemic (increased total
body water and Na+ content) normal total body Na+ content) body water more than Na+ content)

Urine [Na+] usually > 20-30 mmol per L

Urine [Na+] Urine [Na+] Urine [Na+] Urine [Na+]


> 20-30 mmol per L < 20 mmol per L < 20 mmol per L > 20-30 mmol per L

Uosm Uosm Variable


Renal loss (e.g., Extrarenal loss > 100 mOsm per kg H2O < 100 mOsm per kg H2O Uosm Heart failure Renal failure
from diuretics or (e.g., from cirrhosis, nephrosis,
mineralocorticoid vomiting, diarrhea, hypoalbuminemia
deficiency) third spacing, or
Syndrome of Primary polydipsia,
bowel obstruction)
inappropriate anti- low solute intake Reset
diuretic hormone (beer potomania osmostat
secretion, hypo- syndrome)
thyroidism, adrenal
insufficiency,
stress, drug use

Figure 6. Algorithm for the evaluation of hyponatremia. The presence or absence of symptoms associated with hyponatremia should first be
assessed. Subsequently, Posm is measured to exclude both isotonic and hypertonic hyponatremia. If hypotonic hyponatremia is revealed, evaluate
the ECF volume status and proceed with the differential diagnosis of the hyponatremia. Abbreviations: ECF, extracellular fluid; kg H2O, kilo-
gram of water; L, liter; mOsm, milliosmoles; Posm, plasma osmolality; [Na1], sodium concentration; Na1, sodium ion; Uosm, urine osmolality.
Licensed under a Creative Commons License (CC BY NC ND). Reprinted with permission from reference 28 (Fenske W, St€ork S, Koschker
AC, Blechschmidt A, Lorenz D, Wortmann S, et al.: Value of fractional uric acid excretion in differential diagnosis of hyponatremic patients on
diuretics. J Clin Endocrinol Metab 93: 2991–2997, 2008).

obvious signs of decreased ECF volume status are present (such as be considered when there is no apparent increase or decrease in
decreased BP or dry mucous membranes and skin), insufficient ECF volume (i.e., the patient is euvolemic). Specific hormones
intake (malnutrition), vomiting or diarrhea, loss of Na1 and K1 can be measured to confirm the diagnosis. Urine [Na1] of .20 to
from the gastrointestinal tract or skin due to burns, loss from the 30 mmol/L in the absence of external natriuretic administration is
kidneys due to diuretics (natriuretics), and adrenal insufficiency suggestive of euvolemic hyponatremia. The cutoff value of urine
should be excluded. The syndrome of inappropriate antidiuretic [Na1] .30 mmol/L is less useful when a patient is being given
hormone (SIADH) secretion, mineralocorticoid-responsive hy- exogenous natriuretics, but a diagnosis of SIADH can be made in
ponatremia of the elderly, severe hypothyroidism, glucocorticoid such patients if the fractional excretion of uric acid (FEUA) is $12%
deficiency, hypopituitarism, and adrenal insufficiency should (28).
80 Nephrology Self-Assessment Program - Vol 20, No 2, January 2022

Common Causes of Hyponatremia related to 12

Nonosmotic and/or Inappropriate AVP Release

Plasma AVP concentration (pg/ml)


Type C
In the sections below, we mainly focus on SIADH, drug-induced
hyponatremia, and iatrogenic hyponatremia, related to nonosmotic
and/or inappropriate AVP release. 8
Type A

Syndrome of Inappropriate Antidiuretic Hormone


Secretion versus Syndrome of Inappropriate Antidiuresis 4
SIADH was first reported by Schwartz et al. (29) in 1957 in a Type B
patient with bronchogenic carcinoma as “A Syndrome of Renal
Sodium Loss and Hyponatremia Probably Resulting from Inappro-
Type D
priate Secretion of Antidiuretic Hormone.” Schwartz consulted with 0
F. C. Bartter, who was enrolled in the National Institutes of Health 120 130 140 150
(NIH) at the time, about a case of hyponatremia of unknown cause. Serum [Na+] (mmol/liter)
Coincidentally, Bartter had also observed a similar case. As a result
of this initial case report and collaboration, SIADH is also known as
the Schwartz-Bartter syndrome; the story of SIADH was the final Figure 7. Types of arginine vasopressin (AVP) secretion in the syn-
episode of “Milestones in Nephrology” in the Journal of the Ameri- drome of inappropriate antidiuresis. SIAD presents with hypotonic
can Society of Nephrology (30). In SIADH, AVP secretion is not hyponatremia due to body water retention caused by insufficient free
suppressed despite hypotonicity and hyponatremia, and despite water excretion that is inappropriate for a plasma hypo-osmolality.
euvolemic (normal ECF volume) status. Therefore, it is a disorder There are four types of SIAD. In type A, plasma AVP concentration
that presents with hyponatremia caused by inappropriate antidiure- fluctuates randomly, independently of serum [Na1]. In type B, there
sis. Potential causes are many and include central nervous system is a constant low amount of AVP that is increased further in response
diseases, lung diseases, ectopic AVP-producing tumors, and many to increased serum [Na1]. In type C, osmoregulation is shifted to
drugs. Although the exact epidemiology is unknown, hyponatremia the left, meaning AVP is released despite of a lower set point of
is present in 15% to 30% of all hospitalized patients (8,9,31,32), plasma [Na1] for AVP secretion than in the normal state (reset
suggesting that the number of patients with SIADH is potentially osmostat). In type D, AVP is actually undetectable or low, in low
quite large. serum [Na1]. Abbreviations: AVP, arginine vasopressin; mmol/l,
SIAD is a more general term for any conditions that present millimoles per liter; pg/ml, picograms per milliliter; SIAD, syn-
with hypotonic hyponatremia due to body water retention caused drome of inappropriate antidiuresis; [Na1], sodium concentration.
by insufficient free water excretion that is inappropriate for a low Reprinted with permission from reference 4 (Ellison DH, Berl T:
Posm. This term is currently preferred over SIADH for several rea- Clinical practice. The syndrome of inappropriate antidiuresis. N
sons: (1) a substantial number of cases of SIADH have low AVP Engl J Med 356: 2064–2072, 2007).
concentrations, often below detection limits for AVP radioimmuno-
assay (see type D below), and (2) a genetic disease caused by an
AVP V2 receptor mutation that causes constitutive activation (gain- Drug-Induced Hyponatremia
of-function mutations) of the V2 receptor is indistinguishable from
Hyponatremia is the most frequently observed drug-induced water
SIADH by standard clinical criteria and has been termed nephro-
and electrolyte abnormality. In some cases, hyponatremia is caused
genic SIAD (33–35). This patient group does not basically respond
by natriuretic agents such as thiazide diuretics, but the majority of
to the use of vasopressin receptor antagonists (vaptans) (35). There-
drug-induced hyponatremia is due to impaired urine dilutional abil-
fore, as long as the same criteria are used for the diagnosis, SIADH
ity by nonphysiologic (i.e., inappropriate) antidiuresis as a result of
and SIAD are synonymous and can be used interchangeably. Four
stimulation of AVP secretion or enhancement of AVP effects in the
types of abnormal AVP secretion have been described in patients
kidney. Specific examples include (1) increased AVP secretion from
with SIAD (Figure 7) (4):
the posterior pituitary, (2) enhanced AVP action in the renal
Type A: AVP fluctuates randomly independently of serum medulla, and (3) decreased AVP secretion threshold (reset
[Na1]. osmostat).
Type B: There is a constant low amount of AVP that is Hyponatremia caused by diuretics (natriuretics) such as thia-
increased further in response to increased serum [Na1]. zides is frequently associated with other acid–base imbalance and
Type C: Osmoregulation is shifted to the left (reset osmostat).
electrolyte abnormalities. Diuretic-associated hyponatremia is espe-
Type D: AVP is actually undetectable or low, in low serum
cially common among outpatients. Among them, patients taking
[Na1].
thiazide are about 10 times more frequent than those taking loop
Types A through C are found in approximately 30% of SIAD diuretics, and the hyponatremia usually develops within 2 weeks
cases and include most of the cases due to malignant tumors. Type after initiation of diuretic therapy (36). However, thiazide-induced
D is found in about 5% to 10% of SIAD cases, and the causes are hyponatremia may take 17 to 19 days to develop after the initiation
currently not understood (4). of thiazide therapy (37). This is because thiazides act on the distal
Nephrology Self-Assessment Program - Vol 20, No 2, January 2022 81

Table 2. Pathophysiology and common drugs of drug-induced AVP-related hyponatremia

Increased AVP secretion from the posterior pituitary


Antidepressants: tricyclic antidepressants, selective serotonin reuptake inhibitors (SSRIs), monoamine oxidase inhibitor
Antipsychotics: phenothiazine, butyrophenone
Antiepileptic drugs: carbamazepine, sodium valproate
Antimalignant tumor drug, immunosuppressive drug: vinca alkaloid (vinca alkaloid, vinblastine), platinum preparation (cisplatin,
carboplatin), alkylating drugs (cyclophosphamide intravenous injection, melphalan, ifosfamide), others (methotrexate, interferon
a/g, pentostatin)
Narcotics: morphine
Psychoactive substances: 3,4-methylenedioxymethamphetamine “ecstasy”, amphetamine
Enhanced AVP action in the renal medulla
Antiepileptic drugs: carbamazepine, lamotrigine
Antidiabetic drugs: chlorpropamide, tolbutamide
Antimalignant tumor drug: alkylating drug (cyclophosphamide intravenous injection)
Nonsteroidal anti-inflammatory drugs
Decreased AVP secretion threshold (reset osmostat)
Antiepileptic drug: carbamazepine
Decreased urine dilutional ability in the renal dilutional segment and increased AVP production in the hypothalamus
Natriuretics: thiazide, indapamide, loop diuretics
Miscellaneous
Angiotensin-converting enzyme inhibitors, sulfamethoxazole-trimethoprim, immunoglobulin preparations, proton pump inhibitors,
amiodarone, theophylline, oral bowel preparation for colonoscopy

AVP, arginine vasopressin.

tubules of the renal cortex and do not interfere with the formation considered to be a rare side effect (41). In the observational study
of interstitial osmotic gradients. Risk factors for drug-induced hypo- cited above (40), only nine cases of severe hyponatremia were
natremia include older age, female sex, low body weight, low- believed due to endocrinopathies, all of which resulted from sec-
sodium diet, and concomitant use of drugs that affect water balance. ondary adrenal insufficiency. SIADH and volume disturbances
Drugs that cause hyponatremia are listed in Table 2. They were diagnosed in 35% and 20% of severe hyponatremia cases,
include angiotensin-converting enzyme (ACE) inhibitors, narcotics respectively. Risk factors for hyponatremia included use of
(amphetamine, 3,4-methylenedioxymethamphetamine, immunoglob- CTLA-4 inhibitor, use of diuretics, and non-white race.
ulin preparations, sulfamethoxazole-trimethoprim, proton pump
inhibitors, theophylline, and amiodarone (38). Iatrogenic Hyponatremia
Hyponatremia is a commonly reported adverse effect associ- A decrease in serum [Na1] indicates a decrease in effective osmolality
ated with immune checkpoint inhibitors (ICIs), which have (tonicity), with a few exceptions. Normally, when Posm decreases,
become widely used in cancer immunotherapy in recent years thirst is naturally suppressed, and AVP secretion is also suppressed so
(39,40). In a recent review of 2458 patients who received ICIs, as to produce a dilute urine, which assists in maintaining Posm within
62% experienced hyponatremia (defined as serum [Na1] ,134 normal ranges. Within this physiologic construct, in order to cause
mmol/L) during the first year of treatment, and 6% had severe hyponatremia, there must be (1) abnormal urine dilutional ability
hyponatremia (defined as serum [Na1] ,124 mmol/L) (40). The and (2) abnormal water intake or administration (i.e., drinking more
mechanism underlying the ICI-induced hyponatremia varies. than is needed to maintain normal Posm).
Because hypopituitarism or adrenal insufficiency may occur with The administration of exogenous hypotonic solutions during
ICIs, hyponatremia may be caused by cortisol deficiency in some hospitalization is an important cause that doesn’t occur during
cases. Hypopituitarism due to ICIs has been reported in all anti- normal life. In addition, nonosmotic AVP secretion may be
cytotoxic T-lymphocyte–associated protein 4 (CTLA-4), anti- increased as a result of various physical and mental stresses during
programmed cell death 1 (PD-1), and antiprogrammed death hospitalization, which can be an important factor in the onset of
ligand 1 (PD-L1) antibodies, but it is especially common with hyponatremia. Many inpatients are exposed to pain, vomiting, stress,
the use of anti-CTLA-4 antibodies (41–43). Hyposecretion of and various drugs, all of which can cause nonosmotic AVP secretion.
ACTH causes hypocortisolism, which leads to impaired free When hypotonic fluids are administered as maintenance infusions,
water excretion resulting from the effects on both AVP secretion or when parenteral or enteral feedings are given, the free water con-
and renal water retention (44). Primary adrenal insufficiency due tent may exceed renal excretion ability, resulting in hyponatremia.
to ICIs has been reported in all treatments with anti-CTLA-4, All infusion and nutritional therapies should be selected with consid-
anti-PD-1, and anti-PD-L1 antibodies, but the frequency is eration of body fluid status, and it is necessary to review, on a daily
reported to be only 0.7% in patients with these drugs, which is basis, whether the infusion is appropriate for body fluid homeostasis.
82 Nephrology Self-Assessment Program - Vol 20, No 2, January 2022

Symptoms and Adverse Outcomes of Hyponatremia


Patients with acute hyponatremia, especially when complicated by 4 Acute hyponatremia (<48 hours) is associated
severe neurologic symptoms, face a threat to life because of cerebral with cerebral edema and neurologic symptoms
edema and brain herniation; therefore, aggressive therapeutic inter-
vention is necessary. Chronic hyponatremia is often considered to
including headache, malaise, nausea, vomiting,
be asymptomatic; consequently, it is frequently not treated effec- confusion, convulsions, impaired consciousness,
tively. However, in recent years, it has been understood that even and coma (hyponatremic encephalopathy).
mild chronic hyponatremia can lead to gait instability, falls, frac- 4 Chronic hyponatremia (48 hours) is associated
tures, decreased quality of life, and worsening overall prognosis.
with cognitive impairment, gait instability, osteo-
Acute Hyponatremia porosis, falls, fractures, decreased ADL, and
Acute hyponatremia causes cerebral edema, and patients present increased mortality.
with neurologic symptoms resulting from central nervous system
injury. Symptoms related to acute hyponatremia depend on its
severity and rate of progression and include headache, malaise, nau-
sea, vomiting, confusion, convulsions, impaired consciousness, and
coma (hyponatremic encephalopathy) (5,6). Generally, when the (hazard ratio, 1.21) (50). Decreased bone mineral density may be
serum [Na1] is $125 mmol/L, there are no severe symptoms one of the causes of an increased risk of fractures associated with
(sometimes headache, vomiting, or memory impairment as mild to hyponatremia.
moderate symptoms). However, at 120 to 125 mmol/L, confusion Additionally, patients who were admitted to the hospital with
and loss of appetite appear, and when serum [Na1] drops to 115 to moderate hyponatremia, defined as a serum [Na1] ,130 mmol/L,
120 mmol/L, restlessness or somnolence may be observed. Life- were more than seven times more likely to die in the hospital than
threatening symptoms such as convulsions and coma can occur were control patients (51). Thus, chronic hyponatremia is associated
when serum [Na1] is ,115 mmol/L. However, it is important to with various complications, including cognitive impairment, gait
note that not only the absolute value of serum [Na1], but also the instability, osteoporosis, falls, fractures, decreased activities of daily
rate of decrease in serum [Na1] has a significant impact on the living (ADL), and increased mortality (52).
onset of symptoms and outcomes of hyponatremia. For example, an
acute fall in serum [Na1] to ,125 mmol/L has caused fatal brain
edema (e.g., marathon). Mortality rates of 30% for serum [Na1] of Treatment of Hyponatremia
115 to 120 mmol/L and .50% for serum [Na1] of ,115 mmol/L Treatment goals differ according to the time course and symptoms
have been reported (45). of hyponatremia. Currently, there are no evidence-based uniform
standardized guidelines for treatment of hyponatremia. Many pub-
Chronic Hyponatremia lished guidelines are, in fact, based on expert opinion derived from
Chronic hyponatremia neither results in cerebral edema nor causes observational or clinical studies with small sample sizes. Further-
neurologic symptoms because of the ability of the brain to regulate more, the Hyponatremia Registry revealed that 21.6% of hyponatre-
its volume by excretion of electrolyte and organic solutes (46,47). mic patients were untreated for hyponatremia, suggesting a lack of
However, as noted above, cognitive dysfunction such as attention insight or concern about this condition (53).
deficits and neurologic symptoms, including gait instability, have It is relevant to ask why hyponatremia goes untreated so fre-
been observed in patients with chronic hyponatremia considered to quently. First, hyponatremia is a very heterogeneous condition,
be asymptomatic, and the risk of falls and fractures is increased in and its various causes, symptoms, and difficulty in determining
such patients. Renneboog et al. (48) reported that gait stability was the true ECF volume status together make it challenging to arrive
reduced and the odds ratio of falls was as high as 67 times in at a definitive diagnosis that leads to effective treatment. Second,
patients with relatively mild chronic hyponatremia with an average there is less confidence on the magnitude of the impact of treat-
serum [Na1] of 126 mmol/L. Verbalis et al. (49) demonstrated that ing hyponatremia on clinical outcomes. Third, there is fear that
hyponatremia is associated with significantly increased odds of osteo- overly rapid correction of hyponatremia will lead to increased
porosis using cross-sectional human data, and that hyponatremia risk of the osmotic demyelination syndrome (ODS), which is
markedly reduced bone mass via increased bone resorption using a described later. Finally, there are limited treatment options for
rat model of SIAD. Notably, the group showed that sustained low hyponatremia that have sufficient evidence-based data to verify
ECF [Na1] directly stimulated osteoclastogenesis and that their effectiveness and safety (54).
hyponatremia-induced oxidative stress is involved in the mechanism
(49). The Rotterdam study, a population-based cohort study includ-
ing 5208 individuals who had serum [Na1] assessed at baseline,
Current Recommendations Regarding Rate of Serum
found that hyponatremia was associated with an increased risk of [Na1] Correction
nonvertebral fractures after adjustment for age, sex, and body mass When hyponatremia occurs, both electrolytes and organic osmolytes
index (hazard ratio, 1.39). In the fully adjusted model, hyponatre- begin to move from the inside to the outside of the brain cells as a
mia was also associated with an increased risk of vertebral fractures protection mechanism to prevent cerebral edema. Studies in both
(odds ratio, 1.78) and with significantly higher cumulative mortality experimental animals and humans suggest that this process is usually
Nephrology Self-Assessment Program - Vol 20, No 2, January 2022 83

complete after 48 hours (6). Therapeutic recommendations differ From the results of both the Republic of Korea and the Ireland
depending on the time course (acute/chronic) and symptoms (symp- studies, and as recommended in the United States expert panel rec-
tomatic/asymptomatic), the urgency of treatment, the degree of ommendations and the EU guideline, bolus infusion of 3% NaCl
serum [Na1] to be corrected, and the upper limit of the correction. for acute hyponatremia with severe symptoms is considered both
It is extremely important that serum [Na1] must be increased rap- safe and more effective than continuous infusions (Table 3). How-
idly in acute/symptomatic hyponatremia, whereas serum [Na1] ever, the data from both studies favors treatment using the smaller-
should not be increased rapidly in chronic/asymptomatic hyponatre- volume boluses recommended by the United States expert panel. It
mia for reasons discussed below. is interesting to note that the United States expert opinion panel
Symptoms of hyponatremic encephalopathy (i.e., symptoms recommendations were based on practical experiences with treat-
associated with cerebral edema) are observed in acute hyponatremia ment of exercise-associated hyponatremia (EAH). EAH is a com-
and need to be treated (5,6). Higher-risk groups for hyponatremic mon cause of acute/symptomatic hyponatremia, sometimes with
encephalopathy include children, premenopausal women, postopera- poor outcomes resulting from cerebral edema and brain herniation.
tive patients, patients with central nervous system disorders, patients EAH occurs when exertion results in serum [Na1] ,135 mmol/L,
with lung disease, and hypoxemic patients (55). Recent reports have and symptoms can present up to 24 hours after physical activity.
suggested that increased intracranial pressure may stimulate nonos- Upon screening, #5% to 30% of asymptomatic ultramarathon run-
motic AVP release (56), indicating that an increase in intracranial ners experience hyponatremia by the end of the event. The usual
pressure itself might lead to further exacerbation of hyponatremia. cause of EAH is overhydration with hypotonic fluids such as water
Immediate correction of serum [Na1] is needed in such cases to or sports drinks; however, nonosmotic and inappropriate AVP secre-
avoid the life-threatening consequences of tentorial brain herniation tion caused by pain, stress, exercise, nausea, and hypoglycemia can
and irreversible brain damage. A small, quick increase in the serum predispose to EAH. Severe EAH should be rapidly treated with 100
[Na1] (2–5 mmol/L) is effective in treating acute hyponatremia, mL bolus infusion of 3% NaCl over 10 min, which can be repeated
because reducing cerebral edema will substantially decrease intracere- every 30 min, as necessary, for a total of no more than three doses,
bral pressure (57). Studies have shown that increasing serum [Na1] which can be repeated every 10 minutes for a total of three boluses.
by 5 mmol/L reduces intracranial pressure by 50% (58). These doses increase serum [Na1] by 2 to 5 mmol/L and do not
Expert panel recommendations and consensus guideline were increase the risk of ODS, because this is an acute hyponatremia
published in the United States (5) and Europe (6), respectively. (Figure 8) (5).
According to the United States expert panel recommendations, The situation for patients with chronic/asymptomatic hypona-
tremia is different from that for patients with acute/symptomatic
 For severe symptoms, 100 mL of 3% NaCl infused intrave-
hyponatremia. Patients with a more chronic hyponatremia (defined
nously over 10 min 3 3 as needed.
as $48-hour duration) and mild or no neurologic symptoms are at
 For mild to moderate symptoms with a low risk of herniation,
3% NaCl infused at 0.5 to 2 mL/kg per hour. little immediate risk from complications arising from the hyponatre-
mia itself, but they can experience ODS after overly rapid correction
The rate of correction should not be restricted in patients with of serum [Na1]. There is no indication to rapidly correct the serum
true acute hyponatremia (defined as a duration ,48 hours), nor is [Na1] in these patients, and slower-acting therapies such as fluid
the re-lowering of excessive corrections indicated; however, if there restriction or discontinuation of drugs that limit free water excretion
is any uncertainty as to whether the hyponatremia is chronic versus should be used with correction of serum [Na1] during a 48- to
acute, then the upper limits for correction of chronic hyponatremia 72-hour period. Although these situations have reasonably clear
should be followed (discussed below). treatment indications, most patients have hyponatremia of unknown
A recent prospective study from Ireland reported that duration and varying degrees of neurologic impairment. This group
patients with symptomatic hyponatremia due to SIAD were cor- presents the most challenging treatment decision because hyponatre-
rected more quickly when treated with 3% NaCl bolus infusions mia is sufficiently long to allow some degree of brain volume
than were those treated with conventional continuous intrave- regulation but is not long enough to prevent cerebral edema and
nous infusions of 3% NaCl. Most importantly, a more rapid neurologic symptoms. Most authors recommend prompt treatment
improvement in the Glasgow Coma Scale score was also observed for such patients because of their symptoms, but with methods that
in patients treated with boluses of 3% NaCl. Also of note, four allow a controlled and limited correction of their hyponatremia.
deaths occurred in the continuous intravenous infusion group, Reasonable correction parameters consist of a rate of correction of
whereas no deaths occurred in the bolus infusion group (59). If serum [Na1] that results in a total magnitude of correction that
the death of these four patients was due to hyponatremia, it can does not exceed 10 to 12 mmol/L during the first 24 hours and 18
be inferred that more immediate correction of severely symptom- mmol/L throughout the first 48 hours of correction (5). However,
atic hyponatremia is critically important. the correction goal should be set lower than the limit (i.e., 6–8
In the SALSA randomized clinical trial, published recently mmol/L) to reduce the chance of exceeding the upper limit. If the
from the Republic of Korea, both continuous intravenous infusion correction upper limit is exceeded, consideration should be given to
and bolus infusions of 3% NaCl were reported to be effective and the administration of hypotonic fluid (e.g., dextrose 5% in water
safe treatments for symptomatic hyponatremia. In contrast to the [D5W]) to bring the correction magnitude back to the upper limit
study from Ireland, larger volumes were used for the 3% NaCl (Figure 8) (5).
boluses (150 mL versus 100 mL), which resulted in higher rates of However, the maximum correction rates should be even lower
re-lowering of serum [Na1] to prevent overly rapid correction of (no more than 8 mmol/L in 24 hours) if risk factors for the develop-
hyponatremia (60). ment of ODS are present, including alcoholism, liver disease,
84 Nephrology Self-Assessment Program - Vol 20, No 2, January 2022

Table 3. United States recommendations and European Union guideline for the treatment of hyponatremia.

Subject United States Recommendations European Union Guideline

Acute/symptomatic hyponatremia
Severe symptoms Severe symptoms
Bolus 3% NaCl (100 mL over 10 min 3 3 Bolus 3% NaCl (150 mL over 20 min 3
as needed) 2–3 as needed)
Moderate symptoms Moderate symptoms
Continuous infusion 3% NaCl Bolus 3% NaCl (150 mL over 20 min
(0.5–2 mL/kg/h) once)
Chronic/mild symptomatic hyponatremia
SIAD Fluid restriction (first line) Fluid restriction (first line)
Demeclocycline, urea, or vaptans (second Urea or loop diuretics 1 oral NaCl
line) (second line)
Do not recommend or recommend
against vaptansa
Recommend against lithium or
demeclocycline
Hypovolemic hyponatremia Normal (Isotonic) saline Normal (Isotonic) saline or balanced
crystalloid solution
Hypervolemic hyponatremia Fluid restriction Fluid restriction
Vaptansb Recommend against vaptans
Correction rates Minimum Minimum
Dserum [Na1] 148 mmol/L/d None
Dserum [Na1] 146 mmol/L/d Limit
(high risk of ODS) Dserum [Na1] 110 mmol/L/d
Limits
Dserum [Na1] 11012 mmol/L/d
Dserum [Na1] 18 mmol/L/d
(high risk of ODS)
Management of overly rapid correction Baseline serum [Na1] 120 mmol/L Start once limit is exceeded
Probably unnecessary Consult an expert to discuss infusion
Baseline serum [Na1] <120 mmol/L containing electrolyte-free water
Start re-lowering with electrolyte-free (10 mL/kg BW) with or without 2 mg
water or desmopressin after correction desmopressin IV
exceeds Dserum [Na1] 168 mmol/L/d

Reprinted with permission from reference 3 (Hoorn EJ, Zietse R: Diagnosis and treatment of hyponatremia: Compilation of the guidelines. J Am Soc Nephrol 28: 1340-1349,
2017). BW, body weight; IV, intravenous; ODS, osmotic demyelination syndrome; SIAD, syndrome of inappropriate antidiuresis.
a
Do not recommend when serum [Na1] is <130 mmol/L; recommend against when serum [Na1] is <125 mmol/L.
b
In liver cirrhosis, restrict to patients where potential benefit outweighs risk of worsened liver function.

malnutrition, hypokalemia, and a very low serum [Na1] (#105 active phases of treatment (every 2 to 4 hours for 3% NaCl
mmol/L) (5). In such cases, goals should be commensurately lower administration; every 6 to 8 hours for vasopressin antagonist
(i.e., 4–6 mmol/L), and if the correction rate exceeds the upper lim- administration) to adjust therapy so that the correction stays
its, water should be administered, with or without desmopressin, to within the desired upper limits. It cannot be emphasized enough
bring the correction magnitude back to the upper limit (Figure 8) that it is necessary to correct the serum [Na1] acutely only to a
(5). Recent studies have verified that ODS is uncommon even in safe range rather than to normal concentrations. As a practical
patients corrected for .8 mmol/L per 24 hours (0.5% of patients point, after an acute correction has reached 8 mmol/L, the need
with a serum [Na1] #120 mmol/L), but this complication occurred for continued acute therapy should be carefully assessed, because
more frequently in patients with hypokalemia and chronic alcohol- ongoing correction may result in an overly rapid correction by
ism (e.g., beer potomania) (31). the time the next serum [Na1] is available. In some situations,
It follows from these recommendations that serum [Na1] patients may spontaneously correct their hyponatremia via water
must be carefully monitored at frequent intervals during the diuresis (aquaresis). If the hyponatremia is acute (e.g.,
Nephrology Self-Assessment Program - Vol 20, No 2, January 2022 85

20

4 Acute hyponatremia (<48 hours) with severe symp- 18


Re-lowering Re-lowering Re-lowering
16 unnecessary optional recommended
toms should be treated with 100 mL of 3% NaCl

Change in serum [Na+]


infused intravenously over 10 min 3 3 as needed. 14

in first 24 h, mmol/L
12
Patients with mild to moderate symptoms may be
10
treated with 3% NaCl infused at 0.5 to 2 mL/kg per
8
hour with the goal of increase of 6 to 8 mmol/L. The
6 Goal
rate of correction of chronic hyponatremia (48 Goal
4
hours) should be targeted to correct 6 to 8 mmol/L, 2
and the correction rate should be no greater than 10 0
to 12 mmol/L for the first 24 hours. Acute water
intoxication
Low to moderate
risk of ODS
High
risk of ODS
4 The rate should be lower (4 to 6 mmol/L) for indi-
viduals with risk factors for ODS such as alcohol-
ism, liver disease, malnutrition, hypokalemia, and Figure 8. Goals and limits for correction of hyponatremia. The rate
starting serum [Na1] <105 mmol/L. of correction need not be restricted in patients with true acute hypo-
natremia, nor is re-lowering of excessive corrections indicated. Thera-
peutic re-lowering of serum [Na1] is suggested if the correction
exceeds therapeutic limits in patients with low to moderate risk of
ODS, and is recommended in patients with high risk of ODS. Green
lines indicate the treatment goals for the correction of hyponatremia
psychogenic polydipsia with water intoxication), such patients do
and Red lines and dashed lines indicate the recommended upper limits
not appear to be at risk for ODS. However, if hyponatremia is
to avoid the risk of ODS in patients presenting with serum [Na1]
chronic (e.g., hypocortisolism, natriuretic therapy), intervention
,120 mmol/L who exceed the recommended upper limits of
should be considered to limit the magnitude of correction of
correction in the first 24 hours. Abbreviations: h, hour; mmol/L,
serum [Na1], such as the infusion of hypotonic fluids to match
millimoles per liter; [Na1], sodium concentration; ODS, osmotic
urine output and/or the administration of desmopressin 1 to 2
demyelination syndrome. Reprinted with permission from reference
mg intravenously every 8 hours, using the same therapeutic end-
5 (Verbalis JG, Goldsmith SR, Greenberg A, Korzelius C, Schrier
points as for active corrections.
RW, Sterns RH, et al.: Diagnosis, evaluation, and treatment of hypo-
natremia: Expert panel recommendations. Am J Med 126[Suppl 1]:
Osmotic Demyelination Syndrome
S1–S42, 2013).
Central pontine myelinolysis was first reported in 1959 by Adams et al.
(61). Originally thought to be produced by malnutrition and alcohol-
ism, central pontine myelinolysis was later found to be also produced of thiazide diuretics (5). As can be seen from the Edelman equation, serum
by rapid increases in Posm and was called the ODS (62). Clinical find- [Na1] is affected by total body K1 in addition to Na1. Therefore, it
ings of ODS can appear 1 to 7 days after treatment of chronic hypo- should be remembered that the rapid replacement of K1 in patients with
natremia. Neurologic findings typically show a biphasic pattern: hypokalemia may also result in overly rapid correction of the serum
[Na1] (65).
1. When serum [Na1] rises, the symptoms caused by hyponatre-
mia improve.
2. Following that, new and more severe symptoms of ODS Overly Rapid Correction of Serum [Na1]: Risks
appear. and Prevention
Paradoxically, the patient's condition worsens despite the There is a risk that serum [Na1] will be overly rapidly corrected
improvement of electrolyte abnormality. Thus, ODS is a clinical when the cause of increased AVP secretion improves. Inappropriate
syndrome consisting of biphasic findings after treatment for hypona- AVP secretion results in insufficient excretion of free water into the
tremia (63). Symptoms of the second phase of ODS include dysar- hypertonic urine, and serum [Na1] remains low. However, if AVP
thria, dysphagia, pseudobulbar palsy (pyramidal tract disorder), secretion is decreased as the underlying condition improves, a large
initial flaccid limb palsy, and later transition to spastic limb palsy amount of free water is excreted in the urine, and serum [Na1] may
(corticospinal tract disorder). However, if the site of injury extends rise rapidly. That is, overly rapid correction can result from an unex-
to the pontine tegmentum, it can cause pupillary abnormalities and pected excretion of dilute urine (decrease in urine tonicity) with
ocular motility disorders. In more severe cases, the patient becomes resultant increased urine volume. When AVP secretion is decreased
completely paralyzed and unable to communicate, resulting in a to a level that allows excretion of maximally dilute urine (Uosm
so-called locked in syndrome. The hypothesized pathophysiology is ,100 mOsm/kg H2O), serum [Na1] can rises at a rate exceeding
shown in Figure 9 (64). 2 mmol/L per hour (63).
High-risk patients with ODS include serum [Na1] of #105 mmol/ Consequently, when urine tonicity decreases and urine volume
L, hypokalemia, alcoholism, malnutrition, liver disease, older age, and use increases, it is necessary to monitor urine volume and blood/urine
86 Nephrology Self-Assessment Program - Vol 20, No 2, January 2022

Hyponatremic brain
Decrease in organic osmolyte content
Decrease in protein-folding abilities
Overly rapid correction of hyponatremia

Brain dehydration (Sodium “overshoot”)


Increase in Na+ content and ionic strength

Increase in brain protein aggregate


Increase in astrocyte unfolded protein response
Excessive activation of ER stress

Combined gliosis and astrocyte death Oligodendrocyte death


(apoptosis, necroptosis) Microglial activation
Disruption of astrocyte – oligodendroglial syncitium Brain inflammation and BBB damage

BBB damage
Demyelination

Figure 9. Potential pathophysiology of osmotic demyelination after overly rapid correction of chronic hyponatremia. The schema illustrates the
potential pathophysiology of osmotic demyelination based on current data. Overly rapid correction of chronic hyponatremia induces protein
aggregation in astrocytes, along with unfolded protein response (conformational disorder) and an exaggerated endoplasmic reticulum stress that
leads to astrocyte death. These responses occur in regions susceptible to demyelination before any histologic evidence of myelin damage. Astro-
cytes not only provide trophic support to oligodendrocytes but also are required for maintenance of normal myelin; therefore, astrocyte death is
linked to myelin disruption. Abbreviations: BBB, blood–brain barrier; ER, endoplasmic reticulum. Licensed under a Creative Commons License
(CC BY NC ND). https://creativecommons.org/licenses/by-nc-nd/4.0/ Reprinted with permission from reference 64 (Gankam Kengne F, Dec-
aux G: Hyponatremia and the brain. Kidney Int Rep 3: 24–35, 2017).

analysis more frequently to anticipate subsequent increases in serum low risk of inadvertent overly rapid correction of serum [Na1]; des-
[Na1]. Intravenous desmopressin 2 mg should be considered to stop mopressin can be discontinued once the serum [Na1] has been
free water excretion if the daily upper correction limit for serum [Na1] increased to a safe concentration (generally to 128 mmol/L (66)).
is likely to be exceeded. An example is a patient with severe hyponatre- Regarding the need for re-lowering according to risk factors for the
mia due to volume depletion who may be at risk for overly rapid cor- development of ODS, maximum correction limits are shown in the
rection once volume is restored. In such patients, an acute increase in 2013 United States expert panel recommendations (Figure 8) (5).
output of diluted urine may signal the risk of overly rapid correction. In the clinical practice of treatment of symptomatic hyponatre-
Another example is a patient regularly taking drugs that potentially pro- mia, it is most important to first understand the risk factors for ODS,
mote AVP secretion from the posterior pituitary and/or enhance AVP then to anticipate the possibility of a rapid decrease of AVP secretion
effect in the renal collecting ducts. In such patients, discontinuation of in the treatment course, and finally to achieve increases in serum
those drugs can lead to an increase of free water excretion from the kid- [Na1] to the target range by frequently checking the serum [Na1]
neys, resulting in overly rapid correction of hyponatremia. and fine-tuning the treatment parameters as necessary.
In the event of overly rapid correction, 5% glucose solution
(D5W) should be administered and serum [Na1] should be Conventional, Specific, and Potential Future Treatment
re-lowered quickly to the original target upper limits, but it may be
Options for Hyponatremia
difficult to decrease the increased serum [Na1] if only D5W is
administered. In such a case, desmopressin administration should be To date, there is surprisingly little scientific evidence to support the
considered as per the United States expert opinion recommendations usual clinical treatments for hyponatremia in daily clinical practice.
(5). The protocol of pre-emptive administration of desmopressin From this point of view, we describe the following therapies, includ-
every 6 to 8 hours in combination with a slow infusion of 3% saline ing a few recent randomized controlled trials.
titrated to achieve a 6-mmol/L per day increase in serum [Na1] advo-
cated by Sterns et al. (66) is useful in the treatment of patients with Fluid Restriction. In general, fluid restriction has been the first-line
potentially reversible causes of hyponatremia along with an unpredict- treatment for hyponatremia with mild to moderate reduced serum
able aquaresis. This strategy intentionally creates a state of iatrogenic [Na1], regardless of the underlying cause. However, fluid restriction
SIAD and permits a controlled increase in the serum [Na1] with a commonly has a delayed onset of effect, hinders treatment of
Nephrology Self-Assessment Program - Vol 20, No 2, January 2022 87

underlying disease(s) by other infusion therapies, and can impair the been the placebo-controlled RCT of the vasopressin receptor
patient’s quality of life because of increased thirst (6). Although fluid antagonist tolvaptan. In a study in which tolvaptan was adminis-
restriction is still identified in published guidelines as the first-line tered to patients with hyponatremia for 30 days (SALT-1, SALT-2
therapy for mild to moderate hyponatremia secondary to SIAD (6), study), serum [Na1] increased from 128.5 mmol/L by approxi-
the treatment was ineffective in more than half of the cases in the mately 7 mmol/L after 30 days (73). Furthermore, the efficacy and
United States and European Hyponatremia Registry (67). safety of tolvaptan were confirmed in the SALTWATER open-label
In 2020, Garrahy et al. (68) investigated the efficacy of fluid extension study, in which tolvaptan was administered safely for an
restriction compared with that of no specific treatment in a prospective average of approximately 700 days (74).
randomized controlled trial (RCT). Forty-six patients with chronic
moderate hyponatremia due to SIAD were divided into two groups: Urea. Urea has been identified as an alternative oral treatment for
(1) 1.0 L/d fluid restriction or (2) no specific treatment; and plasma SIAD and other diseases that cause hyponatremia. The main mecha-
[Na1] at days 4 and 30 were compared. On day 4, plasma [Na1] nism of action of urea is to trigger solute-induced osmotic diuresis (75).
increased by 3 mmol/L (IQR 2–4) after 3 days of fluid restriction, Reports from retrospective uncontrolled studies suggest that
compared with 1 mmol/L (IQR 0–3) no specific treatment urea is effective in treating SIAD in patients with hyponatremia due
(P50.005). There was minimal additional rise in plasma [Na1] by to subarachnoid hemorrhage and in patients undergoing critical care
day 30; at day 30, median plasma [Na1] increased from baseline by 4 (76). In addition, recent evidence from a short-term study of a small
mmol/L (IQR 2–6) with fluid restriction, compared with 1 mmol/L cohort of patients with SIAD suggests that urea has an efficacy
(IQR 0–1) with no specific treatment (P50.04). After 3 days, 17% of equivalent to that of vaptans in correcting hyponatremia due to
patients with fluid restriction had a rise in plasma [Na1] of chronic SIAD (70). A retrospective study compared 12 patients with
$5 mmol/L, compared with 4% with no specific treatment (RR, 4.0; SIAD-related hyponatremia (plasma [Na1] ,130 mmol/L) who
95% CI, 0.66 to 25.69; P50.35). After 3 days, 61% of fluid restric- received urea to 12 matched patients who did not receive urea
tion corrected plasma [Na1] to $130 mmol/L, compared with 39% (6 received NaCl tablets; 3 received potassium chloride (KCl);
of no specific treatment (RR, 1.56; 95% CI, 0.87 to 2.94; P50.24). 2 received loop diuretics; 1 received a vasopressin antagonist, and
Although fluid restriction was “statistically” more effective than no spe- 1 received hypertonic saline). Urea-treated patients had a greater
cific treatment in the plasma [Na1] rise at some time points, there was increase in plasma [Na1] (2.5 vs 20.5 mmol/L) at 24 hours but no
not a great difference with respect to absolute increases in serum change over full course of treatment or length of hospital stay (77).
[Na1], supporting previous retrospective observational studies that These reports suggest that urea may be an acceptable alternative
fluid restriction might not be adequate for the treatment of chronic therapy for the treatment of chronic hyponatremia, but RCTs will
hyponatremia in the majority of cases of SIAD (68). be required to show adequate scientific justification for the efficacy
and safety of this treatment for hyponatremia due to SIAD.
Furosemide with NaCl in Combination with Fluid Restriction.
In the EU hyponatremia guideline, the administration of loop diu- Sodium-Glucose Cotransporter-2 Inhibitors in Combination
retics with oral NaCl is recommended as a second-line therapy for with Fluid Restriction. Sodium-glucose cotransporter-2 (SGLT2)
chronic SIAD (6). The use of loop diuretics with high NaCl intake inhibitors are the newest class of oral antihyperglycemic agents that
represents an extension of treatment of acute/symptomatic hyponatre- have been approved for the treatment of diabetes mellitus. SGLT2
mia using 3% NaCl administration (69) to long-term management of inhibitors promote osmotic diuresis by increasing urinary glucose
chronic hyponatremia. Successful cases have been reported (70). excretion, leading to increased free water excretion.
However, the effectiveness of this approach in chronic hypona- Refardt et al. (78) investigated the efficacy of an SGLT-2 inhibi-
tremia is limited by a scarcity of evidence to prove the efficacy of tor in the treatment of hyponatremia. Eighty-eight inpatients with
this approach for long-term treatment. Krisanapan et al. (71) SIAD-induced moderate/severe hyponatremia (,130 mmol/L), in
recently reported an open-label RCT (the EFFUSE-FLUID trial) on addition to fluid restriction (,1000 mL/day), were assigned to either
the efficacy of three conventional treatments for chronic moderate orally administered empagliflozin or placebo daily for 4 days. Patients
and severe hyponatremia. Ninety-one patients with serum [Na1] treated with empagliflozin showed a significantly higher delta median
#130 mmol/L due to SIAD were divided into three groups: (1) plasma [Na1] than did patients receiving placebo (10 versus 7
fluid restriction, (2) fluid restriction 1 loop diuretic, and (3) fluid mmol/L; P50.04, respectively). Additionally, the treatment was well
restriction 1 loop diuretic 1 NaCl supplement; changes in serum tolerated, and no hypoglycemic or hypotensive events occurred in
[Na1] on days 4, 7, 14, and 28 were compared. On day 28, it was patients receiving empagliflozin. This finding suggests that SGLT-2
5.7 6 4.7, 7.8 6 7.2, and 8.6 6 7.8 mmol/L among the three inhibitors may be a potential treatment option for hyponatremia
groups, respectively, indicating that there was no significant differ- (78), but as for many other therapies, larger randomized clinical trials
ence in efficacy among the groups (71). These data therefore to verify efficacy and safety of this approach are lacking.
question the usefulness of this approach for treatment of chronic
moderate and severe hyponatremia.
Conclusion
Vasopressin Receptor Antagonists. Vasopressin V2 receptor antag- For current treatment recommendations of hyponatremia, the sever-
onists (“vaptans”) bind to AVP V2 receptors in the renal collecting ity of neurologic symptoms and the time course of the hyponatremia
ducts, thereby blocking water reabsorption and promoting are of importance in the appropriate selection of treatment strategies
electrolyte-free water excretion (aquaresis) (72). The only high- regardless of the underlying cause. The recommendations for treat-
quality evidence for the effective treatment of hyponatremia has ment are intended to be generalizable, but most of them lack
88 Nephrology Self-Assessment Program - Vol 20, No 2, January 2022

rigorous evidence-based proof, i.e., double-blinded randomized 23. Hillier TA, Abbott RD, Barrett EJ: Hyponatremia: Evaluating the correc-
placebo-controlled studies. Most importantly, therapy of hyponatre- tion factor for hyperglycemia. Am J Med 106: 399–403, 1999 PubMed
24. Aviram A, Pfau A, Czaczkes JW, Ullmann TD: Hyperosmolality with
mia must always be individualized to the patient being treated.
hyponatremia, caused by inappropriate administration of mannitol. Am
J Med 42: 648–650, 1967 PubMed
References 25. Desmond J: Serum osmolality and plasma electrolytes in patients who
1. Adrogue HJ, Madias NE: Hyponatremia. N Engl J Med 342: 1581– develop dilutional hyponatremia during transurethral resection. Can
1589, 2000 PubMed J Surg 13: 116–121, 1970 PubMed
2. Upadhyay A, Jaber BL, Madias NE: Incidence and prevalence of hypona- 26. Levin A, Klassen J, Halperin ML: Challenging consults: Application of
tremia. Am J Med 119[Suppl 1]: S30–S35, 2006 PubMed principles of physiology and biochemistry to the bedside. Osmotic diure-
3. Hoorn EJ, Zietse R: Diagnosis and treatment of hyponatremia: Compila- sis: The importance of counting the number of osmoles excreted. Clin
tion of the guidelines. J Am Soc Nephrol 28: 1340–1349, 2017 PubMed Invest Med 18: 401–405, 1995 PubMed
4. Ellison DH, Berl T: Clinical practice. The syndrome of inappropriate 27. Braun MM, Barstow CH, Pyzocha NJ: Diagnosis and management of
antidiuresis. N Engl J Med 356: 2064–2072, 2007 PubMed sodium disorders: Hyponatremia and hypernatremia. Am Fam Physician
5. Verbalis JG, Goldsmith SR, Greenberg A, Korzelius C, Schrier RW, Sterns 91: 299–307, 2015 PubMed
RH, et al.: Diagnosis, evaluation, and treatment of hyponatremia: Expert 28. Fenske W, St€ork S, Koschker AC, Blechschmidt A, Lorenz D, Wort-
panel recommendations. Am J Med 126[Suppl 1]: S1–S42, 2013 PubMed mann S, et al.: Value of fractional uric acid excretion in differential diag-
6. Spasovski G, Vanholder R, Allolio B, Annane D, Ball S, Bichet D, et al.; nosis of hyponatremic patients on diuretics. J Clin Endocrinol Metab 93:
Hyponatraemia Guideline Development Group: Clinical practice guide- 2991–2997, 2008 PubMed
line on diagnosis and treatment of hyponatraemia. Nephrol Dial Trans- 29. Schwartz WB, Bennett W, Curelop S, Bartter FC: A syndrome of renal
plant 29[Suppl 2]: i1–i39, 2014 10.1093/ndt/gfu040 PubMed sodium loss and hyponatremia probably resulting from inappropriate secre-
7. Henry DA: In the clinic: Hyponatremia. Ann Intern Med 163: ITC1– tion of antidiuretic hormone. Am J Med 23: 529–542, 1957 PubMed
ITC19, 2015 PubMed 30. Schwartz WB, Bennett W, Curelop S, Bartter FC: A syndrome of renal
8. DeVita MV, Gardenswartz MH, Konecky A, Zabetakis PM: Incidence sodium loss and hyponatremia probably resulting from inappropriate
and etiology of hyponatremia in an intensive care unit. Clin Nephrol 34: secretion of antidiuretic hormone. 1957. J Am Soc Nephrol 12: 2860–
163–166, 1990 PubMed 2870, 2001 PubMed
9. Flear CT, Gill GV, Burn J: Hyponatraemia: Mechanisms and manage- 31. George JC, Zafar W, Bucaloiu ID, Chang AR: Risk factors and out-
ment. Lancet 2: 26–31, 1981 PubMed comes of rapid correction of severe hyponatremia. Clin J Am Soc Nephrol
10. Kleinfeld M, Casimir M, Borra S: Hyponatremia as observed in a 13: 984–992, 2018 PubMed
chronic disease facility. J Am Geriatr Soc 27: 156–161, 1979 PubMed 32. Nagler EV, Haller MC, Van Biesen W, Vanholder R, Craig JC, Webster
11. Natkunam A, Shek CC, Swaminathan R: Hyponatremia in a hospital AC: Interventions for chronic non-hypovolaemic hypotonic hyponatrae-
population. J Med 22: 83–96, 1991 PubMed mia. Cochrane Database Syst Rev 6: CD010965, 2018 PubMed
12. Brunsvig PF, Os I, Frederichsen P: Hyponatremi. En retrospektiv studie 33. Feldman BJ, Rosenthal SM, Vargas GA, Fenwick RG, Huang EA, Mat-
med henblikk på forekomst, årsak og mortalitet. [Hyponatremia. A retro- suda-Abedini M, et al.: Nephrogenic syndrome of inappropriate antidiu-
spective study of occurrence, etiology and mortality] Tidsskr Nor Laegefo- resis. N Engl J Med 352: 1884–1890, 2005 PubMed
ren 110: 2367–2369, 1990 PubMed 34. Levtchenko EN, Monnens LA: Nephrogenic syndrome of inappropriate
13. Anderson RJ, Chung HM, Kluge R, Schrier RW: Hyponatremia: A pro- antidiuresis. Nephrol Dial Transplant 25: 2839–2843, 2010 PubMed
spective analysis of its epidemiology and the pathogenetic role of vaso- 35. Decaux G, Vandergheynst F, Bouko Y, Parma J, Vassart G, Vilain C:
pressin. Ann Intern Med 102: 164–168, 1985 PubMed Nephrogenic syndrome of inappropriate antidiuresis in adults: High phe-
14. Robinson AG: Disorders of antidiuretic hormone secretion. Clin Endocri- notypic variability in men and women from a large pedigree. J Am Soc
nol Metab 14: 55–88, 1985 PubMed Nephrol 18: 606–612, 2007 PubMed
15. Mack GW, Weseman CA, Langhans GW, Scherzer H, Gillen CM, Nadel 36. Byatt CM, Millard PH, Levin GE: Diuretics and electrolyte disturbances in
ER. Body fluid balance in dehydrated healthy older men: Thirst and 1000 consecutive geriatric admissions. J R Soc Med 83: 704–708, 1990
renal osmoregulation. J Appl Physiol 76: 1615–1623, 1985 PubMed PubMed
16. Farrell MJ, Zamarripa F, Shade R, Phillips PA, McKinley M, Fox PT, 37. Barber J, McKeever TM, McDowell SE, Clayton JA, Ferner RE, Gordon
et al.: Effect of aging on regional cerebral blood flow responses associated RD, et al.: A systematic review and meta-analysis of thiazide-induced
with osmotic thirst and its satiation by water drinking: A PET study. hyponatraemia: Time to reconsider electrolyte monitoring regimens after
Proc Natl Acad Sci U S A 105: 382–387, 2008 PubMed thiazide initiation? Br J Clin Pharmacol 79: 566–577, 2015 PubMed
17. Juul KV, Bichet DG, Nielsen S, Nørgaard JP: The physiological and 38. Liamis G, Milionis H, Elisaf M: A review of drug-induced hyponatremia.
pathophysiological functions of renal and extrarenal vasopressin V2 Am J Kidney Dis 52: 144–153, 2008 PubMed
receptors. Am J Physiol Renal Physiol 306: F931–F940, 2014 PubMed 39. Cantini L, Merloni F, Rinaldi S, Lenci E, Marcantognini G, Meletani T, et al.:
18. Dunn FL, Brennan TJ, Nelson AE, Robertson GL: The role of blood Electrolyte disorders in advanced non-small cell lung cancer patients
osmolality and volume in regulating vasopressin secretion in the rat. treated with immune check-point inhibitors: A systematic review and
J Clin Invest 52: 3212–3219, 1973 PubMed meta-analysis. Crit Rev Oncol Hematol 151: 102974, 2020 PubMed
19. Gheorghe G, Ilie M, Bungau S, Stoian AMP, Bacalbasa N, Diaconu CC: 40. Seethapathy H, Rusibamayila N, Chute DF, Lee M, Strohbehn I, Zubiri
Is there a relationship between COVID-19 and hyponatremia? Medicina L, Faje AT, Reynolds KL, Jhaveri KD, Sise ME. Hyponatremia and
(Kaunas) 57: 55, 2021 PubMed other electrolyte abnormalities in patients receiving immune checkpoint
20. Swart RM, Hoorn EJ, Betjes MG, Zietse R: Hyponatremia and inflam- inhibitors. Nephrol Dial Transplant Dec 29: gfaa272, 2020 PubMed
mation: The emerging role of interleukin-6 in osmoregulation. Nephron, 41. Barroso-Sousa R, Barry WT, Garrido-Castro AC, Hodi FS, Min L, Krop
Physiol 118: 45–51, 2011 PubMed IE, et al.: Incidence of endocrine dysfunction following the use of differ-
21. Edelman IS, Leibman J, O’Meara MP, Birkenfeld LW: Interrelations ent immune checkpoint inhibitor regimens: A systematic review and
between serum sodium concentration, serum osmolarity and total meta-analysis. JAMA Oncol 4: 173–182, 2018 PubMed
exchangeable sodium, total exchangeable potassium and total body water. 42. Caturegli P, Di Dalmazi G, Lombardi M, Grosso F, Larman HB, Lar-
J Clin Invest 37: 1236–1256, 1958 PubMed man T, et al.: Hypophysitis secondary to cytotoxic T-lymphocyte-associ-
22. Adrogue HJ, Madias NE: The challenge of hyponatremia. J Am Soc ated protein 4 blockade: Insights into pathogenesis from an autopsy
Nephrol 23: 1140–1148, 2012 PubMed series. Am J Pathol 186: 3225–3235, 2016 PubMed
Nephrology Self-Assessment Program - Vol 20, No 2, January 2022 89

43. Faje A, Reynolds K, Zubiri L, Lawrence D, Cohen JV, Sullivan RJ, et al.: 61. Adams RD, Victor M, Mancall EL: Central pontine myelinolysis: A hith-
Hypophysitis secondary to nivolumab and pembrolizumab is a clinical erto undescribed disease occurring in alcoholic and malnourished
entity distinct from ipilimumab-associated hypophysitis. Eur J Endocrinol patients. AMA Arch Neurol Psychiatry 81: 154–172, 1959 PubMed
181: 211–219, 2019 PubMed 62. Sterns RH, Riggs JE, Schochet SS Jr: Osmotic demyelination syndrome
44. Yatagai T, Kusaka I, Nakamura T, Nagasaka S, Honda K, Ishibashi S, following correction of hyponatremia. N Engl J Med 314: 1535–1542,
et al.: Close association of severe hyponatremia with exaggerated release 1986 PubMed
of arginine vasopressin in elderly subjects with secondary adrenal insuffi- 63. Sterns RH: Adverse consequences of overly-rapid correction of hypona-
ciency. Eur J Endocrinol 148: 221–226, 2003 PubMed tremia. Front Horm Res 52: 130–142, 2019 PubMed
45. Dineen R, Hannon MJ, Thompson CJ: Hyponatremia and Hypernatre- 64. Gankam Kengne F, Decaux G: Hyponatremia and the brain. Kidney Int
mia. In: Endocrinology: Adult and Pediatric 7th Ed., edited by J. Larry Rep 3: 24–35, 2017 PubMed
Jameson, Leslie J De Groot, David M. de Kretser, Linda C. Giudice, 65. Berl T, Rastegar A: A patient with severe hyponatremia and hypokalemia:
Ashley B. Grossman, Shlomo Melmed, John T. Potts, and Gordon C. Osmotic demyelination following potassium repletion. Am J Kidney Dis
Weir, J. Larry Jameson, Leslie J De Groot, David M. de Kretser, Linda 55: 742–748, 2010 PubMed
C. Giudice, Ashley B. Grossman, Shlomo Melmed, John T. Potts, and 66. Sood L, Sterns RH, Hix JK, Silver SM, Chen L: Hypertonic saline and
Gordon C. Weir, Philadelphia, Elsevier, 2016, pp 1953–1962 desmopressin: A simple strategy for safe correction of severe hyponatre-
46. Verbalis JG, Drutarosky MD: Adaptation to chronic hypoosmolality in mia. Am J Kidney Dis 61: 571–578, 2013 PubMed
rats. Kidney Int 34: 351–360, 1988 PubMed 67. Verbalis JG, Greenberg A, Burst V, Haymann JP, Johannsson G, Peri A,
47. Verbalis JG: Adaptation to acute and chronic hyponatremia: Implications Poch E, Chiodo JA 3rd, Dave J. Diagnosing and treating the syndrome
for symptomatology, diagnosis, and therapy. Semin Nephrol 18: 3–19, of inappropriate antidiuretic hormone secretion. Am J Med 129: e9-537,
1998 PubMed 2016 PubMed
48. Renneboog B, Musch W, Vandemergel X, Manto MU, Decaux G. Mild 68. Garrahy A, Galloway I, Hannon AM, Dineen R, O'Kelly P, Tormey WP,
chronic hyponatremia is associated with falls, unsteadiness, and attention O'Reilly MW, Williams DJ, Sherlock M, Thompson CJ. Fluid
deficits. Am J Med 119: e1–e8, 2006 PubMed restriction therapy for chronic SIAD: Results of a prospective random-
49. Verbalis JG, Barsony J, Sugimura Y, Tian Y, Adams DJ, Carter EA, et al: ized controlled trial. J Clin Endocrinol Metab 105: dgaa619, 2020
Hyponatremia-induced osteoporosis. J Bone Miner Res 25: 554–563, 2010 PubMed
PubMed 69. Hantman D, Rossier B, Zohlman R, Schrier R: Rapid correction of
50. Hoorn EJ, Rivadeneira F, van Meurs JB, Ziere G, Stricker BH, Hofman hyponatremia in the syndrome of inappropriate secretion of antidiuretic
A, et al: Mild hyponatremia as a risk factor for fractures: The Rotterdam hormone. An alternative treatment to hypertonic saline. Ann Intern Med
Study. J Bone Miner Res 26: 1822–1828, 2011 PubMed 78: 870–875, 1973 PubMed
51. Berghmans T, Paesmans M, Body JJ: A prospective study on hyponatrae- 70. Soupart A, Coffernils M, Couturier B, Gankam-Kengne F, Decaux G:
mia in medical cancer patients: Epidemiology, aetiology and differential Efficacy and tolerance of urea compared with vaptans for long-term treat-
diagnosis. Support Care Cancer 8: 192–197, 2000 PubMed ment of patients with SIADH. Clin J Am Soc Nephrol 7: 742–747, 2012
52. Terzian C, Frye EB, Piotrowski ZH: Admission hyponatremia in the elderly: PubMed
Factors influencing prognosis. J Gen Intern Med 9: 89–91, 1994 PubMed 71. Krisanapan P, Vongsanim S, Pin-On P, Ruengorn C, Noppakun K: Effi-
53. Greenberg A, Verbalis JG, Amin AN, Burst VR, Chiodo JA 3rd, Chiong cacy of furosemide, oral sodium chloride, and fluid restriction for treat-
JR, et al: Current treatment practice and outcomes. Report of the hypo- ment of syndrome of inappropriate antidiuresis (SIAD): An open-label
natremia registry. Kidney Int 88: 167–177, 2015 PubMed randomized controlled study (The EFFUSE-FLUID Trial). Am J Kidney
54. Laville M, Burst V, Peri A, Verbalis JG: Hyponatremia secondary to the Dis 76: 203–212, 2020 PubMed
syndrome of inappropriate secretion of antidiuretic hormone (SIADH): 72. Yamamura Y, Nakamura S, Itoh S, Hirano T, Onogawa T, Yamashita T,
Therapeutic decision-making in real-life cases. Clin Kidney J 6[Suppl 1]: et al: OPC-41061, a highly potent human vasopressin V2-receptor antag-
i1–i20, 2013 PubMed onist: Pharmacological profile and aquaretic effect by single and
55. Moritz ML, Ayus JC: Hospital-acquired hyponatremia–why are hypo- multiple oral dosing in rats. J Pharmacol Exp Ther 287: 860–867, 1998
tonic parenteral fluids still being used? Nat Clin Pract Nephrol 3: 374– PubMed
382, 2007 PubMed 73. Schrier RW, Gross P, Gheorghiade M, Berl T, Verbalis JG, Czerwiec FS,
56. Keller WJ, Mullaj E: Antidiuretic hormone release associated with et al.; SALT Investigators: Tolvaptan, a selective oral vasopressin
increased intracranial pressure independent of plasma osmolality. Brain V2-receptor antagonist, for hyponatremia. N Engl J Med 355: 2099–
Behav 8: e01005, 2018 PubMed 2112, 2006 PubMed
57. Battison C, Andrews PJ, Graham C, Petty T: Randomized, controlled 74. Berl T, Quittnat-Pelletier F, Verbalis JG, Schrier RW, Bichet DG,
trial on the effect of a 20% mannitol solution and a 7.5% saline/6% dex- Ouyang J, et al.; SALTWATER Investigators: Oral tolvaptan is safe and
tran solution on increased intracranial pressure after brain injury. Crit effective in chronic hyponatremia. J Am Soc Nephrol 21: 705–712, 2010
Care Med 33: 196–202, discussion 257–258, 2005 PubMed PubMed
58. Koenig MA, Bryan M, Lewin JL 3rd, Mirski MA, Geocadin RG, Stevens 75. Sterns RH, Silver SM, Hix JK: Urea for hyponatremia? Kidney Int 87:
RD: Reversal of transtentorial herniation with hypertonic saline. Neurol- 268–270, 2015 PubMed
ogy 70: 1023–1029, 2008 PubMed 76. Decaux G, Andres C, Gankam Kengne F, Soupart A: Treatment of euvo-
59. Garrahy A, Dineen R, Hannon AM, Cuesta M, Tormey W, Sherlock M, lemic hyponatremia in the intensive care unit by urea. Crit Care 14:
et al: Continuous versus bolus infusion of hypertonic saline in the treat- R184, 2010 PubMed
ment of symptomatic hyponatremia caused by SIAD. J Clin Endocrinol 77. Rondon-Berrios H, Tandukar S, Mor MK, Ray EC, Bender FH, Kley-
Metab 104: 3595–3602, 2019 PubMed man TR, et al: Urea for the treatment of hyponatremia. Clin J Am Soc
60. Baek SH, Jo YH, Ahn S, Medina-Liabres K, Oh YK, Lee JB, et al: Risk Nephrol 13: 1627–1632, 2018 PubMed
of overcorrection in rapid intermittent bolus vs slow continuous infusion 78. Refardt J, Imber C, Sailer CO, Jeanloz N, Potasso L, Kutz A, et al: A
therapies of hypertonic saline for patients with symptomatic hyponatre- randomized trial of empagliflozin to increase plasma sodium levels in
mia: The SALSA randomized clinical trial. JAMA Intern Med 181: 81– patients with the syndrome of inappropriate antidiuresis. J Am Soc Neph-
92, 2021 PubMed rol 31: 615–624, 2020 PubMed
90 Nephrology Self-Assessment Program - Vol 20, No 2, January 2022

Article
Pathophysiology, Evaluation, and Treatment
of Hypernatremia
Solomiia Savedchuk, MD, and Ruediger W. Lehrich, MD
Duke University Medical Center, Department of Medicine, Division of Nephrology, Durham, North Carolina

Activation of this network triggers a coordinated set of responses


Learning Objectives that restores fluid balance. These responses include behavioral mech-
anisms that motivate drinking by activating the cingulate and insular
1. To discuss recent advances in the physiology that governs
cortex, and physiologic mechanisms that activate neuroendocrine
osmoregulation and thirst activation leading to water and sodium retention by the kidneys.
2. To describe the incidence, risk factors, and prognostic value of
Homeostatic and Anticipatory Thirst
hypernatremia
Recent studies make a distinction between two types of thirst:
3. To review recent advances in the diagnostic approaches to and homeostatic thirst and anticipatory thirst (5). Homeostatic thirst is
treatment of hypernatremia and diabetes insipidus stimulated in response to an established water deficit. Homeostatic
thirst is induced by the following three main mechanisms.
First, an increase in [Na1] is sensed by sodium-specific recep-
Physiology of Osmoregulation, Thirst, and Urinary tors (Nax channels), which are expressed in the glial cells of the SFO
Concentration and the OVLT, initiating neural signals that are capable of activating
Hypernatremia is defined as a serum sodium concentration ([Na1]) thirst-encoding neurons. Furthermore, the activation of Nax chan-
.145 mmol/L. In clinical practice this occurs from a loss of nels leads to an increase in the firing rate of inhibitory (g-aminobu-
electrolyte-free water, a loss of hypotonic fluid, or a gain of hyper- tyric acid)-ergic neurons in the SFO, leading to the inhibition of salt
tonic fluid (1). Overall control of water balance is achieved by the intake behavior. In addition to this central mechanism, afferent fibers
regulation of several behavioral and physiologic responses orches- in the hepatic and mesenteric branches of the vagus nerve detect
trated to maintain the volume and osmolarity of extracellular fluid changes in [Na1], relaying signals to the nucleus tractus solitarius,
near an ideal set point. If such behavioral and physiologic responses which then projects into thirst-promoting regions. Additionally,
are impaired, hypernatremia develops. The two main physiologic peripheral signals may be directly or indirectly relayed to the thala-
principles that govern water and sodium homeostasis are thirst and mus, from which thalamic neurons transmit sensory information to
urinary concentration. anterior cingulate and insular cortex to promote thirst (5).
The second mechanism to induce homeostatic thirst involves a
change in blood osmolality, which is detected by SFO and OVLT
Regulation of Thirst
directly via osmoreceptors. Using stretch-sensitive properties,
Thirst is the major behavioral regulator of water intake. Similar to changes in cell volume after intracellular dehydration in the setting
any other behavioral response, it involves complex neuronal interac- of increased plasma osmolality are detected. Stimulation leads to a
tion. Understanding of these interactions originated in the 1950s nonselective cation current, which induces cell depolarization and
with Bengt Anderson, who discovered that the infusion of hyper- subsequent activation of MnPO (6). As opposed to all other
tonic saline solution into the anterior hypothalamus of goats stimu- neurons, SFO and OVLT neurons do not adapt to hypo- or hyper-
lated drinking and water retention (2). Subsequent studies identified osmolality by losing or generating osmoles, and their change in vol-
the anterior wall of the third ventricle as crucial in its role to pro- ume is maintained as long as the osmotic stimulus persists (4).
voke thirst. The foremost brain structure responsible for sensing and Third, changes in effective blood volume are indirectly moni-
regulating internal water balance within this region is the lamina ter- tored by pressure receptors. Low-pressure cardiopulmonary barore-
minalis, which contains three main nuclei: the subfornical organ ceptors in the atria, pulmonary arteries, and vena cava detect blood
(SFO), the organum vasculosum of the lamina terminalis (OVLT), volume. High-pressure baroreceptors in the wall of the aortic arch
and an integratory structure, the median preoptic nucleus (MnPO) and carotid sinuses detect arterial pressure. In the setting of low
(3). The SFO and OVLT both lack the normal blood-brain barrier; blood volume and/or blood pressure, these baroreceptors send inhib-
therefore, the populations of neurons within these nuclei have itory inputs from the periphery via afferent fibers of cranial nerves
unconstrained access to the circulation. By contrast, MnPO does IX and X to the nucleus tractus solitarius, activating the sympathetic
not sense input from the blood and only receives feedback from the nervous system and activating thirst mechanisms. Reduced kidney
SFO and the OVLT. Recent studies have shown that osmoreception blood flow stimulates renin release from the juxtaglomerular appara-
is hierarchically organized, with MnPO neurons as the major output tus, which converts angiotensinogen to angiotensin (Ang) I, which
neurons with integrated inputs from SFO and OVLT (3,4). is subsequently converted to Ang II by the angiotensin-converting

nephsap.org Copyright # 2022 by the American Society of Nephrology


Nephrology Self-Assessment Program - Vol 20, No 2, January 2022 91

enzyme. Ang II causes afferent arteriole and systemic vasoconstric- Hospital-Acquired Hypernatremia
tion and is a potent stimulation of SFO via Ang II receptor type 1 The development of hypernatremia in the hospital setting is more
(5,7). By contrast, hypervolemia inhibits thirst through the humoral common than community-acquired hypernatremia. A large retro-
mechanism of atrial natriuretic peptide, which is released when spe- spective study of 46,634 patients from Italy focusing on hospital-
cialized cardiac myocytes are stretched. Atrial natriuretic peptide acquired dysnatremias reported the incidence of hypernatremia to
stimulates natriuresis in the kidneys and is an important inhibitor of be 4.3%. This study specifically excluded intensive care unit (ICU)
thirst (5). patients and patients with dysnatremias on admission (9).
In contrast to thirst in response to an established water deficit, A prospective cohort from Australia enrolled 180 hospitalized
anticipatory thirst occurs with an impending water deficit. Thirst in general medicine patients who experienced moderate hypernatremia
response to food intake, or prandial thirst, is stimulated during eat- defined as a serum [Na1] of 150 to 159 mmol/L. Approximately
ing to ensure that adequate water is available for food ingestion and 10% of patients experienced progression to severe hypernatremia
digestion and also to counteract the increase in plasma osmolality (serum [Na1] .160 mmol/L). Risk factors for progression were
due to absorption of osmolytes in food. Conversely, dehydration CKD stage, rising creatinine, and administration of intravenous
potently suppresses food intake when water is unavailable (5). In the normal saline solution. The authors stress that patients with CKD
context of anticipatory thirst, presomnial thirst helps prevent dehy- and AKI may have an increased risk of severe hospital-acquired
dration during prolonged sleep. Activation of neurons in the supra- hypernatremia and that hospital-acquired hypernatremia is often
chiasmatic nucleus, which is the center of circadian regulation, in iatrogenic (10).
the hours immediately preceding sleep, projects to the OVLT,
which in turn activates thirst-regulating centers. This increase in pre- Intensive Care Unit Hypernatremia
somnial drinking prevents the development of small rises in plasma Hypernatremia is particularly common in ICU patients. As in other
osmolality that could otherwise occur during sleep (5). settings, impaired thirst or inability to drink water is the driving fac-
Cessation of water intake once thirst results in drinking water tor for hypernatremia. In the ICU setting, this is a result of sedation,
is just as important to prevent overhydration. In fact, the unpleas- intubation, mechanical ventilation, or altered mental status. In
ant emotion associated with thirst rapidly declines in part because patients able to drink, restriction of oral water is frequently ordered
of the activation of specific neurons in the MnPO, thus avoiding for patients with a high risk of aspiration or gastrointestinal issues,
overhydration (8). Furthermore, recent studies have discovered such as gastrointestinal bleeding, obstruction, or ileus. Excessive
anticipatory thirst satiety in thirsty mice once they begin drinking, renal or other insensible fluid losses (skin losses with sweating in the
and demonstrated SFO neuron inhibition with the onset of drink- setting of fever, respiratory losses with mechanical ventilation) with-
ing before any changes in the composition of blood occurs. This out adequate free water replacement are also common contributors
rapid feedback is triggered by the detection of water in the oral cav- to the increased risk of hypernatremia. Last, medication-induced
ity and enables the neurons of the SFO to closely monitor water diuresis, osmotic catharsis with lactulose, iatrogenic hypernatremia
ingestion. Therefore, SFO neurons control water intake in real time from hypertonic saline solution, mannitol, bicarbonate, or large
by comparing the level of physiologic need through the level of amounts of intravenous fluids are common in ICU patients. Unsur-
blood osmolality and the amount of water ingested via checking prisingly, the presence of hypernatremia on admission or develop-
signals from the oropharynx. This model explains how drinking ment of hypernatremia in the ICU setting is associated with higher
can rapidly quench thirst (2,5). mortality and increased length of stay (11,12).
Early intravenous fluid resuscitation is a necessary tool to
Urinary Concentration improve hemodynamic stability in critically ill patients admitted to
the ICU. A retrospective study of 844 ICU patients receiving intra-
As thirst leads to ingestion of hypotonic or electrolyte-free water,
venous fluids compared the incidence of hypernatremia in patients
urinary concentration allows for the retention of free water. Arginine
receiving 5% dextrose in water versus saline diluents, with the results
vasopressin (AVP) mediates water retention in the kidney by render-
showing significantly higher rates of hypernatremia in the saline
ing the collecting duct permeable to water. Water retention is then
diluents group (27.4% versus 14.6%, P,0.001) (13).
driven by an osmotic gradient that favors water to flow from the
Once hemodynamic stability has been achieved after shock, a
lumen to the interstitial space (1). Urinary concentration and AVP
positive fluid balance secondary to excess fluid accumulation is often
action are described elsewhere in this issue of NephSAP.
observed in critically ill patients in the ICU. Pharmacologic diuresis
is frequently used in this setting. In a recent single-center, non-
randomized study, volume overloaded patients were subjected to a
Incidence of and Risk Factors for Hypernatremia protocolized diuresis regimen and compared with a historical cohort.
Clinically relevant hypernatremia is less common than hyponatre- A multidisciplinary team assessed the fluid balance every 8 hours.
mia. It is a clinical problem affecting various populations but is The protocolized diuresis regimen was associated with a significant
most often encountered in hospitalized patients and most frequently decrease in net cumulative fluid balance at 72 hours after shock reso-
affects critically ill hospitalized patients. But it also occurs in patients lution and reduced in-hospital mortality (0.16% versus 0.05%,
who present to EDs as community-acquired hypernatremia. Of P50.008). However, it was also associated with a significant
those patients, the elderly are at a particular risk. Salt intoxication, increase in hypernatremia and hypokalemia (14).
as well as hypernatremia which develops during high-intensity activ- Hypernatremia might also be associated with glucocorticoid
ity can be categorized as community-acquired hypernatremia. therapy in critically ill patients with refractory septic shock. A nested
92 Nephrology Self-Assessment Program - Vol 20, No 2, January 2022

case-control study in Japan investigated the relationship between Salt Intoxication


ICU-acquired hypernatremia and glucocorticoid therapy in critically An unusual cause of community-acquired hypernatremia is acciden-
ill patients. ICU-acquired hypernatremia was associated with high- tal or malicious salt poisoning, and several cases are being described
dose glucocorticoid therapy in a duration- and response-specific each year (22–25). Serum sodium concentrations as high as 238
manner (15). mmol/L have been described in this setting (24), with causes ranging
Hospitalized children usually receive isotonic solutions in accor- from soy sauce or bay salt ingestion to eating disorders. It has to be
dance with the guidelines of the American Academy of Pediatrics to recognized that salt intoxication in contrast to other forms of hyper-
prevent hyponatremia (16). In critically ill children this approach is natremia is not due to the loss of water but rather to single or multi-
less established. To study electrolyte disturbances including hyperna- ple massive salt ingestions. The change in serum tonicity in chronic
tremia in critically ill children, a randomized trial was conducted to hypernatremia allows for the formation of idiogenic osmoles coun-
compare the effect of a “plasma-like” isotonic solution and moder- terbalancing the osmotic gradient, which becomes particularly
ately hypotonic solutions. The “plasma-like” isotonic fluid contained impactful in brain cells. The lack of idiogenic osmoles in acute salt
Na 140 mmol/L and K 5 mmol/L in 5% dextrose; the control fluid ingestion has implications regarding therapy. Rapid lowering of the
contained Na 80 mmol/L and K 20 mmol/L in 5% dextrose. serum sodium concentration with substantial free water administra-
Hypernatremia developed in only a small number of patients tion and extracorporeal removal is paramount (22).
(1.3%) in the group that received isotonic fluid (16).
Athletic Activity as a Risk Factor for Hypernatremia
Community-Acquired Hypernatremia
Altered hydration status is a potentially serious complication that
A large retrospective study of 10,558 patients presenting to the ED can develop during high-intensity exercise activities. Exercise-
in a single center in Japan between 2015 and 2017 demonstrated a associated hyponatremia due to overhydration and inappropriate
prevalence of community-acquired hypernatremia of 1.5% in elderly vasopressin secretion was first reported in the 1980s, and it is now a
patients (age .65) without CKD and #3.5% in elderly patients well-recognized disorder. Hypernatremia is a more intuitive sequela
with CKD (P,0.001). This was about twice as high as in the non- of high-intensity exercise activities and results from hypotonic fluid
elderly group (age ,65). The incidence of community-acquired losses. A recent observational study of 266 ultra-marathon runners
hypernatremia was 0.8% in nonelderly patients without CKD and carefully delineated the incidence of exercise-associated hypo- and
2.0% when CKD was present. The authors conclude that hyperna- hypernatremia. The runners were recruited in 2017 and 2018 before
tremia in the elderly population is not uncommon and is more fre- running 50-mile races in the desert climates of Chile, Namibia, and
quent in elderly patients with underlying CKD (17). The concept Mongolia. Weight change and serum sodium concentration upon
of hypernatremia as a function of aging is supported by recent finishing the race were reviewed. Eleven runners (6.3%) experienced
experimental studies showing that elderly, but otherwise healthy, exercise-associated hyponatremia EAH. and 30 runners (17.3%)
men experience reduced osmotic thirst compared with young men experienced hypernatremia. Predictably, hypernatremic runners were
(controls) after hypertonic saline solution infusion. Additionally, more often volume depleted with a loss of weight during the mara-
when perceived thirst was assessed by a visual analogue scale, it was thon. Also, unsurprisingly, hypernatremia occurred more often in
significantly less expressed in older patients (18). races during hot weather than in races in colder temperatures (odds
Dysnatremias are also associated with patients who present to ratio [OR], 8.8; 95% CI, 2.9 to 39.5) (26).
the ED with AKI. In a cross-sectional analysis of all patients who Another study of 81 randomly selected runners of a 2017 Cali-
presented to a single ED, the Acute Kidney Injury Network (AKIN) fornia half marathon showed no cases of hyponatremia at the end of
classification was used to establish the presence and stage of acute the marathon, but instead discovered six cases of asymptomatic
renal failure. Of a total of 65,489 ED presentations, 1692 patients hypernatremia (7.4%). Whereas hyponatremia has been associated
had either AKIN stage 1, 2, or 3. Even though dysnatremias were with high fluid intake and with races performed in cool weather,
quite common, with 24.5% of patients being affected, the majority such as the Boston Marathon in spring, hypernatremia tends to hap-
of patients had hyponatremia (23.6%), and only a small fraction pen in marathons that take place in warmer weather (27).
were found to be hypernatremic (1.4%) (19).
Seasonal variations in the incidence of hypernatremia are well
described, with hot weather being considered a classic risk factor for Prognosis of Hypernatremia
hypernatremia. A recent retrospective study from Ireland conducted Serum sodium and mortality are often associated in a U-shaped rela-
during a 10-day heat wave in 2017 and 2018 showed the incidence tionship, with mortality rising with both hypo- and hypernatremia.
of hypernatremia to be 2.5 times higher in samples drawn during This was demonstrated again in an analysis of 1-year out-of-hospital
the heat wave than in the control period (3.6% during heat wave mortality in a cohort of 55,901 hospitalized patients. The nadir of
versus 1.4% in the control period) (20). By contrast, a retrospective the U-shaped relationship was in the normal serum sodium range,
study conducted in Japanese ED patients showed a higher preva- at 136 to 142 mmol/L (28). A similar relationship was also observed
lence of hypernatremia in the elderly group during the winter in in-hospital mortality in patients with AKI (29).
months (2.9% in winter versus 2.4% in summer, P50.004). This Although the focus is often on hyponatremia, recent studies
correlation was not observed in the nonelderly control group. This suggest that hypernatremia might be a better predictor for
finding was attributed to potentially less significant care being given in-hospital mortality compared with the predictive value of hypona-
to the patient's hydration status in winter months than during sum- tremia. A recent single-center study involving 6400 hospitalized
mer months (21). patients investigated the association between admission serum
Nephrology Self-Assessment Program - Vol 20, No 2, January 2022 93

sodium and in-hospital mortality. Compared with normonatremia, CI, 1.38 to 2.00), and fluctuating [Na1] (OR, 4.61; 95% CI, 3.61
severe hypernatremia ([Na1] .155 mmol/L) had the odds ratio of to 5.88). This again underscored higher mortality rates in patients
11.1 for in-hospital mortality (95% CI, 3.83 to 32.21; P,0.001), with hypernatremia than in patients with hyponatremia. Of note,
whereas the odds ratio for in-hospital mortality in patients with patients with fluctuating [Na1] trajectory had the greatest 1-year
severe hyponatremia ([Na1] ,125 mmol/L) was 3.9 (95% CI, 1.73 mortality (HR, 2.1; 95% CI, 1.89 to 2.33) compared with patients
to 8.93; P50.001) (30). In another recent study of 879 ED patients with borderline high serum sodium (HR, 1.18; 95% CI, 1.11 to
presenting with sepsis, the prognostic value of [Na1] on the mortal- 1.26) (Figure 1). The authors note that not a single serum sodium
ity rates was examined. Unsurprisingly, patients with dysnatremia or a weighted sodium average, but rather the in-hospital serum
had significantly higher mortality rates. However, when dysnatremic [Na1] trajectory, is the most useful parameter to predict in-hospital
patients were further subdivided into hypo- and hypernatremic, as well as 1-year mortality (32).
hypernatremia was associated with a significantly higher mortality Further characterizing the incidence and outcomes of hyperna-
rates at 7 and 30 days than was hyponatremia. The hazard ratio tremia, a recent analysis of 19,072 unselected hospitalized patients
(HR) for death in the hypernatremia and hyponatremia groups was studied the crude relationship between hypernatremia, defined as a
3.5 (95% CI, 1.58 to 7.82; P50.002) and 2.14 (95% CI, 1.17 to serum [Na1] .142 mmol/L, and mortality. In this unusually low
3.92, P50.014), respectively (31). sodium cutoff, community-acquired hypernatremia was diagnosed
Although single serum sodium levels have been shown to be in as many as 21% of hospitalized patients, whereas hospital-
associated with patient outcomes, directional changes in serum acquired hypernatremia developed in 25.9% of hospitalized patients.
[Na1] offer additional information regarding the outcomes in hospi- Both were associated with increased in-hospital mortality (OR,
talized patients. A carefully conducted recent single-center cohort 1.67; 95% CI, 138 to 2.01) and (OR, 3.17; 95% CI, 2.45 to 4.09),
study investigated in-hospital [Na1] trajectories and their association respectively. Also, both were associated with a greater likelihood of
with in-hospital and 1-year mortality outcomes. A total of 43,000 discharge to a long- or a short-term care facility, as well as increased
patients were categorized into five groups: stable normonatremia, length of hospital stay (33).
corrected hyponatremia, uncorrected hyponatremia, borderline high Unsurprisingly, the poor prognostic value of hypernatremia
serum sodium, and fluctuating serum sodium. After adjustment for holds true in several more specific care settings. In a retrospective
admission serum sodium, significantly higher odds of in-hospital study, the outcomes in elderly patients with underlying CKD (age
mortality were noted in patients with uncorrected hyponatremia .75 years, GFR ,60 ml/min per 1.73 m2) with hypo- and hyper-
(OR, 1.33; 95% CI, 1.06 to 1.67), corrected hyponatremia (OR, natremia were compared with those in normonatremic elderly
1.5; 95% CI, 1.02 to 2.2), borderline high [Na1] (OR, 1.66; 95% patients with CKD. In this study, the results suggested that only

Figure 1. In-hospital serum sodium trajectories and mortality. A) Serum sodium in-hospital trajectories were generated by the use of group-
based trajectory modeling. A total of 43,000 patients were categorized to have corrected hyponatremia, stable normonatremia, uncorrected hypo-
natremia, borderline high serum sodium, and fluctuating serum sodium. B) One-year mortality was worst in patients with fluctuating serum
sodium and best in patients with stable normonatremia. Of patients with dysnatremia, those with high borderline serum sodium had the most
favorable 1-year mortality (32). Used with permission from reference 32 (Chewcharat A, Thongprayoon C, Cheungpasitporn W, Mao MA,
Thirunavukkarasu S, Kashani KB: Trajectories of serum sodium on in-hospital and 1-year survival among hospitalized patients. Clin J Am Soc
Nephrol 15: 600–607, 2020).
94 Nephrology Self-Assessment Program - Vol 20, No 2, January 2022

hypernatremia was associated with increased risk of death, differing combination, TWA of chloride, but not sodium, remained indepen-
from previously shown data indicating a U-shaped curve relationship dently associated with in-hospital mortality (41).
between dysnatremias and mortality (34). In a retrospective cohort Whether hypertonic saline infusions are an effective and safe
in Japan of terminally ill cancer patients, hypernatremia emerged, strategy to reduce ICP and improve outcomes still remains uncer-
alongside leukocytosis and hypoalbuminemia, as a significant predic- tain; one effort to shed light on this question is currently being
tor for poor prognosis (35). Expectedly, admission and discharge investigated. A multicenter, randomized, controlled, open-label,
[Na1] were also a poor prognostic marker for patients who were hos- two-arm study is being conducted, which compares standard care
pitalized with congestive heart failure (36). Inpatient palliative care with continuous hyperosmolar therapy plus standard care in patients
consultation in patients with dementia can be an important segue to who experienced traumatic brain injury 24 hours prior. The primary
engage in goals of care discussions with families. In a recent study, outcome is the score on the Extended Glasgow Outcome Scale at
hypernatremia on admission was recognized as an important predic- 6 months. Safety data such as the degree of hypernatremia or hyper-
tor for inpatient palliative care consultation alongside the presence of chloremia will be of utmost interest to nephrologists (42).
pressure ulcers and being bedbound (37). Clearly, clinicians should AKI in a patient with a traumatic brain injury requiring renal
instinctively recognize the importance of hypernatremia in judging replacement therapy represents a therapeutic challenge because of
overall prognosis. the increased risk of cerebral edema with intermittent conventional
hemodialysis. Dialysis can potentially compromise the therapeutic
goal of hyperosmolality by acutely decreasing the serum osmolality
Permissive Hypernatremia and by causing intradialytic hypotension. Therefore, in such a popu-
Hypernatremia is associated with increased mortality in patients in lation of patients, continuous renal replacement therapy (CRRT)
medical and surgical ICUs; however, in neurosurgical and neurologic has emerged as the preferred modality. A recently published article
ICUs hypernatremia is often a component of the treatment of cere- on therapeutic hypernatremia management during CRRT discusses
bral edema with intravenous mannitol or hypertonic saline solution options of achieving hypernatremia in patients receiving CRRT.
(38). Hyperosmolar solutes acutely increase the systemic serum
Unfortunately, the sodium concentrations of commercially available
osmolality. When combined with blood-brain barrier impermeability
replacement fluids are lower than the desired hypernatremia levels
to mannitol and sodium, water extraction from brain tissue into the
for patients with elevated ICP. Therefore, induction and mainte-
intravascular compartment should occur as a result of the flow of
nance of hypernatremia can be accomplished by use of an adjustable
water following a concentration gradient. Currently, the Brain
infusion of hypertonic saline solution during CRRT. In a recent
Trauma Foundation Guidelines for the management of severe trau-
review the authors discuss strategies like using a separate pump to
matic brain injury recommend using 3% saline solution rather than
deliver 3% saline solution during CRRT as a preferred option to
mannitol as initial treatment for traumatic brain injury with elevated
using the post–filter replacement fluid pump, among other alterna-
intracranial pressure (ICP), mostly because of its predictable response
tives (43).
when targeting serum [Na1] between 150 and155 mmol/L (39).
Although interventions with hypertonic saline are clearly of
physiologic benefit for the treatment of intracranial hypertension,
the high mortality associated with ICU-acquired hypernatremia Diabetes Insipidus
challenges this concept. There are very few studies on the safety of Diabetes insipidus is characterized by hypotonic polyuria caused by
hypertonic saline solution in patients with elevated ICP after trau- impaired synthesis (central DI) or action (nephrogenic DI) of the
matic brain injury. A recent retrospective analysis investigated antidiuretic hormone (ADH)/AVP (Figure 2).
in-hospital mortality, length of stay, and frequency of tracheostomy Central DI is more common than nephrogenic DI. It is most
and percutaneous endoscopic gastrostomy placements in hypernatre- often an acquired disorder that is caused by disruption of the neuro-
mic and normonatremic adults with a diagnosis of traumatic brain hypophysis, whereas hereditary forms are less common and are
injury who required mechanical ventilation, ICP monitoring, or cra- caused by mutation of the AVP gene. Nephrogenic DI is most com-
niotomy. Hypernatremic patients had a higher mortality rate, longer monly the result of an adverse effect of various drugs, or of electro-
length of stay, higher rates of tracheostomy, and higher rates of lyte disorders such as hypercalcemia and hypokalemia. In much rarer
endoscopic gastrostomy placements, suggesting poor functional out- instances, nephrogenic DI is due to a mutation in the AVP-2 recep-
comes in those who did survive. Because severe traumatic brain tor (hereditary nephrogenic DI). A third, rare form of DI, termed
injury is often associated with the development of central diabetes gestational DI, can occur during pregnancy and results from the
insipidus (DI), its presence could be a confounding factor in the enzymatic breakdown of endogenous AVP by increased placental
assessment of the association of ICU-acquired hypernatremia and vasopressinase levels in pregnancy (44).
outcomes in this patient population (40).
Another retrospective analysis that included 458 patients with
moderate to severe traumatic brain injury investigated the associa- Medication-Induced Diabetes Insipidus
tion between hypernatremia and hyperchloremia and mortality rates. Lithium remains the most common cause of nephrogenic DI.
Over the entire duration of an ICU stay, the time-weighted average Whereas glomerular damage appears after years of lithium treat-
(TWA) of sodium and chloride was calculated. Unadjusted mortal- ment, tubular dysfunction can happen within weeks after lithium
ity rates were near linearly increasing for both the TWA of sodium initiation. Lithium inhibits adenylate cyclase, with a resultant
and chloride, and were the highest for serum [Na1] of .160 decrease in the formation of cAMP. By doing so, lithium impairs
mmol/L and serum [Cl-] .125 mmol/L. When evaluated in the fusion of aquaporin storage vesicles with a luminal wall in the
Nephrology Self-Assessment Program - Vol 20, No 2, January 2022 95

Central diabetes
insipidus
Destruction of
p Pituitary
neurohypophyseal
AVP release
neurons or
mutations in AVP

Primary
polydipsia
Excessive
fluid intake

Renal Nephrogenic
resistance diabetes insipidus
to serum AVP Decreased response
to AVP or mutations
in AVPR2 or AQP2

n AVP degradation

Gestational
diabetes insipidus
Excessive activity
p Free water reabsorption
of placental
nSolute-free water diuresis
vasopressinase

Figure 2. Pathophysiology of diabetes insipidus (DI). There are three principle causes of DI. A decrease in arginine vasopressin (AVP) release or
central DI is caused by destruction of neurohypophyseal neurons or a mutation in the AVP gene. A decreased response to AVP or nephrogenic
DI is due to renal unresponsiveness to AVP. Increased AVP degradation or gestational DI is caused by excessive activity of the placental vaso-
pressinase. Of note, the excessive fluid intake in primary polydipsia similarly to central DI causes a decrease in AVP release (44). Reprinted with
permission from reference 44 (Christ-Crain M, Bichet DG, Fenske WK, Goldman MB, Rittig S, Verbalis JG, Verkman AS: Diabetes insipidus.
Nat Rev Dis Primers 5: 54, 2019).

collecting duct. It renders the collecting duct impermeable to water Two recently published articles raise concerns about
and hence causes hypo-osmolar polyuria (44). medication-associated occurrences of DI in the ICU setting. A retro-
Up to 20% of patients treated with lithium may experience spective study of 35 patients receiving sevoflurane showed a 20%
nephrogenic DI, which can persist even after lithium has been incidence of nephrogenic DI during the ICU stay. There is an asso-
stopped. As long as the patient has access to water, hypernatremia is ciation between the development of nephrogenic DI and the con-
not common. However, if an additional insult impairs the access to centration of sevolfurane, as well as the duration of exposure. All
water or a mitigating factor is present, hypernatremia occurs. A recent patients with nephrogenic DI were sedated for more than 72 hours
retrospective study investigated cofounding factors leading to hyper- with mean end-tidal concentration of sevoflurane .1.0 vol % (47).
natremia in patients previously treated with lithium. Concomitant Another recent single-center case series from the UK reported the
infections were identified as the dominant cause of hypernatremia, development of central DI after discontinuation of vasopressin infu-
accounting for 37% of all episodes. This was followed by intoxica- sion for treatment of shock, with the exact cause being unclear (48).
tions other than lithium, which were identified as a confounding fac-
tor in 15% of patients in this cohort. In patients .65 years of age,
infections were identified in 51% of patients taking lithium who Diagnosis of Diabetes Insipidus
experienced hypernatremia (45). In routine clinical practice, the measurement of urine osmolarity
There are currently no known strategies to prevent lithium- and urine volume aids in establishing the cause of hypernatremia.
induced DI in patients who require it. In mice, however, statins Careful assessment of urinary output, daily weight, and accurate
have been shown to increase the expression of membrane aquaporin balance reporting in hospitalized patients is often neglected. Addi-
2 in the renal tubules. Cross-sectional studies showed that the use of tionally, assessment of extracellular fluid volume in clinical prac-
statins is associated with a lower risk of nephrogenic DI. Currently, tice is challenging and can be misleading. Therefore, the need for
a randomized control trial of atorvastatin for lithium-induced neph- novel diagnostic markers for the differential diagnosis of hyperna-
rogenic DI is being conducted in Canada (46). tremia arises.
96 Nephrology Self-Assessment Program - Vol 20, No 2, January 2022

To briefly review the diagnosis of DI, the diagnostic criterion independently of the copeptin measurements. Combining serum
standard in clinical practice is the indirect water deprivation test. copeptin levels with the hypertonic saline infusion test reliably dif-
Patients are deprived of fluid intake while serum sodium and osmo- ferentiated between central DI and primary polydipsia (Figure 3).
lality, along with urine volume and urine osmolality, are carefully Of note, the hypertonic saline infusion test did not aid in differenti-
monitored. This test is required to last for 17 hours or until the ating between complete and partial central DI (51).
serum sodium concentration exceeds 150 mmol/L. At that time, Hypertonic saline stimulation of AVP release, however, is
exogenous AVP is administered, and its effect on urine osmolality is poorly tolerated, with #70% of patients experiencing vertigo, head-
measured. This test indirectly establishes the lack or effect of AVP. ache, or malaise. Another recent study tested the diagnostic value of
Needless to say, this test is cumbersome in clinical practice. Unfor- AVP-stimulated copeptin levels in the workup of DI. In a prospec-
tunately, previous attempts to incorporate direct measurements of tive diagnostic study, a development cohort was recruited to derive
AVP levels were largely unsuccessful because the assay had technical the cutoffs for copeptin to discriminate between patients with cen-
limitations and never made its way into routine clinical practice. tral DI and those with primary polydipsia, and later the diagnostic
Another biomarker with potential to aid in the diagnostic workup performances of these cutoffs were validated in the validation
of hypernatremia was found with the discovery of copeptin in the cohort. All patients had a baseline copeptin checked, and patients
mid-1970s. Copeptin is the C-terminal segment of the AVP precur- whose levels were consistent with nephrogenic DI were excluded.
sor peptide and mirrors the AVP concentration. It can be easily The median copeptin values for those with primary polydipsia dou-
measured and provides a reliable surrogate marker for AVP. How- bled after AVP stimulation, from 3.6 pmol/L (IQR 2.4–5.7 pmol/L)
ever, it wasn’t until the last decade that its potential as a diagnostic at baseline to a maximum of 7.9 pmol/L (IQR 5.1–11.8 pmol/L)
marker for DI was further investigated. Copeptin is released in at 120 minutes (P,0.0001). By contrast, in patients with DI,
response to the same stimuli as AVP. The main osmotic stimulus is copeptin increased only slightly after AVP stimulation (median 2.1
an increase in serum osmolarity. Reduction in arterial blood volume pmol/L (IQR 1.9–2.7 pmol/L) at baseline) to a maximum of 2.5
and pressure is the primary nonosmotic stimulus. A normal range of pmol/L (IQR 1.9–3.1 pmol/L) at 60 minutes (P50.0013). In the
plasma copeptin levels has been established in two large clinical trials subgroups of patients with complete and partial DI, only a small
evaluating healthy volunteers under normal osmotic conditions change in copeptin concentrations was observed for those with
where plasma copeptin levels ranged from 1.0 to 13 pmol/L, with a complete DI (from 1.9 pmol/L (IQR 1.6–2.4 pmol/L) to a maxi-
median concentration of 4.2 pmol/L. Previous studies have shown mum at 60 minutes of 2.2 pmol/L (IQR 1.6–28); P50.0032),
that nephrogenic DI can be easily diagnosed by the use of a single whereas for those with partial DI, copeptin concentrations
baseline copeptin level of .21.4 pmol/L without prior thirsting. increased from 2.5 pmol/L (IQR 2.2–3.0 pmol/L) to a maximum
However, baseline copeptin values in the other entities (i.e., central at 120 minutes of 3.3 pmol/L (2.9–3.9; P50.0035) (52).
DI and primary polydipsia) largely overlapped (49). Outlined above are the recent developments in the diagnosis of
A recent prospective multicenter observational study conducted patients with hypotonic polyuria, which may change the decades-
in Switzerland assessed the value of baseline copeptin in 92 patients old algorithm currently used but have not entered routine clinical
with [Na1] .155 mmol/L (50). Copeptin levels varied according practice yet.
to underlying pathology, where the highest levels were seen in
patients with nephrogenic DI, and the lowest median levels occurred
in those with central DI. Median levels of copeptin differed signifi- 4 A single copeptin level of >21.4 pmol/L without
cantly in patients with central DI (3.39 pmol/L, IQR 1.99–3.9
prior thirsting identifies patients with nephrogenic
pmol/L), salt overload (53.19 pmol/L, IQR 34.39–85.95 pmol/L),
dehydration-induced hypernatremia (54.95 pmol/L, IQR diabetes insipidus, but it does not distinguish
35.01–71.70), and nephrogenic DI (77.75 pmol/L); (P50.001). between central diabetes insipidus and primary
No difference in copeptin levels was seen between cases of polydipsia.
dehydration-induced hypernatremia and salt overload (P50.64).
Additionally, in this study low urea values were highly specific and 4 A copeptin level obtained after hypertonic infusion
sensitive for diagnosis of central DI. This phenomenon is caused by reliably differentiates central diabetes insipidus
net absorption of the urea, which is dependent on the renal action from primary polydipsia.
of ADH. In vitro experiments in rats show that the inner medullary
collecting tubule urea permeability rises in response to AVP; there-
4 An increase in AVP-stimulated copeptin levels may
fore, urea levels might reflect AVP action in the nephron and may identify patients with primary polydipsia.
differentiate patient's DI from dehydrated states (50).
Inasmuch as baseline copeptin levels are unable to differentiate
between central DI and primary polydipsia, a recent prospective
multicenter study added a hypertonic saline infusion test to the algo-
rithm to more accurately differentiate between central DI and pri- Treatment of Hypernatremia
mary polydipsia. A total of 156 patients with hypotonic polydipsia Chronic hypernatremia for clinical purposes is defined as increased
underwent the water deprivation test as well as the hypertonic saline serum [Na] for a duration of .48 hours or unknown in the case
infusion test (measurement of copeptin after a hypertonic saline of community-acquired hypernatremia. Adaptation of brain cells
infusion was done after the [Na1] reached at least 150 mmol/L). to chronic hypernatremia is characterized by an increase in intracel-
The underlying cause of hypotonic polydipsia was established lular osmolality by accumulating organic osmoles. With the rapid
Nephrology Self-Assessment Program - Vol 20, No 2, January 2022 97

Stimulated Copeptin Levels in Response to the Hypertonic


Saline Infusion and Water-Deprivation Tests in Patients with
Hypotonic Polyuria.

W Fenske et al. N Engl J Med 2018;379:428-439.

Figure 3. Patients with hypotonic polyuria and copeptin levels after hypertonic saline infusion test or water deprivation test. (A) Copeptin levels
are significantly higher in patients with primary polydipsia after the hypertonic saline infusion than in patients with central diabetes insipidus.
This is not the case after the water deprivation test. (B) Neither the hypertonic saline infusion test nor the water deprivation test was able to dis-
tinguish between complete and partial diabetes insipidus. DI, diabetes insipidus (51). Reprinted with permission from reference 51 (Fenske W,
Refardt J, Chifu I, Schnyder I, Winzeler B, Drummond J, Ribeiro-Oliveira A Jr, Drescher T, Bilz S, Vogt DR, Malzahn U, Kroiss M, Henzen
CE, Fischli S, Tonjes A, Mueller B, Schopohl J, Flitsch J, Brabant G, Fassnacht M, Christ-Crain M: A copeptin-based approach in the diagnosis
of diabetes insipidus. N Engl J Med 379: 428–439, 2018).

correction of hypernatremia, the extrusion of these accumulated correction were made for both children and adults. In this context it is
organic osmoles is delayed, potentially leading to subsequent cere- important to note that the data governing the rate of correction for
bral edema. Cerebral edema is particularly dangerous in young chil- hypernatremia are less robust than the data that are used to recom-
dren, whose brains fill the whole cranial vault, unlike those of mend a rate of correction in patients with hyponatremia (53).
adults. The cornerstone of treatment of hypernatremia is correction of
Guidelines recommend a correction of hypernatremia of no the free water deficit by administration of free water, orally or intra-
more than 8–10 mmol/L per 24 hours. with a maximal rate of 0.5 venously with 5% dextrose solution. The free water deficit can be
mmol/L per hour in chronic hypernatremia. These recommendations calculated by the following formula: Free water deficit5% of total
were derived from observations in the pediatric population of hyper- body water (as fraction of 1)3body weight (kg)3[(sodium concen-
natremic newborns. There, a sodium correction of 1.02260.1278 tration/140)21], where % total body water is 60% for adult male,
mmol/L per hour led to rehydration seizures in 9 newborns, compared 50% for adult female, 50% for elderly male, and 45% for elderly
with 22 newborns with a sodium correction of 0.61860.083 mmol/L female. The change in serum sodium for each liter of administered
per hour, who remained seizure free. There were no seizures in 27 free water can be calculated by this formula: change in serum
newborns who received correction by ,0.5 mmol/L per hour. Based sodium5[(infusate sodium)2(serum sodium)]/[total body water
on these retrospective studies, recommendations on hypernatremia (L)11].
98 Nephrology Self-Assessment Program - Vol 20, No 2, January 2022

Rapid correction was defined as a rise in the [Na1] of .0.5 mmol/


L per hour, or .12 mmol/L per day. Slow correction was defined as
a rise in [Na1] of ,0.5 mmol/L per hour and ,10 mmol/L per
day. The in-hospital mortality was similar between patients with
rapid versus slow correction. This was the case in patients who had
hypernatremia on admission and patients with hospital-acquired
hypernatremia (in-hospital 30-day mortality 25% versus 28%,
P50.8, in the admission hypernatremia group; 44% versus 40%,
P50.50, in the hospital-acquired hypernatremia group) (Figure 4).
Furthermore, there were no cases of cerebral edema in the 78
patients with serum sodium correction of .12 mmol/L/day. The
authors argue that fears of more rapid correction of hypernatremia
are probably overstated (56).
Severely burned ICU patients represent a unique subset of
hypernatremic patients, inasmuch as high ongoing free water losses
frequently occur in this patient population. Conventional treatments
with free water replacement and nutritional support are often subop-
timal in the correction of hypernatremia. A recent single-center ret-
rospective study investigated the implementation of continuous
venovenous hemofiltration (CVVH) if hypernatremia failed to cor-
rect after 24 hours of conventional therapy. A total of 13 patients
were enrolled. At the beginning of CVVH, the sodium concentra-
tion in the replacement fluid was 8 mmol/L lower than the serum
[Na1] and was reduced by 2 mmol/L every 4 hours subsequently.
With a serum reduction rate of 0.6 mmol/L per hour, all patients
experienced a successful correction of hypernatremia within 48
hours and showed no complications (57).
There are controversial opinions on the choice of anticoagula-
tion during CVVH in patients with severe hypernatremia and
Figure 4. Thirty-day survival in patients with hypernatremia depend- bleeding risk. Infusion of regional citrate anticoagulation (RCA)
ing on the rate of correction of the serum sodium concentration. solution can theoretically result in significant sodium load (408
A) Patients with hypernatremia on admission. B) Hospital-acquired mmol/L for a 4% sodium citrate solution); therefore, hypernatre-
hypernatremia. The 30-day survival between slow correction group mia was considered one of the potential contraindications to
(,0.5 mmol/L per hour) and rapid correction group (.0.5 mmol/L citrate anticoagulation. A recent retrospective study evaluated the
per hour) was similar (56). Used with permission from reference 56 efficacy and safety of RCA versus no-anticoagulation CVVH in
(Chauhan K, Pattharanitima P, Patel N, Duffy A, Saha A, Chaudh- acute severe hypernatremia patients with increased bleeding risk.
ary K, Debnath N, Van Vleck T, Chan L, Nadkarni GN, Coca SG: The mean rate of sodium reduction was 0.9560.39 mmol/L per
Rate of correction of hypernatremia and health outcomes in critically hour in the no-anticoagulation group and 0.9260.28 mmol/L per
ill Patients. Clin J Am Soc Nephrol 14: 656–663, 2019). hour in the RCA group, challenging the prior notion to avoid

As recently reviewed, current recommendations for rate of cor-


rection of hypernatremia likely led to undertreatment of hypernatre- 4 A growing body of evidence suggests that prior
mia in adults. For instance, in a retrospective analysis of a cohort of guidelines for managing hypernatremia were too
hypernatremic patients that investigated treatment success, 24 hours conservative and led to undertreatment of
after initiation of hypernatremia treatment the sodium levels wors-
ened in 34% of patients and remained unchanged in 8% of patients.
hypernatremia.
After 3 days, hypernatremia normalized in only 32% of patients 4 Inadvertent rapid correction of hypernatremia is
(54). Additionally, there may be evidence that extremely slow rates much less likely than that of hyponatremia.
of correction of chronic hypernatremia are associated with higher
mortality in adults (55). 4 More rapid correction of hypernatremia in adults
Addressing the rate of correction for patients with hypernatre- appears to be less harmful than was previously
mia, a recently published study challenged the classic hypernatremia thought.
correction guidelines by comparing more rapid correction with slow
correction in 449 critically ill patients with severe hypernatremia.
Nephrology Self-Assessment Program - Vol 20, No 2, January 2022 99

RCA use in hypernatremia. The mortality rate was similar in both among all donors. The authors speculate that critically ill patients
groups (58). with respiratory diseases are less harmed by the otherwise detrimental
effect of hypernatremia in the ICU patient population, which was
previously reported. The authors surmise that hypernatremia in the
Donor and Recipient Hypernatremia in Solid Organ lung transplant donor (unlike than in other solid organ donors) pro-
Transplantation duces a neutral effect on lung transplantation outcomes and argue
For several solid organ transplants, such as heart, liver, kidneys and that donor hypernatremia should not exclude such donors (64).
pancreas, donor hypernatremia has been found to be associated with
reduced graft survival. Cell lysis and graft damage, as well as post-
Kidney and Pancreas
transplantation edema, of the hypernatremic graft in the recipient
with normal or low serum sodium levels have been proposed to be A recent revision of Eurotransplant data on which donor factors
responsible for some of the pathogenic effects of donor hypernatre- were associated with nontransplantation of donor pancreas found
mia (59). hypernatremia as one of the frequent causes of pancreas graft refusal.
While the detrimental effects of sodium dysregulation are well However, a subsequently published study on the association of
known at a cellular level, its effect on patient-centered outcomes has donor sodium levels with postoperative morbidity and pancreas graft
not been consistently demonstrated. Here, we review the most loss in simultaneous pancreas-kidney transplantation failed to find a
recent studies on the association of hypernatremia with solid organ difference between hyper- and normonatremic donors (59).
transplantation outcomes. In kidney transplantation, out of 11 observational studies
included in a recent systemic review that analyzed the effect of
Heart donor hypernatremia on renal graft function, five studies found an
association between donor hypernatremia and recipient graft func-
Donor hypernatremia has been associated with primary graft failure
in heart transplant patients. A retrospective analysis of data from the tion, whereas six did not. Of the five studies concluding an associa-
University of Vienna on outcomes of heart transplantation (rejected tion between donor sodium and graft dysfunction, most defined
by University of Vienna but accepted elsewhere) showed survival hypernatremia as serum sodium .155 mmol/L. Among six studies
rates of only 46% at 3 years (60). Similar results were reported in a that did not find an association between donor sodium and graft
retrospective cohort of 200 heart transplant recipients. Donor hyper- function, two studies included donor cohorts with serum sodium
natremia was associated with higher incidence of primary graft fail- levels between 145 and 150 mmol/L, three studies included donor
ure. Patients who receive a heart transplant from a donor with a cohorts with serum sodium between 150 and 155 mmol/L, and one
[Na1] .159 mmol/L were significantly more likely to have pri- pediatric study included donors with the most extreme mean serum
mary graft failure (incidence of graft failure 63.3% versus 32.4%, sodium of 163610 mmol/L. This systematic review raises the point
P,0.001) (61). that evidence regarding the effect of donors’ sodium levels and trans-
plantation outcomes is inconclusive and that labeling organs from
Liver hypernatremic donors as marginal may be unsubstantiated and lead
to the discard of a valuable and limited resource (65).
Before liver transplantation, recipients typically have hyponatremia
reflective of the severity of the liver cirrhosis. Pretransplantation
hyponatremia in a recipient is a poor prognostic factor. The liver References
transplantation procedure itself requires a large amount of blood 1. Seay NW, Lehrich RW, Greenberg A: Diagnosis and management of dis-
products, fluid resuscitation, and albumin infusion, which, in com- orders of body tonicity-hyponatremia and hypernatremia: Core curricu-
bination with potentially rapid intraoperative ceasing of endogenous lum 2020. Am J Kidney Dis 75: 272–286, 2020 PubMed
2. Zimmerman CA, Leib DE, Knight ZA: Neural circuits underlying thirst
ADH, can lead to a rapid change in the [Na1]. The size of the
and fluid homeostasis. Nat Rev Neurosci 18: 459–469, 2017 PubMed
intraoperative delta [Na1] is associated with a prolonged hospitali- 3. Augustine V, Gokce SK, Lee S, Wang B, Davidson TJ, Reimann F,
zation and higher early mortality (62). Hypernatremic donors in Gribble F, Desseroth K, Lois C, Oka Y: Hierarchical neural architecture
liver transplantation are considered expanded criteria donors. In a underlying thirst regulation. Nature 555: 204–209, 2018 PubMed
recent retrospective analysis, the degree of donor hypernatremia per 4. Bichet DG: Regulation of thirst and vasopressin release. Annu Rev Physiol
se had little impact on short-term graft survival. However, the overall 81: 359–373, 2019 PubMed
survival rate of the grafts from hypernatremic donors rather 5. Gizowski C, Bourque CW: The neural basis of homeostatic and anticipa-
tory thirst. Nat Rev Nephrol 14: 11–25, 2018 PubMed
depended on exposure time from hypernatremia to reperfusion, cold
6. Prager-Khoutorsky M: Mechanosensing in hypothalamic osmosensory
ischemia time, and MELD score (63). neurons. Semin Cell Dev Biol 71: 13–21, 2017 PubMed
7. Kinsman BJ, Simmonds SS, Browning KN, Wenner MM, Farquhar
Lung WB, Stocker SD: Integration of hypernatremia and angiotensin II by the
In a recent Dutch study analyzing all consecutive lung transplanta- organum vasculosum of the lamina terminalis regulates thirst. J Neurosci
tions from 1996 to 2016, donor hypernatremia was not associated 40: 2069–2079, 2020 PubMed
8. Allen WE, DeNardo LA, Chen MZ, Liu CD, Loh KM, Fenno LE, Ram-
with primary graft dysfunction or long-term survival. Donors were
akrishnan C, Deisseroth, LL: Thirst-associated preoptic neurons encode
categorized as normonatremic (,145 mmol/L), moderately hyper- an aversive motivational drive. Science 357: 1149–1155, 2017 PubMed
natremic (146 to 154 mmol/L), and severely hypernatremic (.155 9. Lombardi G, Ferraro PM, Calvaruso L, Naticchia A, D’Alonzo S, Gam-
mmol/L). Short-term graft function defined as the duration of baro G: Sodium fluctuations and mortality in a general hospitalized pop-
mechanical ventilation, as well as 10-year mortality, was similar ulation. Kidney Blood Press Res 44: 604–614, 2019 PubMed
100 Nephrology Self-Assessment Program - Vol 20, No 2, January 2022

10. Ranjan R, Lo SC, Ly S, Krishnananthan V, Lim AKH: Progression to 29. Gao XP, Zheng CF, Liao MQ, He H, Liu YH, Jing CX, Zeng FF, Chen
severe hypernatremia in hospitalized general medicine inpatients: An QS: Admission serum sodium and potassium levels predict survival
observational study of hospital-acquired hypernatremia. Medicina (Kau- among critically ill patients with acute kidney injury: A cohort study.
nas) 56: E358, 2020 PubMed BMC Nephrol 20: 311, 2019 PubMed
11. Shirazy M, Omar I, Abduljabbar D, Bousselmi K, Alkhaja M, Chaari 30. Chi WC, Patel S, Cheung NW: Admission sodium levels and hospital
A, Kauts V, Hakim K: Prevalence and prognostic impact of hypernatre- outcomes. Intern Med J 51: 93–98, 2021 PubMed
mia in sepsis and septic shock patients in the intensive care unit: A sin- 31. Castello LM, Gavelli F, Baldrighi M, Salmi L, Mearelli F, Fiotti N,
gle centre experience. J Crit Care Med (Targu-Mures) 6: 52–58, 2020 Patrucco F, Bellan M, Sainaghi PP, Ronzoni G, Di Somma S, Lupia E,
PubMed Muiesan ML, Biolo G, Avanzi GC: Hypernatremia and moderate-to-
12. Breen T, Brueske B, Sidhu MS, Murphree DH, Kashani KB, Barsness severe hyponatremia are independent predictors of mortality in septic
GW, Jentzer JC: Abnormal serum sodium is associated with increased patients at emergency department presentation: a sub-group analysis of
mortality among unselected cardiac intensive care unit patients. J Am the need-speed trial. Eur J Intern Med 83: 21–27,2020 PubMed
Heart Assoc 9: e014140, 2020 PubMed 32. Chewcharat A, Thongprayoon C, Cheungpasitporn W, Mao MA, Thiru-
13. Aoyagi Y, Yoshida T, Uchino S, Takinami M, Uezono S: Saline versus navukkarasu S, Kashani KB: Trajectories of serum sodium on in-hospital
5% dextrose in water as a drug diluent for critically ill patients: A retro- and 1-year survival among hospitalized patients. Clin J Am Soc Nephrol
spective cohort study. J Intensive Care 8: 69, 2020 PubMed 15: 600–607, 2020 PubMed
14. Bissell BD, Laine ME, Thompson Bastin ML, Flannery AH, Kelly A, 33. Tsipotis E, Price LL, Jaber BL, Madias NE: Hospital-associated hyperna-
Riser J, Nehra JA, Potter J, Morris PE: Impact of protocolized diuresis tremia spectrum and clinical outcomes in an unselected cohort. Am J
for de-resuscitation in the intensive care unit. Crit Care 24: 70, 2020 Med 131: 72–82.e1, 2018 PubMed
PubMed 34. Grangeon-Chapon C, Dodoi M, Esnault VL, Favre G: Osmotic stress
15. Imaizumi T, Nakatochi M, Fujita Y, Yamamoto R, Watanabe K, Mae- and mortality in elderly patients with kidney failure: A retrospective
kawa M, Hamawaka T, Katsjjno T, Maruyama S: Glucocorticoid study. Clin Interv Aging 14: 225–229, 2019 PubMed
treatment is associated with ICU-acquired hypernatremia: A nested case- 35. Seo MS, Hwang IC, Jung J, Lee H, Choi JH, Shim JY: Hypernatremia
control study. Clin Exp Nephrol 25: 131–139, 2021 PubMed at admission predicts poor survival in patients with terminal cancer: A
16. Lehtiranta S, Honkila M, Kallio M, Paalane N, Peltoniemi O, Pokka T, retrospective cohort study. BMC Palliat Care 19: 94, 2020 PubMed
Renko M, Tapianen T: Risk of electrolyte disorders in acutely ill children 36. Vicent L, Alvarez-Garcia J, Gonzalez-Juanatey JR, Rivera M, Segovia J,
receiving commercially available plasmalike isotonic fluids: A randomized Worner F, Bover R, Pascual-Figal D, Vazquez R, Cinca J, Fernandez-
clinical trial. JAMA Pediatr 175: 28–35, 2021 PubMed Aviles FF, Selles MM: Prognostic impact of hyponatremia and hyperna-
17. Imai N, Shibagaki Y: The prevalence of dysnatremia in the elderly tremia at admission and discharge in heart failure patients with preserved,
patients without CKD. Am J Emerg Med 37: 499–501, 2019 PubMed mid-range, and reduced ejection fraction. Intern Med J 51: 930–938,
18. Begg DP: Disturbances of thirst and fluid balance associated with aging. 2020
Physiol Behav 178: 28–34, 2017 PubMed 37. Sharda N, Zietlow K, Wong S, Kuchibhatla M, Johnson KS: Character-
19. Woitok BK, Funk GC, Walter P, Schwarz C, Ravioli S, Lindner G: Dys- istics and outcomes of dementia patients who receive inpatient palliative
natremias in emergency patients with acute kidney injury: A cross- care consultation. J Am Geriatr Soc 68: 2027–2033, 2020 PubMed
sectional analysis. Am J Emerg Med 38: 2602–2606, 2020 PubMed 38. Mangat HS: Hypertonic saline infusion for treating intracranial hyper-
20. Brennan M, Murray O, O'Shea PM, Mulkerrin EC. Increased rates of tension after severe traumatic brain injury. Crit Care 22: 37, 2018
hypernatraemia during modest heatwaves in temperate climates. QJM: PubMed
monthly journal of the Association of Physicians. 2020;113:266–270. 39. Tsaousi G, Stazi E, Cinicola M, Bilotta F: Cardiac output changes after
21. Imai N, Sumi H, Shibagaki Y: Impact of age on the seasonal prevalence osmotic therapy in neurosurgical and neurocritical care patients: A sys-
of hypernatremia in the emergency department: A single-center study. tematic review of the clinical literature. Br J Clin Pharmacol 84:
Int J Emerg Med 12: 29, 2019 PubMed 636–648, 2018 PubMed
22. Blohm E, Goldberg A, Salerno A, Jenny C, Boyer E, Babu K: Recogni- 40. Hoffman H, Jalal MS, Chin LS: Effect of hypernatremia on outcomes
tion and management of pediatric salt toxicity. Pediatr Emerg Care 34: after severe traumatic brain injury: A nationwide inpatient sample analy-
820–824, 2018 PubMed sis. World Neurosurg 118: e880–e886, 2018 PubMed
23. Hancher J, Fisher J, Shenvi C. Severe hypernatremia and gastric dilation 41. Ditch KL, Flahive JM, West AM, Osgood ML, Muehlschlegel S: Hyper-
from chronic eating disorder and intentional salt ingestion. AmJ Emerg chloremia, not concomitant hypernatremia, independently predicts early
Med. 2020;38:1700.e1–1700.e03. mortality in critically ill moderate-severe traumatic brain injury patients.
24. Park MS, Park HJ, Choi HS, Kim CS, Bae EH, Ma SK, Kim SW, Kim Neurocrit Care 33: 533–541, 2020 PubMed
M: Extremely severe hypernatremia caused by wrong belief in a patient 42. Roquilly A, Lasocki S, Moyer JD, Huet O, Perrigault PF, Dahyot-Fize-
with cervical cancer. Electrolyte Blood Press 18: 16–18, 2020 PubMed lier C, et al.; COBI group: COBI (COntinuous hyperosmolar therapy
25. Sakamoto A, Hoshino T, Boku K, Hiraya D, Inoue Y: Fatal acute hyper- for traumatic Brain-Injured patients) trial protocol: A multicentre rando-
natremia resulting from a massive intake of seasoning soy sauce. Acute mised open-label trial with blinded adjudication of primary outcome.
Med Surg 7: e555, 2020 PubMed BMJ Open 7: e018035, 2017 PubMed
26. Lipman GS, Burns P, Phillips C, Jensen J, Little C, Jurkiewcz C, Jarrett 43. F€ul€op T, Zsom L, Rodrıguez RD, Chabrier-Rosello JO, Hamrahian M,
B, Walker A, Mansfield N, Krabak BJ: Effect of sodium supplements Koch CA: Therapeutic hypernatremia management during continuous
and climate on dysnatremia during ultramarathon running. Clin J Sport renal replacement therapy with elevated intracranial pressures and respira-
Med 31: e327–e334, 2020 PubMed tory failure. Rev Endocr Metab Disord 20: 65–75, 2019 PubMed
27. Martinez-Cano JP, Cortes-Castillo V, Martinez-Villa J, Ramos JC, Uribe 44. Christ-Crain M, Bichet DG, Fenske WK, Goldman MB, Rittig S, Ver-
JP: Dysnatremia among runners in a half marathon performed under balis JG, Verkman AS: Diabetes insipidus. Nat Rev Dis Primers 5: 54,
warm and humid conditions. BMJ Open Sport Exerc Med 4: e000351, 2019 PubMed
2018 PubMed 45. Ott M, Forssen B, Werneke U: Lithium treatment, nephrogenic diabetes
28. Thongprayoon C, Cheungpasitporn W, Petnak T, Ghamrawi R, Thiru- insipidus and the risk of hypernatraemia: A retrospective cohort study.
navukkarasu S, Chewcharat A, Bathini T, Vallabhajosyula S, Kashani Ther Adv Psychopharmacol 9: 2045125319836563, 2019 PubMed
KB: The prognostic importance of serum sodium levels at hospital dis- 46. Fotso Soh J, Torres-Platas SG, Beaulieu S, Mantere O, Platt R, Mucsi I,
charge and one-year mortality among hospitalized patients. Int J Clin Saury S, Renaud S, Levinson A, Andreazza AC, Mulsant BH, M€ uller D,
Pract 74: e13581, 2020 PubMed Schaffer A, Dols A, Cervantes P, Low NC, Herrmann N, Christensen
Nephrology Self-Assessment Program - Vol 20, No 2, January 2022 101

BM, Trepiccione F, Rajji T, Rej S: Atorvastatin in the treatment of hypernatremia: A retrospective study of 13 cases. Int J Artif Organs 43:
lithium-induced nephrogenic diabetes insipidus: The protocol of a ran- 416–421, 2020 PubMed
domized controlled trial. BMC Psychiatry 18: 227, 2018 PubMed 58. Zhao L, Ma F, Yu Y, Li Y, Wang Y, He L, Zhou M, Tian X, Jing R, Li
47. Honore PM, De Bels D, Barreto Gutierrez L, Redant S, Gallerani A, L, Li L, Huang C, Bai M, Sun S: Regional citrate anticoagulation versus
Boer W: Sevoflurane and nephrogenic diabetes insipidus on the rise: no-anticoagulation for continuous venovenous hemofiltration in acute
Copeptin to the rescue? Crit Care 23: 304, 2019 PubMed severe hypernatremia patients with increased bleeding risk: A retrospec-
48. Rana H, Ferguson N, Dicpinigaitis PV: Diabetes insipidus after discon- tive cohort study. Blood Purif 49: 44–54, 2020 PubMed
tinuation of vasopressin infusion for septic shock. J Clin Pharm Ther 43: 59. Sanchez-Hidalgo JM, Rodrıguez-Ortiz L, Arjona-Sanchez A, Ruiz-Rabelo
287–290, 2018 PubMed J, Salamanca-Bustos JJ, Rodrıguez-Benot A, Marquez-Lopez FJ, Brice~ no-
49. Refardt J, Winzeler B, Christ-Crain M: Copeptin and its role in the diag- Hidalgo J: Pancreas donor hypernatremia: Is it really a risk factor for
nosis of diabetes insipidus and the syndrome of inappropriate antidiure- simultaneous pancreas-kidney transplantation? Transplant Proc 50:
sis. Clin Endocrinol (Oxf) 91: 22–32, 2019 PubMed 676–678, 2018 PubMed
50. Nigro N, Winzeler B, Suter-Widmer I, Schuetz P, Arici B, Bally M, 60. Aliabadi-Zuckermann AZ, G€okler J, Kaider A, Riebandt J, Moayedifar R,
Refardt J, Betz M, Gashi G, Urwyler SA, Burget L, Blum CA, Bock A, Osorio E, Haberi T, Angleitner P, Laufer G, Forsythe J, Knezevic I, Skoric
Huber A, M€ullwe V, Christ-Crain M: Copeptin levels and commonly B, Erasmus M, van Cleemput J, Caliskan K, De Jonge N, Szabolcs Z,
used laboratory parameters in hospitalised patients with severe hyperna- Prodan Z, Wasler A, Bara C, Udovicic M, Sandhaus T, Garbade J, Ruh-
traemia - The “Co-MED study”. Crit Care 22: 33, 2018 PubMed parwar A, Schoenrath F, Hirt S, Antretter H, Schulz U, Richter M, Thul
51. Fenske W, Refardt J, Chifu I, Schnyder I, Winzeler B, Drummond J, J, Barten MJ, Haneya A, Aleksic I, Efiert S, Berchtold-Herz M, Smits J,
Ribeiro-Oliveira A Jr, Drescher T, Bilz S, Vogt DR, Malzahn U, Kroiss M, Zuckermann AO: Donor heart selection and outcomes: An analysis of
Henzen CE, Fischli S, Tonjes A, Mueller B, Schopohl J, Flitsch J, Brabant over 2,000 cases. J Heart Lung Transplant 37: 976–984, 2018 PubMed
G, Fassnacht M, Christ-Crain M: A copeptin-based approach in the diagno- 61. Finger MA, Cipullo R, Rossi Neto JM, Dos Santos CC, Contreras CA,
sis of diabetes insipidus. N Engl J Med 379: 428–439, 2018 PubMed Chaccur P, Dinkhuysen JJ, de Souza R, Dias França JI, Lin-Wang HT:
52. Winzeler B, Cesana-Nigro N, Refardt J, Vogt DR, Imber C, Morin B, Donor hypernatremia and smoking addiction contribute to primary graft
Popovic M, Steinmetz M, Sailer CO, Szinnai G, Chifu I, Fassnacht M, failure in heart transplantation. Clin Transplant 33: e13693, 2019 PubMed
Christ-Crain M: Arginine-stimulated copeptin measurements in the dif- 62. Mihaylov P, Nagai S, Ekser B, Mangus R, Fridell J, Kubal C: Prognostic
ferential diagnosis of diabetes insipidus: a prospective diagnostic study. impact of peritransplant serum sodium concentrations in liver transplan-
Lancet 394: 587–595, 2019 PubMed tation. Ann Transplant 24: 418–425, 2019 PubMed
53. Sterns RH: Evidence for managing hypernatremia: is it just hyponatremia 63. Zhou ZJ, Chen GS, Si ZZ, Li Q, Bin YY, Qi HZ, Li J-Q: Prognostic
in reverse? Clin J Am Soc Nephrol 14: 645–647, 2019 PubMed factors influencing outcome in adult liver transplantation using hyperna-
54. Santillanes G, Rose E: Evaluation and management of dehydration in tremic organ donation after brain death. Hepatobiliary Pancreat Dis Int
children. Emerg Med Clin North Am 36: 259–273, 2018 PubMed 19: 371–377, 2020 PubMed
55. Jongejan M, Vleming LJ: Time to speed up-the rate at which we correct 64. Oude Lansink-Hartgring A, Hessels L, de Vries AJ, van der Bij W, Ver-
hypernatremia. Eur J Intern Med 83: 105–106, 2020 PubMed schuuren EAM, Erasmus ME, Nijsten MWN: Donor hypernatremia is
56. Chauhan K, Pattharanitima P, Patel N, Duffy A, Saha A, Chaudhary K, not related with the duration of postoperative mechanical ventilation,
Debnath N, Van Vleck T, Chan L, Nadkarni GN, Coca SG: Rate of primary graft dysfunction, or long-term outcome following lung trans-
correction of hypernatremia and health outcomes in critically ill Patients. plantation. Ann Transplant 23: 500–506, 2018 PubMed
Clin J Am Soc Nephrol 14: 656–663, 2019 PubMed 65. Basmaji J, Hornby L, Rochwerg B, Luke P, Ball IM: A systematic review
57. Huang C, Liu Y, Li L, Liu H, Zhang P: Continuous veno-venous hemo- of donor serum sodium level and its impact on transplant recipients. Int
filtration in the treatment of severely burned patients with acute J Organ Transplant Med 11: 43–54, 2020 PubMed
102 Nephrology Self-Assessment Program - Vol 20, No 2, January 2022

Article
Pathophysiology, Evaluation, and Treatment of Hypokalemia
Tushar Chopra, MD, FASN
Division of Nephrology, University of Virginia School of Medicine, Charlottesville, Virginia
Thomas D. DuBose Jr., MD, FASN
Wake Forest School of Medicine, Section on General Internal Medicine, Winston-Salem, North Carolina and Division
of Nephrology, University of Virginia School of Medicine, Charlottesville, Virginia

the daily K1 intake is excreted in the urine, whereas 5% is excreted


Learning Objectives by the gastrointestinal tract to maintain the steady state.
The sensor that evokes the mechanisms that maintain extracellu-
1. To summarize the mechanisms underlying internal and
lar potassium has not been completely clarified. A traditional view
external potassium balance that focuses on renal excretion is that an increase in extracellular
2. To define the pathophysiology of hypokalemia potassium concentration provoked by dietary intake directly and
indirectly (via aldosterone) stimulates excretion by the collecting
3. To examine the consequences of reduced potassium intake
duct. However, an alternative hypothesis is that the activation of
and hypokalemia enteric sensors located in the splanchnic bed increases potassium
secretion even in the absence of an increase in the extracellular potas-
sium (2). Several recent studies have suggested that enteric sensors
Introduction participate in the shift of K1 into skeletal muscle (3), as well as renal
The defense of body potassium (K1), homeostasis requires coordi- secretory processes upon K1 entry into the gastrointestinal tract (2).
nation of external balance (dietary intake) with absorption and The sensitivity, magnitude, and physiologic significance of enteric
excretion by the kidney and gastrointestinal tract, as well as internal sensors in the regulation of TBK homeostasis have not been fully elu-
balance through the asymmetric distribution of K1 between the cidated. Moreover, it is possible that intracellular shifts in response to
intracellular space (intracellular fluid [ICF]) and the extracellular enteric sensors may be elicited by hormones or neuropeptides that
space (extracellular fluid [ECF]). are known to be derived from the gastrointestinal tract.
This topic reviews the mechanisms underlying the maintenance
of internal and external potassium balance and the pathophysiology Cell Shifts
of hypokalemia. We emphasize the causes and significant conse-
quences for lower values for plasma K1 concentration or [K1]p. Transmembrane potential difference results from the unequal dis-
tribution of ions (particularly Na1 and K1) between intracellular
and extracellular compartments. Maintenance of the asymmetric
K1 concentration gradient across the cell membrane ([K1]in .
Total Body Potassium Distribution [K1]out) is essential to preserve the transmembrane potential dif-
Total body K1 (TBK) is approximately 55 mmol/kg body wt for a ference and equal to approximately 90 mV. This potential differ-
70-kg person (or approximately 3675 mmol) and is distributed pri- ence is important for cellular function in all cells but particularly
marily to the ICF compartment (98%). The ICF distribution is for nerve cells, cardiac muscle, and skeletal muscle. Factors that
largely in skeletal muscle (3000 mmol), red blood cells (200 mmol), affect potassium distribution are shown in Table 1. Catecholamines
and the liver (200 mmol). The driving force for K1 to shift into and insulin cause transient intracellular shifts of serum potassium to
cells is the more negative intracellular voltage. Accordingly, a steady help regulate the distribution of TBK and to protect against wide
state is maintained with a [K1] in skeletal muscle cells that is swings in the [K1]p (4). After ingestion of a meal, insulin release
approximately 35-fold higher than in the ECF. The [K1]p, the com- stimulates K1 uptake in skeletal muscle cells by the Na1, K1-
ponent that represents the K1 distribution to the ECF compart- ATPase, which translocates from intracellular stores to the cell sur-
ment, accounts for only 2% of TBK (1). face. Translocation allows for a highly regulated short-lived effect.
The cellular uptake of potassium into cells is also promoted by
b-adrenergic stimulation, xanthines, and alkalemia. In contrast,
Regulation of Potassium Homeostasis mineral acid acidemia, hypertonicity, a-adrenergic agonists, b2-
Internal and external potassium balance is maintained by a highly adrenergic antagonists, and glucagon increase K1 exit into the ECF.
regulated system involving three integrated components: (1) cellular During exercise, catecholamine release via b2-stimulation lessens the
shifts; (2) absorption and excretion by the kidney; and (3) to a lesser increase in extracellular K1 concentration that would otherwise
extent, transport in the gastrointestinal tract (1). Assuming a daily result from the normal K1 release by contracting muscle. Pharmaco-
dietary intake in the recommended range, approximately 95% of logic stimulation of b2-receptors with exogenous b2-agonists can

nephsap.org Copyright # 2022 by the American Society of Nephrology


Nephrology Self-Assessment Program - Vol 20, No 2, January 2022 103

Table 1. Shifts of K1 between extracellular fluid and intracellular fluid

Drives K1 into Cells to Lower [K1]p Drives K1 out of Cells to Increase [K1]p

Insulin release or administration Mineral acid acidosis (not organic endogenous acids)
b2-adrenergic agonists Hypertonicity
a1-adrenergic antagonists Hyperglycemia
Stress-related epinephrine release Sucrose
Drugs Mannitol
Theophylline Drugs
Albuterol a1-adrenergic agonists
Terbutaline b2-adrenergic antagonists
Xanthines Glucagon
Anabolic state Digoxin overdose
Rapid advancement of lymphoma or leukemia Succinylcholine
Treatment of pernicious anemia Extreme exercise
Crush and tissue injury
Hypokalemic periodic paralysis Rhabdomyolysis
Acquired (thyrotoxicosis) Hemolysis
Familial (periodic hypokalemic paralysis) Tumor lysis syndrome
Drug induced
Barium
Cesium salts
Thyroxine
Chloroquine
Alkalosis (minimal impact on [K1]p)

result in symptomatic hypokalemia in patients with asthma, heart is primarily accomplished through the paracellular pathway and is
failure, and exposure to heroin contaminated with clenbuterol (5,6). proportionate to Na1 and water absorption in this segment.
Tocolytic agents, such as ritodrine and terbutaline, in preterm labor
may be associated with severe hypokalemia by stimulating the cellu- Thick Ascending Limb of Henle (TALH). K1 reabsorption in the
lar uptake of potassium (7). TALH is mediated by the apical Na1, K1-2Cl2 cotransporter
In addition to factors that affect Na1, K1-ATPase, the distri- (NKCC2) (Figure 2). The K1 secretory channel on the apical mem-
bution of K1 across the cell membrane and the maintenance of the brane of the TALH recycles K1 at the apical membrane in series with
transmembrane potential are also regulated by Na1/H1 exchanger operation of NKCC2, generating a lumen-positive potential difference
(NHE1) and the K1 channel. NHE1 increases Na1 entry into the “potassium leak,” which is an important driving force for the passive
cell and thereby secondarily increases the activity of the Na1, K1- reabsorption of K1, Ca12, and Mg12 through the paracellular path-
ATPase. Aldosterone affects the transcellular distribution of K1 in way. Because solute is reabsorbed avidly, but water is not, the ascend-
muscle cells by increasing the activity of the NHE1. ing limb of the loop of Henle is characterized by a progressive decline
in the concentration of Na1, Cl2, and K1 (thus, the designation
“cortical diluting segment”). On arrival of tubule fluid at the distal
Renal Potassium Absorption and Secretion
convoluted tubule 1 (DCT1), most (approximately 90%) of the fil-
The ability to conserve potassium is limited. As a result, even with tered load of K1 has already been absorbed so that only approximately
extreme dietary K1 restriction, although the urine [K1] declines sig- 10% of the filtered load of K1 reaches the distal nephron (Figure 1).
nificantly, it is not rendered K1 free. This contrasts with the ability
of the kidney to diligently conserve Na1 during extreme dietary Distal Convoluting Tubule (DCT). The DCT1 segment is charac-
Na1 restriction. In contrast, with higher dietary intake of K1, the terized by the emergence of the sodium-chloride cotransporter (NCC)
kidney exhibits extraordinary capacity to excrete excess K1 to main- on the apical membrane, which is responsible for Na1 and Cl2
tain potassium balance. absorption as well as the delivery of solute to DCT2 and the collect-
ing duct (Figure 3). NCC is inhibited specifically by thiazide diuretics,
Proximal Tubule. The majority of the filtered load of K1 is reab- and its function is diminished in Gitelman syndrome through a loss-
sorbed in the proximal tubule and the thick ascending limb of of-function mutation of NCC. Gitelman syndrome is characterized
Henle (TALH). ECF K1 is freely filtered by the glomerulus by severe hypokalemia, metabolic alkalosis, salt craving, hypomagnese-
and then avidly reabsorbed by the proximal tubule and TALH mia, and hypocalciuria and is discussed at greater length below.
so that only a small amount of the daily filtered load of K1 reaches Changes in dietary potassium intake and the plasma [K1]
the distal nephron (Figure 1). Reabsorption in the proximal tubule evoke changes in NCC-mediated NaCl transport in DCT1 and
104 Nephrology Self-Assessment Program - Vol 20, No 2, January 2022

Secretion DCT2,
Reabsorption PCT CNT, CD

Filtered Load K+
10% Filtered Load
700 mEq/day
K+ to DCT1

Reabsorption
Reabsorption CD:
TALH
H+, K+-ATPase

Normal [K+] Hypokalemia


Excrete 10 –15% of Excrete <1% of
Filtered Load Filtered Load

Figure 1. Segmental potassium transport: 80%–90% of filtered load is reabsorbed in proximal convoluted tubule (PCT) and thick ascending
limb of Henle (TALH); approximately 10% of filtered load is delivered to distal convoluted tubule 1 (DCT1). Secretion is highly regulated to
control potassium excretion that occurs predominantly in aldosterone-responsive cells of distal convoluted tubule 2 (DCT2), connecting tubule
(CNT), and collecting duct (CD). H1, K1-ATPase secretes H1 and reabsorbs K1, and is upregulated by hypokalemia (see text).

subsequently, epithelial sodium channel (ENaC) activity in kidney has significantly enhanced our understanding of the regula-
aldosterone-responsive cells in late distal tubule (DCT2) and cortical tory mechanism of overall K1 homeostasis by the kidney.
collecting duct, which adjust Na1 absorption and thereby, K1
secretion by renal outer medullary K1 (ROMK) and big potassium Potassium Absorption in Distal Convoluted Tubule 2
(BK) channels channels (8). (DCT2) and Collecting Duct (CD) by the H1, K1-ATPases
Recent studies from the laboratory of Wang et al. (9) showed
Two isoforms of hydrogen-potassium ATPase a-subunits (H1, K1-
that a specialized K1 conductance channel on the basolateral mem-
ATPases) are expressed in the apical membrane of a-intercalated cells of
brane of DCT1 (Kir4.1) (Figure 3) acts as an intrarenal sensor of
the collecting duct, HKa1 (gastric) and HKa2 (colonic), and are
[K1]p and forms a heterotetramer (Kir4.1/Kir5.1). The Kir4.1/Kir5.1
robustly upregulated by dietary K1 deficiency in experimental animal
heterotetramer signals through WNK kinases (WNK4, SPAK, and
models (Figure 4) (12). The upregulation of these ATPases represents
OSR1), forming “WNK bodies” that modify NCC activity. Accord-
ingly, Na1 absorption and K1 secretion in the more distal nephron important physiologic adaptive responses, which occur in synchrony
segments (DCT2, CNT, and CCD) are in response to changes in with a decrease in the activity of the ROMK and BK potassium secre-
Na1 delivery to the aldosterone-responsive principal cells. For exam- tory channels to maximally conserve K1 in hypokalemic states.
ple, low-K1 intake activates Kir4.1 channel activity, which stimulates Pregnancy is an important example of physiologic adaptation
the WNK-SPAK kinase pathway and increases NCC activity in by the kidney to maximally conserve K1. It is widely recognized that
DCT1 cells. When there is a reduction in [K1]p, the WNK kinases gestational K1 retention, typically occurring in late pregnancy, is essen-
phosphorylate the downstream kinases SPAK and OSR1, which phos- tial for fetal development. West et al. (13) recently reported synergistic
phorylate and activate NCC. The increase in NCC activity causes an decreases in the cellular expression of potassium secretory channels
increase in Na1 and Cl2 reabsorption by DCT1 that decreases Na1 ROMK and BKa in a rodent model of late pregnancy. These changes
delivery downstream to aldosterone-sensitive cells. The reduction in were concomitant with increases in the mRNA expression of the colonic
Na1 delivery to DCT2 and CD reduces Na1 absorption by ENaC, subunit of the H1, K1-ATPase (HKa2). A decrease in expression of
which reduces transepithelial voltage and decreases the driving force the BK channel as well as a parallel increase in HKa2 was also observed
for K1 secretion by ROMK and BK potassium channels (10). This in the distal colon. The combination of changes in the kidney and colon
series of events protects the [K1]p from further decline during hypo- during pregnancy is consistent with avid potassium retention (13).
kalemia by reducing the secretion of K1 by principal cells.
This K1-sensing and signaling pathway is supported by con-
struction of a kidney-specific loss-of-function mutation of Kir4.1 in
Role of Hypokalemia in Maintenance of
mouse kidney that recapitulates the classical features of Gitelman Metabolic Alkalosis
syndrome (e.g., hypokalemia and hypomagnesemia) (11). Apprecia- In hypokalemic, hypochloremic metabolic alkalosis, the upregula-
tion of the sensor mechanism function of Kir4.1/Kir5.1 in the tion of HKa2 by hypokalemia contributes to the maintenance of
Nephrology Self-Assessment Program - Vol 20, No 2, January 2022 105

H+ Cl –
Na+ K+
2K+
3Na+

Na+ Cl –
2Cl –
K+ Na+
3HCO3–
Cl –
HCO3 –

K+

Figure 2. Potassium reabsorption in the thick ascending limb of Henle (TALH) is driven by the (1) transepithelial potential as a result of
operation of the apical Na1-K1-2Cl2 cotransporter (NKCC2) in parallel with the renal outer medullary K1 (ROMK) secretory channel (leak
pathway). Transport is passive through the paracellular pathway along with Mg12 and Ca12.

metabolic alkalosis because of continual secretion of H1 into the higher value for [K1] in serum is because clot formation in collected
tubule fluid. The secretion of H1 by HKa2, in response to the pre- blood samples causes release of [K1] from platelets.
vailing hypokalemia, is maladaptive for the prevailing metabolic The reporting of “normal” values for plasma or serum [K1]
alkalosis because it compromises the ability of the kidney to excrete diverges somewhat according to the reporting laboratory, especially for
the excess bicarbonate load. Therefore, hypokalemia “maintains” the the upper and lower range values. Some reported high “normal values”
alkalosis and must be corrected to repair the metabolic alkalosis. (e.g., 5–5.5 mEq/L) are too generous. This review endorses a normal
Additionally, the hypokalemia increases the production and excre- range for [K1]p equal to 3.6–4.9 mEq/L (or millimoles). Clinicians are
tion of ammonium to increase urinary net acid excretion that also advised to become acquainted with the “normal” or reference range
serves to maintain the metabolic alkalosis until the hypokalemia is values for [K1]p in their local clinical chemistry laboratory.
corrected (14,15). This topic is discussed in greater depth in the Because the ECF content of K1 is relatively small and is sensi-
topic on alkalosis in this issue of nephSAP. tive to cell shifts, it follows that the plasma [K1] is not a reliable
index of TBK. For example, in hypokalemia, a plasma [K1] of
Clinical Approach to the Hypokalemic Patient either 3 or 2 mmol/L is approximately equal to a TBK deficit of
The concentration of K1 in plasma is maintained in a narrow range 200 or 500 mmol, respectively.
(3.6–4.5 mEq/L), although published normal ranges vary. Hypokale- Hypokalemia is reported in approximately 3% of unselected
mia is defined as a [K1]p ,3.5 mEq/L, whereas a [K1]p of ,2.5 patients admitted to the hospital; however, 20% may develop
mEq/L should be considered severe, as values below this limit may hypokalemia during admission as a consequence of infection or
be associated with muscle weakness, paralysis, cardiac arrhythmia, prescribed medication, especially diuretics, or preexisting malnu-
and rhabdomyolysis. Values between 2.5 and 3.0 indicate substantial trition, alcoholism or eating disorders, and fever. Although often
hypokalemia requiring diagnosis and treatment. Values for [K1]p are modest in severity, hypokalemia can be life threatening. Hypoka-
0.5 mEq/L lower than serum values in the absence of hemolysis. The lemia represents a medical emergency when either cardiac
106 Nephrology Self-Assessment Program - Vol 20, No 2, January 2022

Na+
NCC 2K+
Cl–
3Na+

K+
Kir4.1

Cl–

ClC-K2

DCT1

NCC inhibited by thiazides and inactive in


Gitelman syndrome

Figure 3. Distal convoluting tubule 1 (DCT1) is the site of Na1-Cl2 cotransport and regulation of sodium-chloride cotransporter (NCC) activ-
ity, and is crucial in the regulation of K1 secretion in downstream segments through delivery of Na1 and Cl2 distally to the aldosterone-
responsive distal tubule (ARDT) of the distal convoluting tubule 2 (DCT2), connecting tubule (CNT), and collecting duct (CD). The sensor
for feedback regulation of plasma [K1] is Kir4.1 on the basolateral membrane of the DCT1 cell. With hypokalemia, Kir4.1 forms tetrameric
Kir4.1/5.1 to phosphorylate WNK and SPAK/OSR1 kinases that then phosphorylate NCC and the Na1, K1-ATPase. Activation of NCC
increases Na1 and Cl2 cotransport by DCT1, therefore reducing the delivery of Na1 downstream and subsequently limiting K1 secretion by
the ARDT. The opposite series of events occurs with an increase in the [K1]p (see in this nephSAP issue: Terker, AS; Ellison, DH: Pathophysi-
ology, Evaluation and Treatment of Hyperkalemia. nephSAP V20N2, Article 4).

arrhythmia or muscle weakness is detected. Patients with moder- Low Dietary K1 Intake
ate or severe acute hypokalemia with symptoms such as muscle Dietary potassium restriction is unusual as a solitary cause of chronic
weakness or cardiac arrhythmia require intravenous administra- hypokalemia, but it can occur with starvation and malnutrition or
tion of KCl emergently. chronic avoidance of potassium-rich foods as is common among
Transient hypokalemia is usually due to cell shifts, whereas sus- patients with CKD.
tained hypokalemia suggests inadequate dietary intake or excessive Potassium intake is generally low among adults in the United
K1 loss from the kidney or gastrointestinal tract. The clinical history States. Although the recommended daily dietary intake of potassium
and physical examination with careful assessment of ECF volume for healthy adults is approximately 4700 mg, several studies have
status, drug and dietary history, and determination of the acid-base revealed that ,2% of United States adults reach this goal (16). Food
status will allow the etiology of the hypokalemia to be ascertained in insecurity is increasingly recognized as a cause of mild or borderline
most cases (Flowchart 1). hypokalemia because of poor access to a healthy diet that includes
daily intake of fruits and vegetables, and this is associated with poor
Causes of Hypokalemia cardiovascular outcomes (17,18) and development of CKD (19–22).
Hypokalemia may be caused by low potassium intake, redistribu- As noted previously, human kidneys do not conserve potassium
tion, extrarenal potassium loss, and renal potassium loss. effectively even in the face of a chronically low average daily intake.
Nephrology Self-Assessment Program - Vol 20, No 2, January 2022 107

alcohol abuse given its association with QT prolongation and ven-


4 Dietary potassium intake evokes coordinated tricular arrhythmias (24). Such changes have been suggested to play
mechanisms that maintain extracellular potas- a role in the increased risk of sudden death reported in patients with
sium concentration within a narrow range. The chronic alcohol abuse.
sensor(s) may include the extracellular potassium Redistribution of Potassium
concentration itself or enteric sensors. As noted previously, the cellular uptake of potassium into cells is
4 Decreased potassium intake and [K1]p increase promoted by insulin, B-adrenergic stimulation, xanthines, and alka-
activity of the thiazide-sensitive cotransporter lemia. Pathologic stimulation of b2-receptors can result in symp-
tomatic hypokalemia. A common example is the severe symptomatic
(NCC), which decreases potassium secretion by
hypokalemia (including hypokalemic paralysis) that is observed with
ENaC. Decreases in ROMK and BKa and increases the hyperadrenergic state that accompanies alcohol withdrawal syn-
in the H1, K1-ATPase likely conserve potassium dromes (25). Another example of hypokalemia from pathologic
in late pregnancy. b-adrenergic stimulation is with the drug clenbuterol, which is a
rapidly acting b2-adrenergic agonist with prolonged duration that is
approved in veterinary medicine. Hypokalemia has been reported in
Some have speculated that the inefficient conservation of potassium association with clenbuterol used illicitly to increase muscle mass or
results from the evolution of kidneys during the Paleolithic era cut heroin (26).
when the diet of our hunter-gatherer ancestors provided as much as Hypokalemic periodic paralyses include acquired (i.e., associated
15,000 mg of potassium per day (21). with thyrotoxicosis) and inherited forms. The inherited disorders are
Low potassium intake of itself has been correlated with hyper- characterized by muscle weakness or paralysis due to the sudden move-
tension, mortality, and cardiovascular risk, possibly because a higher ment of K1 into cells and caused by channelopathies (of calcium or
potassium intake tends to counteract the effects of a high sodium sodium channel). All inherited forms are autosomal dominant with
intake on these outcomes (17,18,20). The studies are discussed incomplete penetrance. The most commonly observed mutation is in
below in the section on complications and outcomes. the muscle calcium channel a-1 subunit gene (CACNA1S). The a-1
subunit is the pore for movement of calcium into the T tubule and
Potassium Intake in CKD Patients contains the dihydropyridine binding site. Mutations reduce the cal-
As in the non-CKD population, hypokalemia in CKD patients usu- cium current. Reduced sarcolemma ATP-sensitive K1 channel activity
ally indicates chronic nutritional deficiency (inadequate intake of underlies the altered K1 fluxes through muscle cells and depolarization
fruit and vegetables) or a complication of drug (e.g., diuretic) ther- of fibers (27). Less common mutations involve the gene encoding the
apy. Because hypokalemia also carries a higher risk for more rapid skeletal muscle sodium channel (SCN4A) and the K1 channel subunit
progression of CKD, with or without diuretic use, recognition of gene KCNE3 (R83H mutation), which cause the abnormal sodium
inadequate dietary intake is important (22). channel to create a gating pore in the voltage sensor, allowing cation
There are no randomized clinical trials that have identified the leakage in the resting state (28).
ideal daily intake of potassium or even the ideal plasma [K1] for Patients with familial hypokalemic periodic paralysis are
patients with CKD at any specific stage. Unfortunately, patients most characteristically men age 20–40 who present with weak-
with even early-stage CKD are often instructed, inappropriately, to ness in the setting of conditions associated with release of cate-
adhere to a “renal failure or kidney diet” and restrict dietary K1. cholamines or insulin, such as stress, high-carbohydrate meals,
Such a broad generalization may be detrimental. For most hyperten- and exercise. Patients with the familial form are typically younger
sive patients with stage 1 or 2 CKD, we recommend a daily intake than those with acquired hypokalemic paralysis. Attacks often
of 4 g of K1 per day (or 102 mmol) and do not recommend dietary occur during the rest period immediately following exertion and
potassium restriction until kidney disease is more advanced (stage may be preceded by muscle cramps and aches. Hypophosphate-
3b and greater). However, dietary K1 should be restricted in mia and hypomagnesemia may occur with acute attacks and
patients with earlier-stage CKD who have a history of documented result from transcellular shifts into the intracellular compart-
hyperkalemia (e.g., hyporeninemic hypoaldosteronism). The mis- ment. The carbonic anhydrase inhibitor, acetazolamide, reduces
match between nutritional requirements encoded into the human the number of attacks in patients with familial hypokalemic
genome during evolution and the current substandard average daily paralysis, possibly related to the induction of metabolic acidosis.
intake of K1-rich foods is incriminated in various chronic diseases, However, although generally effective, a small number of patients
such as hypertension, cardiovascular disease, osteoporosis, and neph- given acetazolamide may demonstrate an exacerbation of symp-
rolithiasis. An increase in dietary intake of K1 is associated with toms (29). The variability of response to acetazolamide may be
reduced BP, decreased risk of stroke, improved bone health, and dependent on the specific mutation. One hypothesis is that the
reduced risk of nephrolithiasis (23). specific mutation (as an example, the R83H mutation in the K1
Alcohol abuse is a common cause of hypokalemia, as well as channel subunit gene KCNE3) decreases channel conductance,
hypomagnesemia and hypophosphatemia. Although starvation is and the decrease in conductance is made worse by low pH. This
clearly a feature of hypokalemia associated with alcohol abuse, alco- hypothesis would explain why the paralytic attacks occur after
hol per se can also adversely affect skeletal muscle membrane integ- exercise (because skeletal muscle intracellular pH drops with
rity. Hypokalemia is of particular concern among individuals with exertion) and why symptoms worsen after acetazolamide.
108 Nephrology Self-Assessment Program - Vol 20, No 2, January 2022

Flowchart 1. Flowchart: Approach to diagnosis of hypokalemia.


Reprinted with permission from reference 15 (Hamm L, DuBose TD Jr: Disorders of acid-base balance. In: Brenner & Rector's The Kidney,
11th Ed., edited by Yu A, Chertow GM, Luyckx VA, Marsden PA, Skorecki K, and Taal MW, Philadelphia, Elsevier, 2020, pp 559–592).

The acquired form of hypokalemic periodic paralysis occurs in The acute attacks are treated with intravenous KCl in
association with thyrotoxicosis, with the highest incidence observed nondextrose-containing saline solutions and nonselective b-adrenergic
among Asians and men (30). Patients present with a sudden onset of blockers. Dextrose will cause insulin release and worsen K1 shifts.
generalized weakness, which can last hours to days and is accompanied Patients may develop rebound hyperkalemia, which may be severe
by signs and symptoms of thyrotoxicosis (low TSH, high T4 or T3, and life threatening if KCl is given in excess or too rapidly. One
tachycardia, fine tremor, weight loss, and heat intolerance). The age of approach is to reduce or stop intravenous administration and switch
onset is older than with the familial form, typically after 20 years. The to oral KCl 20 mEq every 2 hours with a maximum dosage of 90
potassium fluxes are provoked by increased release of catecholamines mEq in 24 hours while continuing to monitor the [K1]p. Propranolol
or insulin and observed in settings such as stress, high-carbohydrate both inhibits the peripheral conversion of T4 to T3 and blocks cate-
meals, and rest after exercise. Hypophosphatemia and hypomagnese- cholamines, and it is continued until hypokalemia is corrected. Treat-
mia are common during acute attacks. The underlying mechanisms ment includes antithyroid drugs until definitive treatment (such as
may be that excess thyroid hormone during thyrotoxicosis increases radioactive iodide or thyroidectomy) is provided. Periodic paralysis
Na1, K1-ATPase activity, which drives K1 intracellularly. does not recur after the patient becomes euthyroid.
Nephrology Self-Assessment Program - Vol 20, No 2, January 2022 109

Lumen Interstitial space

AE1
H+-ATPase Cl–
H+ HCO3–
2
1
H+ + HCO3–
H+ H+
4
3 CA II

CO2 + H2O KCC4


H+
5 Cl–
K+ K+
H+, K+-ATPase

Figure 4. Examples of cellular transport abnormalities causing nongap metabolic acidosis of classical dRTA (classical dRTA or type 1 renal tubu-
lar acidosis [RTA]). (1) Abnormality of the gene (SLC4A1) that encodes the Cl2/HCO32 exchanger expressed on the basolateral membrane may
be inherited as either an autosomal dominant or autosomal recessive defect. The latter is associated with early-onset sensorineural deafness, and
the severity of the metabolic acidosis is less than for defects of the H1-ATPase. (2) Abnormality of one of the genes (ATP6V0A4) that encodes
either the A1 subunit of the H1-ATPase, usually associated with deafness, or ATPV1B1 that encodes the B1 subunit. Either may be inherited
(autosomal dominant). Most adult patients with the diagnosis of dRTA have acquired dRTA. One example of acquired dRTA is caused by auto-
antibodies in Sj€ogren syndrome (dry eyes and dry mouth), manifest as (a) lymphocytic infiltration of the salivary gland and impairment of salivary
secretion and (b) NAG acidosis and hypokalemia with defective H1 secretion. The latter is because autoantibodies associated with Sj€ogren syn-
drome disrupt the insertion of the H1-ATPase into the apical membrane, causing a marked decrease in H1 secretion. The clinical features of
dRTA are reviewed in the text. (3) Carbonic anhydrase II deficiency is autosomal recessive abnormality of the CA2 gene that encodes the intracel-
lular enzyme present in the proximal and distal renal tubules and in other organs. It is associated with RTA, osteopetrosis, mental retardation,
recurrent bone fractures, growth failure, and recurrent nephrolithiasis. (4) Permeability defect or “gradient defect” (e.g., usually acquired by
administration of amphotericin B [antifungal agent]; may be inherited rarely). (5) Abnormality of H1, K1-ATPase.

Extrarenal K1 Loss polypeptide, which causes a watery diarrhea syndrome similar to


Extrarenal potassium loss may be cutaneous or gastrointestinal. that of vasoactive intestinal polypeptideoma, and with excessive
Cutaneous loss of K1 sufficient to cause hypokalemia may occur in secretion of K1 by a villous adenoma (33).
the setting of intense exercise in a hot humid environment (31) Ogilvie syndrome is characterized by diarrhea, hypokalemia due
when large volumes of sweat can be lost each day. to high excretion of K1 in stool, and a dilated colon and has been
Gastrointestinal loss is a common cause of hypokalemia and is reported in elderly patients after nongastrointestinal surgery. Mea-
observed with high-volume secretory diarrhea. Hypokalemia has also surement of the rectosigmoid potential difference in a patient with
been reported with the paraneoplastic syndrome of watery diarrhea, Ogilvie syndrome was reported as 213 mV (lumen negative) (34),
hypokalemia, and achlorhydria in patients with neuroendocrine suggesting active K1 secretion along with active Na1 reabsorption
tumor and is related to secretion of vasoactive intestinal polypep- across the colonic mucosa. The pseudo-obstruction resolves slowly
tide (32). Hypokalemia may also be observed with pancreatic after K1 replacement.
110 Nephrology Self-Assessment Program - Vol 20, No 2, January 2022

As in the kidney, K1 is both absorbed and secreted in the tubular acidosis [RTA]), ketoanions (as in ketoacidosis), and penicil-
colon. Colonic K1 absorption occurs via the colonic isoform of lin derivatives. These conditions are usually associated with normal
H1, K1-ATPase (HKa2), which is upregulated by hypokalemia as or low volume status and BP. Poorly reabsorbed anions increased
in the collecting duct (35). K1 secretion occurs via a luminal the electronegativity of the lumen, which increases the driving force
calcium-activated BK potassium channel that is upregulated by high for potassium excretion into the lumen.
dietary K1 intake (36,37). The evaluation of these disorders requires assessing the ratio of
Bowel cleansing solutions are occasionally associated with hypo- the plasma aldosterone concentration to plasma renin activity. Imag-
kalemia, hyperphosphatemia, hypocalcemia, and hypernatremia, ing of the renal arteries (doppler, CTA, or MRA), renal vein sam-
particularly in older individuals or those with the presence of bowel pling, and adrenals imaging may yield a diagnosis. In the setting of
obstruction, poor gut motility, and kidney disease. low renin and aldosterone, genetic analysis is required to
differentiate.

Increased Renal K1 Excretion Hypokalemia and Nongap Metabolic Acidosis


There are generally two causes of increased kidney potassium excre- The defect in proximal RTA (type 2 RTA) impairs bicarbonate
tion. These include increased activity of ENaC and increased reabsorption in the proximal tubule and is manifest as a reduction
sodium delivery to the distal nephron. in the threshold for bicarbonate reabsorption causing self-limited
Increased ENaC activity may be due to increased aldosterone bicarbonaturia, nongap metabolic acidosis, hypokalemia, and vol-
(hyperaldosteronism) or be aldosterone independent. Hyperaldosteron- ume depletion. Secondarily, volume depletion activates the RAAS.
ism may be secondary to increased renin or primary with suppressed Increased aldosterone levels coupled with increased distal delivery of
renin concentration. Increased ENaC activity in the absence of increased Na1 and HCO32 cause renal K1 wasting; additionally, HCO32
renin or aldosterone is associated with low-renin and low-aldosterone behaves as a poorly reabsorbed anion in the distal nephron (15). In
concentrations. The conditions are often classified as high renin/high the steady state when the filtered load of HCO32 declines to the
aldosterone, low renin/high aldosterone, and low renin/low aldosterone proximal tubule threshold level (15–17 mEq/L), this reduced
(Table 2 and Flowchart 1). All conditions are associated with volume amount can be absorbed, and bicarbonaturia subsides; consequently,
expansion, hypertension, and usually, hypokalemia and alkalosis. renal K1 secretion also declines. Nevertheless, most patients with
High renin/high aldosteronism is caused by renal artery stenosis proximal RTA require chronic K1 replacement. Unfortunately,
and less commonly, renin-secreting tumor. Low renin/high aldoste- treatment of the metabolic acidosis with bicarbonate improves the
ronism is due to adrenal adenoma, bilateral adrenal hyperplasia, and acidosis but exacerbates the degree of hypokalemia by enhancing K1
glucocorticoid-remediable aldosteronism. excretion because of higher distal Na1 and HCO32 delivery. There-
Low renin/low aldosterone causes include Cushing disease or fore, a combination of potassium and base precursor may be
syndrome, 11b-hydroxylase deficiency, 17a-hydroxylase deficiency, provided as potassium citrate (K1-Shohl solution or K1-citrate)
Liddle syndrome, and decreased activity of 11b-hydroxysteroid in proximal RTA.
dehydrogenase enzyme type II (either familial due to autosomal Most patients with classical distal RTA present with a nongap
recessive inheritance or acquired due to licorice/glycyrrhizin acid). metabolic acidosis with hypokalemia (type 1 RTA). Distal RTA
may be the result of an inherited (autosomal recessive) defect in the
Increased Distal Sodium Delivery genes that encode acid-base transporters in the distal nephron. For
Conditions that increase sodium delivery include diuretics, poorly example, either of two subunits of the apical H1-ATPase transporter
reabsorbed anions such as bicarbonate (vomiting or type 2 renal (ATP6V0A4 or ATPV1B1) of a-intercalated cells may be defective

Table 2. Hypokalemia due to increased epithelial sodium channel (ENaC) activity

Serum Renin and Aldosterone Levels Causes

High renin/high aldosterone Renal artery stenosis


Hemangiopericytoma
Low renin/high aldosterone Adrenal adenoma
Bilateral adrenal hyperplasia
Glucocorticoid remediable aldosteronism
Low renin/low aldosterone 11b-hydroxylase deficiency
17a-hydroxylase deficiency
Liddle syndrome
Apparent mineralocorticoid excess (familial due to
11b-hydroxysteroid dehydrogenase II deficiency)
Apparent mineralocorticoid
Excess (acquired) due to licorice/glycorrhyzic acid
Cushing disease
Nephrology Self-Assessment Program - Vol 20, No 2, January 2022 111

(Figure 4). Patients who present in adolescence, with an autosomal acidosis, should be 220 mEq/L or more. In contrast, failure to
dominant pattern of inheritance and often with accompanying sen- increase NH41 excretion in the face of a nongap metabolic acidosis is
sorineural hearing loss, typically have a defect in SLC4A1 (the gene a sentinel feature of RTA and is implied by a positive UAG: U
that encodes the basolateral HCO32/Cl2 exchanger [SLC26A7] in [Na11K1].U [Cl2]. Unfortunately, calculation of the UAG failed
the outer medullary collecting duct). In addition to nongap meta- to consistently represent the measured urine [NH41] in the AASK
bolic acidosis, the phenotype includes volume depletion, growth trial (41). The concentration of urine NH41 can be estimated from
retardation, osteomalacia, nephrolithiasis, and secondary hyperaldos- the urine osmolar gap (UOG) and is equal to the UOG/2. The
teronism with increased renal K1 excretion. The most common UOG is the difference between the measured urine osmolality and
acquired form of distal RTA is that associated with Sj€ogren syn- the calculated urine osmolality. Estimated urine [NH41] concentra-
drome. The autoantibodies generated in Sj€ogren syndrome interfere tions exceeding 75 mEq/L or more are anticipated if renal tubular
with the normal trafficking of the H1-ATPase to the apical mem- function is intact and may be more accurate than the UAG. Never-
brane, thus preventing its activation. Kidney biopsies of Sj€ogren syn- theless, the most accurate analysis is to request the clinical laboratory
drome patients with classical dRTA that have been immunostained to measure the urine [NH41]. Unfortunately, some clinical laborato-
specifically for the H1-ATPase antibody have revealed coalescence ries resist performing urine [NH41] analysis, although urine [NH41]
of the H1-ATPase in intracellular compartments (38). A third can be measured accurately if the urine sample is diluted appropriately
potential defect causing dRTA involves loss of function of the H1, (1:100) for analysis using a standard autoanalyzer ammonia assay
K1-ATPase that would impair K1 reabsorption in the collecting (42). We recommend measurement of urine ammonium concentra-
duct to increase renal K1 excretion. Such an abnormality has been tion by clinical laboratories rather than reliance on imprecise surro-
proposed as an explanation for the combined presentation of severe gates, such as the UAG or the UOG, which when compared with
hypokalemia and nongap metabolic acidosis in an infant, but a measured values, do not correlate reliably or reproducibly with actual
defect in the H1, K1-ATPase per se has not been unequivocally measured values for urine [NH41] (41,43).
demonstrated (39). Finally, dRTA can be acquired as a result of a In summary, patients with hypokalemic nongap metabolic aci-
“gradient defect.” An example of this type of dRTA is that due to dosis in combination with an alkaline urine pH as well as evidence
nephrotoxicity caused by administration of amphotericin B. In of an abnormally elevated level of renal potassium excretion and a
amphotericin B induced distal RTA, K1 wasting is the result of an positive UAG should be suspected of having distal RTA (type 1
increase in permeability of the collecting duct a-intercalated cell RTA). Under normal circumstances, ammonium salts are the major
causing K1 to leak from the cell to the lumen increasing K1 secre- solutes contributing to urine osmolality. Pitfalls include other
tion, whereas the normally secreted H1 leaks back from the lumen osmoles, such as Hippurate anions that increase the UOG by .150
into the cell across the leaky apical membrane, consequently decreas- mosmol/kg with nongap metabolic acidosis. If there is a suspicion of
ing net H1 secretion (15). The locations of the cellular transporter toluene abuse, a UOG should be obtained. If the UOG is .150
abnormalities responsible for the various causes of classical dRTA mosmol/kg, the nonanion gap acidosis is due to the excretion of
are reviewed in Figure 4. All varieties of untreated classical distal other osmotically active substances (e.g., toluene intoxication with
RTA are associated with a decrease in urinary ammonium (NH41) Hippurate excretion).
excretion. Type 4 RTA, probably the most commonly observed
form of RTA, is usually a result of hyporeninemic hypoaldosteron- Hypokalemia with Metabolic Alkalosis, Volume
ism or obstructive nephropathy and the only type of RTA associated Depletion, and Normal or Low BP—High Urine [Cl]:
with hyperkalemia. Bartter Syndrome and Gitelman Syndrome
Toluene inhalation, although uncommon, may cause severe Bartter and Gitelman syndromes are hereditary disorders character-
hypokalemia concurrent with a nongap metabolic acidosis. Symp- ized by hypokalemic metabolic alkalosis.
toms are caused by the toluene metabolite, hippuric acid, and the Bartter syndrome is characterized by severe hypokalemia that may
renal excretion of its sodium salt. Chronic exposure may lead to a be associated with rhabdomyolysis and flaccid paralysis (15,44,45).
persistent dRTA (15). Toxicity from transdermal exposure was Five types of defects in transporters of the TALH can account for the
recently described in a case report (40). phenotypes of Bartter syndrome. Bartter syndromes are caused by
recessive mutations of transport proteins in the TALH cell involved in
The Urine Anion Gap and the Urine Osmolar Gap NaCl and K1 transport. The “type I defect” is of the SLC12A1 gene
The urine anion gap (UAG) is a clinical method to estimate NH41 that encodes NKCC2. The type II defect impairs the KCNJ1 gene
excretion and is commonly used in assessment of nongap metabolic that encodes the potassium channel on the apical membrane, ROMK.
acidosis. The UAG is zero to slightly positive in healthy individuals The type III defect impairs the CLCNKB gene that encodes the baso-
on a typical Western diet. The appropriate response by the normal lateral ClC-Kb channel. The type IV defect involves the BSND gene
kidney to an increase in dietary protein that increases net acid that encodes the intracellular accessory protein Bartten that binds ClC-
production is to significantly increase NH41 excretion. The UAG cal- Ka and ClC-Kb. Alternatively, type IV-B is represented by a combined
culated as the U[Cl2]–(U [Na1]1[K1]) is an arithmetic estimate of defect in CLCNKA and CLCNKB genes that encode both the ClC-Ka
urine [NH41], and by convention, a negative UAG indicates the and ClC-Kb channel proteins. The decreased NaCl and fluid reab-
presence of [NH41] in the urine. For example, the normal kidney sorption in the thick limb is similar to effects of a loop diuretic.
responds to nongap metabolic acidosis of nonrenal origin (e.g., diar- Types I and II Bartter syndrome are severe and usually present
rhea) by significantly increasing urine [NH41]. Therefore, the UAG as polyhydramnios in pregnancy or as hypokalemia and weakness in
is negative when ammonium is present in urine and with nonrenal premature infants. Affected infants with types 1 and 2 may have
112 Nephrology Self-Assessment Program - Vol 20, No 2, January 2022

transient hyperkalemia due to loss of function of ROMK in the col- hypomagnesemia-related paralysis and prolongation of the QT
lecting duct. However, hypokalemia occurs as infants mature as a interval with malignant arrhythmias (48,49). Consensus guidance
result of increased K1 secretion via maxi-K1 (or BK) channels in was published by Kidney Disease Improving Global Outcomes
the DCT and CD because of an increase in tubule fluid flow. This (KDIGO) in 2017 (50). The KDIGO group suggested that
model is supported by studies in an ROMK-deficient mouse model Gitelman syndrome be suspected in the setting of chronic hypo-
of type II Bartter syndrome (46). kalemia with urinary K wasting, metabolic alkalosis, hypomagne-
The most common phenotype, type III Bartter syndrome, is semia, hypocalciuria, high plasma renin, fractional chloride excre-
less severe than types I and II and presents later in life. The disease tion .0.5%, low or normal BP, and a normal ultrasound.
is due to a reduced activity of ClC-Kb (the basolateral Cl2 channel Although patients typically respond well to emergent K1 replace-
in the TALH) but with a degree of preservation of ClC-Kb activity. ment, chronic management can be challenging. Patients are typi-
Some patients exhibit features of Gitelman syndrome (hypomagne- cally managed chronically with potassium and magnesium sup-
semia and hypocalciuria). The specific transport defects for Bartter plementation, amiloride, and spironolactone. This regimen is
syndrome (TALH) and Gitelman syndrome are compared and con- usually successful, especially initially, but may be overwhelmed
trasted in Figure 5. because patients with Gitelman syndrome commonly crave salt.
Gitelman syndrome is caused by a mutation of SLC12A3, The increased salt and fluid intake enhances Na1 delivery to the
which encodes the thiazide-sensitive NCC in DCT1. Patients pre- distal nephron and drives K1 secretion by the unaffected CCT
sent with hypokalemia, hypomagnesemia, hypocalciuria, and normal principal cells. Therefore, in patients with salt craving and exces-
BP with mild hypovolemia. A 2006 report demonstrated complete sive dietary salt intake, reducing salt intake is needed because
absence of NCC immunostaining on renal biopsies of two individu- continued delivery of a high-NaCl load to the distal nephron and
als with Gitelman syndrome (15,47). collecting duct aggravates K1 secretion. Accordingly, we agree
Most patients with Gitelman syndrome come to the with the KDIGO recommendation that ad libitum salt intake
attention of nephrologists because of difficult hypokalemia with should be encouraged but only in patients with evidence of lower
severe muscle weakness. Gitelman syndrome has been asso- salt intake and not in patients with an established high-salt intake
ciated with severe complications, including hypokalemia- and (50).

A B

H+
Na+ Cl–
Na+ 2K+
K+ NCC
Cl– 3Na+

Na+
2Cl– NKCC2 2K+
K+ 3Na+
I

K+ Cl–
Cl–
II III

TALH Cell DCT Cell


Bartters Gitelmans

Figure 5. Contrasting transporter abnormalities responsible for Bartter syndrome versus Gitelman syndrome. (A) Bartter syndrome types I, II,
and III are inherited in the autosomal recessive manner. Genes responsible are I. SLC12A1, II. KCNJ1, and III. CLCNKB. (B) Gitelman syn-
drome is an inherited (autosomal recessive) defect of one of the genes (SLCl2A3 or CLCNKB) that encodes the protein NCC, the apical Na1/
Cl2 cotransporter.
Nephrology Self-Assessment Program - Vol 20, No 2, January 2022 113

Assessment of Renal Potassium Excretion


4 Increased potassium excretion by the kidney is
Measurement of renal K1 excretion confirms whether potassium
loss is renal or extrarenal. Renal K1 excretion can be measured via a caused by increased activity of ENaC and
24-hour urine collection, but it is often more convenient to measure increased sodium delivery to the distal nephron.
a spot urine K1/creatinine ratio. A spot urine K1/Cr ratio ,13 4 Increased sodium delivery results from diuretics
mEq/g (1.5 mEq/mmol) of creatinine suggests extrarenal loss of
potassium, whereas a urine K1/Cr ratio .13 mEq/g (1.5 mEq/ (or inherited disorders, such as Bartter and Gitel-
mmol) of creatinine suggests excessive K1 excretion by the kidney. man syndromes, or vomiting), poorly reabsorbed
Because the urine K1/Cr ratio can be affected by the rate of creati- anions, or type 2 RTAs.
nine excretion, a low body mass index or loss of muscle mass can
4 Bartter and Gitelman syndromes are caused by muta-
reduce the rate of creatinine excretion and falsely elevate the urine
K1/Cr ratio. tions of transporter proteins in the TALH loop and the
Use of the urine K1/Cr ratio was reported to differentiate thiazide-sensitive NCC in DCT1. There is overlap in
between 30 patients with hypokalemic periodic paralysis (redistribu- clinical presentation, and the diagnosis should be
tion hypokalemia) versus ten patients with nonhypokalemic periodic
confirmed by genetic testing when possible.
paralysis due to renal K1 loss (11 versus 36 mEq/g creatinine or 1.3
versus 4.1 mEq/mmol creatinine, respectively) (51).
The transtubular K1 gradient (TTKG) was previously pro-
movements and diarrhea that will increase potassium excretion by
posed to evaluate renal K1 excretion. Unfortunately, the assump-
the colon. Intravenous magnesium increases serum magnesium lev-
tions made in calculating the TTKG are not entirely correct. The
els, but the patient will lose 50% of the administered magnesium
first assumption is that there is only water reabsorption (i.e., without
into the urine.
solute transport) in the medullary collecting duct. The reabsorption
Amiloride reduces renal Mg12 wasting in patients with Gitel-
of any solute, such as Na1 or urea, would lower urine osmolality,
man or Bartter syndrome. Amiloride should only be given as amilor-
resulting in an overestimate of the gradient for K1 secretion in the
ide (Midamor) and not with a thiazide in combination. The dose of
upstream collecting duct. Second, for the TTKG to provide an
amiloride may be advanced slowly from 5 to 15–20 mg/d.
accurate estimate, conditions should be optimal for K1 secretion at
Angiotensin-converting enzyme inhibitors help reduce aldosterone
the time of measurement. Thus, the urine Na1 should be .25
levels and are used as adjunctive therapy, but because of possible
mEq/L to ensure adequate Na1 delivery to the collecting duct.
hypotension, they should be used in very small doses. In Bartter syn-
Urine osmolality should be equal to plasma osmolality. A higher
drome, nonsteroidal anti-inflammatory drugs have been used to
urine osmolality suggests the presence of circulating vasopressin,
which stimulates K1 secretion in the collecting duct. If these condi- ameliorate the effect of increased prostaglandin E2 in the loop of
tions are not met, the TTKG cannot be used reliably. Because of Henle.
these limitations and as clinicians obtain more experience with the For chronic treatment of hypokalemia in classical distal RTA,
urine K1/Cr ratio, the use of the TTKG will probably be discarded. administration of alkali as K1 citrate is preferred to correct both the
nongap acidosis and the hypokalemia. After the hypokalemia is cor-
Treatment of Hypokalemia rected, either sodium citrate (modified Shohl solution) or sodium
bicarbonate tablets (650 mg per tablet) may be initiated. With cor-
Potassium should be given either orally or intravenously as the KCl
rection of the acidosis, the hypokalemia often subsides, and K1
salt. Potassium citrate should not be used unless the patient has met-
administration may be discontinued in adults. Correction of acidosis
abolic acidosis (as in patients with proximal RTA). Potassium citrate
is important for normal growth in children to inhibit progression of
may increase K1 excretion because the citrate is converted to bicar-
CKD in both adults and children. Finally, with the increase in urine
bonate, which raises the plasma bicarbonate concentration and
citrate and decrease in urine [Ca12] excretion, the frequency of
increases excretion.
nephrolithiasis declines dramatically.
KCl should be given slowly (i.e., ,10 mmol/h in dilute saline
solution) to patients with extreme muscle weakness or cardiac
arrhythmias. To prevent rebound hyperkalemia, a conservative goal
is to increase the [K1]p to 3.4 mEq/L using intravenous KCl and Complications and Outcomes
then, to use oral KCl. In a 2007 survey of 140 hospitalized patients Hypokalemia can cause a variety of clinical manifestations due to
with hypokalemia, 16% developed therapy-induced hyperkalemia alterations in the excitability of neuromuscular tissues (Table 3). In
(52). In the face of life-threatening complications, more aggressive addition, both hypokalemia and decreased dietary potassium intake
KCl administration may be required using central intravenous have been associated with adverse long-term outcomes. Both acute
access, frequent measurement of [K1]p, and continuous electrocar- complications and long-term outcomes are discussed below.
diographic monitoring.
As noted previously, treatment of the hypokalemia due to with
Bartter or Gitelman syndrome involves both potassium and magne- Acute Complications
sium replacement. Oral magnesium replacement, at least initially, is Muscle Weakness. Muscle weakness is a common symptom of
preferable in asymptomatic patients. Oral magnesium therapy may hypokalemia and is caused by hyperpolarization of the cell mem-
not be tolerated because of the development of frequent bowel brane (53).
114 Nephrology Self-Assessment Program - Vol 20, No 2, January 2022

Rhabdomyolysis. Severe hypokalemia may cause rhabdomyolysis Table 4. Potential harmful effects of chronic hypokalemia
especially when associated with alcohol intoxication and alcohol- in CKD
associated malnutrition. A possible mechanism is that decreased
accumulation of K1 in the interstitial space limits blood flow to Harmful Effect
skeletal muscle and increases muscle breakdown. Chronic alcohol
abuse is also associated with acquired compromise of the integrity of Worsening of hypertension
skeletal cell muscle membrane and renders it susceptible to damage. Expansion of renal fibrosis and enhanced cyst formation
Acceleration of progression of CKD
Cardiac Arrhythmia Ventricular arrhythmias
Cardiac arrhythmias are seen in some patients with hypokalemia. In ESKD: Intradialytic hypotension
EKG changes include T-wave flattening, U waves, and ST-segment
depression. Hypokalemia causes direct electrophysiological effects
The mechanism of thiazide-induced hyperglycemia may be due
such as resting membrane hyperpolarization; reduced repolarization
to decreased insulin release from the pancreatic b-cell because ATP-
reserve; inhibition of Na1, K1 ATPase; and suppression of K1 con-
sensitive K1 channels modulate insulin secretion (59).
ductance channels, which predispose the myocardium to early after True hypokalemia is associated with higher rates of all-cause
depolarizations (54). mortality and higher rates of major cardiovascular events and hospi-
talization. Several studies have documented a “U-shaped” relation-
Long-Term Outcomes ship between the serum [K1] and mortality in patients with CKD,
Hypokalemic Nephropathy (Kaliopenic Nephropathy). Chronic cardiovascular disease, or diabetes; that is, higher mortality is
hypokalemia causes a tubulointerstitial lesion characterized by observed similarly for both hypokalemia and hyperkalemia in the
tubular atrophy and interstitial fibrosis, an interstitial macrophage lowest and highest ranges, respectively (60,61). In general urban
infiltrate and renal cysts (55). Affected patients have polyuria, pro- populations, mild chronic hypokalemia has been associated with a
teinuria, and often mildly decreased GFR. Kidney injury is caused higher incidence of hypertension, poorly controlled hypertension,
by ischemia, complement activation related to increased ammonia- cardiovascular disease, progression of established kidney disease, and
genesis, and effects of angiotensin II and endothelin. nephrolithiasis (Table 4) (62,63).
Studies have been conflicting in CKD patients. A recent analy-
sis of data from the Korean Cohort Study of outcomes in 1821
New-Onset Diabetes. Hypokalemia likely underlies the association
patients with CKD demonstrated a higher risk of CKD progression
between new-onset diabetes mellitus and use of thiazide diuretics. In
(defined as a 50% decrease in eGFR) among individuals with the
a systematic review including 50 trials that compared thiazides with
lowest quartile of urinary potassium to creatinine compared with
other drugs or placebo, hypokalemia was associated with the devel-
those in the highest quartile (64). These result are consistent with
opment of hyperglycemia, such that there was an approximately
some (65–67), although not all (68), previously published data. The
10-mg/dl increase in glucose for every 1-mEq/L decrease in K1 (56).
Chronic Renal Insufficiency Cohort study reported higher incidence
The prevention of hypokalemia with K1 supplements decreases
of ESKD in individuals with high potassium excretion (68). The
the risk of thiazide-induced glucose intolerance (57). More recently,
reason for conflicting results from these studies is not clear (20).
a randomized trial examined whether the addition or substitution of
amiloride to thiazide treatment would prevent the glucose intoler-
ance associated with thiazide diuretics (58). This trial included 145 References
patients assigned to amiloride, 146 assigned to hydrochlorthiazide, 1. DuBose TD Jr: Regulation of potassium homeostasis in CKD. Adv
and 150 assigned to combination therapy with amiloride and hydro- Chronic Kidney Dis 24: 305–314, 2017 PubMed
2. Youn JH: Gut sensing of potassium intake and its role in potassium
chlorthiazide. At 12 and 24 weeks, 2-hour glucose and oral glucose
homeostasis. Semin Nephrol 33: 248–256, 2013 PubMed
tolerance tests were significantly lower in the amiloride and the 3. McFarlin BE, Chen Y, Priver TS, Ralph DL, Mercado A, Gamba G,
amiloride/hydrochlorthiazide groups compared with the hydrochlor- et al.: Coordinate adaptations of skeletal muscle and kidney to maintain
thiazide group. extracellular [K1] during K1-deficient diet. Am J Physiol Cell Physiol
319: C757–C770, 2020 PubMed
4. Rabelink TJ, Koomans HA, Hene RJ, Dorhout Mees EJ: Early and late
Table 3. Consequences of severe hypokalemia
adjustment to potassium loading in humans. Kidney Int 38: 942–947,
1990 PubMed
Consequence 5. Wong CS, Pavord ID, Williams J, Britton JR, Tattersfield AE: Broncho-
dilator, cardiovascular, and hypokalaemic effects of fenoterol, salbutamol,
Severe muscle weakness—serum K1 ,3.0 mEq/L and terbutaline in asthma. Lancet 336: 1396–1399, 1990 PubMed
Acute rhabdomyolysis, myoglobinuria—serum K1 ,2.5 mEq/L 6. Clausen T: Hormonal and pharmacological modification of plasma
potassium homeostasis. Fundam Clin Pharmacol 24: 595–605, 2010
Cardiac arrhythmia
PubMed
Respiratory muscle weakness, respiratory arrest 7. Braden GL, von Oeyen PT, Germain MJ, Watson DJ, Haag BL: Rito-
Ileus drine- and terbutaline-induced hypokalemia in preterm labor: Mecha-
Acquired diabetes insipidus nisms and consequences. Kidney Int 51: 1867–1875, 1997 PubMed
8. Thomson MN, Cuevas CA, Bewarder TM, Dittmayer C, Miller LN, Si
Kaliopenic nephropathy
J, et al.: WNK bodies cluster WNK4 and SPAK/OSR1 to promote
Nephrology Self-Assessment Program - Vol 20, No 2, January 2022 115

NCC activation in hypokalemia. Am J Physiol Renal Physiol 318: F216– 28. Jiang D, Gamal El-Din TM, Ing C, Lu P, Pomes R, Zheng N, et al.:
F228, 2020 PubMed Structural basis for gating pore current in periodic paralysis. Nature 557:
9. Wang MX, Cuevas CA, Su XT, Wu P, Gao ZX, Lin DH, et al.: Potas- 590–594, 2018 PubMed
sium intake modulates the thiazide-sensitive sodium-chloride cotrans- 29. Statland JM, Fontaine B, Hanna MG, Johnson NE, Kissel JT, Sansone
porter (NCC) activity via the Kir4.1 potassium channel. Kidney Int 93: VA, et al.: Review of the diagnosis and treatment of periodic paralysis.
893–902, 2018 PubMed Muscle Nerve 57: 522–530, 2018 PubMed
10. Zhang C, Wang L, Su XT, Zhang J, Lin DH, Wang WH: ENaC and 30. Kung AW: Clinical review: Thyrotoxic periodic paralysis: A diagnostic
ROMK activity are inhibited in the DCT2/CNT of TgWnk4PHAII mice. challenge. J Clin Endocrinol Metab 91: 2490–2495, 2006 PubMed
Am J Physiol Renal Physiol 312: F682–F688, 2017 PubMed 31. Knochel JP, Dotin LN, Hamburger RJ: Pathophysiology of intense phys-
11. Malik S, Lambert E, Zhang J, Wang T, Clark HL, Cypress M, et al.: ical conditioning in a hot climate. I. Mechanisms of potassium depletion.
Potassium conservation is impaired in mice with reduced renal expression J Clin Invest 51: 242–255, 1972 PubMed
of Kir4.1. Am J Physiol Renal Physiol 315: F1271–F1282, 2018 PubMed 32. Onozawa M, Fukuhara T, Minoguchi M, Takahata M, Yamamoto Y,
12. Codina J, DuBose TD Jr: Molecular regulation and physiology of the Miyake T, et al.: Hypokalemic rhabdomyolysis due to WDHA syndrome
H1,K1 -ATPases in kidney. Semin Nephrol 26: 345–351, 2006 caused by VIP-producing composite pheochromocytoma: A case in
PubMed neurofibromatosis type 1. Jpn J Clin Oncol 35: 559–563, 2005 PubMed
13. West CA, Welling PA, West DA Jr, Coleman RA, Cheng KY, Chen C, 33. Lepur D, Klinar I, Mise B, Himbele J, Vranjican Z, Barsic B: McKit-
et al.: Renal and colonic potassium transporters in the pregnant rat. Am J trick-Wheelock syndrome: A rare cause of diarrhoea. Eur J Gastroenterol
Physiol Renal Physiol 314: F251–F259, 2018 PubMed Hepatol 18: 557–559, 2006 PubMed
14. Codina J, Delmas-Mata JT, DuBose TD Jr: Expression of HKalpha2 34. van Dinter TG Jr, Fuerst FC, Richardson CT, Ana CA, Polter DE, For-
protein is increased selectively in renal medulla by chronic hypokalemia. dtran JS, et al.: Stimulated active potassium secretion in a patient with
Am J Physiol 275: F433–F440, 1998 PubMed colonic pseudo-obstruction: A new mechanism of secretory diarrhea. Gas-
15. Hamm L, DuBose TD Jr: Disorders of acid-base balance. In: Brenner & troenterology 129: 1268–1273, 2005 PubMed
Rector’s The Kidney, 11th Ed., edited by Yu A, Chertow GM, Luyckx VA, 35. Codina J, Pressley TA, DuBose TD Jr: Effect of chronic hypokalemia on
Marsden PA, Skorecki K, and Taal MW, Philadelphia, Elsevier, 2020, pp H(1)-K(1)-ATPase expression in rat colon. Am J Physiol 272: F22–F30,
559–592 1997 PubMed
16. Cogswell ME, Zhang Z, Carriquiry AL, Gunn JP, Kuklina EV, Saydah 36. Sausbier M, Matos JE, Sausbier U, Beranek G, Arntz C, Neuhuber W,
SH, et al.: Sodium and potassium intakes among US adults: NHANES et al.: Distal colonic K(1) secretion occurs via BK channels. J Am Soc
2003-2008. Am J Clin Nutr 96: 647–657, 2012 PubMed Nephrol 17: 1275–1282, 2006 PubMed
17. Mente A, O’Donnell MJ, Rangarajan S, McQueen MJ, Poirier P, Wiel- 37. del Castillo JR, Burguillos L: Pathways for K1 efflux in isolated surface
gosz A, et al.; PURE Investigators: Association of urinary sodium and and crypt colonic cells. Activation by calcium. J Membr Biol 205: 37–47,
potassium excretion with blood pressure. N Engl J Med 371: 601–611, 2005 PubMed
2014 PubMed 38. Cohen EP, Bastani B, Cohen MR, Kolner S, Hemken P, Gluck SL:
18. O’Donnell M, Mente A, Rangarajan S, McQueen MJ, Wang X, Liu L, et al.; Absence of H(1)-ATPase in cortical collecting tubules of a patient with
PURE Investigators: Urinary sodium and potassium excretion, mortality, and Sjogren’s syndrome and distal renal tubular acidosis. J Am Soc Nephrol 3:
cardiovascular events. N Engl J Med 371: 612–623, 2014 PubMed 264–271, 1992 PubMed
19. DeSalvo KB, Olson R, Casavale KO: Dietary guidelines for Americans. 39. Simpson AM, Schwartz GJ: Distal renal tubular acidosis with severe
JAMA 315: 457–458, 2016 PubMed hypokalaemia, probably caused by colonic H(1)-K(1)-ATPase defi-
20. DuBose TD Jr: Inadequate dietary potassium and progression of CKD. ciency. Arch Dis Child 84: 504–507, 2001 PubMed
Clin J Am Soc Nephrol 14: 319–320, 2019 PubMed 40. Tran KM, Hinther K, Bueti J, Karpinski M, Hingwala J: Toluene toxic-
21. Sebastian A, Cordain L, Frassetto L, Banerjee T, Morris RC: Postulating ity: A case report of transdermal exposure causing hypokalemic paralysis.
the major environmental condition resulting in the expression of essential Can J Kidney Health Dis 6: 2054358119871594, 2019 PubMed
hypertension and its associated cardiovascular diseases: Dietary impru- 41. Raphael KL, Gilligan S, Ix JH: Urine anion gap to predict urine ammo-
dence in daily selection of foods in respect of their potassium and sodium nium and related outcomes in kidney disease. Clin J Am Soc Nephrol 13:
content resulting in oxidative stress-induced dysfunction of the vascular 205–212, 2018 PubMed
endothelium, vascular smooth muscle, and perivascular tissues. Med 42. Raphael KL, Yee J: It is really time for ammonium measurement. Adv
Hypotheses 119: 110–119, 2018 PubMed Chronic Kidney Dis 25: 297–300, 2018 PubMed
22. Kieneker LM, Eisenga MF, Joosten MM, de Boer RA, Gansevoort RT, 43. Kirschbaum B, Sica D, Anderson FP: Urine electrolytes and the
Kootstra-Ros JE, et al.: Plasma potassium, diuretic use and risk of devel- urine anion and osmolar gaps. J Lab Clin Med 133: 597–604, 1999
oping chronic kidney disease in a predominantly White population. PubMed
PLoS One 12: e0174686, 2017 PubMed 44. Pela I, Materassi M, Seracini D, Lavoratti G, Bettinelli A: Hypokalemic
23. Palmer BF: Regulation of potassium homeostasis. Clin J Am Soc Nephrol rhabdomyolysis in a child with Bartter’s syndrome. Pediatr Nephrol 20:
10: 1050–1060, 2015 PubMed 1189–1191, 2005 PubMed
24. Facchini M, Sala L, Malfatto G, Bragato R, Redaelli G, Invitti C: Low- 45. Duman O, Koyun M, Akman S, G€ uven AG, Haspolat S: Case of Bartter
K1 dependent QT prolongation and risk for ventricular arrhythmia in syndrome presenting with hypokalemic periodic paralysis. J Child Neurol
anorexia nervosa. Int J Cardiol 106: 170–176, 2006 PubMed 21: 255–256, 2006 PubMed
25. Chen WH, Yin HL, Lin HS, Chen SS, Liu JS: Delayed hypokalemic 46. Bailey MA, Cantone A, Yan Q, MacGregor GG, Leng Q, Amorim JB,
paralysis following a convulsion due to alcohol abstinence. J Clin Neuro- et al.: Maxi-K channels contribute to urinary potassium excretion in the
sci 13: 453–456, 2006 PubMed ROMK-deficient mouse model of Type II Bartter’s syndrome and in
26. Centers for Disease Control and Prevention (CDC): Atypical reactions adaptation to a high-K diet. Kidney Int 70: 51–59, 2006 PubMed
associated with heroin use–five states, January-April 2005. MMWR Morb 47. Jang HR, Lee JW, Oh YK, Na KY, Joo KW, Jeon US, et al.: From
Mortal Wkly Rep 54: 793–796, 2005 PubMed bench to bedside: Diagnosis of Gitelman’s syndrome – defect of sodium-
27. Tricarico D, Servidei S, Tonali P, Jurkat-Rott K, Camerino DC: Impair- chloride cotransporter in renal tissue. Kidney Int 70: 813–817, 2006
ment of skeletal muscle adenosine triphosphate-sensitive K1 channels in PubMed
patients with hypokalemic periodic paralysis. J Clin Invest 103: 675–682, 48. Pachulski RT, Lopez F, Sharaf R: Gitelman’s not-so-benign syndrome. N
1999 PubMed Engl J Med 353: 850–851, 2005 PubMed
116 Nephrology Self-Assessment Program - Vol 20, No 2, January 2022

49. Morita R, Takeuchi K, Nakamura A, Tajima T, Kuroda Y: Gitelman’s syn- underlies a spectrum of diabetic phenotypes. Diabetes 55: 2957–2964,
drome with mental retardation. Intern Med 45: 211–213, 2006 PubMed 2006 PubMed
50. Blanchard A, Bockenhauer D, Bolignano D, Calo LA, Cosyns E, 60. Chen Y, Chang AR, McAdams DeMarco MA, Inker LA, Matsushita K,
Devuyst O, et al.: Gitelman syndrome: Consensus and guidance from a Ballew SH, et al.: Serum potassium, mortality, and kidney outcomes in
Kidney Disease: Improving Global Outcomes (KDIGO) Controversies the Atherosclerosis Risk in Communities Study. Mayo Clin Proc 91:
Conference. Kidney Int 91: 24–33, 2017 PubMed 1403–1412, 2016 PubMed
51. Lin SH, Lin YF, Chen DT, Chu P, Hsu CW, Halperin ML: Laboratory 61. Collins AJ, Pitt B, Reaven N, Funk S, McGaughey K, Wilson D, et al.:
tests to determine the cause of hypokalemia and paralysis. Arch Intern Association of serum potassium with all-cause mortality in patients with
Med 164: 1561–1566, 2004 PubMed and without heart failure, chronic kidney disease, and/or diabetes. Am J
52. Crop MJ, Hoorn EJ, Lindemans J, Zietse R: Hypokalaemia and subse- Nephrol 46: 213–221, 2017 PubMed
quent hyperkalaemia in hospitalized patients. Nephrol Dial Transplant 62. Crews DC, Kuczmarski MF, Miller ER 3rd, Zonderman AB, Evans MK,
22: 3471–3477, 2007 PubMed Powe NR: Dietary habits, poverty, and chronic kidney disease in an
53. DuBose TD Jr: A 42-year-old woman with flaccid paralysis. Am J Kidney urban population. J Ren Nutr 25: 103–110, 2015 PubMed
Dis 54: 965–969, 2009 PubMed 63. Liu Y, Kuczmarski MF, Miller ER 3rd, Nava MB, Zonderman AB,
54. Weiss JN, Qu Z, Shivkumar K: Electrophysiology of hypokalemia and Evans MK, et al.: Dietary habits and risk of kidney function decline in
hyperkalemia. Circ Arrhythm Electrophysiol 10: e004667, 2017 PubMed an urban population. J Ren Nutr 27: 16–25, 201727771303
55. Harada K, Akai Y, Iwano M, Nakatani K, Nishino T, Fujimoto T, et al.: 64. Kim HW, Park JT, Yoo TH, Lee J, Chung W, Lee KB, et al.; KNOW-
Tubulointerstitial macrophage infiltration in a patient with hypokalemic CKD Study Investigators: Urinary potassium excretion and progression
nephropathy and primary Sj€ogren’s syndrome. Clin Nephrol 64: 387– of CKD. Clin J Am Soc Nephrol 14: 330–340, 2019 PubMed
390, 2005 PubMed 65. Kieneker LM, Bakker SJ, de Boer RA, Navis GJ, Gansevoort RT, Joosten
56. Zillich AJ, Garg J, Basu S, Bakris GL, Carter BL: Thiazide diuretics, MM: Low potassium excretion but not high sodium excretion is associ-
potassium, and the development of diabetes: A quantitative review. ated with increased risk of developing chronic kidney disease. Kidney Int
Hypertension 48: 219–224, 2006 PubMed 90: 888–896, 2016 PubMed
57. Palmer BF: Metabolic complications associated with use of diuretics. 66. Korgaonkar S, Tilea A, Gillespie BW, Kiser M, Eisele G, Finkelstein
Semin Nephrol 31: 542–552, 2011 PubMed F, et al.: Serum potassium and outcomes in CKD: Insights from the
58. Brown MJ, Williams B, Morant SV, Webb DJ, Caulfield MJ, Cruick- RRI-CKD cohort study. Clin J Am Soc Nephrol 5: 762–769, 2010
shank JK, et al.; British Hypertension Society’s Prevention and Treat- PubMed
ment of Hypertension with Algorithm-based Therapy (PATHWAY) 67. Leonberg-Yoo AK, Tighiouart H, Levey AS, Beck GJ, Sarnak MJ: Urine
Studies Group: Effect of amiloride, or amiloride plus hydrochlorothia- potassium excretion, kidney failure, and mortality in CKD. Am J Kidney
zide, versus hydrochlorothiazide on glucose tolerance and blood pressure Dis 69: 341–349, 2017 PubMed
(PATHWAY-3): A parallel-group, double-blind randomised phase 4 trial. 68. He J, Mills KT, Appel LJ, Yang W, Chen J, Lee BT, et al.; Chronic
Lancet Diabetes Endocrinol 4: 136–147, 2016 PubMed Renal Insufficiency Cohort Study Investigators: Urinary sodium and
59. Koster JC, Remedi MS, Masia R, Patton B, Tong A, Nichols CG: potassium excretion and CKD progression. J Am Soc Nephrol 27: 1202–
Expression of ATP-insensitive KATP channels in pancreatic beta-cells 1212, 2016 PubMed
Nephrology Self-Assessment Program - Vol 20, No 2, January 2022 117

Article
Pathophysiology, Evaluation, and Treatment of Hyperkalemia
Andrew S. Terker, MD, PhD,1 and David H. Ellison, MD, FASN2
1
Division of Nephrology and Hypertension, Vanderbilt University Medical Center, Nashville, Tennessee
2
Division of Nephrology and Hypertension, Department of Medicine, Oregon Health and Science University, Portland,
Oregon

approximately 2% of the total body potassium, or 70 mmol, exists


Learning Objectives in the extracellular compartment, and precise control of the distribu-
tion between intracellular and extracellular compartments is impera-
1. To summarize the mechanisms underlying internal and
tive to ensure proper cell function, particularly for electrically excit-
external potassium balance able cells such as cardiac and skeletal muscle and neurons. This
2. To define the pathophysiology of hyperkalemia distribution is maintained by a system that ultimately requires activ-
ity of the Na1/K1 adenosine triphosphate (ATP)ase, which uses
3. To summarize current approaches to acute and chronic
energy to actively pump potassium into cells against its electrochem-
hyperkalemia ical gradient. The tendency of potassium to lead out of cells, via
membrane channels, leads most cells to have a membrane voltage
oriented with the cell interior negative, relative to the cell exterior.
Introduction Disruption of this system can result in pathologic aberrations in
Potassium is the predominant cation inside of cells, and it plays plasma potassium and secondarily affect cell membrane potential,
essential roles, especially in setting the resting potential of most cells which increases the risk of life-threatening cardiac arrhythmias.
throughout the body. Potassium homeostasis requires coordination
of external balance, derived primarily via gastrointestinal absorption
and renal excretion, and internal balance, meaning the asymmetric
distribution of K1 between the intracellular space and the extracel- Regulation of Potassium Homeostasis
lular space. This article briefly reviews aspects of normal internal Daily dietary potassium intake generally ranges between 50 and 150
and external potassium balance and then discusses the pathophysiol- mmol, which is roughly equivalent to the entire extracellular pool.
ogy of hyperkalemia. Avoiding significant and sustained aberrations in both total and
plasma potassium levels requires a quick and efficient system to
maintain homeostasis. The potassium homeostatic mechanism
Cardiovascular Outcomes and Mortality involves coordinated regulation between its major components,
Population-based studies have consistently demonstrated a strong including the gastrointestinal tract, which absorbs dietary potassium;
correlation between dietary potassium intake and poor cardiovascu- skeletal muscle, which harbors the majority of intracellular potas-
lar outcomes. In most analyses, the mortality risk related to plasma sium and is involved in intracellular/extracellular redistribution; and
potassium concentration is U shaped, with excess in both the hypo- the kidney, which is the predominant site of potassium excretion,
kalemic and hyperkalemic ranges (1). A 2018 meta-analysis of the most of which derives from secretion along the aldosterone-sensitive
CKD Prognosis Consortium included patient-level data from 27 distal nephron (ASDN). Coordination of these components is
international cohorts (10 general population, 10 with CKD, 7 high accomplished via hormonal control, including contributions from
cardiovascular risk) (1). At 6.9 years, the lowest all-cause mortality aldosterone, insulin, adrenergic stimuli, and plasma potassium itself.
risk was associated with potassium 4 to 4.5 mEq/L. Compared with
potassium of 4.2 mEq/L, the associated hazard ratio was 1.22 (95%
CI, 1.15 to 1.29) for potassium 5.5 mEq/L and 1.49 (95% CI, 1.26 Gastrointestinal Absorption
to 1.76) for potassium 3.0 mEq/L. There were similar U-shaped Dietary potassium intake can vary widely across populations as well
associations for cardiovascular mortality and ESKD. as temporally in a given individual. In general, potassium consumed
in the diet is well absorbed.

Total Body Potassium Distribution


Total body potassium abundance has been estimated to be about 55 Renal Potassium Reabsorption and Secretion
mmol/kg, or between 3000 and 4000 mmol for a 70-kg person, Proximal Tubule. Potassium is freely filtered by the glomerulus
with approximately 98% of the cation found intracellularly. Most of and then reabsorbed along the proximal tubule (PT), where approxi-
the intracellular potassium resides in skeletal muscle, given the abun- mately 60%–65% of total reabsorption occurs. Reabsorption occurs
dance of this tissue in the body, although significant amounts are primarily via a paracellular pathway wherein potassium flux occurs
also in other tissues, including red blood cells and liver. Only secondary to bulk water reabsorption.

Copyright # 2022 by the American Society of Nephrology nephsap.org


118 Nephrology Self-Assessment Program - Vol 20, No 2, January 2022

Thick Ascending Limb of Henle. Approximately 20%–25% of the expressed ROMK and BK (Maxi K) channels. Extensive work has
filtered potassium load is reabsorbed along the thick ascending limb clearly shown that this system is activated by aldosterone via miner-
of Henle (TAL). The reabsorptive pathway along this segment alocorticoid receptor signaling (5,6), which is why this nephron
depends on apical activity of the sodium-potassium-2 chloride segment is often referred to as the aldosterone-sensitive distal neph-
cotransporter (NKCC2). The renal outer medullary potassium ron (ASDN).
channel (ROMK or Kir1.1) is expressed and active along the apical The secretory capacity of these segments is exceptional, with
plasma membrane of the TAL and permits a portion of the potas- animal models demonstrating minimal changes in plasma potassium
sium that traverses the NKCC2 to recycle across the apical mem- levels after the consumption of heavily potassium-loaded diets
brane; coupled with basolateral chloride efflux, via ClC chloride (#5% wt/wt). It has become clear that under typical dietary condi-
channels, this generates a net-positive lumen potential. This positive tions, the CNT is the most active potassium secretory site, whereas
potential then drives paracellular potassium reabsorption along with the CD typically becomes active primarily when aldosterone secre-
other cations, such as Ca21 and Mg21. When this mechanism fails, tion is stimulated by depletion of the extracellular fluid volume or
as in Bartter syndrome or during furosemide administration, by hyperkalemia (7).
patients present with hypokalemia caused by renal potassium losses. During states of potassium depletion, potassium can be reab-
Although potassium is reabsorbed along the TAL under most condi- sorbed along this segment; a-intercalated cells are the major cells
tions, recent experiments show that the transport direction may responsible for potassium reabsorption. They express the hydrogen-
reverse when animals consume a low-sodium/high-potassium diet, potassium ATPase (H1-K1 ATPase), for which two isoforms exist:
in which case the TAL becomes a potassium-secreting segment (2). HKa1 and HKa2. In models of experimental hypokalemia, H1-K1
Under these conditions, furosemide administration becomes potas- ATPase abundance increases (8) to maximally conserve total body
sium sparing. Whether similar effects occur is humans has not potassium.
been studied.

Distal Convoluted Tubule. Studies have failed to consistently dem- Integrated Renal Potassium Excretion: The
onstrate meaningful quantities of potassium reabsorption or secre- Potassium Switch
tion along the distal convoluted tubule (DCT). The contribution of
During the past several years, an increased understanding of renal
this segment is, however, integral to maintaining potassium bal-
potassium handling has emerged, in large part driven by the molecu-
ance—a finding highlighted by dysregulated potassium homeostasis
lar solutions to monogenic diseases of potassium retention and potas-
in Mendelian tubulopathies affecting the DCT such as Gitelman
sium wasting. Gitelman syndrome, an autosomal recessive disease,
syndrome and familial hyperkalemic hypertension (also called pseu-
typically presents with profound hypokalemia (see the NephSAP
dohypoaldosteronism type 2 or Gordon syndrome). Substantial
work over the past decade has demonstrated that this segment’s role article on hypokalemia) and is caused by mutations in the gene
in potassium reabsorption is a consequence of its ability to act as a encoding NCC, expressed exclusively along the DCT (9). Yet, as
modulator of sodium flow to the more distal connecting segment noted above, this segment of the nephron does not transport substan-
and collecting duct (CD), major sites of sodium-dependent potas- tial amounts of potassium, which suggests that the mechanisms of
sium secretion. Inasmuch as these more distal segments require potassium wasting are indirect.
sodium reabsorption to generate a lumen-negative transepithelial The mirror image disease, familial hyperkalemic hypertension
voltage to secrete potassium, the DCT can increase or decrease (also known as pseudohypoaldosteronism type 2 or Gordon syn-
potassium excretion by allowing more or less sodium to flow dis- drome) is a monogenic disease characterized by hyperkalemia, which
tally. The DCT accomplishes this by modulating apical sodium can be severe (10). Yet, once again, we now know that the muta-
reabsorption via the sodium-chloride cotransporter, which is regu- tions that cause the disease do so primarily by activating NCC,
lated by a molecular signaling pathway (WNK-SPAK) that is known again having effects primarily along the DCT, which does not trans-
to be potassium sensitive and allows this system to sense and port potassium.
respond to aberrations in plasma potassium levels (3,4). (See The resolution to this apparent paradox is the potassium switch
“Integrated renal potassium excretion: The potassium switch” below (Figure 2), which links the DCT with the ASDN (primarily CNT
for additional details.) and CD) and aldosterone. Work has made clear that a dominant
factor modulating NCC is the plasma potassium concentration;
Connecting Tubule and Collecting Duct. The connecting tubule when plasma potassium concentration is high, NCC is “turned off”
(CNT) and the CD are targets of aldosterone and act as the pre- (dephosphorylated). When the plasma potassium concentration is
dominant sites for active potassium secretion in the body. After low, NCC is “turned on” (4). Hyperkalemia therefore both stimu-
reabsorption along the PT and TAL, approximately 10% of the fil- lates aldosterone secretion by the adrenal glands AND inhibits
tered potassium load reaches the CNT. Along the CNT and CD, NCC. This means that more sodium flows into a nephron segment
net potassium transport can range from nearly complete reabsorp- (the CNT/CD) that is primed to secrete potassium by aldosterone.
tion of the remaining filtered load (approximately 10%) to net secre- The converse situation is also relevant. When plasma potassium is
tion. Under normal conditions, potassium secretion predominates low, NCC is activated. This limits sodium delivery to the ASDN,
and is responsible for the majority of potassium excreted in the thereby limiting potassium secretion and therefore potassium excre-
urine. This system relies on a net negative lumen potential created tion. These monogenic diseases (Gitelman syndrome and familial
by apical sodium reabsorption via the epithelial sodium channel hyperkalemic hypertension) cause abnormalities in plasma potassium
(ENaC) to drive electrogenic potassium secretion via apically concentration because they break this switch (11).
Nephrology Self-Assessment Program - Vol 20, No 2, January 2022 119
120 Nephrology Self-Assessment Program - Vol 20, No 2, January 2022

The potassium switch discovery appears to resolve the


“aldosterone paradox,” the observation that the same hormone,
aldosterone, can mediate primarily kaliuresis when induced by
hyperkalemia, and primarily sodium retention, when induced pri-
marily by angiotensin II (via extracellular volume depletion) (17).
We can now see that during hyperkalemia, aldosterone concentra-
tions are high, activating the epithelial sodium channel along the
CNT/CD, generating a lumen-negative voltage. Yet, NCC is turned
off, meaning that there is little salt reabsorption along the DCT. In
this situation, Na flows distally, leading to robust potassium secre-
tion, but sites of sodium reabsorption are mostly shifted from more
proximal (DCT) to more distal (CNT/CD); net sodium reabsorp-
tion is not stimulated.
By contrast, when extracellular fluid volume is low, angio-
tensin II is stimulated. In this case, NCC is activated by angio-
tensin II, while at the same time, aldosterone activates the CNT/
Figure 2. Potassium switch. The ‘potassium switch’ involves the
CD. In this case, sodium delivery to the CNT/CD is low, and
potassium-induced interaction between the distal convoluted tubule
therefore, even though the segment is activated, transport there is
(DCT) and the connecting tubule (CNT). When dietary K1 intake
is high (left panels), elevated plasma potassium stimulates aldosterone limited, limiting potassium secretion. In contrast, because
secretion. As the NCC is turned off by high K1, sodium delivery to sodium reabsorption is stimulated all along the distal nephron,
the activated CNT is high and sodium exchange for potassium is net sodium retention occurs.
robust. In contrast, when dietary K1 intake is low, NCC is stimu-
lated directly, but aldosterone secretion is inhibited. Most sodium is
therefore reabsorbed electroneutrally along the DCT, preventing
Diagnosis and Evaluation
potassium loss. Used with permission from reference 11 (Ellison Although flame photometry ushered in the era of electrolyte mea-
DH, Terker AS, Gamba G: Potassium and its discontents: new surement in the 1940s, nearly all laboratories now determine potas-
insight, new treatments. J Am Soc Nephrol 27: 981–989, 2016). sium concentration using ion-sensitive electrodes. Three different
methods of collection are currently used to estimate the concentra-
The signaling pathway that underlies the potassium switch has tion of extracellular potassium in plasma, serum, or whole blood.
also been clarified by monogenic diseases. NCC is activated by phos- Each approach has strengths and limitations, and all three are used
phorylation along its amino terminal cytoplasmic domain (12). The clinically.
kinase that is primarily responsible is SPAK, which is expressed Serum is collected into tubes that often have red tops and
broadly along the distal nephron (13). SPAK itself is activated contain a clotting activator, like silica. The collected blood is
when it is phosphorylated. In this case, the kinase responsible is allowed to clot at rest, and then potassium is measured in the
primarily WNK4, produced by one of the genes that, when clear serum component. When blood clots, a certain amount of
mutated, leads to hyperkalemic hypertension (14). The WNK potassium is released from red cells; this is in vitro hemolysis
kinases are sensitive to intracellular chloride concentration (15), and should be distinguished from hemolysis related to phlebot-
which turns out to be the link with plasma potassium. When omy or in vivo hemolysis, resulting from disease. Additionally,
plasma potassium is high, it increases intracellular chloride con- some potassium is released from platelets and leukocytess.
centration secondarily, which turns off WNK kinases (4,16). These processes account for the fact that serum potassium con-
Conversely, when the plasma potassium concentration is low, centrations typically exceed plasma or whole blood measure-
that reduces intracellular chloride concentration, leaving WNKs ments by 0.1–0.7 mM (18). Spectrophotometric measurement
in a chloride-unbound state, which activates them. of hemolysis is typically used to detect hemolysis when serum

Figure 1. Algorithm for treating moderate or severe hyperkalemia in adults. *IV 1 g calcium gluconate (10 ml of 10% solution [repeated up to
3 times, if necessary after 5 minutes], each containing 93 mg elemental calcium, 2.3 mmol) or calcium chloride (10 ml of 10% solution, 273
mg elemental calcium, 6.8 mmol). †IV regular insulin 5 units plus 25 g glucose (50 ml of 50%) is as effective as albuterol (salbutamol) 10 mg
nebulized; insulin and albuterol may have an additive effect. Beware of hypoglycemia. IV bicarbonate (1 amp of 50 ml of 8.4% solution, Na 50
mmol, HCO3 50 mmol) over 15 minutes. **Potassium binders: sodium polystyrene sulphonate 15–60 g orally/rectally (do not give with sorbi-
tol) or zirconium cyclosilicate 10 g 3/d (patiromer not advisable because onset of action is 7 hours). This guidance is suggestive because there are
limited data on onset of action with no head-to-head studies between potassium binders. ‡Hemodialysis is the modality of preference. Potassium
homeostasis and management of dyskalemia in kidney diseases: conclusions from a Kidney Disease: Improving Global Outcomes (KDIGO)
Controversies Conference. Used with permission from reference 18 (Clase CM, Carrero JJ, Ellison DH, Grams ME, Hemmelgarn BR, Jardine
MJ, Kovesdy CP, Kline GA, Lindner G, Obrador GT, Palmer BF, Cheung M, Wheeler DC, Winkelmayer WC, Pecoits-Filho R; Conference
Participants: Potassium homeostasis and management of dyskalemia in kidney diseases: conclusions from a Kidney Disease: Improving Global
Outcomes (KDIGO) controversies conference. Kidney Int 97: 42–61). License (CC BY NC ND).
Nephrology Self-Assessment Program - Vol 20, No 2, January 2022 121

measurements are performed in laboratories, with a recom- were these approaches able to predict serious arrhythmias more accu-
mended threshold of ,2%, but higher values are common, rately than humans examining ECG tracings, they could become
especially when blood is obtained from intravenous catheters very useful in emergency settings.
(19).
The other approaches to determine the amount of potassium in Causes and Risk Factors
circulating blood use plasma and whole blood. Plasma is collected In approaching a patient with an elevated blood potassium concen-
into heparinized tubes and obtained by centrifugation; the potas- tration, the first question is whether the measurement reflects a true
sium concentration is obtained with an ion-sensitive electrode auto- increase in circulating levels. Given that 95% of body potassium is
analyzer. By contrast, whole blood techniques measure potassium inside of cells, and that methods for measurement depend on blood
immediately after the sample is obtained, without further processing; collection, it is not surprising that cell leakage can artificially elevate
these methods determine potassium concentrations that are essen- potassium; this phenomenon is called pseudohyperkalemia. Hemo-
tially those in plasma and are used commonly in emergency depart- lysis during or after blood collection is the most common cause. A
ments and intensive care units. Patients who have substantial leuko- falsely elevated potassium level may occur with fist clenching during
cytosis (.503109/L) (20) or thrombocytosis are at risk for phlebotomy, mechanical trauma, tourniquet use .1 minute, and
pseudohyperkalemia, when serum measurements are used. during blood clotting, but, as noted, most laboratories include spec-
In fact, determination of whole blood potassium is used trophotometric analysis of hemolysis and have cutoffs of 1%–2%.
increasingly, owing to its speed and accessibility. It is often
Additionally, patients with elevated leukocyte or platelet counts may
argued that serum potassium is the criterion standard, inasmuch
have release from those formed elements. In general, if formed ele-
as the American Society of Clinical Pathology has the most
ments are increased (as discussed above), or if there is suspicion of
clearly developed quality control metrics for its use. In many
hemolysis, the potassium should be assessed by use of the converse
recent trials, whole blood methods have been used routinely
technique (serum versus whole blood or plasma). If there is uncer-
(21). In many series, it has been noted that the value obtained
tainty, treatment is often provided.
from whole blood point-of-care testing is often 0.3–0.5 mM
A second question to address is the duration of hyperkalemia.
lower than that obtained from serum in the laboratory (22).
Causes that involve shifting of potassium from inside of cells are
Overall, these data raise the possibility that whole blood potas-
typically acute and do not involve a change in body potassium con-
sium is a more accurate measure of the relevant physiologic vari-
tent. By contrast, chronic hyperkalemia, with or without a tempo-
able than is serum (23).
rary increase, requires an impairment of potassium secretion along
In the acute setting, an important goal of treating hyperkalemia
the ASDN. Most common cases of hyperkalemia, at least in patients
is to prevent arrhythmias. Unfortunately, arrhythmias are not closely
correlated with serum or blood potassium concentrations. Several who do not have ESKD, reflect a congruence of factors that impair
well-described electrocardiographic (ECG) changes occur in the set- renal potassium secretion. Although high potassium intake may con-
ting of hyperkalemia, including tall “peaked” T waves, a prolonged tribute, its role may have been overemphasized in the past, and there
P-R interval, intraventricular blocks, widening of the QRS com- has been a reconsideration of dietary recommendations for patients
plexes, eventually resulting in “sine wave” patterns, ventricular fibril- with CKD or other hyperkalemic risk factors (29). In view of the
lation, and asystole. Some work has suggested that ECG changes are cardioprotective effects of diets high in potassium, especially those
reliable and predictive of more serious arrhythmias (24), but the that emphasize fruits and vegetables, many authorities now advocate
associated findings have not included such early signs as peaked T that patients at risk for hyperkalemia focus on eating a healthy diet,
waves. without undue focus on potassium restriction; this may help address
An interesting approach explored recently has used artificial the low adherence to typical CKD diets observed in practice (30).
intelligence and deep learning algorithms to detect subtle ECG It is not surprising that the major risk factors for hyperkalemia
changes that are associated with hyperkalemia. Such approaches include kidney failure (acute or chronic), use of drugs that block the
have been termed bloodless blood tests and have used ECG leads in renin/angiotensin/aldosterone system (RAAS), and acidosis. Other
the dialysis unit (25,26). Recently, approaches using routine ECGs common drug-induced causes are numerous, and they have been
have also been shown to correlate well with the results of serum reviewed (31). Patients with ESKD are at special risk. On the basis
potassium measurements and to exceed the performance of experi- of epidemiologic evidence, other frequently identified factors include
enced cardiologists at detecting hyperkalemia (27,28). In one study male sex, lower body mass index, malignancy, and diabetes (32).
a deep learning model was trained to detect potassium .5.5 mEq/L Whereas these risk factors increase the prior probability of true
using .1.5 million ECGs from 449,380 patients and tested in a val- hyperkalemia, other important, but less common, bona fide causes
idation cohort including 61,965 CKD patients (28). The area under of hyperkalemia exist. These include hyperkalemic periodic paralysis,
the receiver operating curve ranged from 0.853 to 0.883. Using a an autosomal dominant disorder in which episodes of flaccid weak-
high sensitivity operating point to maximize its utility as a screening ness and intermittent myotonia can be precipitated by cold, exercise,
tool, the negative predictive value was .99%. This is as good as, or fasting, or the ingestion of small amounts of potassium. The most
better than, many other commonly used screening tools, including common abnormality in hyperkalemic periodic paralysis is a point
mammography and colonoscopy for detecting breast cancer and mutation in the SCNA4 gene for the a subunit of the skeletal mus-
colorectal cancers, respectively. cle cell sodium channel (a channel that is molecularly distinct from
Of course, the utility of such approaches currently is limited ENaC in the kidney) (33).
because of the ease of measuring blood potassium concentrations Hypoaldosteronism can result from Addison disease or be iso-
clinically and the lack of implemented software. On the other hand, lated. Isolated hypoaldosteronism, often a part of hyporeninemic
122 Nephrology Self-Assessment Program - Vol 20, No 2, January 2022

hypoaldosteronism, was previously diagnosed commonly in patients


with chronic diabetes or interstitial nephritis; it is often viewed as
synonymous with type 4 renal tubular acidosis, but some cases of
hyperkalemic acidosis have distal resistance to aldosterone, rather
than aldosterone insufficiency, and have been termed hyperkalemic
distal RTA (34)
Pseudohypoaldosteronism type 1 is usually diagnosed early in
life and presents with severe salt wasting and hyperkalemia. It can
result from genetically defective ENaC channels and can be inher-
ited in an autosomal recessive manner or from mutations in the
mineralocorticoid receptor itself, and either be inherited in an auto-
somal dominant manner or be de novo (35). The recessive form is
sometimes referred to as systemic and presents with severe lifelong
salt losing. The form resulting from mutations in the mineralocorti- Figure 3. KDIGO-recommended classification of hyperkalemia.
coid receptor is sometimes called the renal form and is milder; many *indicates that hyperkalemia is defined as either .5.0 or .upper
times, salt supplementation becomes unnecessary by 1–3 years of limit of normal potassium for the laboratory. Potassium homeostasis
age. Interestingly, the salt wasting is likely secondary to both a defi- and management of dyskalemia in kidney diseases: conclusions from
ciency in ENaC and secondary inhibition of NCC resulting from a Kidney Disease. Used with permission from reference 18 (Clase
hyperkalemia. Animal models have indicated that the salt wasting CM, Carrero JJ, Ellison DH, Grams ME, Hemmelgarn BR, Jardine
can be mitigated by correcting the plasma potassium, thereby reacti- MJ, Kovesdy CP, Kline GA, Lindner G, Obrador GT, Palmer BF,
vating NCC (36), and this phenomenon has recently been reported Cheung M, Wheeler DC, Winkelmayer WC, Pecoits-Filho R; Con-
in humans (37). A third form of pseudohypoaldosteronism type 1 is ference Participants: Potassium homeostasis and management of dys-
acquired and is associated with urinary tract abnormalities and uri- kalemia in kidney diseases: Conclusions from a Kidney Disease:
nary infections (35). Improving Global Outcomes (KDIGO) controversies conference.
Pseudohypoaldosteronism type 2, which is now also referred to Kidney Int 97: 42–61). License (CC BY NC ND).
as familial hyperkalemic hypertension or Gordon syndrome, also
presents with hyperkalemia, but in this case is associated with nor-
now known to disrupt their binding to KLHL3, preventing their
mal BP (early in life) and hypertension later. It occurs most com-
degradation (41). Most mutations of Cullin 3 and KLHL3 are
monly as an autosomal dominant disease, but some cases, discussed
inherited in a dominant manner, but some KLHL3 mutations are
below, can also be recessive. The disease is caused by mutations in
recessive (Table 1).
genes encoding proteins that affect NCC. The first discovered were
mutations in WNK (with no lysine) kinase 1 and WNK4, and they
resulted in increased phosphorylation and activation of NCC (38). Prevention
Inhibition of potassium channels may also be involved, but the phe-
Expert guidelines suggest that plasma potassium should be moni-
notype is mimicked in mice that have activated NCC alone (39).
tored within several weeks of starting or increasing drugs that block
Mutations have more recently been identified in genes that
the RAAS (18).
modulate WNK activity, including Kelch-like 3 and Cullin 3 (40),
which are detected more commonly. These proteins are part of the
Cullin ring ligase complex, which leads to ubiquitylation of proteins, Treatment
targeting them for degradation via the proteasome. The mutations
The decision to treat hyperkalemia, and the urgency of such treat-
appear to disrupt the ability of the complex to bind to WNK kin-
ment, is not standardized. One approach to this decision is to cate-
ases, leading to their accumulation; the WNK kinase mutations are
gorize hyperkalemia according to a consensus-based algorithm that
incorporates both measured potassium and ECG changes and that
classifies hyperkalemia as mild, moderate, and severe (Figure 3). The
indications for controlling hyperkalemia in the long term are dis-
4 The sodium-chloride cotransporter (NCC) in the cussed below.
distal convoluted tubule modulates potassium
Acute Hyperkalemia
excretion by regulating the amount of sodium
A KDIGO controversies conference recently addressed the manage-
delivered to the connecting segment and ment of hyperkalemia (18). The initial management of acute hyper-
collecting duct. kalemia typically includes the use of insulin/glucose and b agonists
4 NCC is dephosphorylated (turned off) when the for potassium .6.5 mEq/L, with use of intravenous calcium if
EKG changes are present; the algorithm also recommends calcium
plasma potassium is high and phosphorylated
use if a repeat serum or blood potassium is above 6.5 mEq/L, even
(turned on) when the plasma potassium is low. in the absence of ECG changes. Bicarbonate may be considered if
acidosis is present. Intravenous loop diuretics and potassium binders
are also considered. Immediate short-term treatment of potentially
Nephrology Self-Assessment Program - Vol 20, No 2, January 2022 123

Table 1. Causes of pseudohyoaldosteronism

Type Mode of Inheritance Gene Mutations Clinical Features

Pseudohypoaldosteronism type IA Dominant Mineralocorticoid receptor Milder


Pseudohypoaldosteronism type IB Recessive Epithelial sodium channel Severe phenotype
Pseudohypoaldosteronis type IIA Dominant WNK 1, WNK4, KLHL3, Hypertension, short stature,
CUL 3 responsive to thiazides

Reprinted with permission from reference 34 (Batlle, D, Arruda, J: Hyperkalemic forms of renal tubular acidosis: clinical and pathophysiological aspects. Adv Chronic Kidney Dis
28: 321–333, 2018).

life-threatening hyperkalemia is based on experience and opinion; administration (42). Its effects have been reported to be additive
although it is relatively standardized (42), wide variation in with insulin, although studies have been small (46). A randomized
approaches persists (43) (Figure 1). control trial comparing nebulized albuterol and insulin with glucose,
versus both treatments combined, is currently in progress (47).
Stabilize the Membrane. For urgent cases, calcium is usually Sodium bicarbonate has been used, but its effects are limited,
administered; calcium chloride contains three times the amount of and it has generally fallen out of favor unless there is systemic meta-
calcium as calcium gluconate, but the gluconate salt is sometimes, bolic acidosis. The minimal effect of bicarbonate has been con-
although not always, preferred as it causes less local irritation. The firmed in several studies (48), although it should be noted that sus-
European Resuscitation Council recommends use of 10 ml 10% cal- tained alkali treatment in CKD has several other benefits and may
cium chloride over 2–5 minutes in hyperkalemic patients with ECG enhance urinary potassium excretion in the long term.
changes (44); the KDIGO conference recommended calcium gluco-
nate, but this would require 30 ml of a 10% solution to obtain the Remove Potassium from the Body. Except when hyperkalemia is
same amount of elemental calcium (18). The effects are short lived the result of an internal shift, removal from the body is the definitive
(30-60 minutes) and doses can be repeated if ECG changes persist. approach. For individuals with kidney function, diuretic-induced
Hyperkalemia depolarizes the resting potential of myocytes, putting kaliuresis is often effective. Although it is often stated that there is
it closer to the threshold potential, increasing the likelihood of aber- little experimental support for the use of diuretics to treat hyperkale-
rant action potentials. Calcium raises the threshold potential, mia in the short term, and there certainly are no large controlled
thereby restoring the difference between it and the resting potential; studies of loop diuretic efficacy in this situation, many smaller physi-
calcium may have other effects on cardiomyocytes, as well. ologically based trials show preserved kaliuresis in situations of low
GFR, as long as urine volumes increase (49,50). Therefore, in the
Shift Potassium into Cells. Insulin shifts potassium into cells pri- absence of extracellular fluid volume depletion, loop diuretics should
marily by stimulating the Na/K ATPase, is clearly effective in lower- be given; using natriuretic (often higher) doses is essential because
ing potassium concentrations, and forms the centerpiece of most potassium clearance correlates with sodium clearance during loop
approaches to acute hyperkalemia; it is usually given with glucose to diuretic administration (51).
avoid hypoglycemia, unless the baseline serum glucose exceeds 250 When there is doubt about the utility of diuresis, or if forced
mg/dL. The insulin effect occurs quickly (10-20 minutes) and can diuresis fails, dialysis is often used. Hemodialysis is the most effective
last four to six hours. The most recent guidelines from the UK approach to rapid potassium removal, with rates of removal inversely
Renal Association are very clear and recommend that all patients proportional to dialysate potassium concentration (52). Each hemo-
with moderate or severe hyperkalemia (potassium .6 mEq/L) dialysis session removes approximately 70–100 mmol of potassium
receive 10 units of insulin in 25 grams of glucose. When the base- (53) The serum potassium concentration typically falls by 1 mmol/L
line serum glucose is less than 126 mg/dl, this should be followed during the first hour of dialysis, when the gradient between the
by 10% glucose at 50 ml/h for 5 hours, with a target glucose of serum and dialysate K1 is highest, then by 1 mmol/L over the next
72-126 mg/dl) (https://renal.org/health-professionals/guidelines/ 2 hours. It reaches a steady state during the last hour of the
guidelines-commentaries). treatment.
Yet, even using this approach, hypoglycemia may occur. A Hemodialysis patients have a high risk of hyperkalemia, which
recent meta-analysis evaluated retrospective cohort studies of lower is associated with mortality and may be responsible for 3%–5% of
dose strategies with low or moderate risk of bias (45). The standard deaths (54). At the same time, there is also speculation that the sud-
approach was considered to be 10 units of insulin and the alternative den reduction in serum potassium that occurs during dialytic
approaches included 5 units, 0.1 units/kg, and ,10 units. Alterna- removal may be just as important (55). The most common time for
tive dosing had lower pooled odds of hypoglycemia and severe hypo- hyperkalemic events in hemodialysis patients is immediately after
glycemia, without detected differences in potassium reduction. the 3-day weekend break (56). The long interdialytic break also cor-
b-Adrenergic agonists, primarily albuterol (also called salbuta- relates with hospitalization (57) and mortality in hemodialysis
mol), can also be used to shift potassium into cells. The drug lowers patients (58). In a large study of 52,734 hemodialysis patients
plasma potassium by approximately the same amount as insulin, receiving three-times-weekly dialysis who underwent 533,889
and its use by nebulizer is generally preferred over intravenous repeated serum potassium measurements over a 2-year period, the
124 Nephrology Self-Assessment Program - Vol 20, No 2, January 2022

serum was .5.5 mEq/L in 20% of all measurements, particularly In the setting of acute hyperkalemia, or during a transition
after the long interdialytic interval (i.e., 24%, 16%, and 20% of from the acute phase of treatment to more sustained approaches,
measurements conducted on Monday, Wednesday, and Friday, sodium zirconium cyclosilicate has the advantage of faster
respectively) (59). action, with potassium lowering beginning as early as 1 hour (11)—
In rigorous analyses, the investigators found that increasingly a result of its acting along both the small and the large intestines.
higher serum potassium levels were associated with incrementally Yet, a recent although somewhat limited study found little advan-
higher short-term hospitalization risk. Serum potassium levels of tage to adding sodium zirconium cyclosilicate to standard treatment,
5.5–6.0 mEq/L obtained on Friday were potently associated with including insulin and glucose (65). The UK Renal Association
higher hospitalization risk within 4 days of measurement (adjusted guidelines do provide sodium zirconium cyclosilicate (level 1B) and
odds ratio, 1.68; 95% CI, 1.22 to 2.30), whereas levels obtained on patiromer (level 1C) recommendations for treating acute
Monday and Wednesday were linked with mild to no greater risk, hyperkalemia.
respectively. Notably, serum potassium levels .6.0 mEq/L were
associated with greater hospitalization costs across all days of mea- Chronic Hyperkalemia
surement. On the basis of data such as these, the UK Renal Associa- There is no consensus regarding the optimal management of chronic
tion Clinical Practice Guideline on Haemodialysis recommends an hyperkalemia, although a common approach was recently summa-
optimal predialysis serum K1 in the range of 4.0–6.0 mmol/L. rized by KDIGO (18).
Although dialysate potassium concentrations ,2 mM remove In the setting of chronic hyperkalemia, the focus has been on
more potassium than those $2 mM , low dialysate potassium con- withdrawing offending drugs, increasing urinary excretion, increas-
centrations (0 or 1 mM) associate with mortality, at least when used ing gastrointestinal removal, and possibly reducing intake (with the
in the long term—an association that is magnified in patients with caveats noted above). Whereas reducing the doses of, or discontinu-
higher predialysis potassium concentrations (60). Whether this asso- ing, drugs that contribute to hyperkalemia has long been a core
ciation reflects confounding by indication is not clear, but very low
component of treatment, drugs that interfere with the RAAS are
dialysate potassium concentrations should typically be avoided, espe-
among the most effective in mitigating both cardiovascular and renal
cially if ECG monitoring is not used.
disease. Given the development of demonstrably safe and effective
Continuous renal replacement therapy (CRRT) is commonly
agents that can reduce plasma potassium, the possibility of combin-
used to treat critically ill patients, many of whom have hyperkale-
ing RAAS inhibition with potassium-binding drugs is now being
mia. Inasmuch as blood flow is lower in CRRT than in hemodialy-
evaluated and is widely used.
sis, potassium clearance is typically less, and therefore hemodialysis
Among the traditional approaches to mitigating hyperkalemia
is preferred as first-line treatment. Nevertheless, there are individuals
in patients taking RAAS blocking agents, and especially with CKD,
in whom hemodynamic considerations suggest the use of CRRT; in
is to increase urine sodium and water excretion with loop diuretics.
such patients, blood flow and replacement fluid/dialysis flow rates
Although lacking randomized controlled data, this approach is
should be maximized initially; if persistent hyperkalemia is a prob-
almost surely effective, as noted above, and inasmuch as potassium
lem, it may signal persistent tissue breakdown/ischemia (61).
excretion can be normalized in patients with Addison disease, who
Gastrointestinal Potassium Removal. Sodium polystyrene sulfo- lack aldosterone, as long as salt and water consumption provides suf-
nate (SPS), still often used with sorbitol, has been a common treat- ficient distal flow. When such patients are hypertensive, as is often
ment for hyperkalemia, but its history is checkered. Even an initial the case with CKD, loop diuretics can treat both the hyperkalemia
report found little effect on plasma potassium concentration beyond and the hypertension.
that of sorbitol alone (62). Meaney et al. (63) found three clinical Another traditional approach is to administer fludrocortisone.
studies of its use, which reported lowering of approximately 1 mM This mineralocorticoid agonist directly stimulates potassium secre-
within 1–3 days. Reports of serious gastrointestinal complications, tion in the ASDN and can be quite effective. It also causes sodium
especially when SPS was administered with sorbitol, caused the U.S. retention, however, and directly counteracts many of the beneficial
Food and Drug Administration (FDA) to recommend discontinua- effects of RAAS blockers, making its use restricted to relatively rare
tion of its use with sorbitol, but even when it is used without, intes- cases of chronic hyperkalemia with salt wasting.
tinal necrosis may occur (64). SPS, however, is still administered The development of new potassium-binding drugs has raised
with sorbitol in many settings. The FDA also recommends that SPS the prospect that preventive treatment with oral potassium-binding
administration be separated temporally from that of other drugs drugs might permit RAAS blockade to be continued in situations
because it can alter drug absorption. Given that there are now safer where it would otherwise require discontinuation. Unlike SPS,
and clearly effective potassium-binding agents, many authorities rec- which has limited efficacy, poor tolerability, and potential safety
ommend against using SPS (64); its price may be lower than the pri- issues, patiromer and sodium zirconium cyclosilicate are generally
ces of the new agents, perhaps fostering continued use. safe and well tolerated. There are now multiple studies showing their
Two newer potassium-binding agents have been approved in efficacy.
the past 10 years: patiromer in 2015 and sodium zirconium cyclosi- Patiromer was the first agent approved (Table 2) and is a pow-
licate in 2018. Each has been shown to be effective in lowering der for oral suspension. It is a cation exchange polymer that is not
plasma potassium, although neither has been compared directly absorbed along the gastrointestinal tract, where it exchanges calcium
with SPS in terms of efficacy. The sites of action, benefits, and side for potassium, especially the colon. In one initial open-label trial,
effects of each, as well as their physical chemistry, differ, although 306 diabetic patients with CKD (eGFR 15–59 ml/min per 1.73
both lead to increased potassium removal through the gut. m2) and serum potassium of 5.1–5.9 mEq/L were randomized to
Nephrology Self-Assessment Program - Vol 20, No 2, January 2022 125

Table 2. Selected characteristics of K1 binding agents for hyperkalemia

Characteristic SPS Patiromer SZC

Approval date 1958 US, 2015; EU, 2017 US, 2018; EU, 2018
Mechanism of action K1 binding in exchange for K1 binding in exchange for K1 binding in exchange for
Na1 in GI tract (" fecal Ca21 in GI tract (" fecal H1 and Na1 in GI tract
excretion) excretion) (" fecal excretion)
Site of action Colon Colon Small and large intestines
Selectivity for K1 Nonselective; also binds Nonselective; also binds Highly selective; also binds
Ca21 and Mg21 Na1 and Mg21 NH41
Onset of action Variable; several hours 7h 1h
Na1 content 1500 mg per 15-mg dose None 400 mg per 5-g dose
Ca21 content None 1.6 g per 8.4-g dose None
Sorbitol content 20,000 mg per 15-g dose 4000 mg per 8.4-g dose No sorbitol content
Dosing 15 g 1–4 times (oral); 30–50 g 8.4 g QD (oral), titrate up to 10 g TID (oral) for initial
1–2 times (rectal) 16.8 g or 25.2 g QD correction of hyperkalemia
(for #48 h), then 5 g QOD
to 15 g QD for maintenance
Serious AEs Cases of fatal GI injury reported None reported None reported
Most common AEs GI disorders (constipation, GI disorders (abdominal GI disorders (constipation,
diarrhea, nausea, vomiting, discomfort, constipation, diarrhea, nausea, vomiting),
gastric irritation), diarrhea, nausea, flatulence), mild to moderate edema
hypomagnesemia, hypomagnesemia
hypokalemia, hypocalcemia,
systemic alkalosis

Reprinted with permission from Elsevier: Palmer BF, Carrero JJ, Clegg DJ, Colbert GB, Emmett M, Fishbane S, Hain DJ, Lerma E, Onuigbo M, Rastogi A, Roger SD, Spinowitz BS,
Weir MR. Clinical Management of Hyperkalemia. Mayo Clin Proc. 96(3): 744–762, 2021. Licensed under CC-BY-NC-ND.

different doses of patiromer and followed up for 1 year. All patients An interesting recent study tested the effects of patiromer on
were also treated with stable doses of an angiotensin-converting serum and stool electrolytes in 27 anuric dialysis patients who had a
enzyme inhibitor or an angiotensin II receptor blocker, or both, tendency toward hyperkalemia at baseline. During 12 weeks, treat-
often in combination with spironolactone. Serum potassium ment with patiromer reduced serum potassium concentration (by
decreased by 20.35 to 20.55 mEq/L at 4 weeks, with initial doses 0.6 mM) and increased stool potassium excretion—effects that
of 4.2–12.6 g twice daily among those with mild hyperkalemia, and reversed when the treatment was discontinued. Additionally, during
from 20.87 to 20.97 mEq/L with initial doses of 8.4–12.6 g twice the treatment phase, serum calcium increased (from 8.9 to 9.1 mg/
daily among those with moderate hyperkalemia. Serum potassium dL) and serum magnesium decreased (from 2.6 to 2.4 mg/dL) com-
concentrations remained in the normal range at 1 year with contin- pared with pretreatment levels, and the changes in serum potassium
ued patiromer therapy. Patiromer discontinuation resulted in an and magnesium were directly related (67).
increase in the serum potassium within 3 days. There were no Patiromer has been shown to reduce plasma potassium concen-
treatment-related serious adverse events in this trial, although consti- trations for longer periods of time, in a variety of settings, and has
pation (6.3% of patients) and hypomagnesemia (8.6% of patients) permitted uptitration of RAAS-blocking drugs (68). The AMBER
was reported. Hypomagnesemia occurred more commonly with study was a phase 2 multicenter, randomized, double-blind, pla-
higher doses of patiromer (66). cebo-controlled study that was designed to determine whether con-
In another early study of 237 patients with CKD who were receiv- comitant use of patiromer would permit greater use of spironolac-
ing RAAS inhibitors and who had serum potassium levels of 5.1 to tone for uncontrolled hypertension in participants with CKD (69).
,6.5 mmol per liter, patiromer was given during a correction phase for Participants were randomly assigned (1:1) to receive either placebo
4 weeks followed by a randomized withdrawal phase. At week 4, 76% or patiromer (8.4 g once daily) in addition to open-label spironolac-
of the patients had reached the target potassium level (,5.1 mM). Sub- tone (starting at 25 mg once daily). The primary endpoint was the
sequently, 107 patients were randomly assigned to patiromer (55 between-group difference at week 12 in the proportion of patients
patients) or placebo (52 patients) for the randomized withdrawal phase. taking spironolactone. Two hundred ninety-five patients were ran-
The median increase in the potassium level from baseline was greater domly assigned to spironolactone in addition to double-blind treat-
with placebo than with patiromer (P,0.001); as in the other study, ment with either placebo (n5148) or patiromer (n5147). At week
mild to moderate constipation was the most common adverse event (in 12, 66% of patients in the placebo group and 86% of patients in
11% of the patients), and hypomagnesemia occurred (in 3%) (66). the patiromer group continued with spironolactone (statistically
126 Nephrology Self-Assessment Program - Vol 20, No 2, January 2022

significant). Adverse events were mostly mild or moderate in severity double-blind, placebo-controlled multicenter study in which
and occurred in 79 (53%) of 148 patients in the placebo group and patients with predialysis hyperkalemia were randomized to receive
82 (56%) of 147 patients in the patiromer group. In a prespecified placebo or sodium zirconium cyclosilicate on nondialysis days over
subgroup analysis of patients with heart failure, the treatment group 4 weeks. One hundred ninety-six patients were randomized to
was able to receive more spironolactone than the placebo group sodium zirconium cyclosilicate or placebo. Significantly more
(70). patients in the treatment group (41.2%) maintained serum potas-
It appears that the approach of using potassium binders to per- sium concentrations below 5 mM than did patients treated with pla-
mit continued RAAS blockade is being used commonly in a variety cebo (1.0%). Rescue therapy to reduce potassium was required in
of healthcare settings. In one study, 288 veterans with hyperkalemia fewer patients receiving zirconium cyclosilicate compared with pla-
who were treated with patiromer were studied after patiromer initia- cebo (2.1% versus 5.1% respectively). There were no differences in
tion. All had heart failure, diabetes, or CKD, but not ESKD, and adverse effects, and the weight gains in the two groups were similar
had a median age of 70 years. Potassium concentration reductions (75).
after patiromer initiation were, on average, 21.0 mM (P,0.001), The sodium load delivered with sodium zirconium cyclosilicate
and RAAS inhibitor therapy was continued in .80%–90% of (Table 2) can be substantial and is believed to be the reason that sus-
patients (71). tained use has been associated with edema in .10% of patients,
A retrospective cohort study used data from a large dialysis pro- especially in those patients with eGFR ,30 ml/min (76). This con-
vider and examined dialysis patients who had a medication order for cern may be one factor leading many cardiologists to prefer
patiromer or SPS or who had laboratory evidence of hyperkalemia. patiromer to sodium zirconium cyclosilicate during sustained use
Patiromer initiators were 2.6 times more likely to have had multiple (D.H. Ellison, unpublished observation). Otherwise, the drug is
prior episodes of hyperkalemia. Sixty percent of patients’ first usually well tolerated, with constipation being the most commonly
patiromer orders remained open after 180 days, with statistically sig- reported side effect.
nificant reductions in serum potassium, averaging approximately One potential benefit of sodium zirconium cyclosilicate is that
20.5 mM. is also removes hydrogen ions and can increase the serum bicarbon-
In nearly all studies, the most common side effects of patiromer ate concentration (77). Acidosis complicating CKD is recognized
use are gastrointestinal intolerance, such as constipation, and hypo- increasingly as having adverse consequences, and a modest improve-
magnesemia, as mentioned above. Because the drug exchanges cal- ment in acid-base balance is likely to be beneficial (78). Roger et al.
cium for potassium, it does not deliver a sodium load, but it does (77) examined data from three placebo-controlled phase 3 trials of
alter divalent mineral metabolism. In addition to hypomagnesemia, sodium zirconium cyclosilicate. The results showed a significant
some recent case reports have detected hypercalcemia in some dose-dependent increase in mean serum bicarbonate of 0.3 to 1.5
patients (72). mM within 48 h of treatment. These changes were maintained over
Sodium zirconium cyclosilicate was approved in 2018 (Table 29 days. With highest sodium zirconium cyclosilicate maintenance
2). It is an insoluble, nonabsorbed cation exchange crystal that doses, the proportions with serum bicarbonate ,22 mM declined
exchanges potassium for sodium. Zirconium silicates have been used from 39% at baseline to 4.9% at 29 days (P50.005).
extensively in medical and dental applications because of their safety;
in the case of sodium zirconium cyclosilicate, the molecule is com-
plexed with sodium. Many studies include a correction phase, often
using higher doses, and then a maintenance phase at a lower, once-
daily dose. Most do not use a placebo group during the initial cor- 4 Both hypo- and hyperkalemia are associated with
rection phase, because leaving patients uncorrected might put them
increased cardiovascular and overall mortality,
at risk. In one early study, treatment reduced serum potassium by
0.46 mM within 48 hours during the uncontrolled correction phase and with ESKD risk, although these associations
(73). After correction, patients were randomized to zirconium or do not define cause.
placebo for 2 weeks, during which serum potassium concentration 4 Patiromer and sodium zirconium cyclosilicate are
was lower in the treatment groups than in the placebo group.
Later studies have supported this efficacy and demonstrated
effective and safe for the treatment of chronic
more prolonged effects. In the HARMONIZE multisite randomized hyperkalemia and allow continued use or
controlled study, 267 patients were treated during an open-label cor- up-titration of RAAS blocking agents
rection phase, followed up, and then randomized to 5 g or 10 g
daily of sodium zirconium cyclosilicate in the controlled sustained
phase. During the first 48 hours, serum potassium declined by 1.28
mmol/L. During the maintenance phase (days 8–29), serum potas-
sium was 10% and 18% lower, with low and high sodium zirco-
References
nium cyclosilicate doses, than with placebo (P,0.001 for both). 1. Kovesdy CP, Matsushita K, Sang Y, Brunskill NJ, Carrero JJ, Chodick
G, Hasegawa T, Heerspink HL, Hirayama A, Landman GWD, Levin A,
More patients had normokalemia with sodium zirconium cyclosili-
Nitsch D, Wheeler DC, Coresh J, Hallan SI, Shalev V, Grams ME;
cate 5 (58.6%) and 10 g (77.3%) versus placebo (24.0%), with the CKD Prognosis Consortium: Serum potassium and adverse outcomes
greatest number of normokalemic days in the 10-g group (74). across the range of kidney function: A CKD Prognosis Consortium
One group of patients at substantial risk of hyperkalemia is meta-analysis. Eur Heart J 39: 1535–1542, 2018 10.1093/eurheartj/
those with ESKD treated with dialysis. The DIALIZE study was a ehy100 PubMed
Nephrology Self-Assessment Program - Vol 20, No 2, January 2022 127

2. Wang B, Wen D, Li H, Wang-France J, Sansom SC: Net K1 secretion in kidney diseases: conclusions from a Kidney Disease: Improving Global
in the thick ascending limb of mice on a low-Na, high-K diet. Kidney Int Outcomes (KDIGO) controversies conference. Kidney Int 97: 42–61,
92: 864–875, 2017 10.1016/j.kint.2017.04.009 PubMed 2020 10.1016/j.kint.2019.09.018 PubMed
3. Cuevas CA, Su XT, Wang MX, Terker AS, Lin DH, McCormick JA, 19. Lowe G, Stike R, Pollack M, Bosley J, O’Brien P, Hake A, Landis G,
Yang CL, Ellison DH, Wang WH: Potassium sensing by renal distal Billings N, Gordon P, Manzella S, Stover T: Nursing blood specimen
tubules requires Kir4.1. J Am Soc Nephrol 28: 1814–1825, 2017 10. collection techniques and hemolysis rates in an emergency department:
1681/ASN.2016090935 PubMed analysis of venipuncture versus intravenous catheter collection techniques.
4. Terker AS, Zhang C, McCormick JA, Lazelle RA, Zhang C, Meermeier J Emerg Nurs 34: 26–32, 2008 10.1016/j.jen.2007.02.006 PubMed
NP, Siler DA, Park HJ, Fu Y, Cohen DM, Weinstein AM, Wang WH, 20. Ranjitkar P, Greene DN, Baird GS, Hoofnagle AN, Mathias PC: Estab-
Yang CL, Ellison DH: Potassium modulates electrolyte balance and lishing evidence-based thresholds and laboratory practices to reduce inap-
blood pressure through effects on distal cell voltage and chloride. Cell propriate treatment of pseudohyperkalemia. Clin Biochem 50: 663–669,
Metab 21: 39–50, 2015 10.1016/j.cmet.2014.12.006 PubMed 2017 10.1016/j.clinbiochem.2017.03.007 PubMed
5. Todkar A, Picard N, Loffing-Cueni D, Sorensen MV, Mihailova M, 21. Friedman PA, Scott CG, Bailey K, Baumann NA, Albert D, Attia ZI, Lade-
Nesterov V, Makhanova N, Korbmacher C, Wagner CA, Loffing J: wig DJ, Yasin O, Dillon JJ, Singh B: Errors of classification with potassium
Mechanisms of renal control of potassium homeostasis in complete aldo- blood testing: the variability and repeatability of critical clinical tests. Mayo
sterone deficiency. J Am Soc Nephrol 26: 425–438, 2015 10.1681/ASN. Clin Proc 93: 566–572, 2018 10.1016/j.mayocp.2018.03.013 PubMed
2013111156 PubMed 22. Bloom BM, Connor H, Benton S, Harris T: A comparison of measure-
6. Terker AS, Yarbrough B, Ferdaus MZ, Lazelle RA, Erspamer KJ, Meer- ments of sodium, potassium, haemoglobin and creatinine between an
meier NP, Park HJ, McCormick JA, Yang CL, Ellison DH: Direct and Emergency Department-based point-of-care machine and the hospital
indirect mineralocorticoid effects determine distal salt transport. J Am laboratory. Eur J Emerg Med 21: 310–313, 2014 10.1097/MEJ.
Soc Nephrol 27: 2436–2445, 2016 10.1681/ASN.2015070815 PubMed 0000000000000076 PubMed
7. Meneton P, Loffing J, Warnock DG: Sodium and potassium handling 23. Hartland AJ, Neary RH: Serum potassium is unreliable as an estimate of
by the aldosterone-sensitive distal nephron: The pivotal role of the distal in vivo plasma potassium. Clin Chem 45: 1091–1092, 1999 PubMed
and connecting tubule. Am J Physiol Renal Physiol 287: F593–F601, 24. Durfey N, Lehnhof B, Bergeson A, Durfey SNM, Leytin V, McAteer K,
2004 PubMed Schwam E, Valiquet J: Severe hyperkalemia: can the electrocardiogram
8. Codina J, Delmas-Mata JT, DuBose TD Jr: Expression of HKalpha2 risk stratify for short-term adverse events? West J Emerg Med 18: 963–
protein is increased selectively in renal medulla by chronic hypokalemia. 971, 2017 10.5811/westjem.2017.6.33033 PubMed
Am J Physiol 275: F433–F440, 1998 10.1152/ajprenal.1998.275.3.F433 25. Yasin OZ, Attia Z, Dillon JJ, DeSimone CV, Sapir Y, Dugan J, Somers
PubMed VK, Ackerman MJ, Asirvatham SJ, Scott CG, Bennet KE, Ladewig DJ,
9. Blanchard A, Bockenhauer D, Bolignano D, Calo LA, Cosyns E, Devuyst Sadot D, Geva AB, Friedman PA: Noninvasive blood potassium mea-
O, Ellison DH, Karet Frankl FE, Knoers NVAM, Konrad M, Lin SH, surement using signal-processed, single-lead ECG acquired from a hand-
Vargas-Poussou R: Gitelman syndrome: consensus and guidance from a held smartphone. J Electrocardiol 50: 620–625, 2017 10.1016/j.
Kidney Disease: Improving Global Outcomes (KDIGO) controversies con- jelectrocard.2017.06.008 PubMed
ference. Kidney Int 91: 24–33, 2017 10.1016/j.kint.2016.09.046 PubMed 26. Dillon JJ, DeSimone CV, Sapir Y, Somers VK, Dugan JL, Bruce CJ,
10. O’Shaughnessy KM: Gordon syndrome: a continuing story. Pediatr Ackerman MJ, Asirvatham SJ, Striemer BL, Bukartyk J, Scott CG, Ben-
Nephrol 30: 1903–1908, 2015 10.1007/s00467-014-2956-7 PubMed net KE, Mikell SB, Ladewig DJ, Gilles EJ, Geva A, Sadot D, Friedman
11. Ellison DH, Terker AS, Gamba G: Potassium and its discontents: new PA: Noninvasive potassium determination using a mathematically
insight, new treatments. J Am Soc Nephrol 27: 981–989, 2016 10.1681/ processed ECG: Proof of concept for a novel “blood-less, blood test”. J
asn.2015070751 PubMed Electrocardiol 48: 12–18, 2015 10.1016/j.jelectrocard.2014.10.002
12. Pacheco-Alvarez D, Cristobal PS, Meade P, Moreno E, Vazquez N, PubMed
Mu~ noz E, Dıaz A, Juarez ME, Gimenez I, Gamba G: The Na1:Cl2 27. Lin CS, Lin C, Fang WH, Hsu CJ, Chen SJ, Huang KH, Lin WS, Tsai
cotransporter is activated and phosphorylated at the amino-terminal CS, Kuo CC, Chau T, Yang SJ, Lin SH: A deep-learning algorithm
domain upon intracellular chloride depletion. J Biol Chem 281: 28755– (ECG12Net) for detecting hypokalemia and hyperkalemia by electrocar-
28763, 2006 PubMed diography: algorithm development. JMIR Med Inform 8: e15931, 2020
13. Piechotta K, Lu J, Delpire E: Cation chloride cotransporters interact 10.2196/15931 PubMed
with the stress-related kinases Ste20-related proline-alanine-rich kinase 28. Galloway CD, Valys AV, Shreibati JB, Treiman DL, Petterson FL, Gun-
(SPAK) and oxidative stress response 1 (OSR1). J Biol Chem 277: dotra VP, Albert DE, Attia ZI, Carter RE, Asirvatham SJ, Ackerman M,
50812–50819, 2002 10.1074/jbc.M208108200 PubMed Noseworthy PA, Dillon JJ, Friedman PA: Development and validation
14. Vitari AC, Deak M, Morrice NA, Alessi DR: The WNK1 and WNK4 of a deep-learning model to screen for hyperkalemia from the electrocar-
protein kinases that are mutated in Gordon’s hypertension syndrome diogram. JAMA Cardiol 4: 428–436, 2019 10.1001/jamacardio.2019.
phosphorylate and activate SPAK and OSR1 protein kinases. Biochem J 0640 PubMed
391: 17–24, 2005 PubMed 29. Wei KY, Gritter M, Vogt L, de Borst MH, Rotmans JI, Hoorn EJ: Die-
15. Piala AT, Moon TM, Akella R, He H, Cobb MH, Goldsmith EJ: Chlo- tary potassium and the kidney: Lifesaving physiology. Clin Kidney J 13:
ride sensing by WNK1 involves inhibition of autophosphorylation. Sci 952–968, 2020 10.1093/ckj/sfaa157 PubMed
Signal 7: ra41, 2014 10.1126/scisignal.2005050 PubMed 30. Kaesler N, Baid-Agrawal S, Grams S, Nadal J, Schmid M, Schneider
16. Su XT, Klett NJ, Sharma A, Allen CN, Wang WH, Yang CL, Ellison MP, Eckardt KU, Floege J, Bergmann MM, Schlieper G, Saritas T:
DH: Distal convoluted tubule Cl2 concentration is modulated via K1 Low adherence to CKD-specific dietary recommendations associates
channels and transporters. Am J Physiol Renal Physiol 319: F534–F540, with impaired kidney function, dyslipidemia, and inflammation. Eur J
2020 10.1152/ajprenal.00284.2020 PubMed Clin Nutr 75: 1389–1397, 2021 10.1038/s41430-020-00849-3
17. Arroyo JP, Ronzaud C, Lagnaz D, Staub O, Gamba G: Aldosterone par- PubMed
adox: differential regulation of ion transport in distal nephron. Physiology 31. Ben Salem C, Badreddine A, Fathallah N, Slim R, Hmouda H: Drug-
(Bethesda) 26: 115–123, 2011 10.1152/physiol.00049.2010 PubMed induced hyperkalemia. Drug Saf 37: 677–692, 2014 10.1007/s40264-
18. Clase CM, Carrero JJ, Ellison DH, Grams ME, Hemmelgarn BR, Jar- 014-0196-1 PubMed
dine MJ, Kovesdy CP, Kline GA, Lindner G, Obrador GT, Palmer BF, 32. Gilligan S, Raphael KL: Hyperkalemia and hypokalemia in CKD: Preva-
Cheung M, Wheeler DC, Winkelmayer WC, Pecoits-Filho R; Confer- lence, risk factors, and clinical outcomes. Adv Chronic Kidney Dis 24:
ence Participants: Potassium homeostasis and management of dyskalemia 315–318, 2017 10.1053/j.ackd.2017.06.004 PubMed
128 Nephrology Self-Assessment Program - Vol 20, No 2, January 2022

33. Jurkat-Rott K, Lehmann-Horn F: Genotype-phenotype correlation and hypoglycemia and potassium reduction. Pharmacotherapy 41: 598–607,
therapeutic rationale in hyperkalemic periodic paralysis. Neurotherapeutics 2021 10.1002/phar.2596 PubMed
4: 216–224, 2007 10.1016/j.nurt.2007.02.001 PubMed 46. Allon M, Copkney C: Albuterol and insulin for treatment of hyperkale-
34. Batlle D, Arruda J: Hyperkalemic forms of renal tubular acidosis: clinical mia in hemodialysis patients. Kidney Int 38: 869–872, 1990 PubMed
and pathophysiological aspects. Adv Chronic Kidney Dis 25: 321–333, 47. Montassier E, Lemoine L, Hardouin JB, Rossignol P, Legrand M: Insulin
2018 10.1053/j.ackd.2018.05.004 PubMed glucose infusion versus nebulised salbutamol versus combination of sal-
35. Tajima T, Morikawa S, Nakamura A: Clinical features and molecular butamol and insulin glucose in acute hyperkalaemia in the emergency
basis of pseudohypoaldosteronism type 1. Clin Pediatr Endocrinol 26: room: protocol for a randomised, multicentre, controlled study
109–117, 2017 10.1297/cpe.26.109 PubMed (INSAKA). BMJ Open 10: e039277, 2020 10.1136/bmjopen-2020-
36. Perrier R, Boscardin E, Malsure S, Sergi C, Maillard MP, Loffing J, Loff- 039277 PubMed
ing-Cueni D, Sørensen MV, Koesters R, Rossier BC, Frateschi S, 48. Allon M, Shanklin N: Effect of bicarbonate administration on plasma
Hummler E: Severe salt-losing syndrome and hyperkalemia induced by potassium in dialysis patients: interactions with insulin and albuterol. Am
adult nephron-specific knockout of the epithelial sodium channel alpha- J Kidney Dis 28: 508–514, 1996 10.1016/s0272-6386(96)90460-6
subunit. J Am Soc Nephrol 27: 2309–2318, 2015 10.1681/ASN. PubMed
2015020154 PubMed 49. Knauf H, Mutschler E: Diuretic effectiveness of hydrochlorothiazide and
37. Adachi M, Tajima T, Muroya K: Dietary potassium restriction attenuates furosemide alone and in combination in chronic renal failure. J Cardio-
urinary sodium wasting in the generalized form of pseudohypoaldoster- vasc Pharmacol 26: 394–400, 1995 10.1097/00005344-199509000-
onism type 1. CEN Case Rep 9: 133–137, 2020 10.1007/s13730-019- 00008 PubMed
00441-0 PubMed 50. Fliser D, Zurbr€ uggen I, Mutschler E, Bischoff I, Nussberger J, Franek E,
38. Wilson FH, Disse-Nicodeme S, Choate KA, Ishikawa K, Nelson- Ritz E: Coadministration of albumin and furosemide in patients with the
Williams C, Desitter I, Milford DV, Lipkin GW, Achard JM, Feely MP, nephrotic syndrome. Kidney Int 55: 629–634, 1999 PubMed
Dussol B, Berland Y, Unwin RJ, Mayan H, Simon DB, Farfel Z. Jeune- 51. Knauf H, Spahn H, Mutschler E: The loop diuretic torasemide in
maitre X, Lifton RP: Human hypertension caused by mutations in chronic renal failure: Pharmacokinetics and pharmacodynamics. Drugs
WNK kinases. Science 293: 1107–1112, 2001 PubMed 41[Suppl 3]: 23–34, 1991 10.2165/00003495-199100413-00005
39. Grimm PR, Coleman R, Delpire E, Welling PA: Constitutively active PubMed
SPAK causes hyperkalemia by activating NCC and remodeling distal 52. Zehnder C, Gutzwiller JP, Huber A, Schindler C, Schneditz D: Low-
tubules. J Am Soc Nephrol 28: 2597–2606, 2017 10.1681/ASN. potassium and glucose-free dialysis maintains urea but enhances potas-
2016090948 PubMed sium removal. Nephrol Dial Transplant 16: 78–84, 2001 10.1093/ndt/
40. Boyden LM, Choi M, Choate KA, Nelson-Williams CJ, Farhi A, Toka 16.1.78 PubMed
HR, Tokhonova IR, Bjornson R, Mane SM, Colussi G, Lebel M, Gor- 53. Bansal S, Pergola PE: Current management of hyperkalemia in patients
don RD, Semmekrot BA, Poujol A, V€alim€aki MJ, De Ferrari ME, San- on dialysis. Kidney Int Rep 5: 779–789, 2020 10.1016/j.ekir.2020.02.
jad SA, Gutkin M, Karet FE, Tucci JR, Stockigt JR, Keppler-Noreuil 1028 PubMed
KM, Porter CC, Anand SK, Whiteford ML, Davis ID, Dewar SB, Betti- 54. Shibata M, Kishi T, Iwata H: Clinical study of complications in dialyzed
nelli A, Fadrowski JJ, Belsha CW, Hunley TE, Nelson RD, Trachtman diabetics. Tohoku J Exp Med 141[Suppl]: 417–425, 1983 10.1620/tjem.
H, Cole TRP, Pinsk M, Bockenhauer D, Shenoy M, Vaidyanathan P, 141.suppl_417 PubMed
Foreman JW, Rasoulpour M, Thameem F, Al-Shahrouri HZ, Radhak- 55. Genovesi S, Boriani G, Covic A, Vernooij RWM, Combe C, Burlacu A,
rishnan J, Gharavi AG, Goilav B, Lifton RP: Mutations in kelch-like 3 Davenport A, Kanbay M, Kirmizis D, Schneditz D, van der Sande F,
and cullin 3 cause hypertension and electrolyte abnormalities. Nature Basile C; EUDIAL Working Group of ERA-EDTA: Sudden cardiac
482: 98–102, 2012 10.1038/nature10814 PubMed death in dialysis patients: different causes and management strategies.
41. Wakabayashi M, Mori T, Isobe K, Sohara E, Susa K, Araki Y, Chiga M, Nephrol Dial Transplant 36: 396–405, 2021 10.1093/ndt/gfz182
Kikuchi E, Nomura N, Mori Y, Matsuo H, Murata T, Nomura S, Asano PubMed
T, Kawaguchi H, Nonoyama S, Rai T, Sasaki S, Uchida S: Impaired 56. Yusuf AA, Hu Y, Singh B, Menoyo JA, Wetmore JB: Serum potas-
KLHL3-mediated ubiquitination of WNK4 causes human hypertension. sium levels and mortality in hemodialysis patients: A retrospective
Cell Rep 3: 858–868, 2013 10.1016/j.celrep.2013.02.024 PubMed cohort study. Am J Nephrol 44: 179–186, 2016 10.1159/000448341
42. Lindner G, Burdmann EA, Clase CM, Hemmelgarn BR, Herzog CA, PubMed
Małyszko J, Nagahama M, Pecoits-Filho R, Rafique Z, Rossignol P, 57. Foley RN, Gilbertson DT, Murray T, Collins AJ: Long interdialytic
Singer AJ: Acute hyperkalemia in the emergency department: A interval and mortality among patients receiving hemodialysis. N Engl J
summary from a Kidney Disease: Improving Global Outcomes Med 365: 1099–1107, 2011 10.1056/NEJMoa1103313 PubMed
conference. Eur J Emerg Med 27: 329–337, 2020 10.1097/MEJ. 58. Fotheringham J, Fogarty DG, El Nahas M, Campbell MJ, Farrington K:
0000000000000691 PubMed The mortality and hospitalization rates associated with the long interdia-
43. Peacock WF, Rafique Z, Clark CL, Singer AJ, Turner S, Miller J, Char lytic gap in thrice-weekly hemodialysis patients. Kidney Int 88: 569–575,
D, Lagina A, Smith LM, Blomkalns AL, Caterino JM, Kosiborod M; 2015 10.1038/ki.2015.141 PubMed
REVEAL-ED Study Investigators: Real world evidence for treatment of 59. Brunelli SM, Du Mond C, Oestreicher N, Rakov V, Spiegel DM: Serum
hyperkalemia in the emergency department (REVEAL-ED): A multicen- potassium and short-term clinical outcomes among hemodialysis patients:
ter, prospective, observational study. J Emerg Med 55: 741–750, 2018 Impact of the long interdialytic interval. Am J Kidney Dis 70: 21–29,
10.1016/j.jemermed.2018.09.007 PubMed 2017 10.1053/j.ajkd.2016.10.024 PubMed
44. Truhlar A, Deakin CD, Soar J, Khalifa GE, Alfonzo A, Bierens JJ, Bie- 60. Ferrey A, You AS, Kovesdy CP, Nakata T, Veliz M, Nguyen DV, Kalan-
rens JJLM, Brattebø G, Brugger H, Dunning J, Hunyadi-Anticevic S, tar-Zadeh K, Rhee CM: Dialysate potassium and mortality in a prospec-
Koster RW, Lockey DJ, Lott C, Paal P, Perkins GD, Sandroni C, Thies tive hemodialysis cohort. Am J Nephrol 47: 415–423, 2018 10.1159/
KC, Zideman DA, Nolan JP; Cardiac arrest in special circumstances sec- 000489961 PubMed
tion Collaborators: European Resuscitation Council Guidelines for 61. Yessayan L, Yee J, Frinak S, Szamosfalvi B: Continuous renal replace-
Resuscitation 2015: Section 4. Cardiac arrest in special circumstances. ment therapy for the management of acid-base and electrolyte imbalances
Resuscitation 95: 148–201, 2015 10.1016/j.resuscitation.2015.07.017 in acute kidney injury. Adv Chronic Kidney Dis 23: 203–210, 2016 10.
PubMed 1053/j.ackd.2016.02.005 PubMed
45. Moussavi K, Garcia J, Tellez-Corrales E, Fitter S: Reduced alternative 62. Flinn RB, Merrill JP, Welzant WR: Treatment of the oliguric patient
insulin dosing in hyperkalemia: A meta-analysis of effects on with a new sodium-exchange resin and sorbitol—A preliminary report. N
Nephrology Self-Assessment Program - Vol 20, No 2, January 2022 129

Engl J Med 264: 111–115, 1961 10.1056/NEJM196101192640302 71. Kovesdy CP, Gosmanova EO, Woods SD, Fogli JJ, Rowan CG, Hansen
PubMed JL, Sauer BC: Real-world management of hyperkalemia with patiromer
63. Meaney CJ, Beccari MV, Yang Y, Zhao J: Systematic review and meta- among United States Veterans. Postgrad Med 132: 176–183, 2020 10.
analysis of patiromer and sodium zirconium cyclosilicate: A new arma- 1080/00325481.2019.1706920 PubMed
mentarium for the treatment of hyperkalemia. Pharmacotherapy 37: 401– 72. Wiederkehr MR, Mehta AN, Emmett M: Case report: Patiromer-
411, 2017 10.1002/phar.1906 PubMed induced hypercalcemia. Clin Nephrol Case Stud 7: 51–53, 2019 10.5414/
64. Parks M, Grady D: Sodium polystyrene sulfonate for hyperkalemia. CNCS109782 PubMed
JAMA Intern Med 179: 1023–1024, 2019 10.1001/jamainternmed.2019. 73. Packham DK, Rasmussen HS, Lavin PT, El-Shahawy MA, Roger SD,
1291 PubMed Block G, Qunibi W, Pergola P, Singh B: Sodium zirconium cyclosilicate
65. Peacock WF, Rafique Z, Vishnevskiy K, Michelson E, Vishneva E, Zver- in hyperkalemia. N Engl J Med 372: 222–231, 2015 10.1056/
eva T, Nahra R, Li D, Miller J: Emergency potassium normalization NEJMoa1411487 PubMed
treatment including sodium zirconium cyclosilicate: A phase ii, random- 74. Zannad F, Hsu BG, Maeda Y, Shin SK, Vishneva EM, Rensfeldt M,
ized, double-blind, placebo-controlled study (ENERGIZE). Acad Emerg Eklund S, Zhao J: Efficacy and safety of sodium zirconium cyclosilicate
Med 27: 475–486, 2020 10.1111/acem.13954 PubMed for hyperkalaemia: The randomized, placebo-controlled HARMONIZE-
66. Bakris GL, Pitt B, Weir MR, Freeman MW, Mayo MR, Garza D, Stasiv Y, Global study. ESC Heart Fail 7: 54–64, 2020 10.1002/ehf2.12561
Zawadzki R, Berman L, Bushinsky DA: AMETHYST-DN Investigators: PubMed
Effect of patiromer on serum potassium level in patients with hyperkalemia 75. Fishbane S, Ford M, Fukagawa M, McCafferty K, Rastogi A, Spinowitz
and diabetic kidney disease: The AMETHYST-DN randomized clinical trial. B, Staroselskiy K, Vishnevskiy K, Lisovskaja V, Al-Shurbaji A, Guzman
JAMA 314: 151–161, 2015 10.1001/jama.2015.7446 PubMed N, Bhandari S: A phase 3b, randomized, double-blind, placebo-con-
67. Amdur RL, Paul R, Barrows ED, Kincaid D, Muralidharan J, Nobakht trolled study of sodium zirconium cyclosilicate for reducing the incidence
E, Centron-Vinales P, Siddiqi M, Patel SS, Raj DS: The potassium regu- of predialysis hyperkalemia. J Am Soc Nephrol 30: 1723–1733, 2019 10.
lator patiromer affects serum and stool electrolytes in patients receiving 1681/asn.2019050450 PubMed
hemodialysis. Kidney Int 98: 1331–1340, 2020 10.1016/j.kint.2020.06. 76. Roger SD, Lavin PT, Lerma EV, McCullough PA, Butler J, Spinowitz
042 PubMed BS, von Haehling S, Kosiborod M, Zhao J, Fishbane S, Packham DK:
68. Leon SJ, Harasemiw O, Tangri N: New therapies for hyperkalemia. Curr Long-term safety and efficacy of sodium zirconium cyclosilicate for
Opin Nephrol Hypertens 28: 238–244, 2019 10.1097/MNH. hyperkalaemia in patients with mild/moderate versus severe/end-stage
0000000000000500 PubMed chronic kidney disease: comparative results from an open-label, Phase 3
69. Agarwal R, Rossignol P, Romero A, Garza D, Mayo MR, Warren S, Ma study. Nephrol Dial Transplant 36: 137–150, 2021 10.1093/ndt/gfz285
J, White WB, Williams B: Patiromer versus placebo to enable spironolac- PubMed
tone use in patients with resistant hypertension and chronic kidney dis- 77. Roger SD, Spinowitz BS, Lerma EV, Fishbane S, Ash SR, Martins JG,
ease (AMBER): A phase 2, randomised, double-blind, placebo-controlled Quinn CM, Packham DK: Sodium zirconium cyclosilicate increases
trial. Lancet 394: 1540–1550, 2019 10.1016/S0140-6736(19)32135-X serum bicarbonate concentrations among patients with hyperkalaemia:
PubMed Exploratory analyses from three randomized, multi-dose, placebo-con-
70. Rossignol P, Williams B, Mayo MR, Warren S, Arthur S, Ackourey G, trolled trials. Nephrol Dial Transplant 36: 871–883, 2021 10.1093/ndt/
White WB. Agarwal R: Patiromer versus placebo to enable spironolac- gfaa158 PubMed
tone use in patients with resistant hypertension and chronic kidney dis- 78. Wesson DE: Sodium zirconium cyclosilicate for hyperkalemia: A collat-
ease (AMBER): Results in the pre-specified subgroup with heart failure. eral acid-base benefit? Nephrol Dial Transplant 36: 756–760, 2021 10.
Eur J Heart Fail 22: 1462–1471, 2020 10.1002/ejhf.1860 PubMed 1093/ndt/gfaa241 PubMed
130 Nephrology Self-Assessment Program - Vol 20, No 2, January 2022

Article
Metabolic Acidosis
Zhabiz Solhjou, MD
Renal Division, Brigham and Women’s Hospital, Boston, Massachusetts
Martina M. McGrath, MBChB, FASN
Renal Division, Brigham and Women’s Hospital, Boston, Massachusetts
Renal Division, Veterans Affairs Boston Healthcare System, Boston, Massachusetts

could be reported as “normal” when as low as 18 and as high as 34.


Learning Objectives The authors argue that this wide variability can result in “hidden”
yet clinically important acid-base disorders and suggest that adhering
1. To detail the diagnosis and management of ketoacidosis in
to a more narrow normal range of 23–29 could prevent missed diag-
specific clinical scenarios noses of milder acid-base disorders (3).
2. To describe the risk factors for and management of drug- Despite perturbations in [HCO32], the sum of circulating cati-
ons must always equal the sum of circulating anions, so that electro-
induced nonanion gap and high–anion gap metabolic acidosis
neutrality is maintained. The difference between these two sums
3. To discuss the significance of metabolic acidosis in CKD and its represents unmeasured circulating anions. The AG, calculated using
impact on CKD progression the equation Na1 (Cl2 1 HCO32), is a readily available tool to
assess acid-base disturbance. However, several points are worthy of
4. To review recent clinical trials of alkali therapy for treatment of
note when interpreting the AG. Depending upon the laboratory
metabolic acidosis in CKD methods used to measure electrolytes, the AG can vary widely
between laboratories. Albumin, with its negative charge, contributes
a large component of unmeasured anions, and variations in serum
Introduction albumin concentration can lead to changes in the AG such that for
Metabolic acidosis is associated with increased mortality and negative every 1-g reduction in serum albumin concentration, the AG falls
clinical outcomes, emphasizing the importance of timely diagnosis by 2.5 mEq/L (4). Differences in serum albumin concentrations
and treatment. Important causes of metabolic acidosis include net may explain much of the interindividual variability observed in the
accumulation of acid arising from excess intake (ingestion of acid- AG and should be considered when estimating a change from nor-
producing food, medications, or toxins, including toxic alcohols), mal AG. Spuriously low AG is seen in several situations, including
increased production (ketoacidosis or lactic acidosis), decreased excre- severe hypoalbuminemia, monoclonal gammopathies (again related
tion (renal tubular acidosis [RTA] types I and IV or CKD), or net to the charge properties of the circulating paraprotein), and severe
loss of bicarbonate ([HCO32]; gastrointestinal losses or RTA type hypertriglyceridemia or hyperparaproteinemia (secondary to pseudo-
II) (1,2). hyponatremia). Lithium is a cation and in high concentrations (.4
Low serum HCO32 is the hallmark of metabolic acidosis, but mEq/L), can lead to reduced AG. Bromide intoxication, such as in
analysis of arterial blood gas is required for accurate diagnosis of a pyridostigmine overdose, can lead to falsely elevated measurement of
specific acid-base disturbance. Measurement of pH, pCO2, and cal- serum chloride concentration, resulting in a low or extremely nega-
culation of [HCO32] (using the Henderson–Hasselbach equation) tive AG. These caveats aside, the AG is an important tool to differ-
from the arterial blood gas can identify chronic respiratory alkalosis, entiate the underlying cause of metabolic acidosis.
mixed metabolic and respiratory acidosis, or other acid-base High-AG metabolic acidosis results from accumulation of
derangements that can also present with low [HCO32]. Full assess- organic acids, whereas renal or GI losses of HCO32 lead to normal
ment of an acid-base disturbance also requires consideration of the AG metabolic acidosis (1,2). Normal AG metabolic acidosis was
adequacy of compensatory mechanisms to maintain normal pH and extensively reviewed in a prior NephSAP issue and is addressed in
calculation of the anion gap (AG). another article within this issue. Here, we provide an update on the
In clinical practice, total serum CO2 (CO2) is measured in latest studies on diagnosis, etiologies, and treatment of high-AG
venous blood and reported as a surrogate of [HCO32]. CO2 is cal- metabolic acidosis and review drug-induced metabolic acidosis.
culated as the sum of [HCO32] 1 dissolved CO2 (pCO2 3 0.031)
and typically runs 1–1.5 mEq/L higher than [HCO32]. In adults,
normal [HCO32] is between 23 and 29 mEq/L, but the reported Lactic Acidosis
“normal” range can vary widely across clinical laboratories. Kraut Lactic acid was identified as part of a glycolysis pathway in the
and Madias (3) reviewed the reporting of venous [HCO32] in 64 1940s, but in recent years, it is increasingly recognized as playing a
laboratories across multiple institutions. This study emphasized the key role in a diverse range of cellular processes (5). The average adult
prominent variation between laboratories, where serum [HCO32] generates around 1500 mM lactic acid daily from muscle (25%),

nephsap.org Copyright # 2022 by the American Society of Nephrology


Nephrology Self-Assessment Program - Vol 20, No 2, January 2022 131

skin (25%), brain (25%), red blood cells (20%), and intestine treatment in lactic acidosis. First, when bicarbonate buffers excess H1,
(10%). This circulates as lactate at physiologic pH and is cleared carbon dioxide is generated and can move intracellularly more freely
through the Cori and Krebs cycles. Lactate is the main precursor for than HCO32, thereby generating an intracellular acidosis. This effect
more than half of renal gluconeogenesis, which typically contributes can be particularly marked in patients given large volumes of intrave-
approximately 10% of overall blood glucose. Physiologic studies sug- nous bicarbonate or those with severe circulatory failure. Second, as
gest that this percentage can rise significantly in settings of stress, bicarbonate level acutely rises, levels of ionized calcium can drop sud-
such as hypoglycemia, fasting, exercise, and high adrenaline states denly, with negative effects on myocardial function. Measures to coun-
(up to 40% of total body gluconeogenesis), using lactate as a key pre- teract these negative effects include optimization of ventilation and
cursor (5). Lactate is also an important source of fuel for cardiomyo- consideration of coadministration of intravenous calcium (11).
cytes, particularly during vigorous exercise (6). It appears to play a
role in angiogenesis and wound healing (5). In the immune system,
lactic acid acts as an energy source for multiple innate and adaptive Ketoacidosis
immune cells and can contribute to immune response modulation Diabetic Ketoacidosis
by reducing the activation of proinflammatory macrophages and NK Large volume resuscitation with intravenous normal saline delivers a
cells and increasing production of anti-inflammatory cytokines (7,8) substantial chloride load, which can result in hyperchloremic acidosis,
Although lactic acid serves important physiologic functions, its
renal vasoconstriction, and hypoperfusion, and has been associated
rise in critically ill patients is associated with increased mortality. Lactic
with increased risk of AKI in several studies over the last decade
acidosis occurs when production exceeds degradation. Sepsis, local or
(12,13). The SMART and SALT-ED trials, published in New
global tissue hypoxia, and hypoperfusion are the most important
England Journal of Medicine in 2018, were pragmatic, open-labeled,
causes of hyperlactatemia in hospitalized patients. A number of other
cluster-randomized trials of balanced crystalloids (Ringer lactate or
causes are also worthy of note. Elevated circulating epinephrine levels,
Plasma-Lyte A) versus normal saline in critically ill patients admitted
as seen in cardiogenic/hemorrhagic shock, extensive trauma, cocaine
to an intensive care unit (SMART) and noncritically ill patients
ingestion, or severe asthma, lead to markedly increased anaerobic gly-
admitted through the emergency department (SALT-ED) (14,15).
colysis. This is an efficient mechanism for rapid production of ATP
Over 29,000 patients were enrolled between the two studies, and in
but also produces large amounts of lactate and resulting lactic acidosis.
both trials, treatment with balanced crystalloid was associated with a
Liver dysfunction in sepsis or fulminant acute liver failure can lead to
significant reduction in the prespecified composite outcome compris-
both increased lactate production and reduced clearance, and it is a
ing death, new need for RRT, and persistent renal dysfunction at 30
poor prognostic sign (9). Nonhypoxemic lactic acidosis primarily arises
days. No difference in mortality as a single end point was observed
due to exposure to drugs or toxins that impair mitochondrial oxidative
(14,15). In SMART, the greatest benefit was observed in patients
phosphorylation, including propofol, metformin, salicylates, nucleoside
with sepsis and higher predicted mortality. Limitations of these stud-
reverse transcriptase inhibitors, cyanide, and toxic alcohol (9).
ies include open-label design and the lack of significant difference for
In high-AG metabolic acidosis, the ratio of changes in AG to
individual components of the composite end point. Based on the
changes in serum bicarbonate (DAG/DHCO32) is used to detect
number of events, the studies were not powered to detect a difference
coexisting acid-base derangements. When high-AG metabolic acido-
for individual components. However, the trend for benefit was simi-
sis is the sole acid-base disorder, an equivalent change in AG and
lar for each component of the composite end point, and it has been
serum bicarbonate is observed. A DAG/DHCO32.1 suggests the
estimated that among patients with sepsis, the number needed to
presence of an additional metabolic alkalosis, whereas DAG/
treat to prevent the composite end point was 20 (16).
DHCO32,1 implies a coexisting normal AG metabolic acidosis.
Normal saline is generally considered the fluid of choice to treat
Lactic acidosis is traditionally thought of as a cause of high-AG aci-
patients who present with diabetic ketoacidosis (DKA). A subgroup
dosis where DAG/DHCO32 is close to one. In lactic acidosis, lactate
analysis of patients enrolled in SMART and SALT-ED was recently
excretion falls due to decreased renal perfusion and function, and
published, examining the effect of normal saline versus balanced
lactate molecules accumulate as unmeasured anions in extracellular
crystalloids in patients admitted with DKA. This secondary analysis
fluid compartment, whereas the H1 buffering occurs intracellularly
included 172 patients: 94 given balanced crystalloids and 78 given
and in bone. This differential distribution of lactate and buffered
saline. The primary outcome was time from emergency department
H1 can lead to an increase in DAG/DHCO32 in lactic acidosis
admission to resolution of DKA. When compared with those treated
(10). In animal models of lactic acid infusion, the ratio of DAG/
with saline, patients treated with balanced crystalloids had earlier
DHCO32 is 1.0 early in the course but increases to 1.6–1.8 after
several hours (4). Similarly, in humans, Rudkin et al. (10) studied resolution of DKA (13 versus 16.9 hours). They also had higher
45 trauma patients with serum lactate of .2.1 mmol/L and showed bicarbonate levels, lower serum chloride, and earlier discontinuation
the presence of DAG/DHCO32 1.86 within the first hours of devel- of intravenous insulin infusion (17). A number of other smaller
opment of hypovolemic shock, suggesting a similar effect. studies have shown similar results, suggesting that there may be ben-
Treatment of lactic acidosis is primarily aimed at reversal of the efit to using balanced crystalloids as an alternative to normal saline
underlying abnormality and prompt restoration of normal tissue perfu- in the management of patients with DKA.
sion. There are no clinical trial data that show reductions in mortality
after treatment with intravenous bicarbonate and limited evidence of
Euglycemic Ketoacidosis with Sodium-Glucose
benefit. In general, treatment with bicarbonate should be limited to Transporter-2 Inhibitors
those patients with severe acidosis, unresponsive to resuscitative meth- For patients with CKD (eGFR .30 ml/min per 1.73 m2) and type
ods. There are several important risks for intravenous bicarbonate 2 diabetes, sodium-glucose transporter-2 (SGLT2) inhibitors and
132 Nephrology Self-Assessment Program - Vol 20, No 2, January 2022

metformin are recommended as first-line oral hypoglycemic agents. solution, and despite stable blood glucose levels, patients were found
SGLT2 inhibitors have been shown to have beneficial effects on a to have elevated b-hydroxybutyrate levels. Mean time from initiation
range of cardiovascular outcomes and significantly reduce the risk of of CRRT to recognition of ketoacidosis was 57 hours. In all cases, a
progression of CKD, development of ESKD, and death (18). supplemental glucose infusion was initiated, which allowed an
SGLT2 inhibitors improve glycemic control by reducing proximal increase in insulin administration, and ketosis resolved over an aver-
tubular reabsorption of glucose, thereby promoting glycosuria and age of 14 hours. The authors hypothesized that in these cases, major
an osmotic diuresis. Urinary glucose loss reduces insulin secretion contributors to ketosis were the lack of caloric intake in critically ill
and promotes glucagon release. This reduced insulin-to-glucagon patients along with glucose loss in glucose-free CRRT, both of which
ratio promotes a switch to lipolysis and promotes hepatic ketoacid promote gluconeogenesis and ketosis. Furthermore, in diabetic
production. Physiologic studies have demonstrated increases in levels patients with underlying insulin deficiency, the resulting “well-
of ketoacids in SGLT2 inhibitor–treated patients with and without controlled” blood sugars led to a reduction in administered exoge-
underlying diabetes (19). However, the occurrence of ketoacidosis nous insulin, increasing their risk of ketosis. A persistent high-AG
in SGLT2 inhibitor–treated patients was rare in initial clinical trials. acidosis in intensive care unit patients treated with CRRT should
Postmarketing surveillance suggests that ketosis occurs much more alert clinicians to search for possible coexisting ketosis.
commonly in clinical practice than initially recognized (20). The
majority (.70%) of such cases present with euglycemic ketoacidosis Drug-Induced Metabolic Acidosis
(glucose ,250 mg/dl), thought to be driven by SGLT2 A large number of drugs used in everyday clinical practice are associ-
inhibitor–mediated ketogenesis (21,22). ated with the development of metabolic acidosis via a variety of
Patients with euglycemic ketoacidosis frequently present with mechanisms. An exhaustive review of all agents causing acidosis and
vague symptoms, such as abdominal pain, nausea, and fatigue. the associated pathophysiology are beyond the scope of this article.
Although the AG is significantly elevated, diagnosis may be delayed However, the compounds and associated clinical features
due to the absence of significant hyperglycemia. Recommended treat- highlighted in Tables 1 and 2 are of particular interest to contempo-
ment is similar to hyperglycemic DKA, although the degree of volume rary nephrology practice.
depletion is significantly less due to the absence of hyperglycemia.
SGLT2 inhibitors are currently Food and Drug Administration
(FDA) approved only for use in type 2 diabetes, and their use is
Sodium Thiosulfate
associated with higher risk of ketoacidosis in patients with type 1 Sodium thiosulfate (STS) is commonly used in the treatment of cal-
diabetes. Other risk factors for euglycemic ketoacidosis include ciphylaxis, despite a lack of strong evidence to support its use. The
female gender, excessive reduction or discontinuation of insulin, mechanism of action of STS is unclear. STS is cleared equally by
major surgery/trauma, intercurrent illnesses that limit the ability to the kidneys and liver, and thus, its clearance declines by half in
eat/drink, alcohol consumption, and low carbohydrate intake, ESKD (28), where its use is frequently associated with increased AG
including ketogenic diets (20,23,24). Euglycemic ketoacidosis has acidosis. STS-induced high-AG acidosis is thought to arise due to
also been reported in patients taking SGLT2 inhibitors who develop accumulation of the metabolite hydrogen sulfide, which can inhibit
coronavirus disease 2019 (25). To reduce the risks of euglycemic mitochondrial cytochrome complex IV and interfere with oxidative
ketoacidosis, SGLT2 inhibitors should be used with caution in phosphorylation and ATP generation, thereby resulting in lactic aci-
patients with type 1 diabetes and discontinued in the setting of dosis (29).
intercurrent illness and 48–72 hours prior to planned major surgery. In a retrospective study of 35 ESKD patients treated with STS,
Decreases in insulin dosage of .20% should be avoided, and 33 (94%) had an increase in mean AG following initiation of STS
patients should be counseled to avoid excessive alcohol intake or treatment. The average change in AG was 6.6 mmol/L on a dose of
ketogenic diets (26). Euglycemic ketoacidosis appears to have a 25 g per hemodialysis session and 2.6 mmol/L after STS 12.5 g per
higher mortality than classic DKA (20), and therefore, SGLT2 treatment. However, the change in AG varied widely, and an
inhibitors should be permanently discontinued in affected patients. increase in AG of 18 mmol/L was observed in several patients (30).
Combination treatment with metformin and an SGLT2 inhibi- The degree of acidosis appears unpredictable, and although most
tor was commonly used in cardiovascular outcome trials and has cases are mild, there have been reports of severe high-AG metabolic
been FDA approved. Both agents can cause metabolic acidosis acidosis presenting with weakness, arrhythmia, seizure, and altered
through differing mechanisms, but to date, there have been no mental status associated with treatment with STS (31,32).
reports of increased risk of metabolic acidosis in patients treated
with both agents. Metformin
Metformin is a first-line agent recommended for treatment of type 2
Continuous RRT and Ketoacidosis diabetes in patients with eGFR .30 ml/min per 1.73 m2, and its
A recent case series described a group of eight critically ill diabetic use has been associated with reduced mortality in patients with
patients who developed euglycemic ketoacidosis while on continuous CKD (33). Generally safe and well tolerated, metformin reduces
RRT (CRRT) (27). Three patients required insulin therapy prior to blood glucose through several mechanisms but primarily through
admission; the others were on oral hypoglycemic agents (no SGLT2 inhibition of hepatic gluconeogenesis (34). Metformin inhibits the
inhibitors) or no treatment. After an initial improvement after start- mitochondrial respiratory chain components and mitochondrial
ing CRRT, patients developed a rising AG and falling serum glycerophosphate dehydrogenase, inhibiting the conversion of glyc-
[HCO32]. All patients were dialyzed using a glucose-free CRRT erol and lactate to glucose. These combined effects reduce
Nephrology Self-Assessment Program - Vol 20, No 2, January 2022 133

Table 1. Drug-induced high–anion gap metabolic acidosis: Mechanisms and clinical features (39)

Risk Factors and Clinical


Agent Drug Class Mechanism of Acidosis Type of Acidosis Features

Propofol Anesthetic agent Disrupts mitochondrial Lactic acidosis Increased risk with high dose and
function and impairs prolonged exposure
oxidative Propofol infusion syndrome:
phosphorylation Estimated incidence
approximately 1% of ICU
patients (77)
Associated with rhabdomyolysis,
hypertriglyceridemia, and AKI
Linezolid Antimicrobial Disrupts mitochondrial Lactic acidosis Increased risk with liver/renal
function and impairs dysfunction, higher doses,
oxidative prolonged exposure
phosphorylation Associated with myelosuppression
and liver toxicity (78)
Lamivudine abacavir Nucleoside/ Disrupts mitochondrial Lactic acidosis Used for treatment of HIV,
Zidovudine tenofovir NRTIs function and impairs Fanconi syndrome hepatitis B, and HIV pre-
Stavudine didanosine oxidative with non-AG exposure prophylaxis
phosphorylation metabolic acidosis Mild increase in lactate .20% of
seen with didanosine, treated patients, severe in
cidofovir, lamivudine, 1%–2% (79)
stavudine, tenofovir Increased risk with liver/renal
dysfunction, alcohol use,
obesity, pregnancy, and
combination therapy
Mortality up to 50% in severe
cases
High volume of distribution,
prolonged intracellular half-life.
NRTIs are less readily
dialyzable, but this may be
considered in severe cases (80)
Acetaminophen Analgesic Depletion of glutathione, High-AG metabolic Seen in chronic ingestion of
accumulation of acidosis acetaminophen, including at
pyroglutamic acid (5- Pyroglutamic acidosis therapeutic doses
oxoproline) Urinary 5-oxoproline elevated
Risk factors include malnutrition,
women, underlying liver/renal
dysfunction, sepsis, and alcohol
abuse
Concomitant use of antibiotics,
such as dicloxacillin/oxacillin,
may increase risk (81)
Treatment with n-acetylcysteine
may supplement cysteine, a
precursor of glutathione
Ethanol Alcohol Ketosis Poor carbohydrate intake, chronic
volume depletion, and insulin
(continued on next page)
134 Nephrology Self-Assessment Program - Vol 20, No 2, January 2022

Table 1. Drug-induced high–anion gap metabolic acidosis: Mechanisms and clinical features (39) (cont.)

Risk Factors and Clinical


Agent Drug Class Mechanism of Acidosis Type of Acidosis Features

High-AG metabolic deficiency. Resolves when


acidosis treated with fluids and nutrition
Ketoacidosis
Methanol Toxic alcohols Generation of acid High-AG metabolic Certain laboratory instruments
Ethylene glycol metabolites (formic acid acidosis differentiate poorly between
and oxalate) High osmolar gap metabolites of ethylene glycol
and lactate and may give falsely
high lactate readings in ethylene
glycol ingestion (82)
Osmolal gap may not identify
small ingestions and may have
closed in those presenting late
Treatment with fomepizole
(alcohol dehydrogenase
inhibitor) and hemodialysis
Propylene glycol Carrier vehicle Lactic acidosis High-AG metabolic Used to administer intravenous
for Generation of acid acidosis lorazepam and diazepam
intravenous metabolites High osmolar gap Metabolized to lactic acid,
infusions pyruvate, and acetate
Increased risk in liver/renal
dysfunction and those at risk of
DKA
Valproate Antiepileptic Mitochrondrial High-AG metabolic Overdose or intoxication is
dysfunction acidosis associated with hepatoxicity,
hyperammonemia,
encephalopathy, and metabolic
acidosis
Emergent hemodialysis is
indicated for severe intoxication,
shock, and cerebral edema.
Valproate is effectively cleared
by dialysis (83)
Salicylates NSAID Disrupts mitochondrial High-AG metabolic Toxicity is also associated with
function and impairs acidosis respiratory alkalosis due to
oxidative stimulation of the medullary
phosphorylation respiratory center
Urinary alkalinization with
intravenous bicarbonate
increases renal clearance; oral
bicarbonate should be avoided
Hemodialysis is indicated for
severe intoxication, including
respiratory distress, altered
mental status, and levels
.90 mg/dl (84)

ICU, intensive care unit; NRTI, nucleotide reverse transcriptase inhibitor; AG, anion gap; DKA, diabetic ketoacidosis; NSAID, nonsteroidal anti-inflammatory drug.
Nephrology Self-Assessment Program - Vol 20, No 2, January 2022 135

gluconeogenesis while also reducing degradation and increasing gen- CKD patients have metabolic acidosis, with increasing prevalence at
eration of lactate (35,36). lower levels of renal function. In the Chronic Renal Insufficiency
Metformin-associated lactic acidosis (MALA) is a rare but life- Cohort study, the prevalence of acidosis was reported as 7% in stage
threatening complication of treatment, with reported incidence of 2, 13% in stage 3, and 37% in stage 4 CKD participants (42,43)
,10/100,000 patient-years. A recently published retrospective, Untreated metabolic acidosis in the CKD population has been
nested case-control study using data from 2003 to 2013 in the Vet- associated with a range of adverse health consequences, including
erans Affairs Healthcare system identified 2662 patients with type 2 loss of bone mineral density, muscle wasting, oxidative stress, and
diabetes, CKD, and a newly dispensed antihyperglycemic agent who negative cardiovascular outcomes (42). Multiple large cohort studies
developed lactic acidosis (defined as lactate .5 mEq/L). A control have reported an association between the presence of metabolic aci-
group of 26,000 patients was matched for demographic characteris- dosis and greater mortality in patients with CKD, ESKD and renal
tics, stage of CKD, and year of analysis. When comparing metfor- transplant recipients. These observations have led to the current rec-
min exposed with nonexposed, those treated with metformin had an ommendations, to treat all patients with a serum [HCO32]
incidence of lactic acidosis of 2/1000 patient-years, with a rate dif- ,22mEq/L with alkali therapy (44).
ference of 0.04/1000 patient-years compared with nontreated refer- Multiple factors have been identified to influence serum
ence group. A significant increase in risk of lactic acidosis was [HCO32] level in patients with CKD, independent of GFR. Factors
observed with worsening renal function, where CKD stages 1 and 2 associated with worsening metabolic acidosis in CKD include hyper-
were at baseline risk, CKD stages 3a and 3b had a two- to three-fold kalemia, albuminuria, smoking, anemia, dietary protein intake, and
increase in risk, and CKD stages 4 and 5 had a five- to seven-fold selected medications (43,45). The typical Western diet is particularly
increase in risk of lactic acidosis (37). Among CKD stages 3a and acid generating and high in protein, and it can result in net acid
3b patients, although there was increased relative risk in metformin- production of up to 1 mEq/kg per day (46). A large intake of animal
treated patients, the absolute risk remains small. The findings of this protein promotes acidosis, and several studies have shown that an
study support the current recommendations that metformin is safe alternative diet of alkali-producing fruits and vegetables can prolong
in patients with eGFR.30 ml/min, but with progressive CKD, the duration of eubicarbonatemia (47,48). Medications are also an
alternative agents may be more suitable. important factor in the development of metabolic acidosis; for
MALA most commonly occurs in the presence of other condi- example, loop diuretics increase serum [HCO32], and others, such
tions that promote accumulation of lactic acid. Important risk fac- as inhibitors of the renin-angiotensin system, promote acidosis (49).
tors include AKI, volume depletion, hepatic impairment, congestive Gardener et al. (50) studied the effect of medications with an acid
heart failure, shock/tissue hypoperfusion, advanced age, and con- salt (i.e., hydrogen chloride) on serum bicarbonate levels in 74
comitant use of radiocontrast agents or other nephrotoxins (36). United States veterans with diabetes and CKD stages 2–4 with eubi-
Higher serum levels of metformin have been associated with poorer carbonatemia. In this cohort, 25% of prescribed medications con-
outcomes, where level .5 mg/ml was associated with up to 30% tained an acid salt, with metformin hydrochloride and gabapentin
mortality compared with 11% in patients with levels ,5 mg/ml hydrochloride having the highest acid load potential. Patients who
(38). However, pH and lactate levels are not predictive of outcome received the highest load of acid from medications had [HCO32]
in MALA (39). 1.7 mEq/L lower and AG 2.2 mEq/L higher than patients receiving
Metformin is a small, nonprotein-bound molecule and is effi- lower acid load (50).
ciently cleared by hemodialysis. Emergent RRT is indicated in cases
of severe MALA and should be initiated in cases where lactate level
is .20 mmol/L, pH#7.0, shock, decreased level of consciousness, Eubicarbonatemic Metabolic Acidosis and Urine Citrate
and failure of supportive measures (40). Additional indications may Experimental data indicate that patients with higher levels of GFR
include the presence of AKI, liver dysfunction, or other significant or serum [HCO32] within the normal range also have evidence of
comorbidities. Due to greater efficiency of clearance, hemodialysis is acid retention (known as eubicarbonatemic acidosis) and reinforce
preferred, but there are many case reports of successful treatment of the concept that low serum [HCO32] occurs later, when buffering
critically ill, unstable MALA patients with CRRTs. RRT should be mechanisms have been overwhelmed. Eubicarbonatemic acidosis has
continued until lactate level is below 3 mmol/L and pH is .7.35. been associated with more rapid CKD progression in a number of
large cohort studies. In patients enrolled in African American Study
Chronic Metabolic Acidosis in CKD of Kidney Disease and Hypertension and the NephroTest cohort
(CKD stages 2–5), eubicarbonatemic patients in the lowest tertile of
Background baseline urine ammonium excretion had a higher odds of developing
With declining renal function, there is a reduction in renal ammo- hypobicarbonatemic metabolic acidosis within a year and greater
niagenesis and a reduced ability to generate bicarbonate in the proxi- risk of progression to ESKD (41,46,51). Conversely, in United
mal and distal tubules. The loss of ability to efficiently excrete States veterans with CKD and CKD patients from the Cleveland
endogenous and exogenous H1 leads to accumulation of H1 Clinic Foundation, those with a high-normal serum [HCO32] had
beyond the buffering capacity of muscle and bone and eventually, to lower risk of mortality or GFR decline than those with low-normal
the development of metabolic acidosis (41). Although multiple fac- serum [HCO32] (42). As a result, an alternative paradigm for the
tors play a role, low GFR and decreased capacity for acid excretion management of metabolic acidosis in CKD has been proposed,
are the key factors in the development of metabolic acidosis in CKD where earlier introduction of alkali therapy can be considered as a
patients (42). Using a cutoff of [HCO32] #22mEq/L, 15% of all measure to delay CKD progression (Figure 1).
136 Nephrology Self-Assessment Program - Vol 20, No 2, January 2022

Measuring urine ammonium excretion is not feasible in stan- upregulated along the course of the tubule to collecting duct, lead-
dard clinical practice, and urine citrate has been suggested as an ing to an increase in urinary acid excretion within hours to days
alternative method to identify patients with H1 retention and eubi- (56). However, many of these compensatory mechanisms also upre-
carbonatemia. With intracellular acidification of renal tubular epi- gulate cellular pathways associated with fibrosis and endothelial cell
thelium, there is increased proximal tubular reabsorption of citrate inflammation.
as a mechanism to regenerate bicarbonate, resulting in hypocitraturia Reduction in interstitial and intracellular pH stimulates local
(41). In a 2019 study, Goraya et al. (52) hypothesized that urine cit- release of angiotensin II, aldosterone, and endothelin-1 (ET-1).
rate measurement could identify patients with acid (H1) retention, Angiotensin II increases proximal tubular acid excretion via upregu-
including those without overt metabolic acidosis. They assigned 66 lation of NH3 and H1-ATPase and increases ammoniagenesis.
patients with albuminuric CKD stages 1 and 2 to a base-producing However, chronic elevation in intrarenal angiotensin II leads to
diet for 30 days. At study end, they observed a reduction in H1 increased local aldosterone, elevated intraglomerular filtration pres-
retention with an increase in urine citrate, suggesting that urine cit- sure, and activation of proinflammatory and profibrotic pathways
rate may reflect changes in H1 retention in nonacidotic patients within the kidney, resulting in worsening proteinuria, fibrosis, and
with CKD (52). In a subsequent study, the same group of investiga- tubular atrophy (56). ET-1 also increases tubular H1 secretion to
tors treated 120 patients with CKD stage 2 and total CO2 .24 reduce acid retention, but sustained levels of ET-1 lead to foot pro-
mEq/L with oral NaCl, NaHCO3, or usual care and measured urine cess effacement and similarly activate local profibrotic and proin-
citrate excretion at baseline and study end. Patients treated with oral flammatory pathways, exacerbating proteinuria and tubulointerstitial
NaHCO3 had no significant change in H1 retention or urine citrate injury (56,57). Increasing evidence suggests that that these compen-
excretion during the 10-year follow-up. In contrast, the NaCl and satory mechanisms are upregulated early in CKD and may have det-
usual care groups had significant increases in H1 retention and rimental effects on CKD progression. In a study of 120 patients
reduction in urine citrate but with little change in plasma CO2 or with macroalbuminuria and CKD stage 2, serum [HCO3] and acid-
AG (both remained within the normal range). The authors suggest base parameters were similar to patients with CKD stage 1. How-
that urine citrate may identify patients with subclinical H1 reten- ever, those with CKD stage 2 showed H1 retention, as assessed by
tion that is not apparent on routine laboratory measurement and reduced net acid excretion following an acid load, and elevated
who may benefit from alkali therapy (47) (Figure 1). However, plasma and urine levels of aldosterone and ET-1 compared with
despite these interesting findings, no studies have identified a rela- CKD stage 1. Plasma and urine aldosterone and ET-1 levels normal-
tionship between urine citrate and renal outcomes to date, and this ized after 30 days of NaHCO3 treatment, suggesting alkali therapy
remains an area of investigation. may be beneficial for such patients (58).
Increased ammoniagenesis is a central mechanism to excrete an
Mechanisms of Injury Associated with Chronic Metabolic acid load. In CKD, ammoniagenesis is initially maintained despite a
Acidosis in CKD reduced number of functional nephrons, due to increases in single-
In observational studies, both eubicarbonatemic and overt metabolic nephron ammonia generation. However, ammonia activates the
acidosis have been associated with progression of CKD (49,53–55). alternative pathway of complement, leading to tubulointerstitial
The kidney has multiple mechanisms to detect and respond to inflammation and fibrosis (59). In studies of nephrectomized rats,
changes in systemic acid-base status. In response to increased acid treatment with NaHCO3 reduced ammonia concentration, and
load, ammoniagenesis increases, and H1 transporters are was associated with reductions in proteinuria, tubular injury and

A Current paradigm B Alternative paradigm C Alternative paradigm


Renal acid excretion (mEq/d)

No alkali therapy No alkali therapy


X X
Total CO2 (mEq/L)

Total CO2 (mEq/L)

Total CO2 (mEq/L)

X–1 X–1
85% of CKD No alkali therapy
Y
No alkali therapy
Treat subclinical
metabolic acidosis <Y
Treat subclinical
metabolic acidosis
22 22 22
21 21 21
15% of CKD 15% of CKD 15% of CKD
Treat metabolic acidosis Treat metabolic acidosis Treat metabolic acidosis

Figure 1. Urinary acid excretion may identify patients with CKD and normal total CO2 who have subclinical metabolic acidosis. (A) The cur-
rent metabolic acidosis treatment paradigm is to withhold alkali until serum total CO2 falls below 22 mEq/L, in which case only 15% of
patients with CKD receive alkali therapy. (B) An alternative paradigm assumes that patients with CKD and total CO2 below a threshold value
(represented as X) have either overt acidosis or subclinical metabolic acidosis and are potential candidates for alkali therapy. (C) Another possible
approach would be to treat patients with CKD with total CO2 of 22 mEq/L and those with subclinical metabolic acidosis defined by a normal
total CO2 and urinary acid excretion below a threshold value (represented as Y) (42).
Nephrology Self-Assessment Program - Vol 20, No 2, January 2022 137

Table 2. Drug-induced nonanion gap metabolic acidosis: Mechanisms and clinical features (39)

Mechanism of Risk Factors and Clinical


Agent Drug Class Acidosis Type of Acidosis Features

Topiramate Antiepileptic Inhibition of carbonic Non-AG metabolic Decreased urinary citrate levels
anhydrase acidosis Increased risk of calcium phosphate
calculi
Sevelamer Phosphate binder Release of chloride in Non-AG metabolic Increased risk in patients with CKD
hydrochloride GI tract leads to acidosis and limited renal HCO32
bicarbonate loss in regeneration
stool
Trimethoprim Antimicrobial Reduced H1 excretion Non-AG metabolic Blockade of Na1 channel on luminal
acidosis surface of principal cell reduces
lumen negative potential and limits
H1 and K1 excretion
Hyperkalemia and acidosis
Cyclosporin Calcineurin inhibitors Reduced H1 excretion Non-AG metabolic Blockade of Na1, K1-ATPase on
Tacrolimus acidosis principle cell, decreasing Na1
reabsorption, and K1 and H1
secretion
Acetazolamide Carbonic anhydrase Inhibition of carbonic Non-AG metabolic Bicarbonaturia
inhibitor anhydrase acidosis Increased risk in elderly, CKD, and
DM
Amiloride Potassium-sparing Reduced H1 excretion Non-AG metabolic Blockade of Na1 channel on luminal
Triameterene diuretics acidosis surface of principal cell reduces
lumen negative potential and limits
H1 and K1 excretion
Hyperkalemia and acidosis
ACEi Inhibitors of the renin- Reduced H1 excretion Non-AG metabolic Aldosterone and angiotensin II
ARB angiotensin- acidosis promote transport of H1 into
Renin inhibitors aldosterone system tubular lumen via H1-ATPase;
Aldosterone inhibited by renin-angiotensin-
antagonists aldosterone system blockade
Reduced Na1 reabsorption reduces
lumen negative potential and limits
H1 excretion via H1-ATPase
Hyperkalemia suppresses renal
ammoniagenesis and excretion of an
acid load

AG, anion gap; GI, gastrointestinal; DM, diabetes mellitus; ACEi, angiotensin converting enzyme inhibitor; ARB, angiotensin receptor blocker.

decreased deposition of C3 and C5–C9 complement components, clinical trial of bicarbonate supplementation in patients with hyper-
compared with controls, further demonstrating the intra-renal effects tensive nephropathy and mean eGFR of 75 ml/min. At 5 years
of correction of acidosis (60). follow-up, patients treated with NaHCO3 had greater preservation of
eGFR and a slower trajectory of GFR decline than control groups
(61). More recently, the UBI study group randomized 740 patients
Treatment of Metabolic Acidosis in CKD from Italy with CKD stages 3–5, metabolic acidosis (serum HCO32
Although the effects of metabolic acidosis on muscle, bone, and .18 and ,24 mEq/L), and eGFR of 30612 ml/min per 1.73 m2
CKD progression have been known for some time, until recently it to either standard care or NaHCO3 treatment with a 3-year follow-
was unclear if treatment with alkali could reverse these effects and up period. Those treated with NaHCO3 had lower incidence of dou-
delay CKD progression. Several recent trials have addressed this ques- bling of serum creatinine (6.6% versus 17%) and significantly lower
tion and demonstrated the benefits of treatment, as described below. rates of development of ESKD (6.9% versus 12.3%) and death
In 2010, Mahajan et al. (61) reported a randomized, controlled (3.1% versus 6.8%) (62) (Figure 2). No significant effect was
138 Nephrology Self-Assessment Program - Vol 20, No 2, January 2022

Figure 2. Time to (A) creatinine doubling, (B) all-cause mortality, and (C) initiation of RRT, according to the Kaplan–Meyer methods (P value
refers to the unadjusted log-rank tests). Analyses are carried out as per intention to treat, and no data imputation on missing data is performed (62).
Nephrology Self-Assessment Program - Vol 20, No 2, January 2022 139

Figure 3. (A) The forest plot shows a slower decline in eGFR at the end of the study period with oral alkali supplementation or reduction of
dietary acid intake. I2 for the combined effect estimate: 39% (95% confidence interval [95% CI], 0% to 66%). df, degrees of freedom; IV,
inverse variance (64). (B) The forest plot shows the potentially reduced risk of ESKD with oral alkali supplementation or reduction of dietary
acid intake. I2 for the combined effect estimate: (95% CI, 0% to 73%). M-H, Mantel–Haenszel (64).
140 Nephrology Self-Assessment Program - Vol 20, No 2, January 2022

observed of NaHCO3 on BP, total body weight, or hospitalizations. dilation during the course of 14 weeks of observation (66). Prior
However, it is noteworthy that this study excluded patients with heart studies have reported that increases in brachial flow–mediated diam-
failure NYHA III and IV and those with BP .150/90 mm Hg. eter are associated with reduced risk of cardiovascular event, but this
In the Bicarbonate Administration to Stabilize eGFR Pilot trial, study was not designed to examine such outcomes. Serum phos-
a multicenter, randomized trial, Raphael et al. (63) assessed the phate and FGF23 also increased in this small study, suggesting that
safety and tolerability of two different NaHCO3 doses, 0.5 or 0.8 the effect of NaHCO3 on MBD parameters, vascular calcification,
mEq/kg lean body wt per day, versus matched placebo over 28 and cardiovascular disease may warrant further investigation in
weeks in 194 adults with eGFR of 20–44 or 45–59 ml/min per larger, longer-term studies.
1.73 m2 with urinary albumin-to-creatinine $50 mg/g and serum There are a number of unanswered questions around alkali
[HCO3] 20–28 mEq/L. Both NaHCO3 doses were well tolerated therapy in CKD, including the most effective dose and the long-
over the study period, with no significant difference in weight, term effect of treatment on renal and cardiovascular outcomes in
diuretic requirement, BP, or hospitalizations across the three groups patients with CKD. There are some concerns around the safety of
by week 28 of follow-up. Mean urinary ammonium excretion was NaHCO3 in CKD, although it has long been argued that chloride-
25% lower in the group that received the high dose of NaHCO3, containing salts have a greater propensity to induce fluid retention
suggesting that this may be an appropriate dose for a future larger and edema than when sodium is given with bicarbonate (42). As
study (63). In this study, no differences in GFR were observed. described above, alkali-producing fruits and vegetables have been
However, a small but statistically significant increase in the urine suggested as a potential therapy for treatment of metabolic acidosis,
albumin-to-creatinine ratio was observed in the high-dose NaHCO3 and this intervention has been associated with increased urine citrate
group. This was an unexpected finding, not observed in the UBI excretion, reduced albuminuria, lower systolic BP, and weight loss,
study or in a meta-analysis discussed below, and is of unclear signifi- along with many other cardiovascular benefits. However, evidence is
cance, but it necessitates future follow-up. limited on the best practice of prescribing these fruits and vegetables,
A 2019 systematic review and meta-analysis of treatment of and hyperkalemia remains a concern with this approach, particularly
metabolic acidosis in CKD using either oral alkali or dietary inter- in those with later stages of CKD. In future studies, investigation of
vention included data from 14 clinical trials, comprising 1394 par- the coadministration of newer potassium binders may facilitate safe
ticipants (64). Four of the studies included patients treated for consumption of alkali-producing fruits and vegetables in the CKD
eubicarbonatemic metabolic acidosis. Eight studies used oral alkali, population (67).
five studies had a dietary intervention, and one study used both. In
the pooled analysis, treatment of metabolic acidosis with oral alkali
supplementation or a reduction of dietary acid intake increased Veverimer
serum bicarbonate levels and resulted in a slower decline in eGFR Veverimer (also known as TRC101) is an orally administered, non-
and reduced risk of progression to ESKD (RR, 0.32; 95% confi- absorbed hydrochloric acid binder being evaluated as a novel therapy
dence interval [95% CI], 0.18 to 0.56) (Figure 3). However, in this for metabolic acidosis in CKD. Veverimer increases serum
analysis, treatment with oral alkali was associated with increased [HCO32] by selectively binding and removing H1 and chloride
edema, necessitating increases in diuretics, worsening hypertension, anions from the GI tract. It acts by sequestering HCl from the
and the need to escalate antihypertensive therapy. The authors cau- stomach, similar to the physiologic effect of vomiting or nasogastric
tion that these studies excluded patients with uncontrolled hyperten- suction, where acid removal stimulates increases in serum
sion and decompensated heart failure, meaning that further research [HCO32]. HCl-bound veverimer is then excreted unchanged in
is needed to establish how generalizable these findings are to a real- feces (68). Veverimer is not an exchange resin; therefore, no sodium,
world CKD population (64). potassium, or calcium is added to the GI tract or systemically
A more recent double-blinded, randomized, controlled trial absorbed.
studied the effect of NaHCO3 therapy in elderly CKD stages 4 and The first human study of veverimer was a randomized,
5 patients .60 years old (mean age 74 years) over 2 years and double-blind, placebo-controlled, multicenter trial of 135 partici-
reported no changes in physical function as assessed by the Short pants with mean baseline eGFR of 35 ml/min per 1.73 m2 and
Physical Performance Battery or secondary measures, such as of mean baseline serum [HCO32] of 17.7 mEq/L, including patients
quality of life and walk distance. However, there are several impor- with hypertension (93%), diabetes (70%), and heart failure (21%)
tant limitations to this study. There were challenges with recruit- (69). Patients were kept in a metabolic unit, fed a controlled diet,
ment and a very high dropout rate. This was a cohort of advanced and treated for 14 days with either placebo or one of four dosing
CKD, with 22% of patients at CKD stage 5 at enrollment. Further- regimens of veverimer (1.5, 3, or 4.5 g twice daily or 6 g once
more, the achieved difference in serum [HCO32] between the two daily). After treatment, patients were followed for 7–14 days.
groups was ,1 mEq/L, suggesting either poor compliance or inade- Treatment with veverimer was associated with a $1.3-mEq/L
quate dosing for the level of GFR. Further studies will be required increase in serum [HCO32] within 72 hours and a mean increase
to evaluate the effect of alkali therapy in older adults (65). of 3.2–3.9 mEq/L by study end. This effect cleared after 14 days
There also is considerable interest is investigating the effect of of termination of therapy.
alkali therapy on cardiovascular disease. In a pilot randomized, cross- The safety and efficacy of veverimer were further studied in a
over study of 18 individuals with CKD (eGFR 15–44 ml/min) and phase III randomized, controlled, multicenter trial, where 217
serum [HCO32] of 16–21 mEq/L, treatment of metabolic acidosis patients with CKD, eGFR of 20–40 ml/min per 173 m2, and serum
with NaHCO3 was associated with improved vascular endothelial [HCO32] of 12–20 mmol/L were randomly assigned to veverimer
function as evaluated by the increased brachial artery flow–mediated (6 g/d) or placebo with unrestricted dietary protein intake, and they
Nephrology Self-Assessment Program - Vol 20, No 2, January 2022 141

were followed for 12 weeks (70). Veverimer treatment was associated In this observational study of 2128 renal transplant recipients,
with significantly higher rates of increase in [HCO32] by 4 mEq/L or 38.8% had metabolic acidosis at 1 year post-transplant. Total
more and higher rates of normalization of [HCO32]. Physical func- CO2 level ,20 mEq/L was associated with increased risk of car-
tion was assessed using ten items from the Kidney Disease Quality of diovascular events (adjusted HR, 2.2; 95% CI, 1.29 to 3.10), and
Life Short Form-36 and the repeat chair stand test, and veverimer- this effect was driven mainly by the increased risk of ischemic car-
treated patients showed a significant increase in physical function diovascular events. For every 1-mEq/L drop in total CO2 below
scores compared with controls. Adverse events were uncommon and 24 mEq/L, the risk of cardiovascular events rose by 17%. Similar
similar in both groups. In a follow-up extension study, 196 patients to other reports, acidosis was also independently associated with
continued on study assignment for a further 40 weeks. Those treated all-cause mortality and with progressively higher risk with wors-
with veverimer had stable [HCO32] out to week 52 and again, ening acidosis (76). Similar to in all CKD patients, treatment
showed significant improvements in measures of physical function as with alkali therapy is recommended for renal transplant recipients
compared with control. The investigators concluded that veverimer is with [HCO32] ,22 mEq/L, but no studies have specifically
safe and effective for treatment of metabolic acidosis of CKD and examined the effect of alkali therapy on outcomes in transplant
resulted in improvements in subjective and objective physical func- recipients. Given the prevalence of acidosis and high rates of com-
tioning (71). As a counterion free hydrochloric acid binder, veverimer plications in this patient group, this is an area ripe for future
may be especially useful in patients for whom sodium loading with investigation.
NaHCO3 may be of concern. However, further studies are required
to evaluate the effect of veverimer in such populations and on other
complications of metabolic acidosis, including bone density, CKD 4 Subgroup analysis of the SMART and SALT-ED tri-
progression, and cardiovascular outcomes.
als suggests that fluid resuscitation with balanced
crystalloid leads to earlier resolution of acidosis in
Metabolic Acidosis and Kidney Transplant DKA when compared with saline therapy.
Outcomes 4 Euglycemic ketoacidosis is an important complica-
The prevalence of metabolic acidosis in kidney transplant recipients
varies widely across studies, with reported rates of 11% to .50%.
tion of treatment with SGLT2 inhibitors. Risk factors
However, the prevalence appears higher than nontransplant CKD, include states of decreased oral intake, rapid reduc-
at all levels of GFR. Several factors specific to transplantation may tions in insulin therapy, major surgery/trauma,
play a role; a single kidney has more limited reserve to handle an decreased carbohydrate intake, and ketogenic diets.
acid load, renal transplantation is commonly associated with immu-
nologically and nonimmunologically mediated tubular dysfunction, 4 Eubicarbonatemic metabolic acidosis occurs
and calcineurin inhibitors are associated with a dose-dependent where total CO2 remains >22 mEq/L but there is
increased risk of metabolic acidosis secondary to tubular dysfunction evidence of H1 retention. It has been associated
and type 4 renal tubular acidosis (72–74) (Table 2). with increased rates of progression to ESKD.
Several recent observational studies have reported an association
between metabolic acidosis in renal transplant recipients and
Reductions in urine citrate may identify patients
reduced patient and allograft survival (74,75). In a large, multicen- with H1 retention and eubicarbonatemic meta-
ter, retrospective cohort study, Park et al. (74) analyzed data from bolic acidosis.
2318 renal transplant recipients who underwent renal transplanta- 4 Untreated metabolic acidosis is associated with
tion between 1997 and 2012, with mean follow-up of 62 months.
Metabolic acidosis was defined as the presence of total CO2 ,22 increased intrarenal angiotensin II, aldosterone, and
mEq/L at 3 months post-transplant. Risk factors for metabolic aci- endothelin-1 levels, which promote proinflammatory
dosis included GFR ,60 ml/min per 1.73 m2, deceased donor and profibrotic pathways within the kidney.
transplantation, and treatment with tacrolimus (versus cyclosporin).
4 Treatment of metabolic acidosis with NaHCO3 has
In adjusted Cox regression models, the presence of metabolic acido-
sis was associated with a significantly increased risk of graft loss (haz- been associated with decreased rates of doubling
ard ratio [HR], 3.48; 95% CI, 2.47 to 4.90), mortality (HR, 3.16; of serum creatinine, delayed progression to ESKD,
95% CI, 1.77 to 5.62), and death-censored graft failure (HR, 3.17; and decreased mortality.
95% CI, 2.12 to 4.73) (74). In a retrospective cohort of 468 trans-
plant recipients, 57 of whom had metabolic acidosis ([HCO32]
4 Veverimer is an orally administered, nonabsorbed
,22 mEq/L) at baseline, Gojowy et al. (75) also reported reduced hydrochloric acid binder. It increases serum
patient and allograft survival at 3-year follow-up. In this study, those [HCO32] by selectively binding and removing H1
with metabolic acidosis had significantly lower GFR at baseline and and chloride anions from the GI tract. Phase III stud-
at 3 years, and the presence of acidosis was associated with relative
ies have shown that it effectively increases [HCO32]
risk of death of 4.11 (95% CI, 1.58 to 10.67) at 3 years.
Djamali et al. (76) examined the association between meta- with improved physical functioning scores.
bolic acidosis and cardiovascular events after renal transplantation.
142 Nephrology Self-Assessment Program - Vol 20, No 2, January 2022

References 20. Palmer BF, Clegg DJ: Euglycemic ketoacidosis as a complication of


SGLT2 inhibitor therapy. Clin J Am Soc Nephrol 16: 1284–1291, 2021
1. Seifter JL, Chang H-Y: Extracellular acid-base balance and ion transport
10.2215/cjn.17621120 PubMed
between body fluid compartments. Physiology (Bethesda) 32: 367–379, 21. Erondu N, Desai M, Ways K, Meininger G: Diabetic ketoacidosis and
2017 PubMed related events in the canagliflozin type 2 diabetes clinical program. Diabe-
2. Matyukhin I, Patschan S, Ritter O, Patschan D: Etiology and manage- tes Care 38: 1680–1686, 2015 PubMed
ment of acute metabolic acidosis: An update. Kidney Blood Press Res 45: 22. Rosenstock J, Ferrannini E: Euglycemic diabetic ketoacidosis: A predict-
523–531, 2020 PubMed able, detectable, and preventable safety concern with SGLT2 inhibitors.
3. Kraut JA, Madias NE: Re-evaluation of the normal range of serum total Diabetes Care 38: 1638–1642, 2015 PubMed
CO2 concentration. Clin J Am Soc Nephrol 13: 343–347, 2018 PubMed 23. Peters AL, Buschur EO, Buse JB, Cohan P, Diner JC, Hirsch IB: Euglycemic
4. Kraut JA, Madias NE: Serum anion gap: Its uses and limitations in clini- diabetic ketoacidosis: A potential complication of treatment with sodium-glu-
cal medicine. Clin J Am Soc Nephrol 2: 162–174, 2007 PubMed cose cotransporter 2 inhibition. Diabetes Care 38: 1687–1693, 2015 PubMed
5. Sun S, Li H, Chen J, Qian Q: Lactic acid: No longer an inert and 24. Kum-Nji JS, Gosmanov AR, Steinberg H, Dagogo-Jack S: Hyperglyce-
end-product of glycolysis. Physiology (Bethesda) 32: 453–463, 2017 mic, high anion-gap metabolic acidosis in patients receiving SGLT-2
PubMed inhibitors for diabetes management. J Diabetes Complications 31: 611–
6. Meyer C, Stumvoll M, Dostou J, Welle S, Haymond M, Gerich J: 614, 2017 PubMed
Renal substrate exchange and gluconeogenesis in normal postabsorp- 25. Vitale RJ, Valtis YK, McDonnell ME, Palermo NE, Fisher NDL: euglycemic
tive humans. Am J Physiol Endocrinol Metab 282: E428–E434, 2002 diabetic ketoacidosis with COVID-19 infection in patients with type 2 diabe-
PubMed tes taking SGLT2 inhibitors. AACE Clin Case Rep 7: 10–13, 2021 PubMed
7. Husain Z, Huang Y, Seth P, Sukhatme VP: Tumor-derived lactate modi- 26. Donnan K, Segar L: SGLT2 inhibitors and metformin: Dual antihyper-
fies antitumor immune response: Effect on myeloid-derived suppressor glycemic therapy and the risk of metabolic acidosis in type 2 diabetes.
cells and NK cells. J Immunol 191: 1486–1495, 2013 PubMed Eur J Pharmacol 846: 23–29, 2019 PubMed
8. Errea A, Cayet D, Marchetti P, Tang C, Kluza J, Offermanns S, et al.: 27. Ting S, Chua H-R, Cove ME: Euglycemic ketosis during continuous
Lactate inhibits the pro-inflammatory response and metabolic reprogram- kidney replacement therapy with glucose-free solution: A report of
ming in murine macrophages in a GPR81-independent manner. PLoS 8 cases. Am J Kidney Dis 78: 305–308, 2021 10.1053/j.ajkd.2020.10.014
One 11: e0163694, 2016 PubMed PubMed
9. Kraut JA, Madias NE: Lactic acidosis. N Engl J Med 371: 2309–2319, 28. Farese S, Stauffer E, Kalicki R, Hildebrandt T, Frey BM, Frey FJ, et al.:
2014 PubMed Sodium thiosulfate pharmacokinetics in hemodialysis patients and
10. Rudkin SE, Grogan TR, Treger RM: The Δ anion gap/Δ bicarbonate ratio healthy volunteers. Clin J Am Soc Nephrol 6: 1447–1455, 2011 PubMed
in early lactic acidosis: Time for another delta? https://kidney360. 29. Jiang J, Chan A, Ali S, Saha A, Haushalter KJ, Lam WL, et al.: Hydro-
asnjournals.org/content/2/1/20. Kidney360 2: 20–25, 2020 gen sulfide–mechanisms of toxicity and development of an antidote. Sci
11. Kraut JA, Madias NE: Lactic acidosis: Current treatments and future Rep 6: 20831, 2016 PubMed
directions. Am J Kidney Dis 68: 473–482, 2016 PubMed 30. Hundemer GL, Fenves AZ, Phillips KM, Emmett M: Sodium thiosulfate
12. Yunos NM, Bellomo R, Hegarty C, Story D, Ho L, Bailey M: Associa- and the anion gap in patients treated by hemodialysis. Am J Kidney Dis
tion between a chloride-liberal vs chloride-restrictive intravenous fluid 68: 499–500, 2016 PubMed
administration strategy and kidney injury in critically ill adults. JAMA 31. Rein JL, Miyata KN, Dadzie KA, Gruber SJ, Sulica R, Winchester JF:
308: 1566–1572, 2012 PubMed Successfully treated calcific uremic arteriolopathy: Two cases of a high
13. Raghunathan K, Bonavia A, Nathanson BH, Beadles CA, Shaw AD, anion gap metabolic acidosis with intravenous sodium thiosulfate. Case
Brookhart MA, et al.: Association between initial fluid choice and subse- Rep Nephrol 2014: 765134, 2014 PubMed
quent in-hospital mortality during the resuscitation of adults with septic 32. Mao M, Lee S, Kashani K, Albright R, Qian Q: Severe anion gap acido-
shock. Anesthesiology 123: 1385–1393, 2015 PubMed sis associated with intravenous sodium thiosulfate administration. J Med
14. Semler MW, Self WH, Wanderer JP, Ehrenfeld JM, Wang L, Byrne Toxicol 9: 274–277, 2013 PubMed
DW, et al.; SMART Investigators and the Pragmatic Critical Care 33. Mariano F, Biancone L: Metformin, chronic nephropathy and lactic aci-
Research Group: Balanced crystalloids versus saline in critically ill adults. dosis: A multi-faceted issue for the nephrologist. J Nephrol 34: 1127–
N Engl J Med 378: 829–839, 2018 PubMed 1135, 2021 10.1007/s40620-020-00941-8 PubMed
15. Self WH, Semler MW, Wanderer JP, Wang L, Byrne DW, Collins 34. Hundal RS, Krssak M, Dufour S, Laurent D, Lebon V, Chandramouli
SP, et al.; SALT-ED Investigators: Balanced crystalloids versus saline V, et al.: Mechanism by which metformin reduces glucose production in
in noncritically ill adults. N Engl J Med 378: 819–828, 2018 type 2 diabetes. Diabetes 49: 2063–2069, 2000 PubMed
PubMed 35. DeFronzo R, Fleming GA, Chen K, Bicsak TA: Metformin-associated
16. Palevsky PM: Intravenous fluids: Finding the right balance. Clin J Am lactic acidosis: Current perspectives on causes and risk. Metabolism 65:
Soc Nephrol 13: 1912–1914, 2018 20–29, 2016 PubMed
17. Self WH, Evans CS, Jenkins CA, Brown RM, Casey JD, Collins SP, et 36. Madiraju AK, Erion DM, Rahimi Y, Zhang XM, Braddock DT, Albright
al.; Pragmatic Critical Care Research Group: Clinical effects of balanced RA, et al.: Metformin suppresses gluconeogenesis by inhibiting mito-
crystalloids vs saline in adults with diabetic ketoacidosis: A subgroup chondrial glycerophosphate dehydrogenase. Nature 510: 542–546, 2014
analysis of cluster randomized clinical trials. JAMA Netw Open 3: PubMed
e2024596, 2020 PubMed 37. Alvarez CA, Halm EA, Pugh MJV, McGuire DK, Hennessy S, Miller
18. Group KDIGO; Kidney Disease: Improving Global Outcomes RT, et al.: Lactic acidosis incidence with metformin in patients with type
(KDIGO) Diabetes Work Group: KDIGO 2020 clinical practice guide- 2 diabetes and chronic kidney disease: A retrospective nested case-control
line for diabetes management in chronic kidney disease. Kidney Int 98[ study. Endocrinol Diabetes Metab 4: e00170, 2020 PubMed
4S]: S1–S115, 2020 PubMed 38. Boucaud-Maitre D, Ropers J, Porokhov B, Altman JJ, Bouhanick B, Doucet
19. Ferrannini E, Baldi S, Frascerra S, Astiarraga B, Heise T, Bizzotto R, et J, et al.: Lactic acidosis: Relationship between metformin levels, lactate con-
al.: Shift to fatty substrate utilization in response to sodium-glucose centration and mortality. Diabet Med 33: 1536–1543, 2016 PubMed
cotransporter 2 inhibition in subjects without diabetes and patients with 39. Pham AQT, Xu LHR, Moe OW: Drug-induced metabolic acidosis.
type 2 diabetes. Diabetes 65: 1190–1195, 2016 PubMed F1000Res 4: F1000, 2015 PubMed
Nephrology Self-Assessment Program - Vol 20, No 2, January 2022 143

40. Calello DP, Liu KD, Wiegand TJ, Roberts DM, Lavergne V, Gosselin S, 60. Nath KA, Hostetter MK, Hostetter TH: Pathophysiology of chronic
et al.; Extracorporeal Treatments in Poisoning Workgroup: Extra- tubulo-interstitial disease in rats. Interactions of dietary acid load, ammo-
corporeal treatment for metformin poisoning: Systematic review and rec- nia, and complement component C3. J Clin Invest 76: 667–675, 1985
ommendations from the Extracorporeal Treatments in Poisoning Work- PubMed
group. Crit Care Med 43: 1716–1730, 2015 PubMed 61. Mahajan A, Simoni J, Sheather SJ, Broglio KR, Rajab MH, Wesson DE:
41. Madias NE: Metabolic acidosis and CKD progression. Clin J Am Soc Daily oral sodium bicarbonate preserves glomerular filtration rate by
Nephrol 16: 310–312, 2021 slowing its decline in early hypertensive nephropathy. Kidney Int 78:
42. Raphael KL: Metabolic acidosis and subclinical metabolic acidosis in 303–309, 2010 PubMed
CKD. J Am Soc Nephrol 29: 376–382, 2018 PubMed 62. Di Iorio BR, Bellasi A, Raphael KL, Santoro D, Aucella F, Garofano L,
43. Raphael KL, Zhang Y, Ying J, Greene T: Prevalence of and risk factors et al.; UBI Study Group: Treatment of metabolic acidosis with sodium
for reduced serum bicarbonate in chronic kidney disease. Nephrology bicarbonate delays progression of chronic kidney disease: The UBI Study.
(Carlton) 19: 648–654, 2014 PubMed J Nephrol 32: 989–1001, 2019 PubMed
44. Kidney Disease: Improving Global Outcomes. KDIGO 2012 Clinical Prac- 63. Raphael KL, Isakova T, Ix JH, Raj DS, Wolf M, Fried LF, et al.: A ran-
tice Guideline for the Evaluation and Management of Chronic Kidney Dis- domized trial comparing the safety, adherence, and pharmacodynamics
ease. Available at: https://kdigo.org/wp-content/uploads/2017/02/KDIGO_ profiles of two doses of sodium bicarbonate in CKD: The BASE Pilot
2012_CKD_GL.pdf. Accessed October 31, 2021 Trial. J Am Soc Nephrol 31: 161–174, 2020 PubMed
45. Gennari FJ, Hood VL, Greene T, Wang X, Levey AS: Effect of dietary 64. Navaneethan SD, Shao J, Buysse J, Bushinsky DA: Effects of treatment
protein intake on serum total CO2 concentration in chronic kidney dis- of metabolic acidosis in CKD: A systematic review and meta-analysis.
ease: Modification of Diet in Renal Disease study findings. Clin J Am Soc Clin J Am Soc Nephrol 14: 1011–1020, 2019
Nephrol 1: 52–57, 2006 PubMed 65. Witham MD, et al.; BiCARB study group: Clinical and cost-effectiveness
46. Chen W, Levy DS, Abramowitz MK: Acid base balance and progression of oral sodium bicarbonate therapy for older patients with chronic kidney
of kidney disease. Semin Nephrol 39: 406–417, 2019 PubMed disease and low-grade acidosis (BiCARB): A pragmatic randomised, dou-
47. Goraya N, Simoni J, Sager LN, Mamun A, Madias NE, Wesson DE: ble-blind, placebo-controlled trial. BMC Med 18: 91, 2020 PubMed
Urine citrate excretion identifies changes in acid retention as eGFR 66. Kendrick J, Shah P, Andrews E, You Z, Nowak K, Pasch A, et al.: Effect
declines in patients with chronic kidney disease. Am J Physiol Renal Phys- of treatment of metabolic acidosis on vascular endothelial function in
iol 317: F502–F511, 2019 PubMed patients with CKD: A pilot randomized cross-over study. Clin J Am Soc
48. Phisitkul S, Khanna A, Simoni J, Broglio K, Sheather S, Rajab MH, et Nephrol 13: 1463–1470, 2018 PubMed
al.: Amelioration of metabolic acidosis in patients with low GFR reduced 67. Clegg DJ, Gallant KMH: Plant-based diets in CKD. Clin J Am Soc
kidney endothelin production and kidney injury, and better preserved Nephrol 14: 141–143, 2019
GFR. Kidney Int 77: 617–623, 2010 PubMed 68. Brady C, Chemaly ER, Lohr JW, Parker MD: Veverimer: An advance in
49. Driver TH, Shlipak MG, Katz R, Goldenstein L, Sarnak MJ, Hoofnagle base therapy for metabolic acidosis. Ann Transl Med 8: 1331, 2020
AN, et al.: Low serum bicarbonate and kidney function decline: The PubMed
Multi-Ethnic Study of Atherosclerosis (MESA). Am J Kidney Dis 64: 69. Bushinsky DA, Hostetter T, Klaerner G, Stasiv Y, Lockey C, McNulty
534–541, 2014 PubMed S, et al.: Randomized, controlled trial of TRC101 to increase serum
50. Gardner J, Tuttle K, Raphael KL: Influence of medications containing bicarbonate in patients with CKD. Clin J Am Soc Nephrol 13: 26–35,
acid salts on serum bicarbonate in CKD. Kidney360 1: 330–336, 2020 2018 PubMed
51. Moranne O, Froissart M, Rossert J, Gauci C, Boffa JJ, Haymann JP, et 70. Wesson DE, Mathur V, Tangri N, Stasiv Y, Parsell D, Li E, et al.:
al.; NephroTest Study Group: Timing of onset of CKD-related meta- Veverimer versus placebo in patients with metabolic acidosis associ-
bolic complications. J Am Soc Nephrol 20: 164–171, 2009 PubMed ated with chronic kidney disease: A multicentre, randomised, double-
52. Goraya N, Simoni J, Sager LN, Madias NE, Wesson DE: Urine citrate blind, controlled, phase 3 trial. Lancet 393: 1417–1427, 2019
excretion as a marker of acid retention in patients with chronic kidney disease PubMed
without overt metabolic acidosis. Kidney Int 95: 1190–1196, 2019 PubMed 71. Wesson DE, Mathur V, Tangri N, Stasiv Y, Parsell D, Li E, et al.: Long-
53. Shah SN, Abramowitz M, Hostetter TH, Melamed ML: Serum bicar- term safety and efficacy of veverimer in patients with metabolic acidosis
bonate levels and the progression of kidney disease: A cohort study. Am J in chronic kidney disease: A multicentre, randomised, blinded, placebo-
Kidney Dis 54: 270–277, 2009 PubMed controlled, 40-week extension. Lancet 394: 396–406, 2019 PubMed
54. Menon V, Tighiouart H, Vaughn NS, Beck GJ, Kusek JW, Collins AJ, 72. Messa PG, Alfieri C, Vettoretti S: Metabolic acidosis in renal transplanta-
et al.: Serum bicarbonate and long-term outcomes in CKD. Am J Kidney tion: Neglected but of potential clinical relevance. Nephrol Dial Trans-
Dis 56: 907–914, 2010 PubMed plant 31: 730–736, 2016 PubMed
55. Dobre M, Yang W, Pan Q, Appel L, Bellovich K, Chen J, et al.; CRIC 73. Ritter A, Mohebbi N: Causes and consequences of metabolic acidosis in patients
Study Investigators: Persistent high serum bicarbonate and the risk of after kidney transplantation. Kidney Blood Press Res 45: 792–801, 2020 PubMed
heart failure in patients with chronic kidney disease (CKD): A report 74. Park S, Kang E, Park S, Kim YC, Han SS, Ha J, et al.: Metabolic acido-
from the Chronic Renal Insufficiency Cohort (CRIC) study. J Am Heart sis and long-term clinical outcomes in kidney transplant recipients. J Am
Assoc 4: e001599, 2015 PubMed Soc Nephrol 28: 1886–1897, 2017 PubMed
56. Wesson DE, Buysse JM, Bushinsky DA: Mechanisms of metabolic acidosis– 75. Gojowy D, Skiba K, Bartmanska M, Kolonko A, Wiecek A, Adamczak
induced kidney injury in chronic kidney disease. J Am Soc Nephrol 31: 469– M: Is metabolic acidosis a novel risk factor for a long-term graft survival
482, 2020 in patients after kidney transplantation? Kidney Blood Press Res 45: 702–
57. Kohan DE, Barton M: Endothelin and endothelin antagonists in chronic 712, 2020 PubMed
kidney disease. Kidney Int 86: 896–904, 2014 PubMed 76. Djamali A, Singh T, Melamed ML, Stein JH, Aziz F, Parajuli S, et al.:
58. Wesson DE, Simoni J, Broglio K, Sheather S: Acid retention accompa- Metabolic acidosis 1 year following kidney transplantation and subse-
nies reduced GFR in humans and increases plasma levels of endothelin quent cardiovascular events and mortality: An observational cohort study.
and aldosterone. Am J Physiol Renal Physiol 300: F830–F837, 2011 Am J Kidney Dis 73: 476–485, 2019 PubMed
PubMed 77. Roberts RJ, Barletta JF, Fong JJ, Schumaker G, Kuper PJ, Papadopoulos
59. Nath KA, Hostetter MK, Hostetter TH: Increased ammoniagenesis as a S, et al.: Incidence of propofol-related infusion syndrome in critically ill
determinant of progressive renal injury. Am J Kidney Dis 17: 654–657, adults: A prospective, multicenter study. Crit Care 13: R169, 2009
1991 PubMed PubMed
144 Nephrology Self-Assessment Program - Vol 20, No 2, January 2022

78. Cervantes CE, Menez S, Monroy Trujillo JM, Hanouneh M: Clinical 82. Poirier-Blanchette L, Simard C, Schwartz BC: Spurious point-of-care lac-
approach to a patient with an acid-base disturbance. Am J Kidney Dis 77: tate elevation in ethylene glycol intoxication: Rediscovering a clinical
A9–A11, 2021 PubMed pearl. BMJ Case Rep 14: e239936, 2021 PubMed
79. Berns JS, Kasbekar N: Highly active antiretroviral therapy and the kid- 83. Patel AR, Nagalli S: Valproate Toxicity. [Updated 2021 Jul 26]. In:
ney: An update on antiretroviral medications for nephrologists. Clin J StatPearls [Internet]. Treasure Island (FL): StatPearls Publishing; 2021
Am Soc Nephrol 1: 117–129, 2006 PubMed Jan. Available at: https://www.ncbi.nlm.nih.gov/books/NBK560898/.
80. Morales A, Danziger J: Management consideration in drug-induced lactic Accessed October 31, 2021
acidosis. Clin J Am Soc Nephrol 15: 1511–1512, 2020 PubMed 84. Juurlink DN, Gosselin S, Kielstein JT, Ghannoum M, Lavergne V,
81. Zand Irani A, Almuwais A, Gibbons H: Acquired pyroglutamic acidosis Nolin TD, et al.; EXTRIP Workgroup: Extracorporeal treatment for
due to long-term dicloxacillin and paracetamol use. BMJ Case Rep 13: salicylate poisoning: Systematic review and recommendations from the
e233306, 2020 PubMed EXTRIP Workgroup. Ann Emerg Med 66: 165–181, 2015 PubMed
Nephrology Self-Assessment Program - Vol 20, No 2, January 2022 145

Article
Metabolic Alkalosis
Namrata Krishnan, MD
Yale School of Medicine, New Haven, Connecticut, and
Veterans Affairs Medical Center, West Haven, Connecticut
Robert Jay Alpern, MD
Yale School of Medicine, New Haven, Connecticut

Buffering
Learning Objectives To define the buffer capacity of the extracellular fluid (ECF), Swan
1. To state the normal homeostatic mechanisms that prevent et al. infused HCO32 into nephrectomized dogs and measured the
change in plasma bicarbonate concentration (HCO32) (1). As was
metabolic alkalosis
typical for such experiments, they framed it as an apparent volume
2. To describe the factors involved in the generation and of distribution (Vd) for HCO32. Whereas such an approach is rela-
maintenance of metabolic alkalosis, including contraction of tively simple for nondecomposable solutes, the approach becomes
effective arterial blood volume and the interactions of distal more complex for HCO32. Although the expected anatomic space
of bicarbonate is the extracellular space, the apparent Vd for infused
delivery of Na1 and water with activated distal H1 secretory
bicarbonate is closer to that of total body water. This is because
mechanisms extracellular buffers (such as phosphate), intracellular buffers (such
3. To list the clinical examples that result in alkalosis by the as hemoglobin), and bone buffers (some which are extracellular and
some which involve changes in cellular membrane transport) all
mechanisms defined above
defend blood pH and HCO32.
4. To describe the approach to evaluation and treatment of All buffers have a pHoptimum at which they are maximally effec-
metabolic alkalosis in common clinical settings tive. As a result, the apparent Vd for HCO32 varies with pH.
Because the body’s ability to prevent a change in HCO32 increases
as blood pH decreases, the apparent Vd for HCO32 increases at a
lower pH (2). Whereas Vd refers to buffering or the prevention of
acute changes in HCO32, the quantitatively most important buffer
Introduction system that defends blood pH is the HCO32/CO2 buffer system,
Metabolic alkalosis is a condition of great clinical importance that the buffer capacity of which is equal to 2.3 HCO32. Thus, whereas
occurs frequently in critically ill patients. It predicts and, in many the defense against changes in blood HCO32 is greatest at low pH
ways, contributes to poor clinical outcomes. It is common in hospi- and HCO32, the defense against changes in blood pH is greatest at
talized patients and correlates with increased morbidity, mostly as a more alkaline pH with a higher HCO32. This is a major reason
marker for severe underlying conditions but also as a cause of why one rarely sees blood pH above 7.5–7.6.
adverse outcomes.
We begin this review with a discussion of the normal homeostatic
Respiratory Compensation
mechanisms that prevent metabolic alkalosis. We then discuss the Acute buffering is followed by respiratory compensation (hypoventi-
pathophysiologic processes that cause these mechanisms to fail, and lation), which returns blood pH toward normal. This is mediated
the evaluation and the treatment of established metabolic alkalosis. by changes in cerebrospinal fluid pH and generally takes a few hours
to evolve.

Renal Excretion
Mechanisms That Prevent Metabolic Alkalosis The major defense against metabolic alkalosis, which prevents it
From a teleologic point of view, the more dangerous a clinical con- from being sustained, is the efficient renal excretion of HCO32.
dition is, the more powerful are the homeostatic mechanisms that Owing to the kidney’s large capacity to excrete HCO32, individuals
prevent it. Because alkalosis is dangerous, homeostatic mechanisms can ingest large amounts of HCO32 without experiencing sustained
ensure that the blood pH rarely rises above 7.6 and is typically metabolic alkalosis. Acute infusions of #1500 mEq of NaHCO3
below 7.5. Increases in blood pH are generally limited in magnitude only transiently increase plasma HCO32 (3). Only if these renal
because of a multitude of defense mechanisms that defend against it, homeostatic mechanisms are disrupted will a patient maintain meta-
including (1) buffering mechanisms that moderate the acute effect bolic alkalosis.
of net alkali addition on blood pH (2), respiratory compensation, The mechanism by which the renal tubule regulates HCO3-
and (3) renal mechanisms that define the steady-state blood pH. secretion is complex. Pitts and Lotspeich showed that if HCO32 is

Copyright © 2022 by the American Society of Nephrology nephsap.org


146 Nephrology Self-Assessment Program - Vol 20, No 2, January 2022

infused into dogs, all of the filtered HCO32 is reabsorbed until a


threshold (approximately 25 mmol/L), is reached at which bicarbo-
naturia occurs, and then HCO32 reabsorption approaches a tubular
maximum (Tm) (4). Normal plasma HCO3- will generally fall
around the threshold. The concept of a Tm for bicarbonate absorp-
tion is useful clinically but is not quite correct. In tubular microper-
fusion studies, proximal tubular HCO3- absorption increases linearly
up to a luminal HCO32 of 40 mEq/L, which is considerably higher
than the normal range of plasma HCO3- (25 mmol/L) (5). In the
absence of changes in luminal HCO32, increases in blood pH and
HCO32 inhibit proximal tubular HCO3- absorption (6). Thus, the
apparent Tm for HCO32 reabsorption actually represents inhibition
of tubular HCO3- absorption by peritubular alkalinity in the proxi-
mal tubule.
A similar phenomenon occurs in the distal nephron, where
increases in luminal HCO32 stimulate HCO32 absorption (7),
whereas alkalosis inhibits H1 secretion and more importantly stimu-
lates HCO32 secretion (8). HCO32 secretion is mediated by the
type B intercalated cell that uses a basolateral membrane H1
ATPase and the apical membrane Cl2/HCO32 exchanger pendrin,
encoded by SClc26a4 (Figure 1). The identification of genes encod-
ing pendrin and other intercalated cell proteins that contribute to
acid–base physiology was recently reviewed (9,10).

Generation and Maintenance of Sustained


Metabolic Alkalosis
There are two requirements for patients to have a sustained meta-
bolic alkalosis: the alkalosis must be generated and then maintained
(11). The generation of metabolic alkalosis refers to the addition of
alkali or, equivalently, the removal of acid from the body. Mainte-
nance of metabolic alkalosis refers to the failure of normal renal
homeostatic mechanisms to correct the alkalosis. Although both pro-
cesses are critical to a sustained metabolic alkalosis, one typically
approaches patients by first addressing the cause of maintenance and
then defining how the alkalosis was generated. Figure 1. Cells critical to H1 transport in the connecting tubule and
collecting duct. These segments contain three cells that are important
Factors That Result in Maintenance of in the generation and maintenance of metabolic alkalosis in many
Metabolic Alkalosis patients. The principal cell is responsible for electrogenic Na1
Factors resulting in the maintenance of metabolic alkalosis include absorption via the epithelial sodium channel (ENaC), and electro-
genic K1 secretion via the renal outer medullary K1 channel
volume contraction, potassium depletion, imbalances in distal sodium
ROMK. Electrogenic H1 secretion is mediated mostly by the H1
delivery and reabsorptive capacity, and defects in bicarbonate secretion.
ATPase in the type A intercalated cell with some contribution from
Volume Contraction. The most important cause of maintained the H1 K1 ATPase. Last, the type B intercalated cell can secrete
metabolic alkalosis is a contracted effective arterial blood volume HCO32, mediated by pendrin, a Cl2/HCO32 exchanger. The roles
(EABV) (12,13). When baroreceptors sense EABV contraction, the these transporters play in the generation, maintenance, and correc-
kidney retains filtered NaCl to return EABV to normal. Other tion of metabolic alkalosis are described in the text. Reprinted with
mechanisms used to maintain EABV include increased thirst and permission from reference 10 (Emmett M: Metabolic acidosis: a brief
secretion of antidiuretic hormone, which maintains EABV at the pathophysiologic review. Clin J Am Soc Nephrol 15: 1848-1856,
expense of hyponatremia. Increases in renal NaHCO3 retention lead 2020).
to defense of EABV at the expense of metabolic alkalosis.
Volume depletion increases the ability of the kidney to retain increases in HCO32 absorption in the proximal and distal tubules
an administered HCO32 load (14). Well-designed animal studies (16,17); and decreased distal Cl2 delivery, which limits Cl/HCO32
have shown that the mechanisms by which a contracted EABV exchange in the distal lumen (9,18,19). Microperfusion studies that
increases renal HCO32 absorption include increased activity of the compared volume-expanded and volume-depleted rats showed that
proximal tubular luminal membrane Na/H antiporter (15); activa- volume expansion also increases proximal tubular HCO32 perme-
tion of the renin-angiotensin system, with angiotensin II-induced ability, resulting in increased backleak when luminal HCO32 is
Nephrology Self-Assessment Program - Vol 20, No 2, January 2022 147

lowered, suggesting that the converse, i.e., volume contraction, may (CCD) (9). The principal cell absorbs Na1 via the epithelial Na1
allow greater net HCO3 reabsorption via decreased backleak (6). channel, ENaC, which generates a lumen-negative voltage. This
Volume depletion decreases GFR in rodents, thereby lowering the drives K1 secretion mostly through the renal outer medullary K1
filtered load of HCO32, which could contribute to the maintenance (ROMK) channel in the same cell, and H1 secretion mediated pre-
of metabolic alkalosis. However most studies suggest that volume dominantly by the H1 ATPase in the type A intercalated cell.
depletion-induced changes in GFR are minimal in human subjects Whereas mineralocorticoids can directly activate many of these
and cannot account for the maintenance of the alkalosis (12). transporters, increased H1 transport seems dependent on increased
Another issue is the role of Cl- concentration and total ECF Cl- Na1 transport, which requires an activated ENaC and luminal Na1
in maintaining metabolic alkalosis. Metabolic alkalosis is always and flow.
hypochloremic. If the anion gap is relatively constant, an increase in Because of this, distal nephron acid excretion is controlled by a
HCO32 in the presence of a normal Na1 level must be accompa- balance between the intrinsic capacity of the distal nephron to
nied by a decrease in Cl2. However low plasma Cl2 per se plays absorb Na1, and distal delivery of Na1 and water (flow rate), both
little role in the maintenance of alkalosis associated with volume which are affected by EABV. By contrast, distal H1 and K1 secre-
contraction. In dogs, diuretic-induced metabolic alkalosis is cor- tion are generally not directly affected by EABV. Whereas a con-
rected by an infusion of NaCl and NaHCO3 in concentrations simi- tracted EABV results in increased mineralocorticoid levels, distal
lar to those in the alkalotic plasma (20). In this setting, the infusion H1 secretion is not altered because distal delivery of Na1 and water
did not directly alter the Cl2 or HCO32; yet, it did correct the is decreased. Thus, secondary hyperaldosteronism does not result in
alkalosis, suggesting it is not Cl2 per se that is critical to the mainte- hypokalemia or metabolic alkalosis (28–30). Similarly, an expanded
nance of metabolic alkalosis but rather the ECF volume. EABV results in increased distal delivery but suppressed mineralo-
A separate issue that has been raised is whether it is volume per corticoid, so that distal H1 and K1 secretion remains unchanged.
se or total ECF Cl2. When NaCl is administered, both parameters Increased distal H1 secretion occurs when both distal delivery of
increase. Although it is difficult to separate volume from total ECF Na1/water and distal nephron Na1 and H1 transporter activity
Cl-, investigators have attempted to do so by using dialysis in rats to are increased. This occurs only when there is a primary change
remove Cl- in the absence of volume changes (isovolemic Cl- deple- in one of these determinants that is unrelated to alterations in
tion), and they found that metabolic alkalosis was maintained (21). EABV. Because this mechanism can generate new alkali, its acti-
However, others have found that the generation of an “isovolemic” vation results in both the generation and the maintenance of
hypochloremic metabolic alkalosis induces extracellular volume metabolic alkalosis.
depletion via shifts of fluid into cells (22). This issue remains unre- The propensity for metabolic alkalosis is further enhanced if
solved because it is very difficult to separate total ECF Cl- from
the increased distal delivery of Na1 is accompanied by distal
ECF volume in a convincing manner. More importantly, it is not
delivery of a nonreabsorbable anion (31). The nonreabsorbable
clear how total ECF Cl- would be sensed. We therefore believe it is
anion prevents proximal Na1 and water reabsorption, which
more likely that the low EABV is the dominant determinant respon-
increases distal delivery, and then, because the nonreabsorbable
sible for maintenance of metabolic alkalosis.
anion cannot follow Na1 across the distal epithelium, Na1
Potassium Depletion. Metabolic alkalosis is almost always associ- absorption generates a more lumen-negative potential that drives
ated with hypokalemia, in part because alkalosis causes renal K1 loss H1 secretion.
and in part because hypokalemia stimulates renal acidification. K1 Defects in Distal Bicarbonate Secretion. Isolated defects in distal
depletion causes intracellular acidosis in renal tubules, which results HCO32 secretion can enhance the magnitude of metabolic alkalosis
in many of the renal effects of acidosis, including increased ammo- and may be able to maintain metabolic alkalosis as a sole mechanism
nia production, renal citrate absorption, and proximal tubular and (9,32). This is exemplified in patients with Pendred syndrome,
distal tubular acidification (23,24). However, hypokalemia sup-
which is caused by a genetic mutation of the pendrin-encoding
presses aldosterone, which will suppress distal acidification (25).
gene, Slc26a4. Pendrin is an apical Cl/HCO32 exchanger on b
Although hypokalemia may contribute to the maintenance of
intercalated cells in the collecting duct and is discussed below.
alkalosis associated with volume contraction, it is not as important
as the volume contraction itself. In sustained hypokalemic metabolic
alkalosis associated with volume contraction, correction of volume Factors That Result in Generation of Metabolic Alkalosis
contraction without correcting hypokalemia leads to bicarbonaturia As noted previously, a sustained metabolic alkalosis requires both
and correction of the metabolic alkalosis (13). On the other hand, maintenance by the kidney, and generation. The generation of met-
in conditions in which aldosterone cannot be suppressed, the correc- abolic alkalosis is caused by the loss of hydrogen ions (H1) or gain
tion of hypokalemia is critical. K1 depletion significantly enhances of alkali. In the absence of H1 loss or alkali gain, potassium deple-
the alkalosis produced by exogenous mineralocorticoid administra- tion can also cause metabolic alkalosis by driving H1 to shift into
tion (26). In addition, the magnitude of metabolic alkalosis in cells (33).
patients with primary hyperaldosteronism correlates with the magni-
External H1 Loss. External H1 loss from the gastrointestinal (GI)
tude of the hypokalemia (27).
tract or the kidneys is the most important generator of metabolic
Enhanced Distal Acidification Due to Imbalances in Distal alkalosis. GI H1 loss is most commonly due to vomiting or nasogas-
Nephron Na1 Absorptive Capacity and Distal Delivery of Na1 tric suction. Less commonly, alkalosis may be generated by the loss
and Water. Figure 1 shows the major cells relevant to H1/HCO32 of acid fluids from the lower GI tract, as with villous adenomas or
transport in the connecting tubule and the cortical collecting duct congenital chloridorrhea.
148 Nephrology Self-Assessment Program - Vol 20, No 2, January 2022

Renal H1 loss is also an important cause of metabolic alkalosis Exogenous Alkali Gain
and occurs when renal acid excretion exceeds metabolic acid genera- Clinical examples of metabolic alkalosis due to exogenous alkali gain
tion. This excess renal acid excretion is due to enhanced H1 secre- include ingestion or infusion of bicarbonate, administration of cit-
tion in the distal nephron. rate with massive transfusions or plasmapheresis, or metabolism of
Although volume contraction contributes to the maintenance retained organic anions after treatment of lactic or keto acidosis with
of metabolic alkalosis, it does not generate alkalosis. As described alkali. In the absence of a stimulus for the kidney to maintain the
above, in the setting of volume contraction, aldosterone increases alkalosis, it will typically be transient (11,35).
the capacity of the distal nephron for Na1 absorption and H1 Milk alkali syndrome occurs in patients who ingest large quan-
secretion, but metabolic alkalosis is not generated because distal tities of calcium and alkali. Although this was seen in the past in
delivery of Na1 and water is low (28–30). Metabolic alkalosis is patients with peptic ulcer disease, more recently it has been noted in
generated when there is a primary increase in distal delivery of Na1 patients being treated for osteoporosis. The metabolic alkalosis is
and water with natriuresis, leading to volume contraction and an generated by alkali ingestion and is then maintained by volume con-
increase in aldosterone, or when there is a primary increase in the traction secondary to hypercalcemia-induced inhibition of thick
transport activity of the distal nephron with increased Na1 absorp- ascending limb NaCl absorption.
tion and volume expansion, which then leads to increased distal
delivery of Na1 and water. Decreased Effective Arterial Blood Volume
Exogenous Alkali Gain. Exogenous alkali gain can occur with the As noted previously, the most common causes of metabolic alkalosis
ingestion or intravenous infusion of HCO32 or other substances are associated with a decreased EABV, typically associated with
that are metabolized to HCO32, such as citrate and other organic decreased ECF volume. The metabolic alkalosis is generated by H1
anions. For the alkalosis to be maintained, there must be continued loss in the GI tract or the kidney and then is maintained by the kid-
addition of alkali, or one of the above maintenance mechanisms ney. Causes include gastric HCl loss (gastric alkalosis), lower GI
must be activated. HCl loss, and increased distal sodium delivery in the setting of acti-
Metabolic Alkalosis in the Absence of H1 Loss or Alkali Gain vated distal nephron transport capacity.
(potassium depletion). Potassium depletion causes H1 to shift into Gastric HCl Loss (Gastric Alkalosis). In gastric alkalosis,
cells, resulting in the generation of metabolic alkalosis (33). In addi- HCl is lost from the stomach by vomiting or nasogastric suction.
tion, ECF volume contraction can theoretically generate a metabolic Gastric fluid contains approximately 100 mEq/L of HCl with
alkalosis by concentrating ECF HCO32 while PCO2 is maintained varying amounts of Na1 and K1. Removal of HCl from the
unchanged. However, this alone does not lead to a clinically significant body generates a metabolic alkalosis, and removal of Na1 leads to
metabolic alkalosis because of the extensive buffering that normally volume contraction, which causes the kidney to maintain the alka-
occurs (34). losis. However, the magnitude of the increase in plasma HCO32
far exceeds that which would be maintained by the level of vol-
ume contraction, resulting in bicarbonaturia. Bicarbonaturia is
4 Major contributors to the maintenance of metabolic associated with increased delivery of NaHCO3 to the distal neph-
ron, which in the presence of an increase in aldosterone (from vol-
alkalosis include volume contraction, conditions ume contraction) enhances distal Na1 absorption with K1 and
that increase distal nephron Na1 and H1 transport H1 secretion (12). The net result is loss of NaHCO3 and
capacity along with increased distal delivery of KHCO3 in the urine, volume depletion, and potassium depletion.
Na1 and water, and defects in bicarbonate secre- An important diagnostic clue is that while patients are volume
depleted they will continue to have a high urinary Na1 with a
tion. Potassium depletion contributes to the main- low urinary Cl- and a urinary pH .7. This phase has been
tenance of alkalosis associated with primary aldo- referred to as the disequilibrium state.
steronism, but to a much lesser degree in alkalosis Once the loss of gastric fluid ceases, bicarbonaturia slows and
associated with volume contraction. then stops as the plasma HCO32 falls to a level that can be main-
tained by the degree of volume contraction and K1 depletion. At
4 Alkalosis is generated via H1 loss from the gastro- this point, referred to as the equilibrium state, urinary Na1, Cl2,
intestinal tract or kidney, exogenous alkali gain, and and pH are low. Administration of NaCl and KCl corrects the fac-
potassium depletion resulting in H1 shift into cells. tors maintaining the alkalosis, leading to bicarbonaturia and return
of plasma HCO32 to normal. As noted above, administration of
NaCl in the absence of correction of K1 deficiency corrects the alka-
losis, demonstrating that volume depletion is more critical to main-
tenance of the alkalosis than is K1 depletion (13). Of course, one
Clinical Conditions That Cause Metabolic Alkalosis would never withhold KCl in this setting, given the sequelae of
Many diverse clinical entities result in metabolic alkalosis. (Table 1). severe K depletion.
In each case the alkalosis must be generated and then maintained if Lower GI HCl Loss. Metabolic alkalosis can be seen with loss of
the alkalosis is to be sustained. Clinically it is often useful to con- HCl from the lower GI tract in conditions such as congenital chlori-
sider the causes based on the mechanisms that generate and main- dorrhea (congenital chloride diarrhea) or villous adenoma. The gen-
tain the alkalosis. eration and maintenance of the metabolic alkalosis is similar to that
Nephrology Self-Assessment Program - Vol 20, No 2, January 2022 149

Table 1. Causes of metabolic alkalosis

Exogenous alkali gain Bicarbonate administration


Citrate in massive transfusions
Retained organic anions
Milk alkali syndrome
Maintained by decreased EABV GI acid loss
Gastric: vomiting, nasogastric suction
Lower GI: congenital chloride diarrhea, villous adenoma
Renal acid loss
Primary increase in distal delivery
Diuretics that function prior to the CCD
Edematous disorders treated with diuretics
Nonreabsorbable anions (penicillins, sulfate, nitrate)
Bartter syndrome
Gitelman syndrome
Mg21 deficiency
Hypercalcemia
Posthypercapneic metabolic alkalosis
Maintained by decreased bicarbonate secretion Cystic fibrosis
Pendred syndrome
Primary increase in activity of the distal nephron High renin/high aldosterone
transport Renovascular hypertension
Malignant hypertension
Scleroderma
Renal vasculitis
Renin-secreting tumor
Low renin/high aldosterone
Adrenal adenoma
Bilateral adrenal hyperplasia
Adrenal carcinoma
Glucocorticoid suppressible hyperaldosteronism
Low renin/low aldosterone
Increase in a non-aldosterone mineralocorticoid receptor agonist
Cushing syndrome
Glycyrrhizic acid
Apparent mineralocorticoid excess: mutation of 11-b-HSD-2
Adrenogenital syndromes
Defective 11-hydroxylation
Defective 17-hydroxylation
Activation of the mineralocorticoid receptor
Pregnancy-induced hypertension
Activation of transport mechanisms downstream of the receptor
Liddle syndrome
Unclear mechanism Postfasting alkalosis

11-b-HSD-2, 11-b-hydroxysteroid dehydrogenase; CCD, cortical collecting duct; GI, gastrointestinal.


seen with gastric alkalosis, with the exception that patients are con- recessive mutation in the Cl/HCO32 exchanger of the ileal and
tinually in the disequilibrium state. colonic luminal membrane, encoded by SLC26A3 (36). In the lower
In contrast to gastric fluid, lower GI fluid is typically high in GI tract, NaCl absorption is mediated by Na/H and Cl/HCO32
HCO32, and increased losses as occur in patients with diarrhea exchangers working in parallel. Under normal circumstances, the
cause metabolic acidosis. Children with congenital chloride diarrhea activity of the Cl/HCO32 exchanger exceeds that of the Na/H
present with profuse watery diarrhea that is high in Cl- and a hypo- exchanger, resulting in stool with a high HCO32. In these patients,
kalemic metabolic alkalosis. This disease is due to an autosomal in the absence of Cl/HCO32 exchange, the unopposed Na/H
150 Nephrology Self-Assessment Program - Vol 20, No 2, January 2022

exchanger results in diarrheal fluid that is high in NH4Cl, with stool thiazide ingestion. The phenotype of Gitelman syndrome is more mild
Cl2 exceeding the sum of Na1 and K1. than that of Bartter syndrome and has onset at an older age (42). By
Villous adenomas are occasionally associated with diarrhea and contrast to Bartter syndrome, Gitelman syndrome presents with hypo-
metabolic alkalosis (37). The pathophysiology is similar to that calciuria rather than hypercalciuria (43).
described above, with Na/H exchange functioning at a rate that Magnesium Depletion. Magnesium deficiency results in hypoka-
exceeds that of Cl/HCO32 exchange. lemia and occasionally metabolic alkalosis that is prevented by spiro-
Increased Distal Delivery of Sodium. Increased distal sodium nolactone (44,45). The absence of hypertension suggests that the
delivery is caused by diuretics, the presence of nonreabsorbable hypokalemia and alkalosis may be due to a primary increase in dis-
anions, Bartter and Gitelman syndromes, magnesium depletion, and tal delivery.
hypercalcemia. Hypercalcemia. Hypercalcemia inhibits thick ascending limb
Diuretic Induced. Renal acid loss is the most common cause of NaCl absorption through calcium sensing receptor inhibition of the
metabolic alkalosis, and among these entities, diuretics are the most apical K1 channel (46). The net effect is similar to that of furose-
common cause. Diuretics that cause hypokalemic metabolic alkalosis mide. However, the incidence of metabolic alkalosis depends on the
inhibit NaCl absorption proximal to the CCD and enhance distal cause of the hypercalcemia. Increases in parathyroid hormone
delivery of Na1 and water to the H1/K1 secreting segments of the inhibit proximal tubule HCO32 absorption, which will ameliorate
distal nephron. Inhibition of Na1 absorption leads to a natriuresis or eliminate the metabolic alkalosis.
that results in volume contraction and high aldosterone levels. In Post–Hypercapneic Alkalosis. Hypercapnea engenders a renal
the presence of high aldosterone levels, high luminal Na1 and flow compensation resulting in an increase in plasma HCO32 and ameliora-
rates in the CCD enhance H1 and K1 secretion. The net result is a tion of the acidemia. The retention of NaHCO3 expands EABV,
metabolic alkalosis generated in the CCD by enhanced H1 secre- resulting in enhanced excretion of NaCl. Euvolemia in this setting is
tion and maintained by the same mechanism, in addition to volume maintained by high total ECF NaHCO3 and low total ECF NaCl.
contraction and hypokalemia stimulating proximal tubular HCO32 Rapid correction of hypercapnea then leads to a primary metabolic
reabsorption. alkalosis resulting from the high plasma HCO32 with a normal
In edematous disorders, decreased EABV is associated with PCO2. The kidney then attempts to correct the alkalosis by excreting
increased extracellular fluid volume. The low EABV in edematous HCO32, which is accompanied by excretion of Na1 and K1, the rela-
disorders such as congestive heart failure causes the kidney to main- tive amounts of which are determined by the level of aldosterone. If
tain the alkalosis, but, as noted above, secondary hyperaldosteronism the patient has a low intake of NaCl, as is typical in this setting, bicar-
does not generate a metabolic alkalosis because distal delivery of bonaturia will cease, resulting in a sustained metabolic alkalosis that is
Na1 and water is low. Thus, an untreated patient with congestive maintained by volume contraction and hypokalemia. Only an increase
heart failure is not expected to have metabolic alkalosis. However, in total ECF NaCl can expand EABV and correct the metabolic alkalo-
these patients are often treated with diuretics that increase distal sis. It should be noted that these patients frequently take diuretics that
delivery of Na1 and water, which in the presence of high aldoste- can worsen the volume contraction and contribute to the generation
rone levels will generate hypokalemia and metabolic alkalosis. and maintenance of metabolic alkalosis. The metabolic alkalosis here is
Distal Delivery of Nonreabsorbable Anions. Filtration of Na1 clinically very important because it serves to suppress respiration.
with a nonreabsorbable anion causes the Na1 to be excreted in the
urine, resulting in volume depletion and increases in aldosterone. In Decreased Distal Bicarbonate Secretion
addition, the nonreabsorbable anion increases distal delivery by inhib- The causes of decreased bicarbonate secretion include Pendred syn-
iting proximal Na1 absorption. Sodium that is delivered to the distal drome and cystic fibrosis.
nephron with a nonreabsorbable anion generates a more lumen- Pendred Syndrome. Bicarbonate secretion by the type B interca-
negative potential, resulting in enhanced H1 and K1 secretion, and lated cell is mediated by apical membrane pendrin functioning in
hypokalemic alkalosis (31,38). Examples of nonreabsorbable anions series with a basolateral membrane H1ATPase (Figure 1). A few
include antibiotics such as carbenicillin, sulfate, and nitrate. patients with Pendred syndrome (mutation of Slc26a4, which enco-
Bartter Syndrome. Patients with Bartter syndrome present with des pendrin) have been reported to experience a more severe meta-
hypokalemic metabolic alkalosis, normal to low blood pressure, bolic alkalosis in response to alkalotic challenges, including thiazide
magnesium depletion, hypercalciuria, and increased plasma renin, treatment in one patient and vomiting in another (9). It remains
angiotensin, and aldosterone (39). The syndrome is due to a num- unclear whether Pendred syndrome can serve as a sole mechanism
ber of inactivating mutations in transporters that mediate thick for maintenance of alkalosis (9).
ascending limb NaCl absorption, including the apical membrane Cystic Fibrosis. HCO32 secretion by the type B intercalated cell
Na-K-2Cl cotransporter, the ROMK K1 channel, the basolateral requires CFTR, the protein responsible for cystic fibrosis (32). The
membrane chloride channels ClC-Ka and ClC-Kb, and the chloride CFTR Cl- channel may be required to recycle Cl- to support pendrin.
channel subunit, Barttin (40). The syndrome can also be caused by Patients with cystic fibrosis have a predisposition to meta-
an activating mutation in the calcium sensing receptor (40). The bolic alkalosis.
syndrome is similar to sustained ingestion of a loop diuretic with
metabolic alkalosis due to a primary increase in distal delivery of
Na1 and water. Increased Distal Transport Activity
Gitelman Syndrome. Gitelman syndrome is caused by inactivating Conditions characterized by a primary increase in the capacity of the
mutations in the distal convoluted tubule NaCl cotransporter (41). distal nephron to absorb Na1 and secrete H1 and K1 cause meta-
Patients present with a syndrome similar to that seen with sustained bolic alkalosis. These conditions are associated with secondary
Nephrology Self-Assessment Program - Vol 20, No 2, January 2022 151

hypertension and are distinguished by measurement of plasma renin Activation of the Mineralocorticoid Receptor. A mutation in the
and aldosterone levels. These conditions are also discussed in the mineralocorticoid receptor has been described that causes constitu-
section on Pathophysiology, Evaluation and Treatment of Hypoka- tive activation and allows it to be further activated by progesterone
lemia in this issue of nephSAP. and other steroids lacking 21-hydroxyl groups (61). In these
High Renin, High Aldosterone. Renovascular hypertension is an patients, pregnancy-induced hypertension develops, but hypokale-
important cause of high renin/high aldosterone but is only occasion- mic metabolic alkalosis does not develop.
ally associated with hypokalemic alkalosis (47). The true incidence is Activation of Transport Mechanisms Downstream of the Recep-
difficult to ascertain because so many patients are taking diuretics. tor. Liddle syndrome is autosomal dominant and results from acti-
Hypokalemic alkalosis with high renin/aldosterone can also be seen vating mutations in the subunits of the luminal membrane Na1
in small vessel vascular disease, as occurs with malignant hyperten- channel, ENaC (62–67). Liddle syndrome can be distinguished from
sion, scleroderma, or renal vasculitis (48). A rare cause of high renin/ the above disorders in that it is not corrected by blockade of the min-
high aldosterone is a renin-secreting tumor (49). The most common eralocorticoid receptor (62–64). However, Na1 channel inhibitors
cause of alkalosis with a high renin and high aldosterone is volume correct the hypertension and the hypokalemic alkalosis (64).
contraction in patients taking diuretics, which is discussed above.
Low Renin, High Aldosterone. A low renin and high aldosterone Unclear Mechanism: Postfasting alkalosis
suggest primary adrenal disorders, which are the most common During starvation, patients experience ketoacidosis. After refeeding,
causes of hypertension with hypokalemic alkalosis. The most com- patients can experience a rebound metabolic alkalosis. In spite of
mon causes include adrenal adenoma and bilateral adrenal hyperpla- much research, it remains unclear how this alkalosis is generated or
sia (27,50,51). Adrenal carcinomas can occasionally secrete maintained (35) Refeeding also can generate NaCl retention and
compounds with mineralocorticoid activity (52). edema, suggesting that homeostatic mechanisms are perceiving a
Glucocorticoid-suppressible hyperaldosteronism is due to a low EABV, which could explain the maintenance of the alkalosis.
genetic crossover between two adjacent genes, CYP11B1, encoding The alkalosis will gradually resolve on its own.
11-b-hydroxylase, and CYP11B2, encoding aldosterone synthase
(51,53). These enzymes control the synthesis of cortisol and aldoste-
rone, respectively. As a result, the 5’-flanking region of 11-b-hydrox- Evaluation
ylase controls the transcription of aldosterone synthase, resulting in The symptoms of metabolic alkalosis often relate to the underlying
regulation of aldosterone synthesis by ACTH, and the administra- cause or to associated volume, potassium, or magnesium depletion.
tion of dexamethasone decreases aldosterone secretion (52). In its severest form, symptoms manifest as cardiac and neuromuscu-
Low Renin, Low Aldosterone. This group of diseases can be lar irritability (arrythmia, paresthesia, muscular spasm, confusion,
divided into three categories (35). seizures) and are exacerbated by hypocalcemia. Hypoventilation due
Increase in a Nonaldosterone Mineralocorticoid Receptor Ago- to depressed respiratory drive, and symptoms of tissue hypoxemia
nist. Approximately 30-40% of patients with Cushing syndrome and cardiac and cerebral ischemia are other serious manifestations
experience hypokalemic alkalosis, which is due to secretion of nonal- (10,35,65,66). The cause is often apparent with a thorough history
dosterone mineralocorticoids as well as the mineralocorticoid activity and physical examination.
of cortisol (54,55). Hypokalemic alkalosis is most common when An elevated serum bicarbonate should be evaluated with a
Cushing syndrome is due to aberrant ACTH secretion (56). blood gas determination to help differentiate primary metabolic
Although mineralocorticoid receptors have a lower affinity for alkalosis versus primary respiratory acidosis with metabolic compen-
glucocorticoids, cortisol circulates at levels that are two to three log sation (Figure 2) (67,68). An arterial blood gas (ABG) is the crite-
orders higher than aldosterone. The mechanism that prevents cortisol rion gold standard for diagnosis. The currently well-established and
from binding to and activating the mineralocorticoid receptor is popular approach introduced by Schwartz and Relman (69) in the
that target cells express the type 2 hydroxysteroid dehydrogenase 1960s explores the physiologic relationship between pH, PaCO2,
(11-b-HSD-2), which converts active cortisol to inactive cortisone. and serum bicarbonate and allows accurate evaluation of acid–base
Failure of this process allows cortisol to activate the mineralocorticoid disorders in informed clinical settings. Standardized base excess
receptor. Most frequently, this occurs in Cushing syndrome when cor- (SBE), or base excess of the ECF, is another method proposed to
tisol levels are excessively high and exceed the activity of 11-b-HSD-2. evaluate acid–base disturbances. For a detailed discussion of SBE,
Glycyrrhizic acid, found in natural licorice or chewing tobacco, inhibits the authors refer the reader to an excellent review by Berend (70).
11-b-HSD-2, allowing normal levels of cortisol to activate the receptor Briefly, the SBE is the amount of strong acid in mmol/L that needs
(57). Last, the autosomal recessive genetic syndrome of apparent miner- to be added to a liter of whole blood to return pH to 7.4 when
alocorticoid excess, due to inactivating mutations of 11-b-HSD-2, will PCO2 is 40 mm Hg using an in vitro assay. By this definition, a
allow cortisol to activate the receptor (57–59). normal SBE is 0 6 2 mmol/L. An SBE .2 mmol/L suggests a base
Nonaldosterone mineralocorticoids can also be increased in the excess that may be indicative of metabolic alkalosis (which could be
adrenogenital syndromes. Defective 11-hydroxylation leads to defec- either primary or secondary [i.e., compensatory]). The standard
tive cortisol synthesis, leading to high levels of desoxycortisol and method advocated by Schwartz and Relman (69) is based on actual
desoxycorticosterone, which have mineralocorticoid activity (60). empiric observations allowing us to predict expected compensatory
Defective 17-hydroxylation prevents synthesis of cortisol, androgens, metabolic response to acute and chronic respiratory acidosis. The
and estrogens, resulting in increases in the synthesis of desoxycorti- concept of SBE is distinct from this standard approach, in that it
costerone and aldosterone. accounts for only the nonrespiratory (metabolic) component of
152 Nephrology Self-Assessment Program - Vol 20, No 2, January 2022

Elevated serum bicarbonate, and low serum chloride

STEP 1 Check blood gas to confirm diagnosis of primary metabolic alkalosis

Assess Effective arterial blood volume (EABV)


STEP 2
(Physical exam; Orthostatic BP and pulse rate; BUN/Cr ratio; serum uric acid; urine electrolytes)

Contracted EABV Expanded EABV (Urine Cl >20meq/L)

Urine Cl >20meq/L Urine Cl <20meq/L Check PRA and PAC

High PRA High PAC Low PRA


x Active diuretic use x Gastric alkalosis; urine pH>7
High PAC Suppressed PRA Low PAC
x Bartter syndrome (NG suction/ vomiting)
x Gitelman x Posthypercapnic alkalosis
x Renal artery stenosis x PHA x Liddle
syndrome x CF and high sweat Cl losses
x Malignant hypertension x GRA x Cushing
x Hypercalcemia x Recent diuretic use
x Renin secreting tumors x CAH x Licorice /AME
x Magnesium x NRA (urine pH<6.5)
x Renal vasculitis x Androgenital
deficiency x Congenital chloride diarrhea
x Scleroderma syndromes**
x Villous adenoma
x Diuretic use*

Figure 2. Proposed algorithm for evaluation of metabolic alkalosis. AME, apparent mineralocorticoid excess; CAH, congenital adrenal hyperpla-
sia; CF, cystic fibrosis; GRA, glucocorticoid remediable or suppressible hyperaldosteronism; NG, nasogastric; NRA, nonreabsorbable anion;
PHA, primary hyperaldosteronism. *Diuretic use: ongoing use of diuretics with persistent volume overload can confuse the clinical picture as
hypervolemic patients tend to be hypertensive and diuretic use can cause an elevation in plasma aldosterone concentration and plasma renin
assay). **Androgenital syndromes such as defective 11-hydroxylation and 17-hydroxylation.

acid–base disturbances and does not factor in the role of respiration Once a diagnosis of primary metabolic alkalosis is made, further
in determining final plasma bicarbonate. The use of SBE, unless evaluation requires consideration of factors responsible for its main-
modified as suggested by Berend (70), would lead to erroneous diag- tenance (Figure 2). Assessment of (EABV) is critical and includes
nosis of primary metabolic alkalosis when the rise in bicarbonate is physical examination, especially postural changes in pulse and BP;
actually due to a compensatory renal response to this disorder (70). laboratory tests such as serum blood urea nitrogen (BUN), BUN/
If used in conjunction with the pH and PCO2, the SBE allows anal- creatinine ratio and serum uric acid, and urinary electrolytes (35). A
ysis of metabolic response to primary respiratory disorders that may low urine Na and Cl suggests chloride depletion and low EABV. A
help identify mixed acid–base disturbances. It is not clear that this paradoxically high urine Na (with a low urine Cl) in a volume-
method adds any value to that described by Schwartz and Relman depleted patient may be related to bicarbonaturia (e.g., in vomiting)
(69), however. or to the presence of another nonreabsorbable anion (e.g., antibiotic,
A venous blood gas (VBG) determination can be a more conve- ketoacid). Urine pH may distinguish between the two; a urine pH
nient alternative to an ABG determination to evaluate acid–base of $7 indicates significant bicarbonaturia, and a pH of #6.5 sug-
abnormalities and shows good correlation with regard to pH (0.03 gests the presence of a nonreabsorbable anion. It’s noteworthy that
to 0.05 pH units lower) and bicarbonate levels (71). Venous PCO2 in metabolic alkalosis, the presence of bicarbonaturia points to the
is usually 3 to 8 mm Hg higher depending on sampling site (central generation of bicarbonate in excess of what the kidney is driven to
venous sample has a higher correlation than peripheral venous sam- maintain. This suggests extrarenal generation of bicarbonate (vomit-
ple) (72) However, there has been concern regarding the accuracy of ing, Cl-secreting diarrhea, nasogastric suction). A high urine Na and
a VBG in patients with circulatory shock. Schrauben et al. (73) Cl in the setting of low EABV indicates active diuretic use, or disor-
recently conducted a multicenter cross-sectional study in critically ill ders such as Bartter syndrome or Gitelman syndrome.
patients with shock (N 5 23) and showed that a central VBG had An elevated urine Cl (and Na) in patients with preserved or
100% sensitivity for diagnosing metabolic acidosis, metabolic alkalo- increased EABV suggests metabolic alkalosis that is caused by a pri-
sis, and respiratory acidosis, and lower sensitivity (71%) for respira- mary increase in Na absorption via the ENaC in the setting of
tory alkalosis. Analysis of ABG results versus VBG results in patients maintained or high distal Na delivery. This ENaC activation may
receiving vasopressors (13/23 patients) yielded similar results. We be due to an intrinsic channel defect (Liddle syndrome) or to a pri-
conclude from these data that a central VBG is an acceptable alter- mary mineralocorticoid excess state, as discussed in the pathophysi-
native to an ABG. ology section. These disorders are characterized by hypertension and
Nephrology Self-Assessment Program - Vol 20, No 2, January 2022 153

accompanying hypokalemia and are causes of treatable hypertension. Alkalemia has several deleterious multisystem effects, including
Further evaluation requires measurement of plasma renin assay and interference with dissociation of oxygen from hemoglobin at the
plasma aldosterone concentration (74). (Table 1, Figure 2). Urine K level of the peripheral tissues, vasoconstriction of vascular smooth
in the face of hypokalemia can help in differentiating between renal muscle tone, and respiratory depression via both central and periph-
K wasting and nonrenal K wasting disorders; the former are typically eral chemoreceptors (35,65,66,80). These effects impair tissue oxy-
associated with primary metabolic alkalosis. gen delivery and cause tissue hypoxia. Cardiac arrythmias due to
In the pediatric population, recurrent metabolic alkalosis should alkalosis-associated hypocalcemia and hypokalemia, alveolar hypo-
raise suspicion for chloride-depleting causes such as Bartter syn- ventilation leading to hypercapnea and hypoxia, delirium, seizures,
drome, Gitelman syndrome, cystic fibrosis (urine Cl will be high), vasoconstriction resulting in myocardial and cerebral ischemia, and
or congenital chloride diarrhea and pyloric stenosis (urine Cl will be electrolyte abnormalities including hypomagnesemia, hypocalcemia,
low). Among adults who present with recurrent alkalosis and low and hypophosphatemia can result, although hypophosphatemia is
urine Cl), surreptitious vomiting should be considered more common with respiratory alkalosis.
(10,70,75,76). Oppersma et al. (85) recently examined the effects of compen-
sated metabolic alkalosis on the neural respiratory drive in 10 young
healthy subjects. Metabolic alkalosis was induced by oral ingestion
of 100 mL of 8.4% sodium bicarbonate solution, three times a day
4 The evaluation of increased bicarbonate requires
for 10 doses, with a demonstrable increase in serum bicarbonate lev-
a blood gas determination. els (25.2 6 0.7 to 29.2 6 0.6 meq/L). The pH remained normal as
4 A central venous blood gas determination is an a result of respiratory compensation. The study demonstrated
acceptable alternative to an arterial blood gas decreased respiratory drive (indicated by diaphragmatic electrical
activity) and suppressed minute ventilation in healthy subjects that
determination. persisted despite increasing inspiratory CO2 pressure, demonstrating
4 Once primary alkalosis is diagnosed, further eval- the hypoventilatory effects of metabolic alkalosis in the face of
uation should be directed at underlying factors hypercapneia even in healthy subjects.
that allow maintenance of alkalosis, particularly a
careful assessment of volume status. Treatment
Indications for Intervention
Based on the available data, evidence for the causality of metabolic
alkalosis on mortality is weak; however, there appears to be a strong
association between metabolic alkalosis and poor patient outcomes
Epidemiology and Outcomes that should raise concern and encourage intervention when arterial
Metabolic alkalosis (pure or combined with other acid–base disor- pH is $7.5, especially in critically ill patients.
ders) is frequently seen in heterogenous clinical settings in both Aggressive correction may be indicated in (1) patients with
adult and pediatric populations. It is common in hospitalized and chronic lung disease who have acute respiratory decompensation
critically ill patients and is associated with poor outcomes (77–84). and impending intubation or those with ventilator dependence due
Alkalosis is likely a marker of severity of the underlying disorder to poor respiratory drive and (2) patients with myocardial ischemia
(82). or cerebral hypoperfusion where alkalemia-induced tissue hypoxia is
Clinical outcomes associated with alkalosis include prolonged a concern.
hospitalization, prolonged duration of mechanical ventilation, and
high mortality (79,81,83,84). Anderson et al. (83) found that the Correct Causes of Generation and Maintenance
mortality rate was 45% in patients with an arterial blood pH of Treatment of metabolic alkalosis must begin with correction of the
.7.5, and mortality increased to 80% at a pH .7.65. A large retro- underlying precipitants, most importantly, factors involved in main-
spective study that used the Multiparameter Intelligent Monitoring tenance (i.e., interfering with renal bicarbonate excretion). Identify-
in the Intensive care Unit II database and included 18,982 critically ing and removing the source of bicarbonate generation or acid loss
ill patients found a high prevalence of metabolic alkalosis (29%), is helpful as well. (Table 2)
defined in this study as a serum bicarbonate level .30 mEq/L (81). Optimization of effective arterial blood volume is central.
There was a U-shaped relationship between serum bicarbonate levels Sodium repletion (as normal saline) expands extracellular volume,
and mortality, with the lowest mortality seen between 25 and 30 improves hypovolemia, downregulates the renin-angiotensin-aldoste-
mEq/L. In a recent retrospective cohort study of 627 critically ill rone system (RAASU), and repletes chloride. Hypokalemia should
septic patients, metabolic alkalosis and late metabolic alkalosis (time be corrected. As noted previously hypokalemia contributes to main-
of onset .48 hours), regardless of severity, independently predicted tenance of metabolic alkalosis, particularly in states when RAAS can-
prolonged intensive care unit (ICU) length of stay (OR 5 6.461 not be suppressed (exogenous mineralocorticoid administration or
[95% CI 4.421 to 9.443, P,0.001]) (79). However, this association states of primary mineralocorticoid excess) and the magnitude of
does not imply cause and effect. In addition, increased mortality metabolic alkalosis correlates with the severity of hypokalemia
due to metabolic alkalosis (at 30 days and 12 months) was not seen (26,27). Potassium chloride (KCl) is the preferred salt because it
in this study. repletes both K and Cl in metabolic alkalosis.
154 Nephrology Self-Assessment Program - Vol 20, No 2, January 2022

Table 2. Summary of treatment of metabolic alkalosis

General measures to correct causes of generation and Discontinue precipitants if possible


maintenance Discontinue nasogastric suctioning
(Trial H2 blockers/PPI if suctioning is necessary)
Treat vomiting
Treat chloride-rich diarrhea
Discontinue diuretics
Discontinue exogenous alkali (blood transfusions, milk-alkali
ingestion, citrate anticoagulation in CRRT)
Optimize volume status
Replete volume and electrolyte deficits (K, Mg) when indicated
Aldosterone antagonists/RAAS blocking agents Slow acting (3-7 days)
Particularly helpful in volume expanded patients with evidence
of mineralocorticoid excess or activated distal Na absorption
(ENaC blockers are used to treat Liddle syndrome because
aldosterone antagonists are not helpful)
Role in treatment of BS and GS
Improves outcomes in systolic heart failure
Caution with hyperkalemia, reduction in GFR
Acetazolamide Effective within 18-24 hours
Diuretic-induced metabolic alkalosis in patients with heart
failure: increases natriuresis and bicarbonaturia
Posthypercapneic metabolic alkalosis: may help with ventilator
weaning
Caution with hypokalemia, hypophosphatemia, and worsening
hypercapnia
Non-sodium based acid solutions (HCl/ammonium chloride/ Role in severe alkalemia especially in patients with EABV
arginine HCl) expansion
Relatively rapid correction (8-12 hours) with HCl
HCl infusions require central venous access
Monitor for hyperkalemia and metabolic acidosis
Renal replacement therapy Role in refractory severe alkalemia and allows precise titration
Conventional HD: dialysate HCO32 cannot be lowered below
approximately 20-24 mEq/L; however, most patients with severe
alkalemia have a much higher serum HCO3, so conventional
HD is effective in treatment
CRRT is effective; however, the correction is slower than
conventional HD (approximately 12 hours versus 4 hours).
Citrate anticoagulation can worsen metabolic alkalosis especially
in patients with liver failure.

CRRT, continuous renal replacement treatment; ENaC, epithelial sodium channel; HCl, hydrochloric acid; HD, hemodialysis; PPI, proton pump inhibitor; RAAS, renin-angiotensin-
aldosterone system. BS, Bartter Syndrome; GS, Gitelman syndrome.

Other measures may include treating primary mineralocorticoid correct severe or refractory metabolic alkalosis, especially in patients
excess with RAAS antagonists, repleting magnesium, and discontinu- with advanced CKD or severe AKI (86). Dialysis allows precise con-
ing diuretic therapy or nasogastric suctioning. In patients for whom trol of serum bicarbonate. Conventional hemodialysis requires a mini-
nasogastric suction cannot be stopped, H2 blockers or proton pump mum dialysate bicarbonate of 20 to 24 mEq/L; however, most
inhibitors can help decrease acid loss and limit generation of alkalosis. patients with severe alkalemia have a serum bicarbonate level that is
much higher and therefore is corrected with conventional dialysis.
Kidney Replacement Therapy CRRT can be prescribed with bicarbonate-free replacement fluid and
Dialysis (intermittent or continuous renal replacement therapy can precisely and effectively treat metabolic alkalosis. However, cor-
(CRRT)) with a low bicarbonate bath may be necessary to rapidly rection with CRRT is slower than conventional hemodialysis (12
Nephrology Self-Assessment Program - Vol 20, No 2, January 2022 155

hours versus 4 hours) (80). Metabolic alkalosis can worsen if citrate of intravenous HCl for treatment of metabolic alkalosis (N 5 30).
anticoagulation is used for CRRT because citrate is metabolized to They describe the use of intravenous HCl as a treatment of choice
bicarbonate in vivo, so reducing bicarbonate in the replacement fluid for patients with diuretic-induced chloride depletion metabolic alka-
and monitoring pH is important (87). losis in whom a trial of acetazolamide had failed; 26/30 of the study
patients had diuretic-induced alkalosis, with a median pH of 7.5
Acetazolamide (7.36 to 7.59) and total CO2 of 34 mmol/L (28–42) The average
Frequently, in clinical practice one might encounter patients with met- HCl infusion rate was 10.5 6 3.7 mEq/h (4 to 16.6 mEq/h). The
abolic alkalosis where removal of precipitants is challenging. Examples average duration of the HCl infusion was 29.0 6 14.6 hours (9 to
include patients with heart failure (HF) who require continued diuretic 68 hours). There was a significant decline in serum bicarbonate lev-
therapy or patients with low EABV but elevated ECF volume (HF els from a median of 34 mM/L to 27 mM/L (P,0.001). No signifi-
and other edematous states) who cannot tolerate volume expansion cant adverse effects were reported, including hyperkalemia or severe
with normal saline. Treatment interventions that directly address the metabolic acidosis, attributed to the treatment. HCl is not commer-
alkalosis may be required in these patients (80). cially available and must be prepared by the institutional pharmacy
Acetazolamide can be used to treat metabolic alkalosis, espe- from a concentrated, sterilized HCl solution using chemically safe
cially in the presence of volume overload, because it has weak equipment and personal protection. CBC, electrolytes, blood gases,
diuretic properties and facilitates bicarbonaturia. Owing to its ability and the infusion site must be closely monitored. There are rare
to decrease serum bicarbonate, acetazolamide can stimulate respira- reported cases of chest wall necrosis and death with the use of HCl
tory drive and counter the hypoventilatory effects of metabolic alka- resulting from malpositioned central catheters and tissue extravasa-
losis (88,89). tion of HCl (92); therefore, it must be infused via a well-positioned
Acetazolamide inhibits the enzyme carbonic anhydrase (89). In central venous catheter using its distal port. Concentrations .0.25
the proximal tubule, carbonic anhydrase is responsible for dehydra- N are not recommended because of the risk of hemolysis. There are
tion of luminal carbonic acid formed from the combination of fil- reports of successfully administering HCl as a buffered lipophilic
tered HCO32 and secreted H1 into CO2 and H2O. It also catalyzes amino acid solution via a peripheral vein (93) or adding it to total
the reaction between cellular H2O and CO2 to form cellular car- parenteral nutrition bags to improve ease of administration and min-
bonic acid, which is the source of secreted H1. Inhibition of proxi- imize local infusion reactions (94).
mal tubular carbonic anhydrase with acetazolamide blocks renal High cation gap-amino acid is a hyperchloremic formulation
acidification and induces bicarbonaturia. The degree of bicarbonatu- that contains significantly less Na1 (Na1: 14 mEq/L versus Cl-:
ria induced is proportionate to the degree of metabolic alkalosis. As 94 mEq/L) and has been used to successfully treat refractory met-
the HCO32 decreases, the clinical effectiveness of acetazolamide abolic alkalosis in a hyponatremic patient without overcorrection
declines in parallel (89). An important adverse effect of acetazol- of low sodium (95). Dosing of acid solutions is based on formulas
amide is hypercapneia (90). RBC carbonic anhydrase is required for using bicarbonate excess and total body water; however, these for-
RBC uptake of CO2 from peripheral tissues as well as uptake of mulas provide very rough estimates of the dose needed. Frequent
RBC CO2 by the lung alveoli. Inhibition of CO2 uptake at these laboratory and blood gas assessments should drive treatment pro-
sites can cause increases in arterial PCO2 and CO2 within peripheral tocols (77).
tissues. Acetazolamide can also increase kaliuresis as a result of Ammonium hydrochloride (NH4Cl) is buffered HCl. Its oral
increased distal sodium bicarbonate delivery. Thus, patients using use is relatively safe; however, intravenously it can cause ammonia
acetazolamide should be monitored for a sudden drop in pH, devel- toxicity, especially in patients with liver disease. It can also raise urea
opment of respiratory acidosis, hyperchloremic metabolic acidosis, levels in patients with renal insufficiency (77). Arginine HCl is
hypokalemia, hypophosphatemia, liver toxicity, and allergic reac- approved for use in the United States as a pituitary stimulant for
tions (89,90). Acetazolamide is typically used in doses of 250 to 500 release of human growth hormone. However, it has been studied,
mg once or twice daily; however, dose adjustment is required with off label, in the treatment of metabolic alkalosis, especially in the
declining estimated GFR because it is eliminated by the kidneys. pediatric population (77). Sierra et al. (96) recently conducted a
single-center retrospective observational study in 82 pediatric
Non-sodium Based Acid Solutions patients (all hospitalized patients included, irrespective of severity of
In severe and refractory metabolic alkalosis, acid solutions (such as illness or ventilator needs) over a 2-year period and found arginine
HCl, NH4Cl, and arginine HCl), which replete acid and chloride HCl (oral or intravenous) to be both effective and safe in the treat-
but have no significant effect on EABV, can be used. ment of metabolic alkalosis (83% of arginine administrations
Infusion of acid-based solutions provides a source of acid and resulted in normalization of abnormal bicarbonate levels) and hypo-
reduces HCO32 in patients with severe refractory metabolic alkalo- chloremia in this population. Severe hyperkalemia has been reported
sis when other treatments are ineffective or contraindicated. HCl with its use in the past (97), but none has been reported recently.
infusion (0.1 or 0.2 N or 100 to 200 mmol Cl-/L concentration in
normal saline or dextrose 5% in water) has been used for this indica- Treatment of Specific Clinical Scenarios
tion. Although its use in the treatment of severe metabolic alkalosis This section will provide guidance on evidence-based treatment of
was reported as early as the 1960s, no randomized controlled trials metabolic alkalosis in certain common clinical scenarios.
(RCTs) have evaluated its use and safety. Guffey et al. (91) con- Diuretic-Induced Metabolic Alkalosis. Diuretic-induced meta-
ducted a single-center retrospective chart review of surgical ICU bolic alkalosis is a common clinical disorder most often seen in the
patients at the University of Rochester who received at least 8 hours setting of HF. Alkalosis correlates with the severity of HF and the
156 Nephrology Self-Assessment Program - Vol 20, No 2, January 2022

use of combined loop and thiazide-like diuretics (80). It results from received conventional therapy. This is the first study that has evalu-
diuretic-induced volume and Cl and K depletion along with the ated the use of acetazolamide as adjunctive therapy for treatment of
underlying pathophysiology of HF (low EABV, RAAS activation, electrolyte and acid–base derangements in Bartter syndrome. Mech-
and increased catecholamines and renal adrenergic tone) (78). In anistically, acetazolamide when coadministered with indomethacin
patients with metabolic alkalosis and diuretic-resistant hypervolemia, and enalapril has been shown to result in a greater degree of inhibi-
where diuretic de-escalation is not possible, several treatment strate- tion of tubuloglomerular feedback and renin inhibition. Although
gies can be attempted. Adding a K-sparing diuretic to a loop diuretic no significant adverse effects of acetazolamide were reported in that
can improve hypokalemia and metabolic alkalosis by antagonizing study, no data are available on the long-term effectiveness and safety
the distal tubular effects of aldosterone, allowing renal bicarbonate of this therapy in Bartter syndrome.
excretion (27). Syndromes of Primary Mineralocorticoid Excess. Primary
Acetazolamide combined with loop and/or thiazide diuretics mineralocorticoid excess syndromes manifest as hypertension, hypo-
has a role in the treatment of diuretic-resistant HF and metabolic kalemia, and metabolic alkalosis. The prevalence of primary hyperal-
alkalosis. There is evidence that acetazolamide increases natriuresis dosteronism is much higher than previously thought, underscoring
while allowing chloride reabsorption and renal bicarbonate excretion the importance of widespread screening. Therapy is determined by
in patients with HF. Wongboonsin et al. (98) performed a meta- the specific cause, but in general K1 repletion, volume optimization,
analysis of nine studies (three RCTs and six cohort studies, N 5 and aldosterone receptor antagonists (spironolactone, eplerenone)
229 HF patients) and found that compared with placebo, acetazol- and/or ENaC blockers (amiloride, triamterene) have an important
amide significantly increased natriuresis (standardized mean differ- role in the medical treatment of these disorders and their electrolyte
ence of 0.67 (95% CI, 0.08 to 1.27). After acetazolamide treatment, derangements (74). Given that Liddle syndrome is due to an activat-
serum HCO32 levels dropped significantly (mean difference (MD) ing mutation in ENaC, direct ENaC blockers (not aldosterone
of 26.42 mmol/L (95% CI, 210.05 to 22.79 mmol/L) with sig- receptor blockers) are required for treatment. Antagonism of the
nificant but small decreases in serum pH (MD of 20.04 (95% CI, aldosterone effect in the kidney decreases principal cell Na1 absorp-
20.06 to 20.02)) and PaCO2 (MD of 22.06 mm Hg (95% CI, tion via ENaC, decreases luminal electronegativity, and conse-
23.60 to 20.53 mm Hg)). This meta-analysis demonstrated the quently reduces distal H1 and K1 secretion. Spironolactone also
safe use of acetazolamide to enhance natriuresis in HF with coinci- blocks the stimulatory effect of aldosterone directly on the H1
dent improvement in metabolic alkalosis. This may be particularly ATPase pump in type A intercalated cells (27).
beneficial in HF with sleep apnea, inasmuch as there was demon- Posthypercapneic Metabolic Alkalosis. Chronic hypercapnia
strable improvement in sleep apnea indices in this study as well. The (as seen in patients with chronic lung disease) results in increased
Acetazolamide in Decompensated heart failure with Volume OveR- renal acidification, chloruresis, and a compensatory increase in
load (ADVOR) trial is a multicenter randomized double-blind pla- plasma HCO32 (68). Following rapid restoration of a normal
cebo-controlled study (actively recruiting) that evaluates the effect of PaCO2, as might occur in a mechanically ventilated patient with
acetazolamide combined with loop diuretic therapy in patients with chronic obstructive pulmonary disease (COPD), plasma HCO32
decompensated HF. Although this trial does not evaluate improve- remains elevated, giving rise to post-hypercapneic metabolic alkalo-
ment in metabolic alkalosis as a primary outcome, the results will sis. This state is exaggerated by volume and Cl- deficiency, which is
inform us of the effectiveness of acetazolamide as a diuretic in HF frequently exacerbated by concomitant use of diuretics. Reduced
and its effect on important patient outcomes. renal function may make it more difficult to correct the alkalosis in
Genetic Disorders Resulting in Secondary Mineralocorticoid these patients. Post-hypercapneic alkalosis (and metabolic alkalosis
Excess. Bartter and Gitelman syndromes typically present as in general) is associated with ventilator dependence and prolonged
“diuretic-like disorders” with hypokalemic metabolic alkalosis with- duration of ICU length of stay, presumably because of its suppres-
out hypertension, although hypertension may be seen in some adults sive effect on central respiratory drive, mediated via central and
with Gitelman syndrome. The standard of care for Bartter and peripheral chemoreceptors (84,88). Acetazolamide has been used as
Gitelman syndromes are RAAS antagonists, nonsteroidal anti- a “respiratory stimulant” in these settings because of its ability to
inflammatory drugs (mediating prostaglandin inhibition), and elec- facilitate renal bicarbonate excretion and rapidly correct alkalemia,
trolyte and volume optimization. especially when saline administration or cessation of diuretics is not
Mazaheri et al. (99) evaluated the adjunctive use of acetazol- clinically appropriate. However, there has been concern over serious
amide as bicarbonate-lowering therapy in children with genetically adverse effects of acetazolamide, including worsening hypercapnia in
proven Bartter syndrome. In a multicenter open-labeled randomized patients with severe chronic lung disease and induction of metabolic
crossover trial (N 5 22; median age 24 months; range 6 to 42 acidosis (90).
months), patients were assigned to two groups, one receiving indo- In a multicenter RCT of 382 mechanically ventilated COPD
methacin, enalapril, and spironolactone and the second group patients, Faisy et al. (100) showed that acetazolamide therapy
receiving the three drugs and acetazolamide (5 mg/kg per day) for 4 resulted in a decrease in serum HCO32 levels and a 16-hour
weeks. The groups were crossed over after a washout period of 2 decrease in duration of mechanical ventilation in the study group
days and treated for an additional 4 weeks. At 4 weeks, treatment compared with placebo; however, the difference did not reach statis-
with acetazolamide resulted in a significant decrease in serum tical significance (95% CI, 236.5 to 4.0 h; P50.17), and the study
HCO32 (22 versus 211 mmol/L, P50.01), fractional excretion of may have been underpowered.
K1, and serum aldosterone and renin levels as well as a significant A recently published systematic meta-analysis (6 RCTs, N 5
increase in serum K1 (10.2 versus 10.8 mmol/L, P50.03) and Cl- 564 patients) by Tanios et al. (101) evaluated the effect of carbonic
(16 versus 19 mmol/L, P50.04), compared with the group that anhydrase inhibitors (mostly acetazolamide) in patients with
Nephrology Self-Assessment Program - Vol 20, No 2, January 2022 157

respiratory failure (mostly COPD) and metabolic alkalosis. Car- 5. Alpern RJ, Cogan MG, Rector FC Jr: Effect of luminal bicarbonate
bonic anhydrase inhibitor therapy resulted in a clinically significant concentration on proximal acidification in the rat. Am J Physiol 243:
F53–F59, 1982 PubMed
decrease in the duration of mechanical ventilation (mean difference,
6. Alpern RJ, Cogan MG, Rector FC Jr: Effects of extracellular fluid vol-
MD 27 h (95% CI, 250 to 24). There was also improvement in ume and plasma bicarbonate concentration on proximal acidification in
blood gas parameters and metabolic alkalosis, with an increase in the rat. J Clin Invest 71:736–746, 1983 PubMed
PaO2 (MD 11.37 mm Hg, 95% CI, 4.18 to 18.56) and a decrease 7. Giebisch G, Malnic G, De Mello GB, De Mello Aires M: Kinetics of
in PaCO2 (MD 24.98 mm Hg, 95% CI, 29.66 to 20.3), serum luminal acidification in cortical tubules of the rat kidney. J Physiol 267:
bicarbonate (MD 25.03 mEq/L, 95% CI 26.52 to 23.54), and 571–599, 1977 PubMed
8. Wesson DE: Dietary HCO3 reduces distal tubule acidification by
pH (MD 20.04, 95% CI 20.07 to 20.01). However, use of car-
increasing cellular HCO3 secretion. Am J Physiol 271: F132–F142,
bonic anhydrase inhibitors did not have any significant effect on 1996 PubMed
mortality (risk ratio (RR) 0.94, 95% CI 0.57 to 1.56) or total 9. Wall SM, Verlander JW, Romero CA: The renal physiology of pendrin-
duration of hospitalization (MD 0.42 days, 95% CI 24.82 to positive intercalated cells. Physiol Rev 100: 1119–1147, 2020 PubMed
5.66). Furthermore, there was a trend toward a higher risk of 10. Emmett M: Metabolic alkalosis: a brief pathophysiologic review. Clin J
adverse events in the carbonic anhydrase inhibitor group (RR Am Soc Nephrol 15: 1848–1856, 2020 PubMed
1.71, 95% CI 0.98 to 2.99), although the ill effects noted were 11. Seldin DW, Rector FC Jr: Symposium on acid-base homeostasis: the
generation and maintenance of metabolic alkalosis. Kidney Int 1: 306–
mild. This study demonstrates the effectiveness of carbonic anhy- 321, 1972 PubMed
drase inhibitor therapy in improving blood gas parameters in 12. Kassirer JP, Schwartz WB: The response of normal man to selective
patients with respiratory failure and metabolic alkalosis, and it depletion of hydrochloric acid: factors in the genesis of persistent gastric
suggests a benefit in decreasing the duration of mechanical ventila- alkalosis. Am J Med 40: 10–18, 1966 PubMed
tion. Unfortunately, it failed to provide information on mortality 13. Kassirer JP, Schwartz WB: Correction of metabolic alkalosis in man
and overall duration of hospitalization. without repair of potassium deficiency: a re-evaluation of the role of
potassium. Am J Med 40: 19–26, 1966 PubMed
In summary, acetazolamide may have a role as a respiratory
14. Kurtzman NA, White MG, Rogers PW: The effect of potassium and
stimulant via its metabolic acidosis–inducing effect; however, extracellular volume on renal bicarbonate reabsorption. Metabolism 22:
close monitoring is required for adverse effects, including 481–492, 1973 PubMed
acetazolamide. 15. Moe OW, Tejedor A, Levi M, Seldin DW, Preisig PA, Alpern RJ: Die-
tary NaCl modulates Na(1)-H1 antiporter activity in renal cortical
apical membrane vesicles. Am J Physiol 260: F130–F137, 1991 PubMed
4 The treatment of metabolic alkalosis requires 16. Liu FY, Cogan MG: Angiotensin II stimulates early proximal bicarbon-
ate absorption in the rat by decreasing cyclic adenosine monophosphate.
removal/reversal of factors that allow its mainte- J Clin Invest 84: 83–91, 1989 PubMed
nance, specifically correction of volume depletion 17. Levine DZ, Iacovitti M, Buckman S, Harrison V: In vivo modulation
of rat distal tubule net HCO3 flux by VIP, isoproterenol, angiotensin
and hypochloremia. II, and ADH. Am J Physiol 266: F878–F883, 1994 PubMed
4 Potassium and magnesium should be repleted if 18. Levine DZ, Vandorpe D, Iacovitti M: Luminal chloride modulates rat
distal tubule bidirectional bicarbonate flux in vivo. J Clin Invest 85:
needed. 1793–1798, 1990 PubMed
4 Severe and refractory cases may require kidney 19. Wesson DE, Dolson GM: Enhanced HCO3 secretion by distal tubule
contributes to NaCl-induced correction of chronic alkalosis. Am J Phys-
replacement therapy, or the use of acetazolamide iol 264: F899–F906, 1993 PubMed
or acid solutions such as hydrochloric acid (HCl), 20. Cohen JJ: Correction of metabolic alkalosis by the kidney after isometric
expansion of extracellular fluid. J Clin Invest 47: 1181–1192, 1968 PubMed
ammonium HCl, and arginine HCl. 21. Galla JH, Bonduris DN, Luke RG: Correction of acute chloride-
depletion alkalosis in the rat without volume expansion. Am J Physiol
244: F217–F221, 1983 PubMed
22. Garella S, Cohen JJ, Northrup TE: Chloride-depletion metabolic alka-
losis induces ECF volume depletion via internal fluid shifts in nephrec-
tomized dogs. Eur J Clin Invest 21: 273–279, 1991 PubMed
References 23. Kunau RT Jr, Frick A, Rector FC Jr, Seldin DW: Micropuncture study of
1. Swan RC, Axelrod DR, Seip M, Pitts RF: Distribution of sodium the proximal tubular factors responsible for the maintenance of alkalosis dur-
bicarbonate infused into nephrectomized dogs. J Clin Invest 34: 1795– ing potassium deficiency in the rat. Clin Sci 34: 223–231, 1968 PubMed
1801, 1955 PubMed 24. Capasso G, Kinne R, Malnic G, Giebisch G: Renal bicarbonate reab-
2. Adrogue HJ, Brensilver J, Cohen JJ, Madias NE: Influence of steady- sorption in the rat. I. Effects of hypokalemia and carbonic anhydrase. J
state alterations in acid-base equilibrium on the fate of administered Clin Invest 78: 1558–1567, 1986 PubMed
bicarbonate in the dog. J Clin Invest 71: 867–883, 1983 PubMed 25. Boyd JE, Palmore WP, Mulrow PJ: Role of potassium in the control of
3. Singer RB, Clark JK, Barker ES, Crosley AP Jr, Elkinton JR: The acute aldosterone secretion in the rat. Endocrinology 88: 556–565, 1971 PubMed
effects in man of rapid intravenous infusion of hypertonic sodium 26. Hulter HN, Sigala JF, Sebastian A: K1 deprivation potentiates the
bicarbonate solution. I. Changes in acid-base balance and distribution renal alkalosis-producing effect of mineralocorticoid. Am J Physiol 235:
of the excess buffer base. Medicine (Baltimore) 34: 51–95, 1955 F298–F309, 1978 PubMed
PubMed 27. Kassirer JP, London AM, Goldman DM, Schwartz WB: On the patho-
4. Pitts RF, Lotspeich WD: Bicarbonate and the renal regulation of acid genesis of metabolic alkalosis in hyperaldosteronism. Am J Med 49:
base balance. Am J Physiol 147: 138–154, 1946 PubMed 306–315, 1970 PubMed
158 Nephrology Self-Assessment Program - Vol 20, No 2, January 2022

28. Seldin DW, Welt LG, Cort JH: The role of sodium salts and adrenal 50. Weinberger MH: Primary aldosteronism: diagnosis and differentiation
steroids in the production of hypokalemic alkalosis. Yale J Biol Med 29: of subtypes. Ann Intern Med 100: 300–302, 1984 PubMed
229–247, 1956 PubMed 51. Sutherland DJ, Ruse JL, Laidlaw JC: Hypertension, increased aldoste-
29. Relman AS, Schwartz WB: The effect of DOCA on electrolyte balance rone secretion and low plasma renin activity relieved by dexamethasone.
in normal man and its relation to sodium chloride intake. Yale J Biol Can Med Assoc J 95: 1109–1119, 1966 PubMed
Med 24: 540–558, 1952 PubMed 52. Alterman SL, Dominguez C, Lopez-Gomez A, Lieber AL: Primary adre-
30. Harrington JT, Hulter HN, Cohen JJ, Madias NE: Mineralocorticoid- nocortical carcinoma causing aldosteronism. Cancer 24: 602–609, 1969
stimulated renal acidification: the critical role of dietary sodium. Kidney PubMed
Int 30: 43–48, 1986 PubMed 53. Lifton RP, Dluhy RG, Powers M, Rich GM, Cook S, Ulick S, et al.: A
31. Stinebaugh B, Miller RB, Relman AS: The influence of non-reabsorbable chimaeric 11 beta-hydroxylase/aldosterone synthase gene causes
anions on acid excretion. Clin Sci 36: 53–65, 1969 PubMed glucocorticoid-remediable aldosteronism and human hypertension.
32. Berg P, Svendsen SL, Sorensen MV, Larsen CK, Andersen JF, Jensen- Nature 355: 262–265, 1992 PubMed
Fangel S, et al.: Impaired renal HCO32 excretion in cystic fibrosis. J 54. Christy NP, Laragh JH: Pathogenesis of hypokalemic alkalosis in
Am Soc Nephrol 31: 1711–1727, 2020 PubMed Cushing’s syndrome. N Engl J Med 265: 1083–1088, 1961 PubMed
33. Cooke RE, Segar WE, Cheek DB, Coville FE, Darrow DC: The extra- 55. Gwinup G, Gantt CL, Hamwi GJ: The production of hypokalemic
renal correction of alkalosis associated with potassium deficiency. J Clin alkalosis with hydrocortisone in subjects with adrenal insufficiency.
Invest 31: 798–805, 1952 PubMed Metabolism 13: 831–836, 1964 PubMed
34. Garella S, Chang BS, Kahn SI: Dilution acidosis and contraction alkalo- 56. Schambelan M, Slaton PE Jr, Biglieri EG: Mineralocorticoid production
sis: review of a concept. Kidney Int 8: 279–283, 1975 PubMed in hyperadrenocorticism: role in pathogenesis of hypokalemic alkalosis.
35. Seldin DW, Giebisch GH: Seldin and Giebisch's The Kidney: Physiology Am J Med 51: 299–303, 1971 PubMed
and Pathophysiology, 5th Ed., edited by Alpern RJ, Caplan MJ, Moe 57. Louis LH, Conn JW: Preparation of glycyrrhizinic acid, the electrolyte-
OW, San Diego, Elsevier Science & Technology; 2013. active principle of licorice: its effects upon metabolism and upon
36. Schweinfest CW, Henderson KW, Suster S, Kondoh N, Papas TS: Iden- pituitary-adrenal function in man. J Lab Clin Med 47: 20–28, 1956
tification of a colon mucosa gene that is down-regulated in colon adeno- PubMed
mas and adenocarcinomas. Proc Natl Acad Sci U S A 90: 4166–4170, 58. Mune T, Rogerson FM, Nikkil€a H, Agarwal AK, White PC: Human
1993 PubMed hypertension caused by mutations in the kidney isozyme of 11 beta-
37. Pilch YH, Kiser WS, Bartter FC: A case of villous adenoma of the rec- hydroxysteroid dehydrogenase. Nat Genet 10: 394–399, 1995
tum with hyperaldosteronism and unusual renal manifestations. Am J PubMed
Med 39: 483–491, 1965 PubMed 59. Wilson RC, Krozowski ZS, Li K, Obeyesekere VR, Razzaghy-Azar M,
38. Clapp JR, Rector FC Jr, Seldin DW: Effect of unreabsorbed anions on Harbison MD, et al.: A mutation in the HSD11B2 gene in a family
proximal and distal transtubular potentials in rats. Am J Physiol 202: with apparent mineralocorticoid excess. J Clin Endocrinol Metab 80:
781–786, 1962 PubMed 2263–2266, 1995 PubMed
39. Bartter FC, Pronove P, Gill JR, MacCardle RC: Hyperplasia of the jux- 60. Biglieri EG, Stockigt JR, Schambelan M: Adrenal mineralocorticoids
taglomerular complex with hyperaldosteronism and hypokalemic alkalo- causing hypertension. Am J Med 52: 623–632, 1972 PubMed
sis: a new syndrome. Am J Med 33: 811–828, 1962 PubMed 61. Geller DS, Farhi A, Pinkerton N, Fradley M, Moritz M, Spitzer A,
40. Hebert SC: Bartter syndrome. Curr Opin Nephrol Hypertens 12: 527– et al.: Activating mineralocorticoid receptor mutation in hyperten-
532, 2003 PubMed sion exacerbated by pregnancy. Science 289: 119–123, 2000
41. Simon DB, Nelson-Williams C, Bia MJ, Ellison D, Karet FE, Molina PubMed
AM, et al.: Gitelman’s variant of Bartter’s syndrome, inherited hypoka- 62. Liddle GW, Bledsoe T, Coppage Jr, WS. A familial renal disorder simu-
laemic alkalosis, is caused by mutations in the thiazide-sensitive Na-Cl lating primary aldosteronism but with negligible aldosterone secretion.
cotransporter. Nat Genet 12: 24–30, 1996 PubMed Trans Assoc Am Physicians 76: 199–213, 1963
42. Gitelman HJ, Graham JB, Welt LG: A new familial disorder character- 63. Rodriguez JA, Biglieri EG, Schambelan M: Pseudohyperaldosteronism
ized by hypokalemia and hypomagnesemia. Trans Assoc Am Physicians with renal tubular resistance to mineralocorticoid hormones. Trans Assoc
79: 221–235, 1966 PubMed Am Physicians 94: 172–182, 1981 PubMed
43. Bettinelli A, Bianchetti MG, Girardin E, Caringella A, Cecconi M, 64. Wang C, Chan TK, Yeung RT, Coghlan JP, Scoggins BA, Stockigt JR:
Appiani AC, et al.: Use of calcium excretion values to distinguish two The effect of triamterene and sodium intake on renin, aldosterone, and
forms of primary renal tubular hypokalemic alkalosis: Bartter and Gitel- erythrocyte sodium transport in Liddle’s syndrome. J Clin Endocrinol
man syndromes. J Pediatr 120: 38–43, 1992 PubMed Metab 52: 1027–1032, 1981 PubMed
44. Francisco LL, Sawin LL, Dibona GF: Mechanism of negative potassium 65. Galla JH: Metabolic alkalosis. J Am Soc Nephrol 11: 369–375, 2000
balance in the magnesium-deficient rat. Proc Soc Exp Biol Med 168: PubMed
382–388, 1981 PubMed 66. Goldring RM, Cannon PJ, Heinemann HO, Fishman AP: Respiratory
45. Shils ME: Experimental human magnesium depletion. Medicine (Balti- adjustment to chronic metabolic alkalosis in man. J Clin Invest 47:
more) 48: 61–85, 1969 PubMed 188–202, 1968 PubMed
46. Wang WH, Lu M, Hebert SC: Cytochrome P-450 metabolites mediate 67. Palmer BF, Alpern RJ: Metabolic alkalosis. J Am Soc Nephrol 8: 1462–
extracellular Ca(21)-induced inhibition of apical K1 channels in the 1469, 1997 PubMed
TAL. Am J Physiol 271: C103–C111, 1996 PubMed 68. Madias NE: Renal acidification responses to respiratory acid-base disor-
47. Bunchman TE, Sinaiko AR: Renovascular hypertension presenting with ders. J Nephrol 23[Suppl 16]: S85–S91, 2010 PubMed
hypokalemic metabolic alkalosis. Pediatr Nephrol 4: 169–170, 1990 PubMed 69. Schwartz WB, Relman AS: A critique of the parameters used in the
48. Laragh JH, Ulick S, Januszewicz V, Deming QB, Kelly WG, Lieberman evaluation of acid-base disorders: “whole-blood buffer base” and
S: Aldosterone secretion and primary and malignant hypertension. J “standard bicarbonate” compared with blood pH and plasma bicarbon-
Clin Invest 39: 1091–1106, 1960 PubMed ate concentration. N Engl J Med 268: 1382–1388, 1963 PubMed
49. Conn JW, Cohen EL, Lucas CP, McDonald WJ, Mayor GH, Blough 70. Berend K: Diagnostic use of base excess in acid-base disorders. N Engl J
WM Jr, et al.: Primary reninism: hypertension, hyperreninemia, and Med 378: 1419–1428, 2018 PubMed
secondary aldosteronism due to renin-producing juxtaglomerular cell 71. Byrne AL, Bennett M, Chatterji R, Symons R, Pace NL, Thomas PS:
tumors. Arch Intern Med 130: 682–696, 1972 PubMed Peripheral venous and arterial blood gas analysis in adults: are they
Nephrology Self-Assessment Program - Vol 20, No 2, January 2022 159

comparable? A systematic review and meta-analysis. Respirology 19: 168– 87. Sigwalt F, Bouteleux A, Dambricourt F, Asselborn T, Moriceau F,
175, 2014 PubMed Rimmele T: Clinical complications of continuous renal replacement
72. Treger R, Pirouz S, Kamangar N, Corry D: Agreement between central therapy. Contrib Nephrol 194: 109–117, 2018 PubMed
venous and arterial blood gas measurements in the intensive care unit. 88. Heming N, Urien S, Faisy C: Acetazolamide: a second wind for a respi-
Clin J Am Soc Nephrol 5: 390–394, 2010 PubMed ratory stimulant in the intensive care unit? Crit Care 16: 318, 2012
73. Schrauben SJ, Negoianu D, Costa C, Cohen RM, Goldfarb S, Fuchs PubMed
BD, et al.: Accuracy of acid-base diagnoses using the central venous 89. Preisig PA, Toto RD, Alpern RJ: Carbonic anhydrase inhibitors. Ren
blood gas in the medical intensive care unit. Nephron 139: 293–298, Physiol 10: 136–159, 1987 PubMed
2018 PubMed 90. Adamson R, Swenson ER: Acetazolamide use in severe chronic obstruc-
74. Ardhanari S, Kannuswamy R, Chaudhary K, Lockette W, Whaley-Connell tive pulmonary disease: pros and cons. Ann Am Thorac Soc 14: 1086–
A: Mineralocorticoid and apparent mineralocorticoid syndromes of second- 1093, 2017 PubMed
ary hypertension. Adv Chronic Kidney Dis 22: 185–195, 2015 PubMed 91. Guffey JD, Haas CE, Crowley A, Connor KA, Kaufman DC: Hydro-
75. Yi JH, Han SW, Song JS, Kim HJ: Metabolic alkalosis from unsus- chloric acid infusion for the treatment of metabolic alkalosis in surgical
pected ingestion: use of urine pH and anion gap. Am J Kidney Dis 59: intensive care unit patients. Ann Pharmacother 52: 522–526, 2018
577–581, 2012 PubMed PubMed
76. Palmer BF: A physiologic-based approach to the evaluation of a 92. Buchanan IB, Campbell BT, Peck MD, Cairns BA: Chest wall necrosis
patient with hypokalemia. Am J Kidney Dis 56: 1184–1190, 2010 and death secondary to hydrochloric acid infusion for metabolic alkalo-
PubMed sis. South Med J 98: 822–824, 2005 PubMed
77. Tobias JD: Metabolic alkalosis in the pediatric patient: treatment 93. Knutsen OH: New method for administration of hydrochloric acid in
options in the pediatric ICU or pediatric cardiothoracic ICU setting. metabolic alkalosis. Lancet 1: 953–956, 1983 PubMed
World J Pediatr Congenit Heart Surg 11: 776–782, 2020 PubMed 94. Finkle D, Dean RE: Buffered hydrochloric acid: a modern method of
78. Cuthbert JJ, Bhandari S, Clark AL: Hypochloraemia in patients with heart treating metabolic alkalosis. Am Surg 47: 103–106, 1981 PubMed
failure: causes and consequences. Cardiol Ther 9: 333–347, 2020 PubMed 95. Ryuge A, Matsui K, Shibagaki Y: Hyponatremic chloride-depletion met-
79. Kre€ u S, Jazrawi A, Miller J, Baigi A, Chew M: Alkalosis in critically ill abolic alkalosis successfully treated with high cation-gap amino acid.
patients with severe sepsis and septic shock. PLoS One 12: e0168563, Intern Med 55: 1765–1767, 2016 PubMed
2017 PubMed 96. Sierra CM, Hernandez EA, Parbuoni KA: Use of arginine hydrochloride
80. Peixoto AJ, Alpern RJ: Treatment of severe metabolic alkalosis in a in the treatment of metabolic alkalosis or hypochloremia in pediatric
patient with congestive heart failure. Am J Kidney Dis 61: 822–827, patients. J Pediatr Pharmacol Ther 23: 111–118, 2018 PubMed
2013 PubMed 97. Bushinsky DA, Gennari FJ: Life-threatening hyperkalemia induced by
81. Liborio AB, Noritomi DT, Leite TT, de Melo Bezerra CT, de Faria arginine. Ann Intern Med 89: 632–634, 1978 PubMed
ER, Kellum JA: Increased serum bicarbonate in critically ill patients: a 98. Wongboonsin J, Thongprayoon C, Bathini T, Ungprasert P, Aeddula
retrospective analysis. Intensive Care Med 41: 479–486, 2015 PubMed NR, Mao MA, et al: Acetazolamide therapy in patients with heart fail-
82. Webster NR, Kulkarni V: Metabolic alkalosis in the critically ill. Crit ure: a meta-analysis. J Clin Med 8: E349, 2019 PubMed
Rev Clin Lab Sci 36: 497–510, 1999 PubMed 99. Mazaheri M, Assadi F, Sadeghi-Bojd S: Adjunctive acetazolamide ther-
83. Anderson LE, Henrich WL: Alkalemia-associated morbidity and mortality apy for the treatment of Bartter syndrome. Int Urol Nephrol 52: 121–
in medical and surgical patients. South Med J 80: 729–733, 1987 PubMed 128, 2020 PubMed
84. Banga A, Khilnani GC: Post-hypercapnic alkalosis is associated with 100. Faisy C, Meziani F, Planquette B, Clavel M, Gacouin A, Bornstain C,
ventilator dependence and increased ICU stay. COPD 6: 437–440, et al; DIABOLO Investigators: Effect of acetazolamide vs placebo on
2009 PubMed duration of invasive mechanical ventilation among patients with chronic
85. Oppersma E, Doorduin J, van der Hoeven JG, Veltink PH, van Hees obstructive pulmonary disease: a randomized clinical trial. JAMA 315:
HWH, Heunks LMA: The effect of metabolic alkalosis on the ventila- 480–488, 2016 PubMed
tory response in healthy subjects. Respir Physiol Neurobiol 249: 47–53, 101. Tanios BY, Omran MO, Noujeim C, Lotfi T, Mallat SS, Bou-Khalil
2018 PubMed PK, et al: Carbonic anhydrase inhibitors in patients with respiratory fail-
86. Huber L, Gennari FJ: Severe metabolic alkalosis in a hemodialysis ure and metabolic alkalosis: a systematic review and meta-analysis of
patient. Am J Kidney Dis 58: 144–149, 2011 PubMed randomized controlled trials. Crit Care 22: 275, 2018 PubMed
160 Nephrology Self-Assessment Program - Vol 20, No 2, January 2022

nephSAP Volume 20, Number 2, January 2022

Article 1: Pathophysiology, Evaluation, and 120 mmol/L, K 4.5 mmol/L, Cl 85 mmol/L, glucose
100 mg/dl, osmolality 250 mOsm/kg
Treatment of Hyponatremia
1. A 25-year-old woman with no significant medical his- Urine: Na 120 mmol/L, K 30 mmol/L, osmolality 500
tory participated in a full marathon for the first time. mOsm/kg H2O
After she completed the race in 5 hours, she experi- Computed tomography of brain: cerebral edema
enced seizures and was transferred to the emergency
department. According to her friend, she had been She received a diagnosis of cerebral edema caused by
drinking water frequently at the water stations during acute hyponatremia resulting from ecstasy.
the marathon.
Which ONE of the following is the MOST appropri-
Vital signs: Glasgow Coma Scale (GCS): E2V2M2 (Eye ate initial treatment for this patient?
response, opens eyes in response to pain; Verbal
A. Loop diuretic (furosemide) IV (20 mg)
response, makes sounds; Motor response, extension to
B. 3% NaCl bolus IV (100 mL over 10 minutes up to
painful stimuli); BP 130/76 mm Hg (supine position),
33)
pulse rate 70 beats/min, respiratory rate 18/min, SpO2
C. 3% NaCl infusion (100 ml/h)
95% (room air), temperature 36.5 C, weight 63 kg (13 kg
D. Tolvaptan (15 mg via nasogastric tube)
more than usual)
E. Urea (30 g via nasogastric tube)
Blood: total protein 7.3 g/dl, albumin 4.2 g/dl, BUN 12
mg/dl, creatinine 0.7 mg/dl, uric acid 3.0 mg/dl, Na 121 3. A 47-year-old man who was a heavy drinker had
mmol/L, K 4.0 mmol/L, Cl 87 mmol/L, glucose 250 received a diagnosis of liver dysfunction for the past 5
mg/dl, osmolality 260 mOsm/kg H2O years. He drank eight to 10 beers per day, and lived
Urine: osmolality 450 mOsm/kg H2O alone, reportedly in poor circumstances. He visited the
clinic when his family noticed gait instability.
Which ONE of the following is the MOST likely cause
Vital signs: GCS: E4V5M6 (Eye response, opens eyes
of this patient’s seizures?
spontaneously; Verbal response, oriented, converses
A. Dehydration normally; Motor response, obeys commands); BP 120/
B. Exercise-associated hyponatremia 60 mm Hg (supine position), pulse rate 70 beats/min,
C. Use of nonsteroidal anti-inflammatory drugs during respiratory rate 14/min, SpO2 98% (room air), tempera-
marathon training ture 36.5 C, weight 59 kg (12 kg compared with a
D. Hyperglycemia month before), conjunctiva icteric
E. Volume depletion
Blood: total protein 6.3 g/dl, albumin 3.0 g/dl, BUN 6
2. A 21-year-old woman with no significant medical his- mg/dl, creatinine 0.5 mg/dl, uric acid 5.5 mg/dl, Na
tory participated in a rave party on a Saturday night. 120 mmol/L, K 2.8 mmol/L, Cl 70 mmol/L, glucose 80
Her friend recommended the use of ecstasy. The mg/dl, osmolality 260 mOsm/kg H2O
patient was dancing and drinking several cocktails, and Urine: Na 18 mmol/L, K 18 mmol/L, osmolality 100
she gradually became confused and drowsy. Unfortu- mOsm/kg H2O
nately, her friend thought that her drowsiness was due
to too much alcohol, and she was left unattended. Sub- He received a diagnosis of chronic hyponatremia with
sequently, when she became unresponsive, she was mild symptoms (gait instability) due to beer potomania,
transported to the emergency department. and his serum [Na1] was corrected. After 24 hours, his
serum [Na1] increased to 132 mmol/L, his K was 3.1
Vital signs: GCS: E1V1M1 (Eye response, does not open mmol/L, and he had no other symptoms.
eyes; Verbal response, makes no sounds; Motor
response, makes no movements); BP 100/60 mm Hg Which ONE of the following is the MOST appropri-
(supine position), pulse rate 120 beats/min, respiratory ate next step in management?
rate 12/min, SpO2 85% (room air), temperature 36.0 C,
A. Continue current treatment until his serum [Na1]
weight 60 kg (13 kg more than usual)
reaches 135 mmol/L
Blood: total protein 7.5 g/dl, albumin 4.5 g/dl, BUN 25 B. Continue the current treatment until all symptoms
mg/dl, creatinine 1.1 mg/dl, uric acid 3.0 mg/dl, Na are eliminated

nephsap.org Copyright # 2022 by the American Society of Nephrology


Nephrology Self-Assessment Program - Vol 20, No 2, January 2022 161

C. Increase K1 replacement for his hypokalemia Vital signs: GCS: E4V5M6 (Eye response, opens eyes
D. Terminate the current treatment and observe the spontaneously; Verbal response, oriented, converses
patient carefully normally; Motor response, obeys commands); BP 100/
E. Terminate the current treatment and re-lower the 60 mm Hg (supine position), pulse rate 65 beats/min,
serum [Na1] with 5% dextrose in water and respiratory rate 14/min, SpO2 95% (room air), tempera-
desmopressin ture 36.2 C, weight 70 kg (22 kg)

4. A 38-year-old woman with no significant medical his- Blood: total protein 6.3 g/dl, albumin 3.0 g/dl, BUN 40
tory experienced severe nausea, vomiting, and diarrhea mg/dl, creatinine 0.9 mg/dl, uric acid 5.0 mg/dl, Na
for 2 days. She was referred to the emergency depart- 160 mmol/L, K 4.5 mmol/L, Cl 120 mmol/L, glucose
ment because of weakness and difficulty in food intake. 100 mg/dl, osmolality 350 mOsm/kg H2O
Vital signs: GCS: E4V5M6 (Eye response, opens eyes Urine: Na 15 mmol/L, K 30 mmol/L, osmolality 120
spontaneously; Verbal response, oriented, converses mOsm/kg H2O
normally; Motor response, obeys commands); BP 90/50
mm Hg (supine position), pulse rate 120 beats/min, Hypernatremia was diagnosed and 5% dextrose in
respiratory rate 16/min, SpO2 99% (room air), tempera- water was administered.
ture 37.8 C, body weight 55 kg (24 kg).
Which ONE of the following is the MOST likely
Blood: total protein 7.0 g/dl, albumin 3.8 g/dl, BUN 50 cause of this patient’s hypernatremia?
mg/dl, creatinine 1.2 mg/dl, uric acid 8.8 mg/dl, Na
A. Aquaresis produced by tolvaptan
128 mmol/L, K 3.5 mmol/L, Cl 90 mmol/L, glucose 80
B. Decreased thirst in response to hypertonicity in the
mg/dl, osmolality 270 mOsm/kg H2O
elderly
Urine: Na 15 mmol/L, K 15 mmol/L, osmolality 450 C. Loss of appetite
mOsm/kg H2O D. Mistakenly continued fluid restriction
E. Partial improvement of syndrome of inappropriate
Volume depletion due to severe vomiting and diarrhea
antidiuretic hormone secretion because of treatment
was diagnosed; normal saline was administered to
of small-cell lung cancer
restore her extracellular fluid volume. Subsequently,
her condition improved (BP 110/70 mm Hg [supine
position]), but serum [Na1] increased to 140 mmol/L
within a day. Article 2: Pathophysiology, Evaluation, and
Treatment of Hypernatremia
Which ONE of the following is the MOST likely 6. A 59-year-old man with a medical history of type 2
cause of this patient’s serum [Na1] increase? diabetes mellitus, hypertension, coronary artery dis-
A. Extracellular fluid volume replacement by normal ease, and bipolar disorder taking lithium is admitted
saline infusion leading to increased free water to the intensive care unit with pneumonia and septic
excretion (aquaresis) shock. On examination, the patient is sedated, receiv-
B. Fluid restriction due to inability to consume fluids ing mechanical ventilation, with an FiO2 40%, positive
orally end-expiratory pressure 8 cmH2O, BP 118/79 mm Hg,
C. Increased water losses from sweating heart rate 91 beats/min. The results of heart and lung
D. Na1 repletion by normal saline infusion examination are unremarkable. There is no jugular
E. Remission of nausea and vomiting, eliminating non- venous distension, or peripheral edema. For the past
osmotic arginine vasopressin secretion 24 hours, his urine output was 1300 ml.

5. An 88-year-old man with syndrome of inappropriate Laboratory evaluation shows serum sodium 155
antidiuretic hormone secretion caused by small-cell mmol/L, potassium 3.7 mmol/L, chloride 123 mmol/L,
lung cancer was being treated with fluid restriction and total CO2 25 mmol/L, BUN 24 mg/dl, serum creatinine
urea, but his serum [Na1] did not improve sufficiently. 1.3 mg/dl (baseline creatinine 1 mg/dl), blood glucose
Moreover, his diet-related quality of life was reduced 170 mg/dl, and urine osmolarity 629 mOsm/kg.
by this treatment. Tolvaptan administration (15 mg/d)
led to an appropriate improvement in serum [Na1] Which ONE of the following answers is MOST likely
within the upper limit of correction. While initiating the correct cause of the patient’s hyponatremia?
tolvaptan, the doctor repeatedly instructed the patient A. Hypernatremia is iatrogenic and requires intrave-
to drink water whenever he felt thirsty. At his initial nous or oral free water supplementation
follow-up visit, the patient reported he was drinking B. Hypernatremia is secondary to lithium-induced
well, with good appetite and an improved diet-related partial nephrogenic diabetes insipidus
quality of life. However, 2 weeks later, he visited a C. Hypernatremia is secondary to osmotic diuresis
clinic because of lethargy. from hyperglycemia
162 Nephrology Self-Assessment Program - Vol 20, No 2, January 2022

D. Hypernatremia is secondary to lithium-induced B. Hypertonic saline infusion test and resultant


complete nephrogenic diabetes insipidus copeptin release will accurately differentiate
between central diabetes insipidus and primary
7. An 86-year-old woman with a medical history of polydipsia
hypertension, multi-infarct dementia, and atrial fibril- C. In the setting of sarcoidosis, nephrogenic diabetes
lation presents from a nursing home with mental insipidus due to hypercalcemia is the most likely
status changes to the emergency department. On pre- cause of hypernatremia
sentation, her BP is 106/58 mm Hg, heart rate is 90 D. Copeptin levels after a water deprivation test will
beats/min (irregular), respiratory rate is 22/min, oxy- accurately differentiate between central diabetes
gen saturation is 97% and weight is 60 kg. On exami- insipidus and primary polydipsia
nation she has dry mucous membranes, she is oriented
to name only, her lung and heart examination results 9. A 40-year-old woman with a medical history of
are unremarkable, and there is no leg edema. schizophrenia presents to the emergency department
with tonic-clonic seizures. She is intubated on
Laboratory evaluation shows serum sodium 176
arrival. On examination, BP is 179/88 mm Hg, and
mmol/L, potassium 3.5 mmol/L, chloride 129 mmol/
heart rate 137 beats/min.
L, bicarbonate 35 mmol/L, creatinine 1.5 mg/dl (base-
line creatinine 1.2 mg/dl), and urine osmolarity 655 Laboratory evaluation shows serum sodium 183
mOsm/kg. mmol/L, serum osmolality 363 mOsm/kg, potas-
sium 3.6 mmol/L, chloride 128 mmol/L, blood glu-
Which is the correct statement regarding the rate of cose 115 mg/dl, BUN 22 mg/dl, creatinine 1.0 mg/
correction and amount of patient’s free water deficit? dl. The patient’s family reports that she might have
A. Assuming minimal ongoing free water losses, the ingested 500 ml of triple-strength Japanese seasoning
infusion of 2 liters of free water would decrease soy sauce 3 to 4 hours before seizure onset.
the serum sodium concentration to approximately
155 mmol/L Which of the following statements regarding the
B. Assuming 45% total body water content in this management of an acute salt intoxication is correct?
elderly woman, the free water deficit is 3.5 L A. Goal serum sodium correction should not exceed
C. A change in the serum sodium concentration of 1 more than 10 mmol/L in 24 hours or 0.5 mmol/L
mmol/L/h is likely safe and should not cause neu- per hour to avoid the development of cerebral
rologic sequelae. edema
D. Assuming minimal ongoing free water losses, the B. Cerebral edema is unlikely to occur in this setting
infusion of 4 liters of free water would decrease of acute hypernatremia, and rapid correction of
the serum sodium concentration to approximately serum Na to baseline should be undertaken
154 mmol/L C. Acute hypernatremia allows for the formation of
idiogenic osmoles counterbalancing the osmotic
8. A 44-year-old man with a medical history of pulmo- gradient between extracellular and intracellular
nary sarcoidosis has his condition evaluated for compartments
increased thirst and urination of 2 weeks duration. He D. Administration of free water at high rates is suffi-
takes maintenance 10 mg of prednisone. He has no cient to treat salt intoxication, and extracorporeal
other contributory medical history and does not take removal can cause too rapid a decrease in the
other medications. On examination, BP is 125/74 mm serum sodium concentration
Hg, heart rate is 72 beats/min, and respiratory rate is
15/min. The lungs and heart examination results are 10. A 55-year-old man with a medical history of hyper-
unremarkable. tension, type 2 diabetes mellitus, hyperlipidemia,
Laboratory evaluation shows serum sodium 147 and ESKD, receiving hemodialysis, is seen for a
mmol/L, potassium 3.8 mmol/L, chloride 110 mmol/ nephrology consultation in the neurologic intensive
L, BUN 16 mg/dl, creatinine 1.1 mg/dl, calcium 8.7 care unit, where he has been admitted for subarach-
mg/dl, albumin 4 g/dl, urine sodium 20 mmol/L, noid hemorrhage. The patient is intubated, is using
and urine osmolarity 115 mOsm/kg. The urine output minimal ventilatory settings, and does not require
exceeds 6 liters per day. any sedation. His GCS score is 7. An external ven-
tricular drain is placed, and the patient is given
What is the correct statement regarding the cause of hypertonic saline infusion. BP 135/75, heart rate 67
and diagnostic workup for hypotonic polyuria? beats/min.
A. The indirect water deprivation test will accurately Laboratory evaluation shows sodium 147 mmol/L,
differentiate between central diabetes insipidus potassium 6.0 mmol/L, chloride 108 mmol/l, total
and primary polydipsia CO2 21 mmol/L, BUN 88 mg/dl, and creatinine
Nephrology Self-Assessment Program - Vol 20, No 2, January 2022 163

10.4 mg/dl. His last dialysis was performed 3 days history. His family informs that over the past few
ago. The patient is full code, and the family would days, he had been painting a new roof in the heat of
like to pursue full medical and surgical manage- summer. On medication review, it is learned that the
ment at this point. patient takes topiramate for migraine prophylaxis and
inhaled tobramycin for Pseudomonas aeruginosa coloni-
What is the optimal decision in regard to the renal zation. History is limited because of altered mental
replacement therapy for this patient? status.
A. Perform intermittent hemodialysis with regular On examination, BP is 90/60 mm Hg and pulse is 100
blood and dialysate flows because the patient is beats/min. Mucous membranes are dry, and poor
starting to experience electrolyte abnormalities, skin turgor is noted. Additional laboratory studies
and rising BUN might contribute to worsening show plasma sodium 135 mEq/L, plasma potassium
mental status 2.2 mEq/L, plasma chloride 86 mEq/L, plasma bicar-
B. Begin continuous venovenous hemodiafiltration bonate 40 mEq/L, plasma BUN 56 mg/dl, plasma cre-
with isotonic bicarbonate replacement fluid and atinine 2.3 mg/dl, plasma uric acid 12.2 mg/dl,
discontinue hypertonic saline infusion plasma creatine kinase 676 U/L.
C. Begin continuous venovenous hemodiafiltration
with standard sodium concentration dialysate and What is the most likely pathophysiologic explana-
replacement fluid; at the same time continue tion for this patient's severe hypokalemia?
hypertonic saline infusion to maintain permissive
A. Inhibition of carbonic anhydrase in the proximal
hypernatremia
tubule
D. Hold off renal replacement therapy at this time
B. Activation of calcium-sensing receptor in the distal
and manage the electrolyte abnormalities
tubule
medically
C. Metabolic alkalosis–induced transcellular shift of
potassium out of the cells
Article 3: Pathophysiology, Evaluation, and D. Loss of potassium in the sweat glands from perspi-
Treatment of Hypokalemia ration with extracellular fluid volume depletion
11. A 22-year old woman is referred for hypokalemia. The and secondary hyperaldosteronism
patient’s history indicates progressive weakness over E. Urinary loss of bicarbonate by the intercalated
the past 3 weeks and weight gain despite exercise and cells
following a healthy balanced diet. The patient does
not describe having diarrhea or vomiting, and she 13. A 36-year-old man is referred to nephrology clinic for
does not have a history of using over-the-counter evaluation of hypokalemia. The patient described pro-
(OTC) medications or herbal supplements. Her body gressive fatigue and leg cramps over the past year
mass index (BMI) is 19 kg/m2, and BP is 120/80 mm since initiating intermittent fasting and adhering to a
Hg. The results of physical examination are unremark- vegetarian diet. He takes magnesium supplements for
able. the leg cramps but denies using other OTC medica-
tions, herbal supplements, or illicit drugs. The patient
Laboratory evaluation shows plasma sodium 133 is sexually active and is taking pre-exposure prophy-
mEq/L, plasma potassium 2.7 mEq/L, plasma chlo- laxis, emtricitabine, and tenofovir to prevent HIV. He
ride 83 mEq/L, plasma bicarbonate 34 mEq/L, plasma is afebrile, BP is 120/80 mm Hg, pulse is 80 beats/
BUN 6 mg/dl, plasma creatinine 0.6 mg/dl, arterial min, and respiratory rate is 16/min. The results of
blood gases pH 7.46 and PCO2 44 mm Hg, urine elec- physical examination are unremarkable.
trolytes (mEq/L) [Na1] 70, urine [K1] 35, urine [Cl2]
,15, urine pH 8. Laboratory data: Plasma sodium 138 mEq/L, potas-
sium 3.4 mEq/L, chloride 114 mEq/L, bicarbonate 15
What is the most likely cause of this patient’s mEq/L, BUN 28 mg/dl, creatinine 0.6 mg/dl, glucose
hypokalemia? 92 mg/dl, uric acid 2.3 mg/dl, serum phosphorus 1.9
mg/dl. Urine studies: pH 6, 11 protein, 21 glucose,
A. Early (“continuous”) vomiting
no erythrocytes or leukocytes.
B. Magnesium deficiency
C. Long-term laxative use The patient was admitted, and an intravenous hypo-
D. Remote diuretic use tonic saline solution (0.45% NaCl) containing 75 mEq
sodium bicarbonate was initiated at a rate of 1
12. A 32-year-old man with a history of cystic fibrosis mEq/kg per h. Spot urine and simultaneous plasma
presents to the emergency department with progres- electrolytes were obtained to measure fractional excre-
sive confusion and is unable to provide any medical tion of HCO32.
164 Nephrology Self-Assessment Program - Vol 20, No 2, January 2022

What would you predict as the expected trend and potassium chloride 20 mEq two times a day for sev-
values for pertinent laboratory data shown in the eral years. The patient is afebrile, with BP 100/70 mm
next series of choices? Hg, pulse 78 beats/min, and respiratory rate 16/min.
A. Plasma K1 #, Plasma HCO32 ", Urine pH ", Frac- He appears euvolemic on examination. No sensory
tional excretion of [HCO32] 2% deficits are noted, and muscle strength in the upper
B. Plasma K1 ", Plasma HCO32 ", Urine pH $, Frac- and lower extremities is 2/4. Electrocardiogram
tional excretion of [HCO32] 15% reveals “U” waves in limb leads II and V1.
C. Plasma K1", Plasma HCO32 #, Urine pH #, Frac- The plasma electrolytes at the time of this visit: Plasma
tional excretion of [HCO32] 2% sodium 139 mEq/L, potassium 2.7 mEq/L, chloride 95
D. Plasma K1 #, Plasma HCO32 ", Urine pH ", Frac- mEq/L, bicarbonate 29 mEq/L, creatinine 0.7 mg/dl,
tional excretion of [HCO32] 15%
magnesium 1.1 mEq/L.
14. A 22-year-old woman is seen in a nephrology clinic The patient is admitted to the hospital for diagnostic
for electrolyte abnormalities. The patient described studies and treatment of hypokalemia. Spot urine stud-
having excessive muscle cramps after ballet lessons ies obtained after stabilization are as follows: K1/Cr
and indicates that she has been vigorously training for ratio 25 mEq/g, Ca12/Cr ratio 0.04 mg/g, fractional
an upcoming dance competition. She denies current excretion of Mg12 5% (high).
use of OTC medications, herbal supplements, or diu-
retics, and she specifically denies vomiting or diar- What is the most likely cause of hypokalemia and
rhea. There is a history of “idiopathic edema” for hypomagnesemia?
which she received hydrochlorothiazide 12.5 mg/d, A. Mutation in CLDN16 gene encoding claudin-16
but she categorically denies current use. Her sister has B. Mutation in SLC12A3 gene encoding NCC in the
a history of Hashimoto thyroiditis, and her mother distal tubule
had a history of stroke. Her BP in the clinic is 110/60 C. Mutation in KCNJ1 gene encoding ROMK in col-
mm Hg, and her BMI is 19 kg/m2. The results of phys- lecting duct
ical examination are unremarkable. D. Mutation in SLC12A1 gene encoding for NKCC2 in
Blood chemistry: Plasma sodium 138 mEq/L, potas- thick ascending limb of Henle
sium 2.7 mEq/L, chloride 95 mEq/L, bicarbonate 32
mEq/L, creatinine 0.6 mg/dl, magnesium 1.3 mg/dl, Article 4: Pathophysiology, Evaluation, and
aldosterone 20 ng/dl (high). Spot urine studies: K1/
Cr ratio 32 mEq/g, Ca12/Cr ratio 0.4 mg/g (high),
Treatment of Hyperkalemia
fractional excretion of Mg12 3% (high). 16. You are following up a 71-year-old man for CKD. He
has long-standing diabetes and hypertension. His
What is the most likely cause of hypokalemia? eGFR is 32 mL/min per 1.73, and his microalbumin-
to-creatinine ratio is 1.7 grams. His serum potassium
A. Abuse of laxatives
is 5.8 mEq/L and takes lisinopril 10 mg daily.
B. Surreptitious diuretic use
C. Herbal medicine containing glycyrrhizic acid
D. Remote vomiting Which of the following is the best approach to the
E. OTC thyroid supplementation management of his hyperkalemia?
A. Leave his regimen unchanged as recent studies
15. A 27-year-old man presents to a nephrology clinic for have shown that low potassium confers higher car-
consultation. The patient was seen in the emergency diovascular risk and mortality than does high
department of a local hospital with profound muscle potassium
weakness, and a [K1] of 2.3 mEq/L was reported. The B. Avoid potassium-binding agents as they all have
patient’s condition improved, and the discharge gastrointestinal side effects and are risky for long-
plasma [K1] was 4.1 mEq/L. This same sequence of term use
events occurred 3 weeks later, and he is referred by C. Initiate patiromer to allow continued treatment
the emergency department physician for evaluation with renin-angiotensin system inhibitors.
of the recurrent hypokalemia. He has been describing D. Add fludrocortisone to reduce potassium in this
increased salt-craving and frequent muscle cramps. patient
His medical history is significant for depression
treated with selective serotonin reuptake inhibitor 17. A 16-year-old boy is referred for evaluation of hyper-
antidepressants. He denies the use of any OTC medi- kalemia detected during emergency department
cations or herbal supplements. He has been taking evaluation of muscle weakness. He notes frequent epi-
magnesium oxide 400 mg four times a day and sodes of muscle weakness since childhood, associated
Nephrology Self-Assessment Program - Vol 20, No 2, January 2022 165

with physical exertion or exposure to cold. Episodes Which of the following is most likely to be an evi-
of weakness have worsened as he has gotten older dence-based approach to her hyperkalemia?
and are associated with muscular pain after episodes. A. Sodium zirconium cyclosilicate should not be cho-
During the most recent episode his potassium was 6.5 sen as it is effective in reducing potassium in non-
mEq/L and was associated with electrocardiographic dialysis CKD patients but not in dialysis patients
changes, with normal chloride, sodium, and bicarbon- B. Sodium zirconium cyclosilicate should be chosen
ate. His condition was subsequently evaluated by his for this patient rather than patiromer as it is less
pediatrician, who noted normal potassium of 4.5 likely to cause edema
mEq/L and normal renin and aldosterone concentra- C. Sodium zirconium cyclosilicate also removes
tions. The patient notes that his late father used to hydrogen ions and may increase serum
describe the exact same problem, as do his uncle and bicarbonate
cousin. In addition, his younger sister is starting to D. Sodium zirconium cyclosilicate works quickly and
have the same symptoms. has been shown to provide added benefit to insu-
lin and glucose in the treatment of acute severe
What is the most likely underlying cause of hyperkalemia
hyperkalemia?
A. A loss of function mutation of the gene encoding
the epithelial sodium channel Article 5: Metabolic Acidosis
B. A mutation of WNK (with no lysine) kinase result- 20. A 74-year-old man with type II diabetes mellitus
ing in increased expression of the sodium-chloride and heart failure with reduced ejection fraction
(NCCT) cotransporter and inhibition of potassium presents to the emergency department with a
channels in principal cells 1-week history of fatigue, vomiting, headache, and
C. A mutation in the SCNA4 gene for the alpha subu- abdominal pain. He is found to have AKI, with
nit of the skeletal muscle cell sodium channel creatinine increased to 2.3 mg/dl from baseline
D. A mutation in genes encoding proteins that modu- 1.2 mg/dl. Other laboratory results show Na1
late WNK activity, including Kelch-like 3 and 134mEq/L, K1 4.0mEq/L, Cl2 98mEq/L, HCO32
Cullin 3 15mmol/L, BUN 68mmol/L, glucose 196mmol/L.
Lactate is 1.5 mEq/L. On examination, he is in
18. You are invited to speak at a symposium sponsored hemodynamically stable condition, with a mildly
by the Federal Emergency Management Agency tender abdomen, but the results of physical exami-
(FEMA) on emergency preparedness in urban centers. nation are otherwise unremarkable. For his diabe-
A FEMA council member has heard that hyperkalemia tes, he takes insulin and another oral antidiabetic
is a major cause of death of victims with crush injury medication that he cannot recall the name of.
sustained as a result of being buried under rubble in He reports that his blood sugars have been very
earthquakes. She asks you how to best determine well controlled, such that his insulin was recently
hyperkalemia, particularly when there are multiple reduced by 50%.
victims and time to treat is limited.
In considering this case, which of the following
What is your response? statements is most accurate?
A. Measuring potassium in serum is faster than mea- A. Metformin is the most likely culprit agent for aci-
suring potassium in whole blood dosis in this case
B. A deep learning algorithm using an electrocardio- B. The recent reduction in insulin dosing may be the
gram is able to exclude hyperkalemia with accept- precipitant for this clinical presentation
able accuracy C. The patient’s near-normal blood glucose excludes
C. Serum potassium is usually lower than plasma ketoacidosis as a cause of this presentation
potassium D. This patient should be given an intravenous infu-
D. Measuring potassium in whole blood is not as sion of isotonic bicarbonate
accurate as measuring potassium in serum or
plasma 21. The internal medicine residency director asks you to
lead a teaching session for medicine residents on
19. A 79-year-old woman with ESKD routinely presents drugs and the kidney. She references a patient recently
on Tuesday mornings with predialysis potassium 6.8 admitted with zidovudine-induced metabolic acidosis
mEq/L. She undergoes dialysis three times weekly for and asks that you include an overview of drug-
4 hours via a fistula. Her Kt/V is 1.7. induced acidosis.
166 Nephrology Self-Assessment Program - Vol 20, No 2, January 2022

With regard to drug-induced metabolic acidosis, A. Switching to a diet of alkali-producing fruits and
which of the following is correct? vegetables may delay progression to hypobicarbo-
A. Topiramate is associated with urine crystals and natemic acidosis
with crystalline stone formation B. In observational studies, up to 40% of patients
B. Valproate is highly protein bound and is not ame- with CKD stage III have metabolic acidosis
nable to removal with hemodialysis C. Decreases in urine citrate suggest H1 retention in
C. Concomitant use of antibiotics has been associated patients with CKD and have been shown to trend
with increased risk of pyroglutamic acidosis from closely with changes in serum [HCO32]
acetaminophen use D. Inhibitors of the renin angiotensin system have
D. Linezolid causes a non-anion gap by blocking car- been shown to be protective against the develop-
bonic anhydrase ment of acidosis

22. A 37-year-old male patient with CKD stage IV comes 24. A 66-year-old woman with a history of hypertension,
for clinic follow-up. At last review, [HCO3] had CKD stage IV, and dyslipidemia is seen at renal clinic.
decreased to 19 mEq/L, and NaHCO3 650 mg twice On review of her laboratory results, total CO2 has
daily was initiated. Today, he reports abdominal decreased to 20 mEq/L. You advise her to increase her
bloating and some nausea after taking the medica- intake of alkali-generating fruits and vegetables, but she
tions. His BP is well controlled at 122/68 mm Hg, and is reluctant to make dietary changes. As an alternative,
on examination he has trace lower limb edema. He you suggest that she start oral bicarbonate therapy.
has read that NaHCO3 may promote fluid retention
and is reluctant to continue taking it. He asks for an Regarding the benefits of alkali therapy in CKD,
explanation why it is necessary. which of the following is correct?
A. In patients with CKD stage III to stage V, treat-
Which of the following is correct with regard to ment of metabolic acidosis has been associated
treatment of metabolic acidosis in CKD? with reduction in progression to ESKD
B. Bicarbonate therapy has not been associated with
A. A meta-analysis of treatment with NaHCO3 for met- improvements in mortality in randomized con-
abolic acidosis did not show any increase in require- trolled trials (RCTs)
ment for antihypertensive or diuretic agents C. Patients with uncontrolled hypertension and
B. Increased ammoniagenesis resulting from acidosis advanced heart failure have been enrolled in RCTs
leads to increases in endothelin-1 (ET-1) levels investigating the use of NaHCO3 in CKD
C. In renal transplant recipients, metabolic acidosis D. In an RCT, treatment with veverimer was associ-
has been associated with increased risk of graft ated with improved [HCO32] but no significant
failure, but not cardiovascular events
improvement in physical function scores
D. Alkali therapy has been shown to reduce urine
ET-1 and aldosterone levels
Article 6: Metabolic Alkalosis
23. A 54-year-old woman with CKD stage III, eGFR 45 25. A 22-year-old woman presents to her primary care
mL/min, secondary to IgA nephropathy comes to physician with weakness and fatigue that has been
clinic for review. She also has type II diabetes mellitus, worsening for the past few months. Her medical his-
obesity with BMI 35 kg/m2, and hypertension. Her tory is significant for mild depression. She does not
laboratory results show BUN 24 mmol/L, creatinine smoke cigarettes daily but drinks alcohol on week-
1.8 mg/dl, Na 135 mEq/L, K 4.3 mEq/L, Cl 102 mEq/ ends. She is currently not sexually active. She denies
L, HCO3 23 mEq/L. Urine microalbumin/creatinine recreational drug use. Medications include a daily
ratio is 200 mg/g. BP is 122/74 mm Hg while she multivitamin. Family history includes hypertension
takes valsartan 160 mg daily, and for her diabetes, she and diabetes in her mother and father.
takes dapagliflozin and insulin and has an HbA1c of
Physical examination: weight 52 kg, pulse 84 beats/
6.8%. She is endeavoring to make lifestyle changes
min, BP 110/70 mm Hg, temperature 37 C. Dentition
and lose weight to improve her overall health but asks
is normal, no pallor, no icterus. Lungs are clear bilater-
if you have other suggestions that may reduce the risk
ally. Heart reveals normal S1 and S2, 2/6 ejection sys-
of CKD progression.
tolic murmur at the left sternal border. Abdomen soft,
bowel sounds present. Extremities have no edema,
With regard to acidosis in early-stage CKD, which of strong peripheral pulses. Neurologic examination
the following is correct? reveals no focal problems.
Nephrology Self-Assessment Program - Vol 20, No 2, January 2022 167

Initial laboratory results He is given bumetanide 2 mg intravenously, three times


a day, with improvement in his initial laboratory results:
Study Result Reference Range
Study Day 1 Day 4 Reference Range
Leukocyte count 6000/ml 4000–11,000
Hemoglobin 12.9 g/dl 12–16 Leukocyte count 6000/ml 8000/ml 4000–11,000
Platelets 189,000/ml 150,000–450,000 Hemoglobin 10.9 g/dl 11.8g/dl 12–16
Serum Platelets 189,000/ml 200,000/ml 150,000–450,000
Sodium 142 mEq/L 136–145 Serum
Potassium 2.9 mEq/L 3.5–5.0 Sodium 142 mEq/L 144 mEq/L 136–145
Chloride 98 mEq/L 98–106 Potassium 3.9 mEq/L 3.1 mEq/L 3.5–5.0
Total CO2 38 mEq/L 23–30 Chloride 108 mEq/L 92 mEq/L 98–106
BUN 8 mg/dl 8–20 Total CO2 28 mEq/L 40 mEq/L 23–30
Creatinine 0.6 mg/dl 0.5–1.1 BUN 20 mg/dl 8 mg/dl 8–20
Random glucose 98 mg/dl Creatinine 1.4 mg/dl 0.6 mg/dl 0.5–1.1
Magnesium 1.8 mg/dl 1.5 mg/dl 1.7–2.2
Urine toxicology screen results are negative. Electro- B-natriuretic 800 pg/ml 450 pg/ml ,100
cardiogram reveals sinus rhythm. Urine electrolytes: peptide
spot urine K1 50 mEq/L, Cl2 8 mEq/L, Na1 52 mEq/
L. Plasma renin activity is elevated. plasma aldoste-
What is the next best step in the management of this
rone concentration is elevated.
patient’s metabolic alkalosis?
On the basis of these findings, of the following, A. Intravenous normal saline
what is the most likely cause of the patient’s hypo- B. Discontinue bumetanide
kalemic metabolic alkalosis? C. Intravenous 0.3 N hydrochloric acid
D. Oral spironolactone
A. Glucocorticoid-remediable hyperaldosteronism
B. Surreptitious vomiting
27. A 75-year-old man is admitted to the intensive care
C. Gitelman syndrome
unit and intubated for acute respiratory failure caused
D. Diuretic abuse
by exacerbation of severe chronic obstructive pulmo-
26. A 75-year-old man is admitted to the cardiac intensive nary disease with altered mental status. He is known
care unit for exacerbation of acute congestive heart to be a chronic CO2 retainer according to previous
failure. His medical history is significant for ischemic blood gas evaluations. You are consulted on day 3 for
cardiomyopathy with an ejection fraction of 45%, evaluation of acid-base derangements.
insulin-dependent diabetes mellitus, hypertension, His medical history is notable for well-controlled
chronic gastritis, and depression. He has a 10-pack- hypertension with a single medication, gastritis, and
year smoking history but quit 10 years ago. He drinks osteoarthritis. Review of his medications includes aspi-
occasionally and has no history of recreational rin, lisinopril, pantoprazole, ipratropium bromide, and
drug use. Home medications include amlodipine 10 albuterol sulfate nebulizer, intravenous methylpred-
mg daily, bumetanide 1 mg twice daily, lisinopril 20 nisolone, intravenous piperacillin-tazobactam, intrave-
mg daily, omeprazole 20 mg daily, metoprolol 25 mg nous propofol, and furosemide.
twice daily, and a multivitamin.
Weight 73 kg, BP 150/54 mm Hg, heart rate 102 beats/
Physical examination results: weight 102 kg, pulse 84 min, temperature 38 C. He is sedated. O2 saturation on
beats/min, BP 124/70 mm Hg, and temperature 40% FiO2 is 96%. Jugular venous distention is absent.
37 C. Dentition is normal, no pallor, no icterus. Jugu- Heart examination reveals tachycardia and a loud S4.
lar venous distension is noted 8 cm above sternal Lungs have decreased breath sounds bilaterally. Abdo-
angle. Lungs have coarse breath sounds and rales men is soft with active bowel sounds. Extremities have
bilaterally. Heart reveals normal S1 and S2, S3 is pre- trace edema and 11 peripheral pulses.
sent, 2/6 ejection systolic murmur is audible at the
left sternal border. Abdomen soft, bowel sounds pre- Arterial blood gases on admission: pH 7.1, PaCO2 92
sent. Extremities have 21 pitting edema, 11 periph- mm Hg, PaO2 72 mm Hg on 6 L/min oxygen via nasal
eral pulses. Neurologic examination reveal no focal cannula; 48 hours later (while still intubated): pH 7.48,
problems. PaCO2 42 mm Hg, PO2 84 mm Hg on 40% FiO2.
168 Nephrology Self-Assessment Program - Vol 20, No 2, January 2022

Laboratory results Peripheral pulses are noted to be weak, extremities are


cold, and skin is mottled. Neurologic examination is
Reference noteworthy for a depressed mental status.
Study Day 1 Day 3 Range
Initial laboratory values
Leukocyte count 12,000/ml 8000/ml 4000–11,000
Hemoglobin 10.9 g/dl 11.8 g/dl 12–16 Study Result Reference Range
Platelets 189,000/ml 200,000/ml 150,000–450,000
Serum Leukocyte count 12,000/ml 4000–11,000
Sodium 142 mEq/L 144 mEq/L 136–145 Hemoglobin 13.3 g/dl 12–16
Potassium 3.8 mEq/L 3.9 mEq/L 3.5–5.0 Platelets 120,000/ml 150,000–450,000
Chloride 102 mEq/L 92 mEq/L 98–106 Serum
Total CO2 34 mEq/L 33 mEq/L 23–30 Sodium 138 mEq/L 136–145
BUN 20 mg/dl 30 mg/dl 8–20 Potassium 4.0 mEq/L 3.5–5.0
Creatinine 1.1 mg/dl 1.2 mg/dl 0.5–1.1 Chloride 103 mEq/L 98–106
Magnesium 1.8 mg/dl 1.5 mg/dl 1.7–2.2 Total CO2 6 mEq/L 23–30
B-natriuretic 300 pg/ml 250 pg/ml ,100 BUN 45 mg/dl 8–20
peptide Creatinine 2.0 mg/dl 0.5–1.1
Random glucose 98 mg/dl
Urinalysis: pH 5.2, specific gravity 1.015, negative for Workup reveals arterial blood gases with PO2 90 mm
protein, blood, glucose. Chest X-ray: hyperinflated lung Hg, pH 7.06, PCO2 18 mm Hg, [HCO32] 5 mEq/L,
fields, mild pulmonary venous congestion. and serum lactate 20 mEq/L. The patient is given
broad-spectrum antibiotics and because of the low pH
What is the most likely cause of this patient’s elevated [HCO32] is administered (5% dextrose in half normal
blood pH on the second blood gas determination? saline with 75 mEq/L NaHCO3).
A. Acute respiratory alkalosis
Over the next 2 days, the patient’s hemodynamics sta-
B. Posthypercapneic metabolic alkalosis
bilize, and arterial blood gases show PO2 92 mm Hg,
C. Intravenous methylprednisolone-induced alkalosis
pH 7.47, PCO2 48 mm Hg, [HCO32] 34 mEq/L. Urine
D. Nasogastric suctioning
pH is 8.
28. A 65-year-old man presents with the sudden onset of
On the basis of the patient’s course, which of the fol-
severe weakness and fever developing over the previ-
lowing is true regarding the patient’s metabolic
ous 24 hours. The patient had been followed up by his
alkalosis?
primary care physician before admission, and earlier
laboratory values and BP have all been normal. Rele- A. It is due to Gitelman syndrome that was not
vant medical history is that the patient has had diffi- apparent on admission
culty urinating, with urinary retention and nocturia. B. It is due to primary hyperaldosteronism that was
Previous workup has revealed benign prostatic hyper- not apparent on admission
trophy. C. It was generated by metabolism of retained lactate
to bicarbonate and is now maintained by volume
Early on the day of admission the patient awoke feel- contraction
ing weak and warm and experienced burning on uri- D. It was generated by metabolism of retained lactate
nation. His temperature was elevated, and he came to to bicarbonate and should spontaneously correct
the emergency department. Physical examination
shows a patient who is pale and confused. Weight 55 29. A 45-year-old woman presents with new onset of
kg, pulse 120 beats/min, BP 70/50 mm Hg, tempera- hypertension but is otherwise healthy. Physical exami-
ture 40 C. Lungs are clear bilaterally. Heart reveals nation results are is noteworthy for a BP of 160/105
normal S1 and S2, abdomen is soft with some supra- mm Hg and are otherwise normal, with no evidence
pubic tenderness and right flank pain tenderness. of volume depletion or edema.
Nephrology Self-Assessment Program - Vol 20, No 2, January 2022 169

Laboratory values are noteworthy for a BP of 155/110 mm Hg and are


otherwise normal, with no evidence of volume deple-
Study Result Reference Range tion or edema.
Laboratory values
Leukocyte count 5000/ml 4000–11,000
Hemoglobin 13.9 g/dl 12–16 Study Result Reference Range
Platelets 230,000/ml 150,000–450,000
Serum Leukocyte count 5500/ml 4000–11,000
Sodium 145 mEq/L 136–145 Hemoglobin 14.2 g/dl 12–16
Potassium 2.5 mEq/L 3.5–5.0 Platelets 200,000/ml 150,000–450,000
Chloride 97 mEq/L 98–106 Serum
Total CO2 34 mEq/L 23–30 Sodium 145 mEq/L 136–145
BUN 8 mg/dl 8–20 Potassium 2.3 mEq/L 3.5–5.0
Creatinine 0.6 mg/dl 0.5–1.1 Chloride 96 mEq/L 98–106
Random glucose 98 mg/dl Total CO2 35 mEq/L 23–30
BUN 8 mg/dl 8–20
Creatinine 0.6 mg/dl 0.5–1.1
A random plasma renin is low, and a plasma aldoste- Random glucose 98 mg/dl
rone is high.

On the basis of these findings, what is the most Random plasma renin and aldosterone are low. Corti-
likely diagnosis? sol levels are normal.
A. Gitelman syndrome
B. Renovascular hypertension
C. Primary hyperaldosteronism On the basis of these findings, what is the most
D. Liddle disease likely diagnosis?
A. Surreptitious diuretic abuse
30. A 35-year old man presents with new onset of hyper- B. Syndrome of apparent mineralocorticoid excess
tension but is otherwise healthy. History was noncon- due to glycyrrhizic acid
tributory with the exception of extensive use of C. Primary hyperaldosteronism
chewing tobacco. The results of physical examination D. Villous adenoma

You might also like